You are on page 1of 290

1000X11 3 READING

m /
c o
o c.
n g
m s
i n g
a k
e 01

ic sp
e
//to
PART 5

tp:
A PART 6

ht {trn: PART 7

Self X1IH Z\±E.

EllAE 2 SVOJAHJ
1, Iftll mo4 i|§ 2ii3L|s? □ Oil
2. Answer Sheet. SS. ^u|SHaL|fi? □ o||
B.Aiiiulsgfcfa? aoil
SE Sm ySE|2J°et 111 $ E)| ABl A|ftL|C|-,
TEST 011 §611 §e!2| il§TS ?Pt6|| § ?•, A's i|(p.20~21 )°£ g iWHi Stsjas ^611

m i-0|t a\X\t70g
EflAS mB # 75gO|Dl, A|S gS 15^ gcj gg n| t^|| ggjg o|6t, A|S®L|C|-.
READING TEST
In this section, you must demonstrate your ability to read and comprehend English. You will be given a
variety of texts and asked to answer questions about these texts. This section is divided into three parts
and will take 75 minutes to complete.
Do not mark the answers in your test book. Use the answer sheet that is separately provided.

PARTS
Directions: In each question, you will be asked to review a statement that is missing a word or phrase.
Four answer choices will be provided for each statement. Select the best answer and mark the
corresponding letter (A), (B), (C), or (D) on the answer sheet.

© PART 5 TW *0I m 11©

101. Delegates attending the international trade 105. The Trescott Chamber of Commerce
m /
convention were provided with overnight
c o
local businessman Brian Larue at a
at Hotel Boswick. ceremony next week.

o c.
(A)
(B)
accommodated
accommodates
(A)
(B) to honor
n g
was honoring

(C)
(D)
accommodating
accommodations
m s
(C)
(D)
will honor
honor

i n g
k
102. Charles Wang submitted a resume that 106. Please review your contract, and feel free

e a
his years of experience in advertising. to ask us questions you might have,

sp
(A) emphasized (A) few
(B) enclosed (B) either
(C) estimated

e ic (C) any

//to
(D) employed (D) much

tp:
103. Beginning next week, Zumwalt, Inc.'s new 107. The manager announced that a few office
smartphone will be available for purchase

ht
supply would be delayed because of
at retailers across Korea and Japan. weather conditions.
(A) authorizing (A) shipment
(B) authority (B) shipments
(C) authorization (C) to ship
(D) authorized (D) shipping

104. Highway construction has been postponed 108. By adding extra flights between Dubai and
indefinitely as Hampshire County residents Istanbul, Euroblue Airlines will increase its
have to the plan. for this route by more than 50
(A) preferences percent.
(B) arrangements (A) endurance
(C) considerations (B) motivation
(D) objections (C) location
(D) capacity

24 se® tisej asowi • ^ Eto®?! MP3 Hackerslngang.com


109. Hamilton Hospital's purchase of a medical 115. A fee of $100 will be to office rental
scanning device will permit doctors to payments that are more than two weeks late.
diagnose patients more than ever (A) applied
before. (B) applying
(A) precise (C) apply
(B) precisely (D) application
(C) precision
(D) preciseness 116. Last month's issue of Wise Finance
an exclusive interview with Chuck Granville,
110. a bank has approved Flynn Co.'s founder of financial firm Granville
business loan, the company can begin its Investments.
planned expansion. (A) subscribed
(A) Even if (B) admitted
(B) As though (C) featured
(C)
(D)
Consequently
Now that
(D) dedicated

m /
117. Broadcasting the
o
latest news 24
c
c.
111. Online reviewers praise Alpha Fashion's hours a day, cable station CPP allows
deliveries and its generous return
policy. o
viewers to keep up-to-date on world events.

g
(A) tangible
(A)
(B)
s
fewer
very n
(B) measured
(C) timely
g m
(C)
(D)
far
more
(D)subsequent

k i n
a
118. The software that New-Tech offers

e
112. Though Bruce Guthrie's concert is planned remove security risks and improve the

sp
August 1, the organizers have said performance of computers.

ic
that this is subject to change. (A) increments
(A) at
e (B) enhancements

//to
(B) for (C) certificates

tp :
(C)
(D)
since
in
(D) exceptions

ht
119. Mr. Evans took a taxi to avoid being late,
113. The public speaking course presents but the theater show had begun by
trainees with opportunities the time he arrived.
themselves in front of a group. (A) usually
(A) will express (B) seldom
(B) to express (C) hourly
(C) are expressing (D) already
(D) expressed
120. Notices residents of possible power
114. The speaker at the Westicon Literature outages were mailed out a week before
Seminar told an story that had the crews began work on the power lines.
entire audience laughing in enjoyment. (A) inform
(A) amusing (B) information
(B) amusement (C) informed
(C) amused (D) informing
GO ON TO THE NEXT PAGE.
(D) amuse

TEST 1 PARTS 25
121. Mr. Clemons' work was so impressive that 126. Local water quality is - to improve
he was made a senior manager only once the sewage treatment center is
six months with the company. upgraded.
(A) on (A) grown
(B) after (B) limited
(C) while (C) bound
(D) owing to (D) acquired

122. In anticipation of an increase in visitors 127. Charles Kapoor was not considered a
during the summer holiday, the Shoreline suitable candidate for a financial analyst
Inn decided to hire additional position he possessed exceptional
housekeeping staff. investment experience.
(A) adversely (A) accordingly
(B) uncontrollably (B) in spite of
(C)
(D)
temporarily
relatively
(C)
(D)
as if
even though
m /
c o
c.
123. Although a graduate degree is a 128. The tour bus did not stop at Sheffield
requirement for the position, none of
who responded to the job announcement
g o
Stadium, so its passengers could only take
pictures as they drove it.
have one.
s
(A) until n
(A)they
(B)these
g m
(B)
(C)
past
onto
(C) themselves
k i n (D) within
(D) those

e a 129. Mercer Incorporated carefully goes over

sp
124. Travel writer Arthur Chaplin will give a its customer surveys and market

ic
short presentation tomorrow the trip research before making important

e
described in his new book, Walking in Peru. decisions.

//to
(A) along (A) as well as

tp :
(B)
(C)
regarding
in exchange for
(B)
(C)
both
between

ht (D) by means of

125. Having written multiple books on the


(D) neither

130. For the purpose of increasing sales,


effects of global warming, Ms. Black is Mendelbaum Electronics is giving a prize
widely regarded as a expert on to js the 100th person to buy a
environmental issues. refrigerator.
(A) naive (A) whichever
(B) notable (B) another
(C) tolerant (C) whoever
(D)mundane (D) someone

26 *|^§MTi|g Hackers.co.kr
PART 6
Directions: In this part, you will be asked to read four English texts. Each text is missing a word,
phrase, or sentence. Select the answer choice that correctly completes the text and mark the
corresponding letter (A), (B), (C), or (D) on the answer sheet.

© PART6 iPof«0l APd 8§

Questions 131-134 refer to the following e-mail,

To: MoRay Productions <inquiries@mcrayprod.com>


From: Laura Hahn <lhahn@delrio.com>
Subject: Proposal
Attachment: Product list

m /
c o
c.
To Whom It May Concern,

My retail business, Delrio, currently needs a new


o
for its products. Delrio mostly sells
g
s n
souvenir items, like canvas bags, pouches, and umbrellas, and I was wondering if you could

g m
produce some goods I've designed for an upcoming trade fair. This event will provide an

k i n
opportunity for my company to attract customers and establish a positive with them.

a
132.

e
It is therefore very important for the merchandise I sell to appeal to my target market.

133.
ic sp
, everything must be convenient to use and made of durable materials. I have attached

e
a list of products I intend to feature and would like a sample of each one.

//to
134.

tp :
Sincerely,

ht
Laura Hahn, Delrio CEO

131. (A) manufacture 134. (A) Perhaps we can discuss the issue
(B) manufacturer |||| further at the trade fair.
(C) manufacturing (B) If I like the results, I'd be willing to work
(D) manufactured with you on a regular basis.
(C) Once I make the design changes, I will
132. (A) nuisance send you the order.
(B) attitude (D) Your quoted prices are higher than
(C) reputation I have budgeted for the items.
(D) condition

133. (A) In this way


(B) in contrast
(C) In particular GO ON TO THE NEXT PAGE.
(D) In reality
TEST 1 PART 6 27
Questions 135-138 refer to the following notice.

Notice to all Northrup Apartment residents:

As you know, we have done some facility renovations for the first time. A children's
135.
playground in the compound has been added, and the old water pipes have been changed.

Also, new walkways have been put in to replace the ones that had holes.

Northrup Apartment management you a detailed report of the amount that was spent
136.
on materials and labor. You all should have had a chance to go over it carefully by now.

Costs will be divided as we earlier during the management-resident meeting. The

management will pay 60 percent of the costs and residents will pay the remainder. To offset
m /
c o
c.
the share you have claimed responsibility for, your monthly amenities and maintenance bills

will increase by $25 beginning next month.


g o
n
138.

m s
135. (A) clever
i n g
138. (A) Those who haven't paid the charge yet
(B)
(C)
repetitive
conservative
a k |||j| will receive a fine.
(B) The additional fee will be charged until

e
sp
(D) extensive the expenses are all paid for.
(C) Please let us know when you will be
(A)
(B)
will send
has sent
e ic using the new services.
(D) This will be imposed after the facility

//to
(C) to be sent improvements have been approved.

tp :(D) is sending

ht (A)
(B)
(C)
declined
agreed
encompassed
(D) attempted

28 eolith^yo^pl mps Hackerslngang.com


Questions 139-142 refer to the following information.

Win a free ski lift ticket from Schuler!

Schuler Gas its customers a chance to win free ski lift tickets. They are worth $149
139.
each and good for the entire day at any of the 15 resorts participating in this promotion.

From now until December 26, simply buy over 38 liters of fuel at any Schuler gas station

nationwide. Then leave the receipt, complete with your contact information on the back, in the

box the exit. Please note that the tickets will be valid for a limited time. .
140. 141.
Furthermore, they cannot be exchanged for cash. Our raffle on January 1 will determine the

winners, who can - the tickets at the station the day after we call them. Alternatively,
m /
c o
c.
winners can arrange to have them mailed.

g o
139. (A)
(B)
has offered
was offering |jlj|
s n
141. (A) You can save more if you buy ski lift
tickets for five or more days.
(C) will have offered
g m
(B) Each one must be used by the date
(D) is offering

k i n and time stamped on it.


(C) Discounts are not applicable if
140. (A) inside

e a coupons are past their expiration

sp
(B) beside dates.
(C) upon (D) The country's ski resorts are among
(D) without

e ic the best the world has to offer.

//to
142. (A) claim

tp : (B)
(C)
deliver
redeem

ht (D) pursue

GO ON TO THE NEXT PAGE,

TEST 1 PART 6 29
Questions 143-146 refer to the following article.

Abbotsburg, May 9 — After a year of construction, the Abbotsburg Community Center will be

unveiled next Friday. Costing nearly $5 million, the center is considered a luxury by

some who contend that the money should have been used to local roads instead.
144.

Most residents, however, believe the center is a Karen Petrowski, has been
145. 146.
appointed the center's director said, "It will provide residents with the vital social, recreational,

and educational opportunities they require." And she added, "We believe that, in time, the

entire community will consider it a worthwhile place."

m /
c o
c.
143. (A) The profitability of the venture could 145. (A) commitment
||||| affect its completion. (B) necessity
(B) It has so far proven quite popular with (C)
g
responsibility
o
people of all ages.
(C) But there was some opposition to the
(D)

s n
coincidence

project when it started.


g m
(A) instead

n
(D) The city has become a model for other (B) who
cities to follow.
k i (C) later

144. (A) repaired


e a (D) also

(B)
(C)
repairing
repairs
ic sp
(D) repair
e
p : //to
ht t

30 Hackers.co.kr
PART7
Directions: In this part, you will be asked to read several texts, such as advertisements, articles, instant
messages, or examples of business correspondence. Each text Is followed by several questions. Select
the best answer and mark the corresponding letter (A), (B), (C), or (D) on your answer sheet.

© PART 7 m -i0! 54-ir

Questions 147-148 refer to the following advertisement.


-r".I-.';

Pick of the Week

Are you moving out of your parents' home for


the first time to work or study and worried about
m /
what you'll eat? Well, don't just rely on expensive takeout food

c o
c.
or the boring university cafeteria. Instead, learn how to make
healthy and nutritious meals for yourself! Celebrity chef
Alexandra Maldini—star of television's Cooking the Italian
g o
s n
Way~\\as just released her latest edition of The Young Person's

g m
Guide to Eating Weil With recipes ranging from simple five-minute
snacks to main courses that will impress your friends, everyone

i n
will find something to love in Maldini's book. You can grab a
k
branch.
e a
copy of it, or her other books, at any Written World Bookstore

ic sp
e
p
(A) : //to
147. What is the advertisement mainly about?
A cooking class
148. What is suggested about Alexandra
Maldini?

ht t
(B)
(C)
A television program
A recipe book
(A)
(B)
She
She
has released several books.
gives lectures at a university.
(D) A school cafeteria (C) She operates a chain of restaurants.
(D) She has retired from a television show.

GO ON TO THE NEXT PAGE^

TEST1 PART? 31
Questions 149-150 refer to the following e-mail.

To: All staff <all(5)bendacorp.com>


From; Marion Jessop <m.jessop@bendacorp.com>
Subject: Quick reminder
Date: February i

Hello everyone,
Please remember to follow the standard procedure when requesting new office supplies. It allows us
to keep track of how much we consume on a regular basis. As a reminder to all employees, be sure to
fill in a detailed requisition form before you make a request. Forms must list the items you wish to
buy, including their quantities and prices. They must also be signed by a departmental supervisor and
by me. Purchases made without an official form will not be reimbursed.
Thank you,
m /
Marion Jessop
c o
Purchasing manager

o c.
n g
149. What is the e-mail mainly about?

m s
150. What is indicated about departmental
supervisors?
(A)
(B)
Placing requests for leave
Relocating to a new office
i ng (A) They are authorized to buy materials
(C)
(D)
Replenishing basic office necessities
Ensuring deliveries are complete
a k on their own.
(B) They are responsible for approving

e
sp
employees' purchases.
(C) They may sign requisition forms on

e ic Ms. Jessop's behalf.


(D) They must work with a budget

//to
determined by Ms. Jessop.

tp :
ht

32 safoj ooi^. yo]op|xr □! yojop| hps Hackerslngang.com


Questions 151-152 refer to the following text messages.

Nicky Candela 11 ;01 A.M.


Jacob, there have been some changes. Could you stop
for some supplies on your way to work?

Jacob Brosky 11:04 A.M.


Oh, sure thing. But I thought that we got everything
yesterday.

Nicky Candela 11:07 A.M.


I just got a call from Ms. Arden's wedding planner. She
wants to add 40 cupcakes to her order. I've already put
m /
them in the oven, but we're all out of decorative silver

c o
c.
beads. Also, we need more yellow icing to mix the right
colors.

g o
Jacob Brosky

s n
11:08 A.M.
OK, so we need at least two bags of silver beads and four
tubes of yellow icing, I'm guessing.

g m
Nicky Candela
k i n 11:10 A.M.

e a
Exactly. We'll finish that this morning and then begin
work on the food items for Mr. and Mrs. Morton's

ic sp
anniversary party.

e
//to
l J

tp :
ht
151. What most likely is Ms. Candela's job? 152. At 11:08 a.m., what does Mr. Brosky most
(A) A store manager likely mean when he writes, "I'm guessing"?
(B) A restaurant critic (A) He would like to confirm the details of
(C) A wedding planner a request.
(D) An event caterer (B) He believes someone already went to
a store.
(C) He could not understand a request.
(D) He wants to check if he already has
some materials.

GO ON TO THE NEXT PAGE.

TEST1 PART 7 33
Questions 153-154 refer to the following ticket.

Seafoam Park

Admit one adult [age 18-65]

PAID : $34.00

Our park opens at 7 a.m.; every day of the week.


Guests may stay in the park until 9 p.m. on
weekdays
and 8 p.m. on weekends.

Parking is free for visitors.

BONUS COUPON
m /
c o
c.
This voucher can be used at Seaview Restaurant

10 percent off any lunch special

g o
This coupon is valid until July 15.

s n
g m
153. When does Seafoam Park close
k i n 154. What information is NOT included on the
on Sundays?

e a ticket?

sp
(A) At 7 p.m. (A) The price of the ticket

ic
(B) At 8 p.m. (B) The expiration date of a voucher
(C) At 9 p.m.
e (C) The location of a parking lot

//to
(D) At 10 p.m (D) The age range of ticket holders

tp :
ht

34 • tHSSMmis Hackers.co.kr
Questions 155-157 refer to the following notice.

o ^ ©
Notice for Aston Towers Residents
This apartment building is nearly 30 years old, and it has been quite some time since it
underwent major improvements. Moreover, the Residents' Association has been asking
management to upgrade the apartments for several months now, and we agree that the time has
come to make these changes. However, this may lead to some disturbances for a short time.
The renovations are planned to take place over three stages;
« Stage One, April 18 to May 11: Renovating the exterior of the building
• Stage Two, May 13 to June 24: Improving the interior corridors
• Stage Three, June 30 to September 1: Necessary improvements to individual units
The first two stages will generate a certain amount of construction noise during the day,
between the hours of 10 a.m. and 4 p.m. For the repairs to individual units, we will be in touch
m /
o
with you shortly to determine the necessary work and arrange dates for repairpersons to visit
c
c.
your unit.
Thank you for your understanding and cooperation.
g o
Aston Towers Management
s n
g m
155. What is the purpose of the notice?
k i n
157. What will the Aston Towers Management

e
(A) To announce the times of a communitya do soon?
event

ic sp
(B) To outline requirements for potential
(A)
(B)
Add a garden to an apartment building
Repair a road that passes a property
renters
e (C) Contact prospective tenants

//to
(C) To tell about upcoming maintenance (D) Schedule visiting times with residents

tp :work
(D) To provide assignment details to

ht
construction workers

156. What is stated about the Residents'


Association?
(A) It has been demanding changes.
(B) It has meetings every month.
(C) It is planning further construction.
(D) It is recruiting volunteers for a project.

GO ON TO THE NEXT PAGE,

TEST 1 PART? 35
Questions 158-161 refer to the following article.

Bluecoats Look to a Brighter Future

Last night the management of the Kansas City Bluecoats finally made the long-expected
announcement that the team will be moving into a new stadium in a suburb just within the city limits
of Kansas City. — [1] —.

The Bluecoats were one of the top teams in American football two decades ago, winning the national
championship once and making it to the final game on another occasion. — [2] —. But recent years
have been more difficult, with disappointing placements in the league and a dwindling fan base that
has been attracted to other more successful sports teams in the region.

Moving will allow for cheaper ticket prices, more parking spaces, and easier access from other areas.
— [3] —. As for the team, it will get to train using state-of-the-art facilities. Work will begin on the

m /
currently unnamed new stadium this month and is expected to be finished within two years.

c o
c.
Kansas City mayor Lester Hickman celebrated the team's move, saying, "This is great news for the

[4] -•
g o
team, and we look forward to helping the Bluecoats rise back up to their previous levels of success."

s n
g m
158. What is the article mainly about?

k i n
160. What is true about the new stadium?
(A) The start of a football season
(B) The foundation of an athletic
e a (A) It will formally open after two months.
(B) It has yet to be given a name.
association

ic
(C) The building of a new suburb sp (C) It will be the site of a championship
game.

e
(D) The relocation of a sports team (D) It is located within Kansas City's

//to
downtown area.

t :
159. What is mentioned about the Kansas City

p
Bluecoats? 161. In which of the positions marked [1], [2],

ht (A) It previously won first place in a [3], and [4] does the following sentence
#
competition. best belong?
(B) It lost one of its corporate sponsors. "Fans and out-of-town supporters will be
(C) Its coach was recruited locally. better served by these changes."
(D) Its players are the highest paid in the
(A) [1]
league.
(B) [2]
(C) [3]
(D) [4]

36 £2^1 {JMqi soijiAj • gotepis 51 Soi^i mps Hackerslngang.com


Questions 162-165 refer to the following online chat discussion.

© Gina Adenan [10:09 A.M. I just got word from the boss. A manufacturing problem
discovered at our Maryland factory will lead to an immediate
recall of all T-20 and T-21 model toasters our company sold in
the past year.
Hal Anderson [10:12 A.M. I heard about that issue earlier this morning. A recall is going to
be a huge undertaking. We'll have to post a notice in some
major newspapers.
Oliver Lee [10:14 A.M. That's right. And we'll also have to put the same information on
our Web site.
Hal Anderson [10:16 A.M. That would be the best way to reach our customers abroad.
Katie, can you draft an announcement if I forward you all of the
details?

m /
Katie Ford [10:19A.M.

c o
Of course. I'll put it together as quickly as I can and get a list of

c.
press contacts from marketing.

o Gina Adenan [10:22 A.M.

g o
Thanks, Katie. Oliver, as you are running the call center, you'll
need to train your staff to handle inquiries regarding the recall.

n
There will be a lot of calls from the public, I'm sure.
s
Oliver Lee [10:25 A.M.

g m
I'll get right on it. We dealt with a similar issue three years ago,
so I know what needs to be done.

o Gina Adenan [10:28 A.M.


k i n
Okay, thanks everyone. It sounds like we should be able to

a
handle everything.

e
ic sp Send

e
//to
162. What is NOT mentioned as a way to 164. What can be inferred about Mr. Lee?
handle the recall?

tp :
(A) Broadcasting the news on television
(A) He has contacts with members of the
press.

ht
(B) Answering calls from customers (B) He will send an announcement to the
(C) Announcing information in some public.
publications (C) He has previously worked on a product
(D) Posting about the issue online recall.
(D) He writes promotional material for the
163. What is suggested about the company? company.
(A) It sells products internationally.
(B) It plans to launch a new toaster model. 165. At 10:25 a.m., what does Mr. Lee most likely
^||| mean when he writes, 'Til get right on it"?
(C) It conducted a press conference.
(D) It runs advertisements in local papers. (A) He will call the marketing department.
(B) He needs to hire more staff.
(C) He intends to consult a former
employee.
(D) He will start instructing the employees.

GO ON TO THE NEXT PAGE^

TEST 1 PART 7 37
Questions 166-168 refer to the following Web page.

www.visitindonesia.com

Home | Tours | Gallery | Contact Us

At Visit Indonesia, we offer a variety of tours to suit everyone visiting our beautiful country. — [1] —.
You can enjoy Indonesia's natural beauty, experience the bustle of cities, or relax on outings to our
ancient sites—all at the most reasonable prices around. Some of our most popular tours are
described below, and you may click on "more" to view a full listing of tours offered.
Hiking in Sumatra
Sumatra is an amazing place with magnificent volcanoes, exciting wildlife, and beautiful scenery.
Our trip will start in the hub city of Medan, and from there we'll trek to an orangutan sanctuary in
Bukit Lawang. We'll spend a night camping in the jungle before ending the trip with a journey to the
top of the Sibayak volcano. — [2] —.

m /
Experience Java

c o
c.
Java is one of the smallest of Indonesia's main islands, but it packs a lot in—and not just in the

g o
famous capital of Jakarta. — [3] —. We'll travel through the mountainous landscape of central Java
before making our way to Yogyakarta, a town containing the world's largest complex of temples,
dating back hundreds of years.
s n
The Beaches of Baii

g m
Bali is famous for its beaches, and we will pick you up from the city of Denpasar and take you

i n
straight to the south of the island. There, you can participate in snorkeling and scuba diving! We

k
a
also run day trips inland to see the rice paddies, lakes, and other attractions of the region. — [4] —.

e
sp
MORE

<g
e ic io

p :
Web page? //to
166. What is the main purpose of the 168. In which of the positions marked [1], [2],
|||| [3], and [4] does the following sentence

ht t (A)
(B)
To advertise travel packages
To collect feedback from customers
best belong?
"Outside the city awaits a world of natural
(C) To provide contact information and historical wonders."
(D) To recruit local tour guides
(A) [1]
167. What is NOT stated as an activity visitors (B) [2]
can do in Indonesia? (C) [3]
(D) [4]
(A) Scuba diving
(B) Visiting temples
(C) Surfing in the sea
(D) Camping in a jungle

38 Hackers.co.kr
Questions 169-171 refer to the following memo.

MEMORANDUM

To: All maintenance staff


From: Janice Lai, Supervisor
Subject: Daily report submissions
Date: December 16
I realize you all have demanding schedules, and we appreciate your hard work. But you
must comply with the policies regarding the completion of your daily reports in a timely
manner. Let me remind you that all report information must be fully entered by the end
of each work day for the hours that you spent on maintenance tasks. In November, I
received several time sheets more than two weeks late, and I still have some that have
yet to be provided.

m /
o
Additionally, for those reports that were submitted on time, I noticed that many were

c
c.
missing important details. Remember that these reports are used not only to track the
number of maintenance tasks, but also to determine how efficient we were at completing
them.
g o
s n
In order to address this issue, a new policy will go into effect next month. I will be
checking your reports weekly to ensure they are complete and posted on time on the

g m
company Web site. If there are any problems, I will follow up on them with you in person.

i n
I appreciate all of your cooperation in this and hope that these issues cease to continue

k
from now on.

e a
ic sp
e
169. Why was the memo written? 171. From when will the new measure be

//to
implemented?
(A) To explain a new log-in system

tp:
(B) To inform workers about regulations (A) From November

ht
(C) To announce a penalty for absences (B) From December
(D) To notify staff of scheduled (C) From January
maintenance (D) From February

170. What is indicated about the daily report?


(A) It must be signed by a supervisor.
(B) It should be drawn up as simply as
possible.
(C) It needs to be submitted in person.
(D) It can be used to evaluate work
efficiency.

GO ON TO THE NEXT PACE.

TEST 1 PART 7 39
Questions 172-175 refer to the following article.

Thoughts on Sharing Economy


By Adriana Solo

In recent years, mobile applications and Web sites have made it


easier and cheaper for people to do things like get a ride, find a place
to stay, or arrange a babysitter. Referred to as "the sharing economy,"
this new Internet-dependent system brings more wage-earning
opportunities to a wider range of people. However, as the practice
of purchasing services from individual people rather than
conventional businesses grows in popularity, some municipalities
have opted to regulate it, or even ban it outright.

Ghorinburg, for instance, has emerged as a strong opponent of the


m /
sharing economy and the local government has prohibited
c o
residents from participating in it. Mayor of the city, Robert

o
Smollet, who became mayor last year in a landslide victory, says c.
n g
that its advantages are not as great as they seem on the surface.

m s
"When you can hire someone to work on a freelance basis for a
lower rate, why bother going to a company that will charge much

i n g
more? Ultimately, the sharing economy can cause businesses to

k
close and the people who work for them to lose their jobs."

a
e
Ghorinburg does not stand alone, as other municipalities across

ic sp
the nation are following suit. In smaller cities, like Browenville,
Marog, and Quentintown, where local businesses play a vital role
e
//to
in the growth of the economy, restrictions have likewise been

:
adopted. Only time will tell whether attitudes remain this way, as

tp technology is certain to spread to even more aspects of our lives.

ht

40 £££! iiSX5! MP3 Hackerslngang.com


172. What is the main topic of the article? 174. What happened last year in Ghorinburg?
(A) The profitability of innovative services (A) A mobile application was introduced.
(B) Consumption patterns among online (B) A new mayor was elected to office.
shoppers (C) A business association held a protest
(C) The adoption of new training programs (D) A ban against the sharing economy
(D) Opposition to an economic trend was lifted.

173. What does the article suggest about the 175. What is mentioned about Browenville?
sharing economy? (A) Its residents are largely unwilling to
(A) It allows workers to charge more for adopt new technologies.
their services. (B) Local businesses are essential to its
(B) It has encouraged charitable giving in economic development.
a local community. (C) The local government encourages
(C) It benefits individuals more than participation in the sharing economy.

m /
traditional companies.

c o
(D) Its employment opportunities have

c.
(D) It can function without the Internet. been declining in recent years.

g o
s n
g m
k i n
e a
ic sp
e
p : //to
ht t

GO ON TO THE NEXT PAGE.

TEST 1 PART 7 41
Questions 176-180 refer to the following advertisement and form.

Bulk Land Supermarket


Make your weekly grocery shopping much quicker and easier by signing up today with Bulk
Land Supermarket! We operate differently from other grocery stores. Unlike most
supermarkets, we have no actual retail outlets. You can simply browse through our online shop
from the comfort of your home. After you place your order, we deliver it right to your door.
We also encourage our customers to buy in bulk. By purchasing large quantities of products,
you can save up to half off each item. To start browsing our wide range of groceries, go to www.
bulkland.com. Choose your ideal delivery time, and everything is done!
This month, we're offering a number of special deals. To begin with, if you book a delivery
before November 15, we'll give you a voucher for 5 percent off your next purchase. In addition,
we offer free shipping on all orders over $100. Finally, all customers who sign up this month and
m /
o
order a 5 kilogram bag of rice will be provided a free storage bin that will keep your grains fresh

c
c.
and dry.

g o
s n
Bulk Land Supermarket Online Order Form
g m
Name: Alexander Johnson
k i n
Customer account number: 054489
e a
Address: 2235 West Grovers Avenue, Phoenix, AZ 85007

Points accumulated: 2,300

ic
Order date: November 11 sp
e
//to
Product Number Product Name Price
P889 Whole wheat pasta, 5kg
:
$18

tp R426 Rice, 3kg $22

ht
T114 Canned chick peas, pack of 12 $15
T526 Canned chopped tomatoes, pack of 12 $11.50
B496 Chunky organic peanut butter, pack of 6 $21.50
Total $88

When would you like your order delivered? Please indicate the date and time below. If we cannot
deliver on your selected date and time, we will contact you as soon as possible to arrange an
alternative time.
Around 10 a.m. on Thursday, November 13

SEND

42 Hackers.co.kr
176. What is stated about Bulk Land 179. What can be inferred about Mr. Johnson?
Supermarket? (A) He has placed orders with Bulk Land
(A) It has a minimum order requirement. Supermarket before.
(B) It introduces new deals every month. (B) He did not get his preferred delivery
(C) It will not deliver to some residences. date.
(D) It does not have any physical stores. (C) He will receive his order on Thursday
night.
177. In the advertisement, the word "comfort" in (D) He will call Bulk Land Supermarket to
paragraph 1, line 4, is closest in meaning confirm his order.
to
180. What special offer is Mr. Johnson eligible
(A) alleviation
for?
(B) service
(C) consolation (A) A coupon for a future purchase
(D) ease (B) A complimentary item of his choice

m /
178. According to the advertisement, what is a
(C)
o
A free delivery to his location

c
c.
(D) An immediate discount on his order
benefit of buying large amounts at Bulk
Land Supermarket?

g o
(A)
(B)
Wider product options
Lower packaging costs
s n
(C) Cheaper prices on products
g m
(D) Better membership incentives

k i n
e a
ic sp
e
p : //to
ht t

GO ON TO THE NEXT PAGE,

TEST 1 PART 7 43
Questions 181-185 refer to the following letter and e-mail.

March 23
Brittany Cosmetics
Carla Danbury
3411 Appleton Way
Portland, OR 97035
Dear Ms. Danbury,
It is that time of year to prepare for the 15th Annual Organic Beauty Product Fair! This year, the event
will be held at the Seacrest Convention Center in Seattle from July 8 to 10.
As your company was a vendor during last year's event in Denver, we would like to offer you a
15 percent discount on booth and equipment rentals for this year's fair. For your convenience, I have
enclosed a complete schedule of events, and included an application form containing information on
pricing.
m /
o
A hotel is attached to the convention center, and all companies with booths at the fair are eligible to

c
c.
reserve rooms at a reduced rate of $120 per night. Please contact us if you want to make a booking.
In addition, free Wi-Fi is offered throughout the center, so you will have access to the Internet from
your booth.
g o
s n
The applications and complete payment are due by May 15. You can remit payment by cash, check, or
bank transfer. If you have any questions or require any further information, please send an e-mail to
toddbanks@tradevent.com.
g m
Sincerely yours,

k i n
Todd Banks
Corporate Event Planner
e a
sp
Tradevent Incorporated •

e ic
TO

p : //to
Todd Banks <toddbanks@tradevent.com>

ht t
FROM
SUBJECT
DATE
Carla Danbury <cdan@brittanycos.com>
Trade Fair
March 25

Dear Mr. Banks,


Thank you for the letter. We would like to be involved in the trade fair again this year, as we received a
lot of product orders during the last event. I mailed you our application this morning along with a check
to pay for our fees. Please send me a receipt by e-mail once the check has been deposited.
Also, I am interested in booking some hotel rooms for my staff and me. We will need three rooms for the
duration of the trade fair. Please let me know if there is anything I have to do to secure the rooms. For
instance, do I need to pay the full amount up front or can I leave a deposit?
Thanks for your assistance, and we look forward to taking part in another successful event.
Regards,
Carla Danbury
Owner, Brittany Cosmetics

44 ^O! yoiopj MP3 Hackerslngang.com


181. What is mentioned about the Seacrest 184. What does Ms. Danbury inquire about?
Convention Center? (A) Extra tickets
(A) It is fully booked for the upcoming (B) Hotel amenities
trade event. (C) A detailed floor plan
(B) It is located far from any accommodation (D) A reservation procedure
facilities.
(C) It provides vendors with complimentary 185. In the e-mail, the phrase "taking part in" in
Internet access. paragraph 3, line 1, is closest in meaning
(D) It hosts the beauty product fair every to
year. (A) supporting
(B) encouraging
182. Where can Ms, Danbury find information (C) joining
on Tradevent Incorporated's prices?
(A) On an application form
(D) competing

m /
(B) In an advertising flyer

c o
c.
(C) On a corporate Web site
(D) In a trade Journal

g o
183. What is implied about Brittany Cosmetics?
(A) It is ineligible for a rental price s n
reduction.
g m
(B) It is in the process of expanding its
range of products.
k i n
e a
(C) It had satisfactory results at an event

sp
in Denver.
(D) It registered for a trade fair online.

e ic
p : //to
ht t

GO ON TO THE NEXT PACE.

TEST 1 PART 7 45
Questions 186-190 refer to the following advertisement, e-mail, and comment card,

Port Royal Cruises!


An unforgettable adventure in historic Jamaica!
Have the adventure of a lifetime by taking a cruise on an incredible replica of
an actual pirate ship! Departing on Mondays, Wednesdays, and Fridays from
beautiful Port Royal, Jamaica, the H.M.S. Lady Blackboard travels to numerous
sites of interest! Featuring gourmet meals and beverages, the ship has a dining
room, lounge, bar, and cafe. And the itinerary also includes a deck party with a
buffet meal created by popular chef, Elijah Bolt. The vessel can accommodate
up to 110 passengers, and rooms are equipped with private bathroom facilities
and televisions. Tickets are just $450 per person for a two-night cruise,
including three daily meals.The ship is also available for lease for corporate or
private functions.To inquire about trip details or to view images of the ship and
m /
its facilities, visit www.portroyalcruise.co.jm.

c o
_1_
o c.
n g
TO
FROM
Evan Baughman <evbau@pealeint.com>
Rayleen Webb <raweb@pealeint.com>
m s
SUBJECT Re: Post-conference activity

i n g
DATE April 30

a k
e
sp
Evan,

e ic
I checked the Port Royal Cruises Web site you mentioned. I agree that it would be a fun activity for
those of us from the company attending the conference in Jamaica in June. I contacted the company to

//to
find out about renting the ship, and the cost is $7,500 per day including meals, staff, accommodation,

tp :and tours. So, a two-day rental would fall within our budget. We could depart on Friday and be back in
Port Royal on Sunday morning, in time for our return flights to Edmonton in the afternoon.

ht If needed, the ship has audiovisual equipment we can use, and Internet access is available. It also has a
paid laundry service, and the cruise line's bus can pick us up at the Mont Drake Resort where the
conference is taking place.
I'll proceed with the booking once you give me your consent.
Rayleen

Port Royal Cruises


Your comments are very important to us...
Please take a few moments to comment on your trip with Port Royal Cruises! The feedback we collect
will be used to improve services and facilities. Your details will not be shared with any third party. After

46 Hackers.co.kr
completing this card, please visit the reception desk on the ship or our administrative office to hand it in.
1. How did you learn about Port Royal Cruises?
It was recommended to me by a colleague.
2. Would you recommend a cruise with us to your friends?
Yes, as well as companies hosting events.
3. What were the best aspects of your trip?
Great onboard facilities, helpful staff, fun sites, excellent gourmet menu from Elijah Bolt, and
comfortable lounge.
4. How could your trip have been better?
The laundry service could have been better. Some of my clothes came back wrinkled. Also, the
shuttle bus could have been less crowded.
5. Would you like to receive regular e-mail newsletters from Port Royal Cruises? YES □ NO 0
m /
NAME: Rayleen Webb
c o
c.
DATE OF TOUR: June 26-27
E-MAIL: raweb@pealeint.com

g o
s n
186. What does Ms. Webb suggest about the
cruise she took?
g m
189. What is indicated about the comment
card?
(A) The onboard facilities seemed
k i n (A) It should be mailed to the cruise
crowded.

e a company.
(B) It can be submitted in person.

sp
(B) It had fewer staff than necessary.
(C) The deck party included very good (C) It was found in the ship's main lounge.
food.

e ic
(D) It cost more than she had anticipated.
(D) It will be sent to the traveler's address
after the trip.

: //to
187. What is mentioned about the H.M.S. Lady

p
190. What did Ms. Webb most likely have to do?

ht t
Blackbeard?
(A)
(B)
It recently underwent a restoration.
It can be viewed in online pictures.
(A)
(B)
(C)
Prepare some audiovisual equipment
Pay extra to have clothes washed
Fill in a newsletter subscription card
(C) It includes a variety of room types. (D) Postpone an excursion to another date
(D) It departs from Port Royal every
Saturday.

188. What is the purpose of the e-mail?


(A) To request specific dates for a trip
(B) To inquire about the cost of a cruise
ticket
(C) To discuss a possible staff activity
(D) To ask for a change to a booking date

GO ON TO THE NEXT PAGE

TEST 1 PART? 47
Questions 191-195 refer to the following list, Web page, and e-mail.

FILMOGRAPHY: Henry Spencer, actor


Run for your Life, role of marathoner Dale Warren
Directed by Aaron Marks
Some Prefer the Breeze, role of doctor Richie Dean, nominated for SPG Award for best supporting
actor
Directed by Larry Loden
Therein Lies the Truth, role of lawyer Abner Cole, nominated for SPG Award for lead actor and won
International Screen Award for lead actor
Directed by Greg Steinfeld
Show me the Way, role of scientist Ellis Charleston, won International Screen Award for lead actor
Directed by Greg Steinfeld

m /
Bandits of Time, role of astronaut Hal Lourdes, won Sci-Fi Award for best supporting actor

c o
c.
Directed by Jerry Bradbury

g o
n
g ms <>
http://www.screenactiontalkshow.com
HOME | SCHEDULED SHOWS |
k i n
SHOW RECORDINGS | CONTACT US

e a
Join Korinna McKay every Wednesday evening at 5 p.m. for Screen Action, a radio program

ic sp
featuring news and interviews about the movie industry on Slick Radio 93.9 FM.
This week on June 7, Korinna welcomes Cheri Oakland to the studio, who will discuss her most
e
//to
recent work, Bandits of Time //. A follow-up to the incredibly successful Bandits of Time released
two summers ago, it includes all the cast members from the first movie. Ms. Oakland will tell us

tp :
about the film and her experience while directing the production. After the interview with

ht
Ms. Oakland, actress Cecily Monroe, who returns as the character Sylvia Slade in Bandits of Time //,
will also join us to discuss the movie. Dial 1-800-555-3944 at the end of the program if you have a
question about the movie that you would like to ask our guests. And if you miss this week's show,
catch it later by clicking on the "SHOW RECORDINGS" tab after the airdate.

Ol lo ii

TO Bob Voorhies <bobv@genericapost.com>


FROM Mamie Hefner <mhef@screenaction.com>
SUBJECT Screen Action : June 7 program
ATTACHMENT Release form
DATE June 8

Dear Mr. Voorhies,


Thank you for calling in to our June 7 program. Your question and comments were very much

48 >jEq| qo^.c^opixh□] yo(op| mps Hackerslngang.com


appreciated by Ms. McKay and our guests.
As you may be aware, Screen Action records all programs and posts them on our site for the
convenience of listeners. Regulations require us to seek the permission of anyone speaking on the
program before we do this, so attached you will find a release form that we kindly ask you to sign
and return to us.
Once again, we are appreciative of loyal listeners like yourself.
Regards,
Marnie Hefner, producer
Screen Action

191. According to the list, what is NOT true


about Henry Spencer?
194. What is suggested about Mr. Voorhies?

m /
o
(A) He was contacted by Korinna McKay

c
c.
(A) He participated in two movies by the previously.
same director. (B) He provided feedback about the June
(B) He has played roles that are different
o
7 show online.
g
occupations.
(C) He performed in leading roles in all
s n
(C) He had a query about Bandits of Time II
during a show.
productions.
(D) He received multiple nominations for
g m
(D) He listened to a rerun of Screen Action.

Therein Lies the Truth.


k i n
195. Why has Ms. Hefner asked Mr. Voorhies

e a
192. What can be inferred about Ms. Oakland?
to sign a document?
(A) To signify his willingness to be

latest film.
ic sp
(A) She worked with Henry Spencer in her interviewed
(B) To allow a program recording to be

e
(B) She received a Sci-Fi Award for one of publicly released

//to
her films. (C) To give permission to post some

tp :
(0) She collaborated with Jerry Bradbury
on a past project.
written questions
(D) To agree to keep information

ht (D) She plans to make a sequel to Bandits


of Time II.
confidential

193. Who most likely is Korinna McKay?


(A) A television producer
(B) A talk show host
(0) A broadcast network executive
(D) An interview guest

GO ON TO THE NEXT PAGE

TEST 1 PART 7 49
Questions 196-200 refer to the following article, e-mail, and form.

Health Note March


The Truth About Supplements? (continued from page 19)
So, ideally, maintaining a varied diet is enough to satisfy the nutritional requirements of adults aged 65 and
older. However, some seniors may find this difficult to do, as they lack time and find preparing nutritious
meals to be too challenging. In such cases, many doctors agree that taking vitamins and other supplements
may be helpful. Even so, physicians believe that these supplements should be carefully regulated so as not to
cause unintended harm. According to a controlled study1 by the Research Institute for Aging, some vitamins
and minerals can be toxic when ingested in large amounts. For this reason, getting the advice of a trusted and
qualified physician before beginning a vitamin regimen is recommended.
1- Health Note was granted advance access to the study, which will soon be available for publication.
*For permission to reprint this article, please write to admin@healthnote.com. Charges apply for publication
of the complete article or 500 words or more. Excerpts with fewer words can be reprinted for free, but must
be attributed to Health Note.
m /
c o
o c.
To: Ernst Hopson <ehopson@psychguide.com>
From: Gayle Brenner <admm@healthnotexom>
n g
Subject: Permission to reprint
Date: April 12
m s
Attachment: Reprint request form

i n g
Dear Mi*. Hopson,
a k
e
We received your e-mail inquiring about reprinting an article excerpt from the March issue of Health Note. I am

ic sp
providing you with a form, which must be filled out completely and sent back to us. Essentially, you will be asked
to cite which parts of the article you would like to reprint in your magazine and state the purpose of your request.

e
//to
You may return the completed document to us by e-mail. Requests made before the 15th of the month will receive a
response by the end of that month. And requests made after that date will receive a response at the beginning of the

tp :
following month. Approval for reprinting is given by the magazine on a case-to-case basis.

ht
Thank you, and we hope to hear from you soon.
Sincerely,
Gayle Brenner
• ..

REQUEST FOR PERMISSION TO REPRINT AN ARTICLE


Date: April 27
Name of Applicant: Ernst Hopson
Address: 1219 Lee Gardens, Arlington, VA 22201
E-mail address: ehopson@psychquide.com
Telephone No.: 555-4189
Name of Magazine: Psychology Guide
(Please provide a link if your magazine is an online publication.)
Link: Not operational

50 Hackers.co.kr
Title of Article: "The Truth About Supplements?"
Publication the article appeared in: Health Note
Issue: March
Which parts of the article do you wish to reprint in your magazine?
I would like to print a portion of the article (200 words) found in the second paragraph of page 5.
What is your purpose for reprinting the article?
Psychology Guide will be devoting an entire upcoming issue to the mental health of seniors. Your article
mentioned various supplements that help with alertness, and we would like to reprint that paragraph as a
short article in the issue.
If my application is approved, I will give appropriate credit to Health Note.
0 I agree to the above.

196. What is the article mainly about? 199. What is indicated about Mr. Hopson's
request?
m /
(A) Some guidelines for ensuring a varied

c o
c.
diet (A) It will not incur a charge for printing.
(B) A disease that commonly affects the (B) It requires that a research study be
elderly conducted.
g o
(C) Some advice regarding the
consumption of supplements
s n
(C) It has to be approved by the Research
Institute for Aging.
(D) A medication proven to be effective for
g m
(D) It is for a newly launched medical
seniors

k i n journal.

197. What does the e-mail suggest about


Health Note?
e a 200. When will Mr. Hopson receive a response?

sp
(A) At the end of March
(A) It does not approve all petitions for (B) At the beginning of April
reprints.

e ic (C) At the end of April

//to
(B) It is published on a weekly basis. (D) At the beginning of May
(C) It has allowed Mr. Hopson to reprint

tp :excerpts before.

ht
(D) It does not provide copies of previous
issues.

198. What is true about Psychology Guide?


(A) It is a new publication.
(B) It is inaccessible to members of the
public.
(C) It has long supported Health Note.
(D) It is not available online.

This is the end of the test. You may review Part 5, 6, and 7 if you finish the test early.

%
p,32A / S-ys p.327 / p.328 / Part 5&6 Sfly H|-S "71
• q# Eiioiqofl Self tf|a aiiM* SSB yi!°| m #0] e«ei ggsfl SAflf

TEST 1 PART 7 51
^2 1000111 3 READING

m /
c o
o c.
n g
m s
i n g
a k
e 02

sp
I TEST

e ic
//to
PART 5

tp : 01 PART 6

ht 0 PART 7

Self *1|H

S2H EII^E 2 shojAHJ


1.^H^o|^§TI^o? □ 0j|
2. Answer Sheet. ^1. Al^m ^u|o^L^? □ o||
3.A|7lll^b|o^o? noil
o= 2^1^001 [[^aj o El|AE^ ARSLpK

^11 i0|»: m\b ^¥£1 70^ %\ _JI _g®NC|V


EH AE iio # ysgolDl. A|^ 2 5^ §3 ^ D^# ?|6|| I^MPh
READING TEST
In this section, you must demonstrate your ability to read and comprehend English. You will be given a
variety of texts and asked to answer questions about these texts. This section is divided into three parts
and will take 75 minutes to complete.
Do not mark the answers in your test book. Use the answer sheet that is separately provided.

PARTS
Directions: In each question, you will be asked to review a statement that is missing a word or phrase.
Four answer choices will be provided for each statement. Select the best answer and mark the
corresponding letter (A), (B), (C), or (D) on the answer sheet.

© PARTS 11&

m /
101. According to Picton Daily News, only

c o
105. The statue on Hastings Avenue that was

c.
damage to the area occurred during the unveiled in a ceremony yesterday by
tropical storm. an ancient Roman sculpture.
(A) moderate (A) inspired
g o
(B)
(C)
moderating
moderately s
(B)
(C) n
to be inspired
was inspired
(D) moderation
g m(D) is inspiring

102. The employees at Barneveld Inc. are


k i n 106. An old building in the Brickford

a
encouraged to build teamwork by assisting

e
neighborhood will be demolished and

sp
on various projects. a luxury apartment.

ic
(A) the other (A) differed from
(B)
e
one another (B) imposed on

//to
(C) other (C) replaced with
(D) another (D) attributed to

tp :
ht
103. Some marketing team members from 107. After he was appointed sales manager,
Game-Tekno will be traveling to Tokyo Bill Haskell achieved improved
the branch opening to finalize between the staff in the sales and
preparations. marketing departments.
(A) although (A) coordinator
(B) next to (B) coordinative
(C) ahead of (C) coordinated
(D) inside of (D) coordination

104. The seminar speaker offered some 108. All potential clients are given brochures
arguments in favor of natural medications containing brief of GL Mutual's
and remedies. insurance policy packages.
(A) convinced (A) distributions
(B) convince (B) operations
(C) convincing (C) summaries
(D) convincingly (D) subtractions

54 Tjjg Hackers.co.kr
109. The Beckford Auditorium can be reached 115. Customers should carefully read the
by walking the second floor corridor descriptions of each item online to
to the end of the hallway. avoid time-consuming product returns.
(A) through (A) was ordered
(B) without (B) ordering
(C) away (C) ordered
(D) under (D) orders

110. Codex Corporation showed a great deal of 116. The release date for HBS Tech's new
about Ms. Khan's recent medical product was a mistake as its
issues and gave her an extended leave. competitor had launched a similar item H
m
i/i
earlier. —i
(A) understand
(B) understandable (A) deliberately g
(C) understood (B) indeed mm
(D) understanding (C)
(D)
instead
evenly
m / B

o
o
UJ
111. Mayor Terrence Miller the volunteers
for the self-sacrificing work they did to 117. All
c. c
directed to Holston Prudential's o
4>

go
assist the community during the disaster. customer service support team are
handled in complete confidence. o
LTI
(A)
(B)
contested
commended (A)
s n
inquiring o
CD
(C) restricted (B)
m
inquired

ing
(D) promised (C) inquiries o
(D) inquire

ak
112. The Parker Fairground off Taylor Street is o
CO

during the summer. e


the perfect place for families and friends

p
118. Country-Link Limited utilizes special
wrapping paper to prevent items

o
LD
(A) to visit
ic s from breaking during delivery. ~T
CD
(B) will visit
e (A) subtle

//to
or
(C) are visiting (B) delicate DH
!>•

tp :
(D) visits (C)
(D)
vigorous
cautious
Kn

nit

ht
ri!
113. The carpenter noted that the old house's oo
roof was run-down and would soon 119. The scheduled meetings for Augustus jaCD
UJ
have to be repaired. Manufacturing's staff were spaced far XI
enough to allow employees to DJo.
(A) rather da
complete their usual tasks.
(B) seldom
(C) never (A) alongside
(D) often (B) beyond
(C) apart
114. editing and rewriting documents, (D) besides
Quill Editing offers translation services in
a number of languages.
(A) On behalf of
(B) In addition to
(C) Due to
(D) In spite of
GO ON TO THE NEXT PAGE,

TEST 2 PARTS 55
120. Bug-Fix Software users of its antivirus 126. The captain of the cruise ship instructed
program to download regular updates and the passengers to stay off the outdoor
scan their computers on a weekly basis. decks winds become severe.
(A) advises (A) in the event that
(B) suggests (B) in keeping with
(C) attracts (C) unless
(D) persists (D) instead of

121. Passengers are asked to store any carry-on 127. Clifton Oil and Gas's annual sales are
luggage in the overhead bins and showing great compared to last year,
fasten their seat belts prior to takeoff. when profits were disappointing.
(A) securely (A) denial
(B) secure (B) consideration
(C) security (C) potential
(D) securing (D) oversight

m /
122. Included in the lease is a strict
c o
128. Most participants have found Bucksmith

c.
requiring tenants to pay for any damages Capital's internship program
caused to the property,
o
because it provides them with relevant

g
banking experience.
(A) prediction
(B) condition
s
(A) n
satisfaction
(C) negotiation
(D)expense
g m
(B)
(C)
satisfactory
satisfyingly

k i n (D) satisfied
123. Manuel Torres,

e a
books have all been
best-selling titles, has released his newest 129. Researchers conducting experiments in

sp
publication on the topic of the national the laboratory are kindly asked to return

ic
election system. equipment to it is stored.
(A) whose
e (A) where

//to
(B) who (B) whether

tp :
(C)
(D)
whom
what
(C)
(D)
when
elsewhere

ht
124. Mr. Norris will take Friday afternoons
off if he has to travel a long distance
130. For coverage of top stories from
around the world, more viewers tune in to
during weekends. Global Syndicated than any other news
(A) accidentally channel.
(B) once (A) renewable
(C) occasionally (B) dependable
(D) lately (C) reachable
(D) adjustable
125. The assistant curator ensures that all
artifacts according to museum
guidelines to avoid deterioration.
(A) were being stored
(B) stored
(C) are stored
(D) will be storing

56 !?esateit!se|a°|JUt-t'oi^lS?!eot?PIHP3 Hackerslngang.com
PART 6
Directions: In this part, you will be asked to read four English texts. Each text is missing a word,
phrase, or sentence. Select the answer choice that correctly completes the text and mark the
corresponding letter (A), (B), (C), or (D) on the answer sheet.

0 PARTS rn*#0! APJ 8g

Questions 131-134 refer to the following e-mail.

To: Christine Lata <clata2@consumermail.com>


From: Dr. Miles Hamilton <mhamilton@hamiltonmedical.com:>

/
Date: March 15
Subject: Retirement

o m
Dear Ms. Lata,

c. c
My 40-year career as a physician is coming to an end as I plan to
131.
g o
from my position on

s n
June 1. Although I look forward to enjoying my golden years, I will miss my work helping patients.

g m
As your doctor, I believe I should suggest an appropriate replacement. , I'd like to

k i n 132.
recommend Dr. Jennifer Jana. She is an excellent physician whose clinic is to my

e a 133.

sp
current office. With your consent, your medical records will be transferred to her on June 2.

ic
, I wish you good health and a happy future.

e
//to
Best regards,

tp :
ht
Dr. Miles Hamilton
Hamilton Medical Clinic

131. (A) step down 134. (A) Your chart shows a marked
(B) move out milk improvement since her last visit.
(C) take over (B) Of course, I am glad that you have
(D) hold off decided to remain with me.
(C) Otherwise, you may retrieve them from
132. (A) Therefore my staff before that date.
(B) Likewise (D) There are many forms of treatment for
(C) Afterward your condition.
(D) Conversely

133. (A) addressable


(B) straight
(C) divided GO ON TO THE NEXT PAGE.
(D) close
TEST 2 PART 6 57
Questions 135-138 refer to the following letter.

April 7

William Randell
118 West Ocean Boulevard
Oceanside, California 92049

Dear Mr. Randell,

Your application has been We are pleased to welcome you to West Coast State
135.
University. In the coming weeks, we will be sending more information about how you should

prepare for your first semester.


136.
you meet the enrollment deadline, you will be allowed

m /
o
to sign up for a full course load and pay your tuition in the last week of July. We will also issue

you a student ID card then.


c. c
Should you wish to have a dormitory room at that point,

g o
we will assign you one a week prior to the start of classes. You will be authorized to move in

shortly
s n
g m
i n
Once again, congratulations on your successful application, and we hope to see you this fall,

k
Sincerely,
e a
ic
Admissions and Financial Aid
West Coast State University sp
e
//to
tp:
135. (A) rejected 137. (A) Now that you have sent the amount,

ht
(B) deferred f||| please wait for confirmation,
(C) replaced (B) We hope that you found your classes
(D) accepted to be both fun and informative.
(C) Once you receive it, you may access
136. (A) Up until all of the facilities on campus.
(B) On condition that (D) The additional charge to your account
(C) Though is for the card you lost.
(D) Rather than
138. (A) before
(B)soon
(C) thereafter
(D) now

58 Hackers,co.kr
Questions 139-142 refer to the following article.

New Transit Options Coming!

City Council yesterday - to approve the proposed subway extension into the Glostrup
139.
zone of Copenhagen. This has encouraged developers to build more apartments there.

Moreover, the creation of the nearby Albertslund Technology Park has added thousands of

new jobs. These ------- opportunities have been the key to attracting more people to the

district. Since development began, however, it the area's transit capacity to its limits,
142.
causing local residents and businesses to campaign for a subway extension. A few council

members opposed the proposal, pointing out the high expense of underground transit.
m /
Nevertheless, the building of the subway extension will proceed at an estimated cost of
c o
€650 million and should be operational within five years.
o c.
n g
139. (A) failed 141. (A)
m seducation
(B)
(C)
reacted
voted
i n g
(B)
(C)
employment
volunteer
(D) hesitated
a k (D) sponsorship

e
sp
140. (A) Tourism to the area has grown in 142. (A) will stretch

ic
|||| recent years thanks to a popular (B) stretches

e
campaign. (C) will be stretched

//to
(B) The population in this area has been (D) has stretched
increasing because of low property

tp :costs.

ht
(C) The district's transportation costs are
increasing at an unprecedented rate.
(D) The closure of the Glostrup line was
announced just two months earlier.

GO ON TO THE NEXT PAGE.

TEST 2 PART 6 59
Questions 143-146 refer to the following memo.

To; All staff


From: Louise Nixon, Director of operations
Subject: VIP event
Date: September 6

We have confirmed a booking for over 200 guests from September 27 to 30. Many of

are important people from respected organizations who anticipate a high level of service

from us.

They will be our only guests over the entire four-day period. For privacy reasons, you are

expected to cooperate with requests from their security personnel and


144. /
from discussing

m
the event outside the hotel.
c o
o c.
We've also agreed that guests will be allowed complimentary use of the spa, gym, and

n g
business center.
140.
s
they should be billed if they order room service or make long-distance

m
i n g
a k
143. (A) whom
e 145. (A) The guests were very pleased with the

sp
(B) them mm service we provided.

ic
(C) their (B) Another group needs to use our small

e
(D) which meeting room on the 28th.

//to
(C) The contract states that their meals
144. (A) to refrain should be provided free of charge.
(B)

tp : refraining (D) Event participants will be entitled to a

ht
(C) refrained discount upon booking.
(D) refrains
146. (A) Specifically
(B) Sometimes
(C) Otherwise
(D) On the other hand

60 VeS2|?.ii!£a)H°Hy.hE!OB7|g^BO|^|MP3 Hackerslngang.com
part?
Directions: In this part, you will be asked to read several texts, such as advertisements, articles, instant
messages, or examples of business correspondence. Each text is followed by several questions. Select
the best answer and mark the corresponding letter (A), (B), (C), or (D) on your answer sheet.

© part 7 54£

Questions 147-148 refer to the following letter.

Martin Bailey
Tallahassee Bike Works
1105 Old Bainbridge Road
Tallahassee, FL 32308
m /
Dear Mr. Bailey,
c o
About one month ago, you contacted our company inquiring
o c.
about our latest bicycle parts brochure. At the time, we were

n g
revising our catalog, and I indicated that I would send you our

m s
brochure and order form for this year when the changes were
completed. I am pleased to inform you that the product booklet

n g
was printed this week, and a copy is enclosed with this letter.

i
k
We appreciate your interest In our merchandise and the previous

a
e
purchases you made for your shop, and look forward to

sp
continuing our partnership during the coming year. I encourage
you to contact me if you have any concerns or questions about

ic
any of our parts. We are also happy to discuss prices for bulk

e
//to
orders or long-term arrangements.
Best Wishes,

tp : Catherine Jenkins

ht
Shipson Bicycle Parts

147. What did Ms. Jenkins send to Mr. Bailey? 148. What is suggested about Tallahassee
(A) A revised contract Bike Works?
(B) A list of products (A) It recently published a promotional
(C) A discount coupon product brochure.
(D) A sales invoice (B) It sent Ms. Jenkins a complete
purchase order form.
(C) It inquired about placing a bulk order
of parts.
(D) It has done business with Shipson
Bicycle Parts before.

GO ON TO THE NEXT PAGE,

TEST2 PART? 61
Questions 149-150 refer to the following notice.

Town of Shakersfield

In an effort to provide rapid service to the Shakersfield area,


roadwork will be taking place next week around Shakersfield
Station. The work will begin on Sunday, May 12 at 7 a.m. and
finish in the late morning of Monday, May 20. The Public Works
Office apologizes in advance for any noise generated while
work is being performed. The main section of town that will be
affected is the area between Mayfield and Morton Roads. The
following local areas may also be impacted:
0
Oakwood Street up through Morton Road
m /
® Pedestrian access to Granderry Plaza

c o
c.
® Route 39 South to New Hammersworth

o
If any questions or concerns should arise with regard to this

g
s
Official at 555-3344 or m_newton@shakersfield.gov. n
notice, please contact Mary Newton, Public Works Senior

g m
k i n
e
149. What is the purpose of the notice? a 150. What is indicated about the Town of

ic sp
(A) To explain the role of a public office
(B) To inform commuters of a station
Shakersfield?
(A) Its subway will be affected by

e
//to
closure construction work.
(C) To state new transit regulations (B) It will post signs for detours during

t :
(D) To announce a construction project

p
roadwork.

ht
(C) It has a plaza that people can walk to.
(D) It is inaccessible to other towns by
highway.

62 Hackers.co.kr
Questions 151-152 refer to the following e-mail.

| FROM: Pierre Grenier <gren@grenierfashions.com>


TO: Elizabeth Monaco <lismon@faedommaii.com>
SUBJECT: Custom order
DATE: July 12

Dear Ms. Monaco,


I just wanted to inform you that we have completed your gown. Specifically, the hemline has now been —i
m
shortened by four centimeters, as you indicated you would not be wearing high heels. in

We are open until 6 p.m. on Fridays and closed on weekends, so please pick up your gown sometime this o
week. We also encourage you to try on the dress one last dme just to make sure everything is as it should
be.
I kindly request that you submit full payment when you next visit my shop. The details are as follows:
m /
7 meters of red brocade silk
c
$580
o
c.
o
Custom gown design $200 4>

go
Labor (sewing, fittings, alterations) $400 o
LTl
Subtotal
+ 10 percent sales tax
s n $1,180
(Total Amount Due) $1,298

m
ing
i so much again for your patronage, and I am sure you'll look great in the gown a O
firm's awards presentation this Saturday night.

ak
O
CO
Sincerely,
Pierre Grenier
pe a
LD

ic
Grenier Custom Fashions
s a

e
: //to
151. Why did Mr. Grenier send the e-mail?

p
152. What will Ms. Monaco do on Saturday?

ht t
(A) To provide a cost estimate for a
custom-made gown
(B) To notify a client that a garment is
(A)
(B)
(C)
Attend an awards ceremony
Try on an outfit at a shop
Pay for tailoring services
ready for pickup (D) Pick up a completed gown
(C) To clarify errors in a shop's billing
statement
(D) To inform a customer of some
scheduling alterations

GO ON TO THE NEXT PAGE,

TEST 2 PART 7 63
Questions 153-154 refer to the following text-message chain.
IB

Fumiko Nakamura [10:50 a.m.]


Philip, I'm about to board my flight and should arrive in Boston
this afternoon. How's everything going?
Philip Akkarat [10:55 a.m.]
OK for the most part. However, a few system errors occurred in
the new computer servers we're using to update the Web site.
I called a technician, and he came in this morning at about 8:00.
Fumiko Nakamura [10:58 A.M.]
Was he able to fix the problem?
Philip Akkarat [10:59 a.m.]
m /
He said the issue was more complicated than he first thought.

c o
c.
He had another job at 10:00 a.m., so he said he'd be back at
2:00 p.m. to finish up.

g o
n
Fumiko Nakamura [11:02 a.m.]

m s
Hmm, but those updates need to be done by 4:00 p.m. Why don't
you ask the staff to use the old servers for now?
Philip Akkarat
i n g [11:03 a.m.]

a
in order to finish in time. k
All right. That will take longer, so we should probably start now

e
sp
Fumiko Nakamura [ll:05 A.M.]

ic
Good. I'll see you in a few hours.

e
p : //to
ht t
153. When will the technician come to complete
a repair?
154. At 11:03 a.m., what does Mr. Akkarat most
|||| likely mean when he writes, "That will take
(A) At 8:00 a.m. longer"?
(B) At 10:00 a.m. (A) He wants to increase the length of a
(C) At 2:00 p.m. project.
(D) At 4:00 p.m. (B) He may ask some staff members to
work overtime.
(C) He feels that a repair job will take too
long to finish.
(D) He thinks that the old servers are
slower than the new ones.

64 gi\o\ sojiuf • y^ MP3 Hackerslngang.com


Questions 155-157 refer to the following information.

fr| PENDLETON ICE RINK

The staff at the Pendleton Ice Rink want everyone to have a safe, enjoyable experience.
17 Accordingly, we ask that visitors please observe the following guidelines:

- No playing with hockey sticks or pucks except on the ice


- No food or drinks are allowed on the ice or in the changing rooms
- No one is permitted on the ice without skates
- No animals are permitted on the rink or in the arena's facilities
- Everyone must keep in motion while on the ice
- Visitors may rent ice skates or bring their own

/
- Everyone must listen to rink attendants and follow their instructions

m
Trained attendants will be skating on the rink at all times. They are easily identifiable as they wear blue jackets

o
c. c
with "Pendleton Ice Rink" printed on them. If you have any questions or concerns, please speak to them.

The Pendleton Ice Rink is open every day from November 10 to March 1, except national holidays. Our hours of

g o
operation are 10 a.m. to 8 p.m. Monday through Friday, and 10 a.m. to 9 p.m. Saturday and Sunday. If you wish to

s n
sign up for one of our weekly ice skating classes taught by an expert coach, please inquire with a staff member
at the information desk or ticket office.

g m
k i n
a
155. What is the main purpose of the information?

e
157. How can visitors register for a class?

sp
(A) To announce the opening of an ice rink (A) By calling the ice rink's main office

ic
(B) To describe revised operation policies (B) By visiting an employee at a counter
to staff
e (C) By finding an attendant on a rink

//to
(C) To notify visiting skaters of a facility's (D) By signing up directly with the coaches
rules

p :
(D) To promote classes on an ice rink

t
ht
156. What is NOT indicated about Pendleton
Ice Rink?
(A) It has uniformed staff that are always
available.
(B) It offers patrons the use of dressing
rooms.
(C) It allows visitors to bring their own
equipment.
(D) It is open to the public 24 hours a day.

GO ON TO THE NEXT PAGE,

TEST 2 PART 7 65
Questions 158-160 refer to the following advertisement.

Get Cash for Your TRASH!

Do you have a home full of things you no longer need or use? Maybe it's clothing from when you
were a couple sizes larger, or some old dishes your grandmother left you that sit unused in a
box. Well, bring all those items in to the Bargain-Bin secondhand goods store nearest you, and
we'll give you money for your junk! With two branches in Beauville at 6254 Leonardo Crescent
and 137 Mount Theresa Avenue, our friendly staff will immediately let you know if your items
have any value to us and give you cash or store credit. It's just as easy as that!

Not only do you get cash for your items, but you can find amazing deals on used goods in both
our stores. We carry a vast selection of clothing, household goods, antiques and collectibles, and

m
home or office furnishings. Moreover, everything in our stores costs only a fraction of what similar/
o
items would cost in brand-new condition. With different stock arriving every day, you never know

c
c.
what sort of treasures you'll find in our stores.

o
Most importantly, our customers feel great knowing that 20 percent of all our profits go to local

g
s n
youth groups. So drop by a Bargain-Bin store today and earn cash, discover incredible savings,
and help contribute to the growth and education of Beauville's young people. For further details,

Ill
call us at 555-3009!

g m
k i n
e a
sp
158. What is the advertisement about? 160. What does the business do with a portion
of its profits?

ic
(A) A refuse recycling center
(B)
e
A used goods business (A) Invest it in independent local

//to
(C) An antiques auction house businesses
(D) A trash removal service (B) Distribute it to Bargain-Bin investors

tp : (C) Donate it to youth organizations

ht
159. What does the advertisement suggest (D) Use it to build educational facilities
about Bargain-Bin?
(A) Its selection of products is constantly
changing.
(B) It operates several outlets across the
state.
(C) It is looking to hire additional
employees.
(D) It rewards frequent shoppers with store
credit.

66 Hackers.co.kr
Questions 161-163 refer to the following notice.

The Winfield Community Chorus will hold open tryouts for new members on January 17 from
6 p.m. to 9 p.m. at Centennial Hall, located at 660 Plum Street in downtown Winfield. We have
openings for women and men and for all types of voices including bass, tenor, alto, and
soprano. — [1] —.
Those who audition must be at least 18 years of age, live in Winfield, and be capable of reading
music or willing to learn. Previous choral experience is certainly appreciated but not necessary
— [2] —, We will judge your ability to sing the correct notes to the tempo and melody of the
audition songs.
Also, we prefer performers that present themselves with professionalism and who can show
the proper facial expressions to communicate the emotions they perform. For concerts, you
m /
c o
will also have to learn lyrics in various languages, including Italian, German, and French,

o c.
Everyone auditioning will receive notification on January 19 about whether or not they have

made up of volunteers and that they will receive no payment. — [3] —.


n g
been accepted. New members must keep in mind that the chorus is a community organization

m s
Join us for some fun with your fellow music-lovers, and take part in loads of exciting
performances! — [4] —.

i n g
a k
e
written?
ic sp
161. For whom was the notice most likely 163. In which of the positions marked [1], [2],
Igfl [3], and [4] does the following sentence

e best belong?

//to
(A) Current members of a music club
(B) Registered participants in a "They are also responsible for purchasing

tp :
competition their own uniforms, the details of which will

ht
(C) Residents interested in performing be provided at a later time."
(D) Students enrolled in a singing class
(A) [1]
162. What is required of those wishing to try out (B) [2]
for the chorus? (C) [3]
(D) [4]
(A) Prior participation in a chorus
(B) Previous foreign language study
(C) Possession of a music degree
(D) Willingness to be a volunteer

GO ONTqTHENEXrPACE.:_

TEST2 PART? 67
Questions 164-167 refer to the following online chat discussion.
Al
osa

Gloria Arden [8:20 A.M.] I've been looking over everyone's progress reports on the software project.
Based on what I've seen, I don't think we'll be able to meet our scheduled
release date.
Isabel Cabrera [8:22 A.M.] You may be right about that. Since Bob and Carol moved over to the new
hardware project, we've had some trouble meeting the initial deadlines.
Gordon Brickyard [8:26 a.m.] Couldn't we just reschedule the release? Pushing it back a month ought to
give us enough time to finish up.
Joe Freemont [8:28 A.M.] But remember that the product is supposed to be available for the holiday
season, which is our peak sales period. If we miss that deadline, we could
lose many potential customers.
Gordon Brickyard [8:31a.m.] My team has a lot of urgent responsibilities, so I think it would be a good

Gloria Arden
idea to hire some independent contractors to help out.
[8:32 a.m.] What are you thinking?
m /
c o
c.
Gordon Brickyard [8:33 a.m. Well, they could help my team finish writing the user manual, for one
thing. And they could probably handle product testing as well.
Gloria Arden [8:35 a.m.] I see. What's your opinion, Isabel?
g o
Isabel Cabrera
n
[8:37 a.m.] It could end up being expensive, but I like the idea. Our graphic designers

s
could certainly use some outside help, too.
Gloria Arden
g m
[8:40 a.m.] Well, it sounds like we should go with Gordon's idea. I'll also bring it up

k i n
during the board meeting on Friday to see what they say.

e a
ic
164. What caused a project to fall behindsp 166. What is suggested about Mr. Brickyard's
schedule?
e team?

//to
(A) The cancellation of product testing (A) It includes independent contractors.

tp :
(B) The alteration of some deadlines (B) It was given one month to complete all

ht
(C) The modification of a work policy of its tasks.
(D) The reassignment of some employees (C) It is responsible for drafting user
instructions,
165. Why most likely is changing the release (D) It is part of the hardware division.
date unacceptable?
167. At 8:32 a.m., what does Ms. Arden mean
(A) It will increase production costs.
when she writes, "What are you thinking"?
(B) It may be rejected by the board.
(C) It might violate a contract. (A) She strongly disagrees with a
(D) It could affect sales of a product. proposal.
(B) She wants more details concerning
a plan.
(C) She does not understand the purpose
of a project.
(D) She prefers to discuss an idea at a
later time.

68 ?ssat£!i!S9)H2|iy,f.EK>ia?IS^901Sf?IMP3 Hackerslngang.com
Questions 168-171 refer to the following article.

Barriston City Unveils New Development Project


By Wendy Ogilvy

The Barriston City Planning Commission announced plans last Thursday to begin work on a revitalization
of the art district on Chestnut Avenue, Long considered the hub of Barriston's art scene, the area has fallen
into disrepair. The project will involve widening the sidewalks, repaying the street, and building a park in
the area. In addition, two information billboards showing maps of the district will be installed on the
thoroughfare for visitors.

But the highlight of the project is a new outdoor staging area which will be located at the corner of
Chestnut Avenue. Designed by local architect Theresa Vergara, the stage will be used to host musical
concerts and theatrical productions, with outdoor seating for up to 500 people.

Also, a portion of the city government's budget has been allocated to the restoration of buildings along
Chestnut Avenue. Inspectors will evaluate the structures to determine what work needs to be carried out.
m /
c o
Proprietors will not be charged, but according to city tax assessor Peter Jones, property tax levels will
increase in the district by 1 percent, starting next year.

o c.
n g
With many proprietors of local businesses in the district reporting dropping sales figures and a decline in
visitors, the commission hopes the project will help draw people back to the area. Evan Sweeten, chair of
s
the planning committee, said, "We believe that this work will help revitalize the area and bring it back to

m
its former glory."

i n g
a k
168. Why was the article written?
e 170. The word "draw" in paragraph 4, line 2, is

sp
closest in meaning to
(A) To publicize the opening of an art
foundation

e ic
(B) To announce a tax increase for all
(A)
(B)
outline
attract

//to
local residents (C) fill

:
(C) To provide details about an urban

tp renewal project
(D) represent

ht (D) To report on plans for a transit


expansion
171. What will most likely be used to pay for the
renovation of some buildings?
(A) City government fund
169. What does the article NOT indicate about (B) Property taxes
Chestnut District? (C) Raised donations
(A) It will be the site of a public (D) Money from corporate sponsors
performance venue.
(B) It will have signs displaying maps of
the entire city.
(C) It is currently in need of some
roadwork.
(D) It has experienced a decline in
business.

GO ON TO THE NEXT PAGE

TEST 2 PART 7 69
Questions 172-175 refer to the following advertisement.

//§*// EZ-R-Printing Services:


Get exactly what you wont!

If your small business requires pamphlets, flyers, business cards, catalogs, or any
other type of promotional material, you can produce precisely what you need
with EZ-R-Printing Services! Our easy-to-use design system allows you to select the
fonts and graphics you like and position them on the page. You can even choose
the type of paper to use.

— [I ] —. Once you're completely satisfied with your document, select the number of
m /
prints you'd like and submit payment by credit card, direct transfer from your

c o
c.
financial institution, or the Paybuddy online transaction service. — [2] —. Fill out a
shipping form, and your items will be delivered anywhere in the US within five

g o
business days. Shipping and handling charges are included in the total cost, so
there are no hidden fees.

s n
g m
— [3] —. For a one-time payment starting at as low as $180, one of our skilled

i n
professionals will create an attractive publication. They can work with you on any

k
printed promotional material until you are satisfied!

e a
sp
In addition, for the month of May only, anyone placing an order of $75 or more will

ic
receive $25 worth of business cards for free. — [4] —.

e
//to
Check out hundreds of sample publications on our site and take a tour of our

:
design system to see how easy and affordable it is to use our services at www.

tp ezrprintingservices.com.

ht

70 Hackers.co.kr
172. What does the advertisement mention 174. What can visitors to EZ-R-Printing Services'
about EZ-R-Printing Services? Web site do?
(A) It provides the services of expert (A) Check out prices
photographers. (B) Settle invoices
(B) It offers free shipping for bulk orders. (C) Print out documents
(C) It allows clients to design their own (D) Find branch listings
documents.
(D) It supplies a wider selection of printing 175. In which of the positions marked [1], [2],
paper than competitors, lUl [3], and [4] does the following sentence
best belong?
173. What will be given to those customers who
"If you're uncertain about your layout and
spend a certain amount?
design skills, you don't have to do it at all."
(A)
(B)
Gift certificates worth $75
Consultations on layout and design
m /
(C) Free delivery services

c o
c.
(D) Complimentary business cards

g o
s n
g m
k i n
e a
ic sp
e
p : //to
ht t

GO ON TO THE NEXT PAGE,

TEST 2 PART? 71
Questions 176-180 refer to the following notice and e-mail.

Recruitment Fairs

It will soon be that time of year again when Cofton College sends staff to university
recruitment fairs around the country to attract new students. Ms. Malkovich will, of
course, be in charge of this as our marketing manager and will be attending all of the
events. Unlike other major private colleges in the UK, we like to emphasize the amount of
personal attention that students receive from their teachers. Thus, not only do we keep
class sizes to manageable proportions, but we also promote healthy social interactions
between students and faculty. For this reason, we try to have at least one member of the
teaching staff attend each recruitment event. The four events we will be attending in the
next month are listed below:
• Thursday, February 10, Mercia Hotel, Birmingham
m /
• Wednesday, February 16, Hartford College, London

c o
c.
• Saturday, February 19, Leeds Metropolitan Convention Centre, Leeds
• Wednesday, February 23, Brighton and Hove Conference Centre, Brighton

g o
n
Please get in touch with me at lsullivan@coftoncoUege.co.uk if you have the time to attend

hourly rate for overtime.


m s
any of these events. Anyone willing to work the weekend event will be paid at the usual

I look forward to hearing from you all.


i ng
Lisa Sullivan
a k
Administration assistant
e
ic sp
e
j|
: //to
From: Andrew Jansen <ajansen@coftoncollege.co.uk>

p
il
|
I
t
To: Lisa Sullivan <lsullivan@coftoncollege.co.uk>

ht
Date: January 25
Subject: Recruitment fairs
i
1 Hello Lisa,
i
| I was glad to see your notice on the departmental bulletin board today, as I always enjoy attending one
|]
j or two events each year to meet potential new students. Of the events you listed, I'd be happy to go to
l!
] the one on February 19. My family lives in the same area the event is taking place, and I'll be visiting
|
| them that weekend. Accordingly, it would be no trouble for me to take a few hours to attend the event.
:|
i r |i
I] Unfortunately, that's all I can offer to do for now, as I teach all day on Tuesdays, Wednesdays, and H
| Thursdays. Please send me all the details of the event as soon as you can.
i I
1 i
S Best Wishes,
ii j
| Professor Andrew Jansen
l;1 i

72 pjCfoiopl MP3 Hackerslngang.com


176. What has Ms. Malkovich most likely been 179. What is indicated about Professor Jansen's
assigned to do? family?
(A) Participate in events to recruit students (A) They will travel with him to London.
(B) Make travel arrangements for a group (B) They are away on vacation in Brighton.
(C) Make a list of the fair participants (C) They are residents of Leeds.
(D) Find appropriate venues for some fairs (D) They recently moved to Birmingham.

177. What is implied about Cofton College? 180. Why is Professor Jansen unable to attend
the other events?
(A) It is holding a recruitment fair on its
campus. (A) He will be busy with work during the
(B) It offers subsidized tuition for some week.
courses. (B) He will be visiting family out of town.
(C) Its course enrollment numbers are
typically low.
(C) He will
(D) He will
m /
be attending a literary event.
be traveling in another country.
(D) Its ratio of teachers to students is high.

c o
178. What is stated about the weekend event?

o c.
(A) Teaching staff are not needed for it.

n g
(B) Staff will be paid overtime for
attending it.
m s
(C) It will host the greatest number of
attendees.
i n g
(D) It will have a modest admission fee.

a k
e
ic sp
e
p : //to
ht t

GO ON TO THE NEXT PAGE

TEST 2 PART? 73
Questions 181-185 refer to the following form and e-mail.

r
liaS i

Full Name: Cindy Norenski Home Phone: 555-2237


Gender: Female Mobile: 555-6923
Birth Date: March 27 E-mail Address: cinnoren5@gladmail.net
Street Address: 46 North Ranch Drive
City: Kansas City
State: MO
Zip Code: 64110
m /
-• ' . • Family Members:
c o
Name Gender Birth Date Age
c.
Relationship

go
Gerald Norenski Male August 13 45 Husband

sn
Jennifer Norenski Female October 7 19 Daughter

gm
Melissa Norenski Female May 6 12 Daughter
Membership Type
n
How did you hear about us?
□ Individual
k i
□ Member Referral
0 Family Plan
e a
□ Advertisement

sp
□ Senior Plan □ Search Engine

ic
□ Group Plan 0 Staff Referral
□ Student Plan
e □ Promotional E-mail

//to
□ Other:

tp :
ht To: Cindy Norenski <cinnoren5@gladmaiI.net>
From: Jerry Hanover <j_hanover@fitprimegym.com>
Date: March 12
Subject: Welcome!

Dear Ms. Norenski,


Thanks for your recent application to Fit Prime Gym. I'm so glad that you decided to follow my
advice and sign up with us. As I've mentioned before, I am certain that your husband and your
children will enjoy all the facilities and activities Fit Prime has to offer.
When we spoke, you said that you were interested in using the pool to swim laps. Well, I just
want to let you know that the scheduled times for that activity have changed. The indoor
swimming pool still opens at 8:00 a.m. every day. However, it now closes at 10:00 p.m. from
Monday to Friday. On weekends, it closes at 9:15 p.m. And please remember to use only the

74 Hackers.co.kr
pool's first two lanes for lap swimming.
I should mention that your daughter, Jennifer, qualifies to take part in our Student Plan, She can
join this program for free while also remaining under your Family Plan program. The Student
Plan comes with two introductory months of personal training sessions for new members.
Please ask her if she is interested so I can register her, if necessary.
Thanks again, and I hope to see you at the gym soon!

Jerry Hanover
Fit Prime Gym Management

m /
o
181. Why did Ms. Norenski fill out the form? 184. What is indicated about Fit Prime Gym's
(A) To enroll her family af a fitness center
pool?

c. c
(A) It only allows lap swimming on
o
(B) To begin teaching classes at a gym
(C) To arrange a tour of a gym
(D) To sign up for weekly swimming
weekdays.

n g
(B) It temporarily shuts down for cleaning
lessons

m s
on Fridays.
(C) It is unavailable during the winter
182. What has NOT been provided on the

i n g months.
form?

a k (D) It closes at an earlier time on Sundays,

e
(A) Ms. Norenski's residential details
185. What is suggested about Ms. Norenski's

sp
(B) The applicant's occupation
membership?

ic
(C) Ms. Norenski's date of birth
(D) The applicant's familial relationships (A) She received an extra discount for
e
//to
renewing a previous membership.
183. What is most likely true about Mr. Hanover? (B) Her husband intends to take part in lap

p :
(A) He just started working as a manager

t
swimming.

ht
at the gym. (C) She plans to switch gym programs to a
(B) He recommended that Ms. Norenski try different one.
the gym. (D) Her older daughter is entitled to
(C) He is certified to provide personal additional benefits.
training to members.
(D) He works a later shift now because of
the new pool hours.

GO ON TO THE NEXT PAGE

TESTS PART? 75
Questions 186-190 refer to the following letter, Web page, and article.

FRANCHISE MASTER
4110 North Boulevard, Tampa, FL 33603
July 11
Daniel Ainge
665 Terrace Drive
Brandon, FL 33511
Dear Mr. Ainge,
Congratulations! You are one step closer to realizing your dream of owning a business. Over the past two
decades, Franchise Master has helped thousands of aspiring entrepreneurs like you find success through
the franchising model.
As a franchisee, you will benefit from proven business practices, receive administrative support, and be able
to predict sales and expenses. You do not need prior experience since all franchisees receive extensive
training. You will fully enjoy the advantages of joining an established company!
m /
o
In addition, you now have the perfect chance to learn more about our partners by coming to the 24th Annual

c
c.
Franchise Expo in Fort Myers, Florida. Many well-known franchises that require the initial investment capital
you said you'd be willing to pay will be participating, such as AirCare Repair and Missy's Ice.

g o
If you have any questions, call us at 555-8965 or stop by our office at the address above.
Sincerely,
s n
Hilary Archer
Consultant
g m
k i n / ■'l '■ \

e a
ic sp
e
24th Annual Franchise Expo

//to
September 6-9, Fort Myers, Florida

tp : Request Info | Exhibitors & Sponsors Press Releases I Contact

ht
Previous I Next

Company Name Industry Initial investment


AirCare Repair Repair services
ShoeSaver Repair services
Barb's Chicken Food retail $32,000
Missy's Ice Food retail $20,000
Muscle Bound Fitness services $38,000
Space Cycle Fitness services $25,000
Clean Sweep Home services $26,500
To view a map of the facility with booth locations for participating franchises, click here. Booths are
limited to one per participant.

76 Ajgoj So^. atQ\^7\M O! mp3 Hackerslngang.com


Tampa Weekly

Is franchising right for you? (continued from page 1)

As I learned when I spoke to experts at the recently held Franchise Expo in Fort Myers, franchising is
not for everyone. Franchise Friendly's Sam Romanek told me, "Franchises may pose fewer risks than
start-ups, but they may not be a good fit if you value total independence and control."

As for choosing the right franchise, FBA's Barbara Allied advises doing your research. "Don't just
look at a company's recent performance," she said. "Find out how the overall industry is doing." For
instance, she cited the promising outlook for home services and a diminishing one for personal fitness.
"Fitness clubs have boomed in recent years," she added, "but the market is now saturated."

m /
186. According to the letter, what is NOT
c o
189. What is suggested about the 24th Annual
a benefit of owning a franchise? Franchise Expo?

o c.
g
(A) Foreseeable costs (A) It is held in a different city every year.
(B)
(C)
Thorough instruction
Financial support
s
sections. n
(B) It grouped industries together into

(D) Established work procedures


g m
(C) It is being sponsored by a charity

n
organization.
187. What can be inferred about Mr. Ainge?
k i (D) It featured fewer than 200 franchise

a
(A) Fie has decided to locate his business
e
owners.

sp
in Fort Myers.
190. Which business might Ms. Allred
(B) Fie budgeted $20,000 to invest in
a company.

e ic recommend?

//to
(C) Fie wants to attend an upcoming (A) Barb's Chicken
franchising event. (B) Missy's Ice

:
(D) Fie has a preference for a certain type

tp
(C) Space Cycle

ht
of industry. (D) Clean Sweep

188. In the letter, the word "model" in paragraph


1, line 3, is closest in meaning to
(A) version
(B) subject
(C) imitation
(D) system

GO ON TO THE NEXT PAGE,

TEST £ PART? 77
Questions 191-195 refer to the following order form and e-mails.

Majestic Porcelain
312 Franklin St, Georgetown, DE 19947
Tel. 555-6973 | www.majestic.com
Order No. 209154-68 Date: April 24
Deliver to: Lorraine Plummer
Customer type: 0 New □ Existing
Address: 31 Longhurst Road, Crawley, UK RH11 9SW
Telephone: 5555-7521
E-mail: l.plummer@britmail.com

Description Quantity Unit price Total price


LV-36291 "Seated Lady with Peacock" figurine 1 "$62.98 $62.98
QD-58792
LV-98201
"Angelic Cherubs" figurine
"Young Lady with Fan" figurine
4
1
$35.00
$42.99
$140.00
$42.99
m /
Subtotal
c o
$245.97
Shipping

oTOTAL c. $110.00
$355,97

n g
Note: You will receive a tracking number from our shipping provider once your order has left the
warehouse. Expect delivery 5 to 7 days after the shipping date.

m s
i n g
a k
e
To: Lorraine Plummer <l.plummer@britmaiI.com>

sp
From: Joel Nesbitt <j.nesbitt@majestic.com>

ic
Subject: Re: Order no. 209154-68

e
Date: May 5

//to
Attachment: Prepaid return shipping label

tp :
Dear Ms. Plummer,

ht I apologize for your recent experience. We always do our best to ensure that every item we ship is
received in good condition by wrapping each piece separately in impact-resistant packaging.
However, damage can still sometimes occur, particularly when items are sent overseas.
The problem is, as indicated on our Web site, that our shipping provider does not offer insurance, so
they don't cover the cost of items damaged during delivery However, we will compensate you at our
expense. Just mail the damaged item (item number LV-98201) back to us, and we will issue you a full
refund. We have attached a prepaid return shipping label for your convenience. Thank you, and we hope
that you will continue to do business with us in the future.
Sincerely,
Joel Nesbitt
Majestic Porcelain

78 Hackers.co.kr
To: Joel Nesbitt <j.nesbitt@majestic.com>
From: Randall Howe <r.howe@majestic.com>
Subject: Shipping providers
Date: May 25
Dear Mr. Nesbitt,
You asked me to look up alternatives to our current shipping provider, PK Logistics. Below is a table I've put
together of different providers. We want to avoid experiencing the sort of loss we had with Order no. 209154-68,
so I think we should go with one that can ensure us that won't happen. It would also be good if the new
provider can promise the same delivery time. Please check the table below, and let me know when you are
free to discuss it.

Shipper Insurance Tracking Fastest delivery


Super Mail Carriers
Corporate Parcel Sen/ice
Yes
No
Yes
Yes
5 days
5 days
Low-cost, delivers year-round
Strong overseas presence
m /
Global Delivery No No 7 days

c o
Discounts for bulk shipments

c.
Overseas Transport Specialists Yes No 8 days Largest branch network in US
Thank you.

g o
Randall Howe

s n
g m
n
191. What is indicated about Ms. Plummer? 194. According to the second e-mail, what
(A) She owns a retail establishment.
k i advantage does Global Delivery have

e a
(B) She has never transacted with Majestic
over other shipping providers?

sp
before. (A) Low-cost damage insurance
(C) She did not have to pay a shipping (B) State-of-the-art tracking technology
charge.
e ic (C) A large number of branches

//to
(D) She will be unable to track the delivery (D) Reduced pricing on large shipments
of her order.

tp:
195. Which shipping provider will Majestic

ht
192. In the first e-mail, what is stated about Porcelain most likely choose?
Majestic Porcelain? (A) Super Mail Carriers
(A) It will not process a request without a (B) Corporate Parcel Service
receipt, (C) Global Delivery
(B) It packs items individually for protection. (D) Overseas Transport Specialists
(C) It recently modified its return policies.
(D) It charges a flat fee to ship
internationally.

193. How much is being offered to Ms. Plummer


as a refund?
(A) $35.00
(B) $42.99
(C) $62.98
(D) $355.97
GO ON TO THE NEXT PAGE.

TEST 2 PART 7 79
Questions 196-200 refer to the following job posting, e-mail, and letter.

Ool
" MARKET ING POSI RONS AVAILABLE: Sav-Tech Incorporated, San Bernardino
Sav-Tech Incorporated is looking for promotions experts for its marketing department. Two full-time
contractual positions are available at our headquarters in San Bernardino, California, and the start date
for the job is February 1. Candidates must possess a degree in marketing, and we will only consider
applications from those with three or more years of experience working in the field.
The selected candidates will work as part of the marketing team. Their duties will include the
development, revision, and implementation of diverse marketing strategies to promote the sale of
Sav-Tech's software applications for mobile phones and tablets.
Send a cover letter and current resume to marketing@savtech.com to apply. Applications will be
accepted until January 15.

m /
c o
c.
TO Mia Choi <miachoi@headmanad.com>
FROM Pradeep Kumar <pradeepkumar®savtech.com>
SUBJECT Possible marketing position
g o
DATE January 14

s n
I
i
Mia,

g m
|
H
k i n
I was having lunch today with a colleague, Rupert Kim, who works in my company's marketing division. He said
that two members of their team are leaving and that his supervisor is looking to hire two new employees.
H
a
I immediately thought of you since I remembered that you were looking for a job exactly like this.

e
sp
I
j) I spoke with the marketing manager, David Bradley, and told him a bit about your experience and current

ic
situation. He seemed quite interested and said you could give him a call at 555-3928 to talk about the job and
;j
e
possibly set up a meeting. He said that there have been a lot of applicants who have the necessary educational

//to
qualifications for the position but few that fulfilled the other requirement. You should probably contact
Mr Bradley today. If he is interested, he will ask you to send a reference.
i
|
tp :
Pradeep

ht
Headman Adwertising Mgpmcf
www.headman.com
January I 8
David Bradley
Sav-Tech
886 N. Arrowhead Avenue
San Bernardino, CA 92401
Dear Mr Bradley
I am sending this letter in regard to Mia Choi, as I understand you are considering her for a position in your
marketing department at Sav-Tech. I would highly recommend Ms. Choi for such a role, as she has been an
exceptional employee over the last six years at Headman Advertising Agency in Los Angeles.

80 ^ ooi-q^ . o. yojopi mpb Hackerslngang.com


I have worked as Ms. Choi's director for several years now, and find her to be intelligent and creative. She has helped
our agency develop many successful promotional campaigns, including work for Gleaming Studios and Holstein Dairy.
Her field of expertise is social media, and she is very knowledgeable about current technological advances in regard
to marketing.
I will miss working with her; but I understand this would be an excellent opportunity for her career I'm sure you'll
appreciate her as much as I have.
Sincerely yours,
Richard Morrison,
Creative Director
Headman Advertising Agency

196. What type of company is Sav-Tech? 199. What is suggested about Mia Choi?

m /
o
(A) A marketing and promotions firm (A) She will send a job application to
(B) A seller of applications for mobile
devices
Mr. Morrison.

c.
(B) She contacted a marketing managerc
(C) An online advertising service for at Sav-Tech.

g o
businesses
(D) A recruitment agency for software
s n
(C) She hopes to relocate to a larger
company in Los Angeles.
programmers

g m
(D) She was the head of a team at
Headman Advertising Agency.
197. What did Mr. Bradley indicate about most

k i n
e
(A) They have insufficient experience ina
of the applicants for the marketing jobs? 200. Who is Richard Morrison?
(A) A former client of Mr. Bradley
marketing.

ic sp
(B) They are not familiar with software
(B)
(C)
A software marketing expert
A proprietor of a software firm
development.
e (D) A current supervisor of Ms. Choi

//to
(C) They are unavailable for the positions

tp :immediately.
(D) They do not have degrees in marketing.

ht
198. What did Mr. Kumar do on January 14?
(A) Had a meeting with a colleague from
Los Angeles
(B) Discussed some job vacancies with
a coworker
(C) Met David Bradley for lunch
(D) Read through the terms of a contract

This is the end of the test. You may review Part 5, 6, and 7 if you finish the test early.

sa-P.B24 / St® p.327 /Sim P.3B5 / Part 5&6 olM U|£M7|


• Si|o|x|oj 2te Self *||a. m m »o| Eflst ggofl aAiia.

TEST 2 PART 7 81
otl^ m lOOOXil 3 READING

m /
c o
o c.
n g
m s
i n g
a k
I TEST
e
sp
OB

e ic
//to
PART 5

tp : 01 PART 6

ht 0 PART 7

Self *l|a e|Ae

S-»H Ell A e *j
, ocHMiggiiL^H oo||
2. Answer Sheet, n oj|
3.A|7l|#^b|o^L|-o7 noii
o- £H||BI^ODI [[itEj o Ei|XEt A|^L|LK

^11 #0® U\t\t A|^ AI^EI 70^ °e! _A| _^L|EK


Ell ae 11^ # ysgoiDi. A|^ ^2 Aj 5^ kef g E 51 aixi afei# ?|6|| a^^|
READING TEST
In this section, you must demonstrate your ability to read and comprehend English. You will be given a
variety of texts and asked to answer questions about these texts. This section is divided into three parts
and will take 75 minutes to complete.
Do not mark the answers in your test book. Use the answer sheet that is separately provided.

parts

Directions: In each question, you will be asked to review a statement that is missing a word or phrase.
Four answer choices will be provided for each statement. Select the best answer and mark the
corresponding letter (A), (B), (C), or (D) on the answer sheet.

O parts w0! A|y

m /
101. Hanson Inc.'s marketing team to

c o
105. Members who renew their newspaper

c.
finalize the television advertisement for the subscription at least one month prior to its
company's newest touch screen monitor. will receive a 5 percent discount.
(A) assembling (A) opening
g o
(B)
(C)
assembled
to assemble
(B)

s
(C) n
expiration
obligation
(D) assembly
g m(D) statement

102. The construction of a new subway line was


k i n
106. Lorasoft's customer service agents are
aimed at overcrowding during rush

e a instructed to handle minor requests

sp
hour. before forwarding calls directly to the
technical support team.

ic
(A) imposing
(B) preparing (A) their
e
//to
(C) relieving (B) them
(D) benefiting (C)they

tp : (D) themselves

ht
103. Ms. Bedford was chosen to represent
Centraine because of her considerable 107. The popularity of the daily television show,
in mergers and acquisitions. Celebrity Update, has declined the
(A) experts years as entertainment news has become
(B) expertly increasingly available online.
(C) expert (A) toward
(D) expertise (B) until
(C) as to
104. The wireless headset by Smartcom's (D) over
representative at the Fentwood Technology
Trade Show will go on the market next 108. Scientists employed by Fluent
month. Pharmaceuticals spend most of their time
(A) demonstrate medications to treat diabetes.
(B) demonstrates (A) developing
(C) demonstrated (B) practicing
(D) demonstrating (C) depicting
(D) signaling
84 iisq S°JiUh 901°^ ni yojopi Mp3 Hackerslngang.com
109. who needs to access information on 115. The latest version of the accounting
the corporate Web site must first obtain software is not with the operating
login credentials from the IT department. system installed on the office computers.
(A) Those (A) rational
(B) Anyone (B) challenged
(C) Most (C) acquainted
(D) All (D) compatible

110. Flintrock Manufacturing employees are 116. Automat CEO Sergei Rostov's speech on
required a training program in trends in the high-tech industry drew an
audience consisting mostly of H
m
equipment safety in order to operate heavy
machinery on the job. entrepreneurs. H

(A) completing (A) aspiring O


(B) complete (B) aspired
(C)
(D)
to complete
completely
(C)
(D)
aspire
aspiration
m / CD
r\j

c o
c.
5
111. Had Mr. Shriver booked his vacation 117. Since acquiring the land on Taylor Avenue CD
4>
package further in advance, he
a much lower price.
g o
five years ago, the proprietor multiple
offers from developers wishing to buy it. CD

n cn

ms
(A) is being paid (A) had been receiving O
(B) will receive CD
(B) would have paid

ing
(C) would have been paid (C) has received CD
(D) to receive

ak
(D) has paid
CD
CO

e
112. Most of the conference participants arrived

p
118. BRE Manufacturing is unlikely to take
chances on new names as brands CD
UD
s
an hour ago, but the keynote speaker is

ic
on his way. are more appealing to its target market.

e
O
(A) still (A) familiar

//to
Of
(B) already (B) proficient 3!

tp :
(C) before (C) sufficient
(D) aggressive

D-
Kn

ht
(D) lately nj>;
r2d
Oo
113. Research indicates that consumers are 119. The cast was interviewed by the press oX
LU
much more likely to purchase products the first performance of the play X)
Father Simon. OJ
Q.
from companies that show for the
environment. OQ
(A) rather
(A) compliance (B) as soon as
(B) designation (C) following
(C) concern (D) between
(D) diversity

114. Mr. Reed made to rent a car, as he


planned to meet with several clients during
his trip.
(A) arranges
(B) arranging
GO ON TO THE NEXT PAGE.,
(C) arranged
(D) arrangements
TEST 3 PARTS 85
120. Ms. Morgan's responsibility as the 125. Mr. Trevors overcame the difficulty of
company's head of operations is to ensure speaking French by improving his
that every department is running smoothly. skills with regular practice.
(A) reputable (A) gradually
(B) principal (B) respectively
(C) ethical (C) intimately
(D) efficient (D) haltingly

121. Home Choice is looking for suppliers who 126. Dong Suk Kim's promotion was approved
can - produce the various parts it after the sudden of the company's
needs for its new line of kitchen appliances. chief executive officer,
(A) afford (A) gratitude
(B) affording (B) departure
(C) affordable (C) routine
(D) affordably (D) adjustment

m /
122. Eastern Automotive is the largest producer 127. The
c o
script for the novel of Winding
of vehicle parts, only Dreier Limited
in terms of annual profits. c.
Road will serve as a screenplay for the
upcoming film.
o
(A) within (A) adapt
n g
(B)
(C)
behind
among
m s
(B)
(C)
adapted
adaptability
(D) beyond

i n g (D) adaptively

123. The owner of White Pearl Restaurant has


ak 128. the run-down parking lot, the rest of
always found conducting regular surveys
e the building's facilities are in excellent

sp
to be a way of obtaining customer condition.
feedback.
(A) reliant
e ic (A)
(B)
Regarding
Namely

//to
(B) reliability (C) Other than

tp :(C) reliable (D) Or else

ht
(D) relying
129. Although there are numerous household
124. students have requested on-campus cleaners being sold, many shoppers
housing, they must wait at least a month simply purchase ones are being
for an official response. promoted.
(A) Once (A) when
(B) Then (B) what
(C) Yet (C) whichever
(D) During (D) which

130. Providing senior managers with


incentive-based compensation is a
solution to their declining productivity,
(A) variable
(B) thriving
(C) discrete
(D) practical

86 Hackers,co.kr
PART 6
Directions: In this part, you will be asked to read four English texts. Each text is missing a word,
phrase, or sentence. Select the answer choice that correctly completes the text and mark the
corresponding letter (A), (B), (C), or (D) on the answer sheet.

© PARTS as #0|ApJ 8S

Questions 131-134 refer to the following e-mail.

To: Collin McCabe <cmccabe1 @ncmail.com>


From: Virginia Payton <virginiapayton@clarkstel.com>
Date: February 22
Subject: Your Clarks Telecom bill
m /
c o
c.
Dear Mr. McCabe,

Please be informed that your monthly Internet bill


131 > o
$48.25 should have been paid by
g
s n
February 15. Payment is due immediately. Otherwise, a fee worth 5 percent of the amount

g m
owed will be added every month until your outstanding balance has been

k i n
At this time, we would like to recommend that you use our automatic payment system. --------

e a lUu.
Simply log into your account, go to "Payment Settings," select "Automatic," and click "Submit."

ic sp
If you feel that there has been a billing error, you
134.
to contact us and we will gladly review

e
//to
your payment history.

tp :
Virginia Payton

ht
Clarks Telecom

131. (A) totaled 133. (A) This may affect when you start
(B) totally |||i| receiving high-speed service.
(C) totaling (B) The money you owe will be paid on the
(D) totals scheduled date through this means.
(C) We have sent a receipt confirming
132. (A) canceled payment of the charges.
(B) settled (D) Your service will be restored once you
(C) insured send the amount that is due.
(D) forgotten
134. (A) encourage
(B) encouraging
(C) are encouraged
(D) will encourage
GO ON TO THE NEXT PAGE..

TEST 3 PART 6 87
Questions 135-138 refer to the following advertisement.

Waterview Valley Ranch is the perfect place to take a horse-riding holiday. Discover the

beauty of the surrounding area and about its past as our knowledgeable guides delight
135.
you with interesting stories of its history. you like riding horses for relaxation or prefer a
136.
challenging ride in the open country, we have all the equipment you'll need to fulfill your

requirements. In addition, should you choose to bring your own horses, we can
137. 138.
them during your stay with us. This is because our stables have ample space, where they will

be well cared for. Simply let us know what your party needs when you call us at 555-7823 to

book your spot.

m /
c o
135. (A)
(B)
learned
learns
o c.
137. (A) Our regularly scheduled guided tours
have been overbooked.
(C) learn
n g
(B) This ensures your experience with us
(D) to learn

m s is both safe and satisfactory.


(C) The livestock we raise is renowned for
136. (A)
(B)
Even
Although
i ng its exceptionally high quality,

k
(D) Your horse-riding gear must meet the
(C)
(D)
Only if
Whether
e a strict standards of the ranch.

sp
138. (A) switch

e ic (B)
(C)
race
house

//to
(D) auction

tp :
ht

88 VS££(?!i!Sa|fie)3Af.EK)i°plS9!E!0)^|MP3 Hackerslngang.com
Questions 139-142 refer to the following announcement.

Important Announcement

Beaumont Hall announces with that the August 17 concert featuring the pianist Sophia

Kaminsky has been canceled. Ms. Kaminsky has suddenly taken ill, and organizers have

deemed it best to ----- the event until further notice. If you purchased your ticket using cash,

you will receive an immediate refund. On the other hand, those who paid for tickets with a

credit card will not get a refund. This is due to card processing regulations.
141.

in addition to being fully refunded, everyone who purchased a ticket will receive a 20 percent

m /
discount when they make a future reservation.

c o
Hopefully, the discount makes up for

the inconvenience. Please accept our sincerest apologies.

o c.
n g
139. (A)
(B)
regrettable
regret lHll
m s
142. (A) We can no longer offer group rates to
those without valid memberships.
(C) regrettably

i n g
(B) We cannot combine this price reduction
with any other special promotion or
k
(D) regretful

140. (A)sponsor
e a discount.
(C) We are committed to starting each of

sp
(B) evaluate our productions in a punctual manner.
(C) confirm
(D) postpone
e ic (D) We have posted the dates and times
for this performance on our Web site.

:
141. (A) previous

p
(B) formal //to
ht t
(C) prompt
(D)dependent

GO ON TO THE NEXT PAGE,

TESTS PART 6 89
Questions 143-146 refer to the following letter.
L
>k
■■ Emma Florins
1119 Parrish Drive
Rockville, MD 20851

Dear Ms. Florins,

You are cordially invited to Speaking Poetry, a recital with three of the literary world's most

----- poets. Adalberto Marte, Marceline Fontes, and Lawrence Faucher will be reading their

recently published works.

Our past
144.
have mainly featured works adhering to a more traditional style. However, we

m /
o
are pleased that the diversity of the genre will be able to shine through this time. Our invited

c
c.
writers discard regular forms and rhyming patterns characteristic of conventional poetry.

o
145.
n g
Focusing on such topics as our rapidly changing society and the impact of technology,

146.
poems are sure to resonate with audience members.

m s
i n g
The recital will take place on October 10 at the Rocheport Arts Center. We hope to see you

a k
e
Sincerely,

ic sp
e
//to
Abraham Schlitz
Rocheport Arts Center

tp :
ht
143. (A)
(B)
imminent
crowded
145. (A) With the right publicity, these poets will
be able to become well known.
(C) widespread (B) There are similarities between
(D) promising traditional and contemporary works.
(C) Instead, they rely on expressive writing
144. (A) performances to address modern issues.
(B) publications (D) Many guests have much preferred the
(C) communications themes of our earlier recitals.
(D) competitions
146. (A) his
(B) its
(C) your
(D) their

90 Hackers.co.kr
PART
Directions: In this part, you will be asked to read several texts, such as advertisements, articles, instant
messages, or examples of business correspondence. Each text is followed by several questions. Select
the best answer and mark the corresponding letter (A), (B), (C), or (D) on your answer sheet.

© PART? 54^

Questions 147-148 refer to the following advertisement.

Are you letting high prices get in the way of your fitness goals?
Everyone is talking about yoga these days—in magazines, on television, and F?
m /
even during chats with friends. But while we've all heard about the many

c o
c.
health benefits that yoga provides, looking around yoga stores can be LJ
intimidating. With even a simple yoga mat costing up to $50 or more, yoga
can seem like an expensive activity to get into!
g o
s n
Well, not anymore. Merchandise at FlexiWorld Yoga ranges from top brand name yoga

g m
equipment to cheaper gear for beginners. And we can afford to offer all of our products at
lower prices than department stores and other retailers because we operate entirely online.

k i n
So if you're thinking of trying out yoga, but don't want to overspend, head to www.
flexiworldyoga.com!

e a
ic sp
e
//to
147. What is the advertisement mainly about? 148. According to the advertisement, how can

tp :
(A) A subscription to a health magazine FlexiWorld Yoga offer reasonable prices?
(A) It retails its merchandise solely online.

ht
(B) A discounted yoga course
(C) An exercise equipment store (B) It buys its goods at a discount.
(D) A series of fitness videos (C) It manufactures its own products.
(D) It has exclusive deals with suppliers.

GO ON TO THE NEXT PAGE^

TESTB PART 7 91
Questions 149-150 refer to the following announcement.

; i
The Ramford Center for the Arts

vl ]
Our center seeks a dynamic individual to join its fundraising team at our main facility in
Houston. Reporting to the executive director, the fundraising associate will be responsible
for requesting donations for the various year-round programs and events we hold at our
center.

Successful candidates need to possess a bachelor's degree in nonprofit management, fine


arts, or a similar field. Relevant experience is preferred but not required.

The Ramford Center for the Arts offers a competitive compensation package, including
medical insurance, generous vacation allowances, pension contributions, and
m /
complimentary admission to all exhibitions for its entire staff.
c o
% '■ . . , , .
Interested applicants should send their cover letter and resume to Andrea Perkins
o c.
(aperkins@ramfordarts.com) by Friday, October 3.

n g
m s
i ng
149. What is the purpose of the announcement?
(A) To inform personnel of a change
a k 150. What is mentioned about the center?
(A) It has achieved its annual fundraising
e
sp
(B) To recruit a new employee goal.
(C) To announce a yearly event (B) It allows its staff to attend exhibits for
(D) To
ic
promote a new program

e
free.

//to
(C) It offers gift packages to visitors.
(D) It only hires staff with previous

tp : experience.

ht

92 ^ £££! 22JI!^. e0O7|£ ^ yoiopl MP3 Hackerslngang.com


Questions 151-152 refer to the following text-message chain

Frank Cameron 2:22 P.M.


2:2:
Have you heard when production on the new laptop will begin?
Liz Cohen 2:24 P.M.
I called Stan Lowell, and he said he can't get things started at the
factory for at least another four weeks.
Frank Cameron 2:26 P.M.
That's not good news. What's causing the delay?
Liz Cohen 2:28 P.M.
He says the plant can't proceed until the engineering department
finishes testing the product. It's taking them longer than anticipated.

m /
Frank Cameron 2:29 P.M.

c o
c.
But we're scheduled to ship the computers to stores at the beginning
of next month. Do you think that it will be possible to meet that
target?

g o
Liz Cohen
Not a chance given the current situation.
s n 2:31 P.M.

Frank Cameron
g m 2:33 P.M.

i n
I see. How do you think the sales representatives will take the news?

k
Liz Cohen

e a 2:34 P.M.
They'll be fine as long as we inform them in advance. At this point,

sp
we just need to focus on shipping the best possible product to

ic
market, even if it is a bit late.

e
p : //to
ht t
151. Why is production delayed?
(A) A device is still in the process of being
152. At 2:31 p.m., what does Ms. Cohen mean
||||| when she writes, "Not a chance"?
tested. (A) She thinks that too few units of an item
(B) A factory was forced to shut down will be produced.
temporarily. (B) She is doubtful that a recently released
(C) A new feature was recommended by product will have high sales.
sales representatives. (C) She doesn't believe a department has
(D) A store has decided to cancel its order sufficient staff to meet a deadline.
of goods. (D) She doesn't think computers will be
shipped on a planned date.

GO ON TO THE NEXT PAGE,

TESTS PART? 93
Questions 153-155 refer to the following memo.

I |
i I
To: All TechBase Staff
From: John Borges
Subject: Speaking event
Date: August 20
1 1
Hello everyone,
I am pleased to announce that we will have an exclusive speaking event here at
the TechBase office on Wednesday, September 12. Elsa Goncalvez, author of the
book The Key Components of Selling, will be coming in to talk about proven
marketing strategies that she has learned over the years. Ms. Goncalvez previously
worked with distinguished enterprises in the same field as our company, including
two of the country's top computer manufacturers, Stentech and Microtrax, so I'm
sure that her advice will be helpful to us as well.
m /
1
Attending the event is not mandatory, but I encourage your participation.
c o 1

c.
Ms. Goncalvez will be focusing on tools and techniques that we can easily adopt,

g
creating effective advertising materials for various forms of media.o
including understanding customer needs, defining unique selling points, and

s n
Everyone who attends the talk will also receive a free copy of The Key
I

ij
g m
Components of Selling. I hope to see many of you there.
i
Best wishes,

k i n
John Borges

e
Administrative associate a
ic sp
e
//to
153. What is indicated about TechBase? 155. What is true about the event?

tp:
(A) It has invited Ms. Goncalvez to speak (A) It will take place at a rented venue.
before. (B) Attendees will be given a copy of a

ht
(B) It works in conjunction with two firms.
(C) It is a computer-related company.
(D) It hired a new full-time marketing
book.
(C) All staff members are obliged to attend
it.
specialist. (D) Participants must read some material
beforehand.
154. What will NOT be discussed during the
talk?
(A) Comprehending customer needs
(B) Producing successful advertisements
(C) Forming effective sales teams
(D) Identifying product selling points

94 Hackers.co.kr
Questions 156-157 refer to the following notice.

Sheltonville Town Council

NOTICE: Residents of Smith Street and


Surrounding Areas

Please be aware that a public forum will be held to talk about the
ongoing work at the corner of Smith Street and Lincoln Avenue, where
an apartment complex is in the process of being constructed. We have m
LQ
been informed that the project is about to enter its building phase, and -H
this is expected to last until the end of September. During this period, it O
is likely that residents will be affected by road closures and increased
levels of noise from the worksite. Additionally, the sidewalk will be
closed oft to pedestrians in the interest of safety.
m / O
m

Representatives from the construction firm of Blakely and Reid will be


c o
c.
on hand to respond to questions from residents. The forum will be o
4>

go
moderated by Joseph Coran, member of the city council representing
O
LTl
District 1 1.

s n
The forum will begin at 6:00 p.m. on Tuesday, July 1 0. It will be held at O
cn

m
the Evergreen Community Center, and everyone in the neighborhood is

ing
o
free to attend.

a k
pe
s
156. What is the main purpose of the notice?

ic
157. What is indicated about Joseph Coran?

e
(A) To warn people of upcoming road (A) He has questions about a building

//to
closures project.
(B) To express worries about pedestrian (B) He will facilitate a public forum.

tp :
safety (C) He is an employee of a construction

ht
(C) To invite participation in a public firm.
discussion (D) He is running for office in Sheltonville.
(D) To solicit suggestions for the use of a
space

GO ON TO THE NEXT PAGE

TEST3 PART? 95
Questions 158-160 refer to the following article.

Belaiar University to Ooen High-Tech Hub

Malaysia's Belajar University yesterday announced that it is beginning to build a new campus in
Malacca City that is expected to be completed by the end of next year. The new campus is part of an
outreach strategy to encourage students from other parts of Malaysia to apply to the prestigious
university, the campuses of which are primarily located in the capital Kuala Lumpur. "Although we
have seen an increase in the number of applicants from outside Kuala Lumpur in recent years," said
university vice-chancellor Abdullah Mohammed, "this new campus is sure to attract even more."

The new campus is also being partially funded by the Malaysian government, which considers it a
key part of its plan to develop the southern part of the country into a high-technology hub. Because of
this, the campus will feature a number of state-of-the-art computer laboratories that will be used to

m /
develop new types of microchips and computer processors. A new high-speed rail link between the

c o
c.
capital and Malacca City is also being constructed by the government as part of the process of
encouraging technology companies to relocate to the smaller town.

g o
s n
Qing Weihai, governor of Malacca State, said, "We welcome this decision by the Malaysian
government and Belajar University and expect the new campus to create thousands of new jobs in the
region by encouraging businesses to move here."

g m
k i n
158. What is stated about Belajar University?
e a 160. What does Qing Weihai anticipate will

sp
(A) It is trying to attract students from happen?

e ic
diverse areas of the nation.
(B) It is celebrating the 100th year since its
(A) A state will receive more foreign
visitors.

//to
establishment. (B) A major road will be reopened to the

tp :
(C) It has recently opened a new students'
union building.
public.
(C) Construction of a school building will

ht
(D) It has the highest entrance standards be expedited.
in the country. (D) Local citizens will have more
employment opportunities.
159. What is being built by the Malaysian
government?
(A) An employee training campus
(B) A wider road to the capital
(C) A fast railway connection
(D) A residence for students

96 a°|;au ■ EtoigPIg 5! E!0®?l MP3 Hackerslngang.com


Questions 161-164 refer to the following e-mail.

TO Carolyn Shah <cshah@qrcbooks.com>


FROM Edwin Lewis <elewis@caspub.com>
SUBJECT Publication information
DATE July 19

Dear Ms. Shah,


I am pleased to tell you that Casper Publishing has come out with an important title that
I think QRC Bookstore will be interested in selling. — [1] —.
The 10th edition of Economics for Business has been completed and contains revised
multimedia content and new exercises in DVD format. We have one version for students and
another for teachers. The students' version is packaged for affordability, while the teachers'
m /
o
version features several premium extras, such as preloaded presentation. — [2] —. Seeing as

c
c.
QRC Bookstore mainly sells university textbooks, it would make perfect sense for your shop to
carry both versions of this publication or any of the other new titles in our catalog. — [3] —.

g o
If you are interested in ordering, download the form on our Web site at www.caspub.com/orders

e-mail or contact me directly at 555-3044.


s n
and send it to us by post. — [4] —. For any further information, feel free to respond to this

Best Regards,
g m
Edwin Lewis

k i n s

e a
161. Why was the e-mail written?

ic sp 163. How can Ms. Shah place a product order?


(A) To
e
request an updated product list (A) By submitting a form online

//to
(B) To give feedback on a purchase (B) By mailing a document
(C)

p
(D)
t : To
To
inquire about a popular book
promote a publication's release
(C)
(D)
By calling a phone number
By responding to an e-mail

ht
162. What is implied about QRC Bookstore?
(A) It is owned by a former professor.
164. In which of the positions marked [1], [2],
mSk [3], and [4] does the following sentence
(B) It sells all of its products online. best belong?
(C) Its customers include university "These may be used in the classroom
students. along with the extensive lesson plans
(D) Its products are discounted for found in each chapter."
teachers.
(A) [1]
(B) [2]
(C) [3]
(D) [4]

GO ON TO THE NEXT PAGE.

TEST 3 PART? 97
Questions 165-168 refer to the following online chat.

Ool

^ Clara Dalton [2:01 P.M.] Good afternoon, everyone. As I'm sure you all heard on Monday, the
board of directors has scheduled the opening of the Erik Masterson art
exhibit for next month. We need to start letting people know about it, and
I'd like to get a sense of how everything's going.
Aaron Cobble [2:04 P.M. I finished designing the personalized invitations for everyone who
attended our last exhibition, but the mailing list hasn't been compiled
yet. George, do you think this might cause any problems?
George Duffy [2:10 P.M. A delay of one day is fine. But the guests should receive their invitations
before Saturday when we publish a press release about the opening. So,
we need to get them to the courier by Thursday at the very latest.
O Clara Dalton [2:12 P.I All right. I'll ask Steven to take care of it. Deborah, how about the
posters?
Deborah Emerson [2:14 P.M. They're ready to be printed. Do you have any thoughts on where
m /
I should tell our volunteers to put them up?

c o
c.
Clara Dalton [2:16 P.M. I've heard that Masterson's work is popular with college students.
Deborah Emerson [2:20 P.M.
o
Oh, what about Randall University? The gallery is within walking distance
of it.
g
Clara Dalton
s n
[2:21 P.M. That sounds great. Maybe we should give a presentation on campus to
promote the exhibit as well.

g m
Deborah Emerson [2:22 P.M. Sure. I'll call the university to check on that.

k i n
e a
ic sp
165. What kind of organization does Ms, Dalton 167. At 2:20 p.m., what does Ms. Emerson

e
most likely work for? mean when she writes, "what about

//to
(A) An art gallery Randall University"?

tp:
(B) A local university (A) She thinks a school may be a good

ht
(C) An advertising agency place to promote an event.
(D) A printing company (B) She believes that she will be able to
recruit student volunteers,
166. When will the event be formally announced? (C) She intends to have some posters
(A) On Monday printed out at a local college.
(B) On Tuesday (D) She would like to try displaying art at
(C) On Thursday Randall University.
(D) On Saturday
168. What is NOT mentioned as a method of
publicizing the opening?
(A) Speaking at a university
(B) Contacting previous visitors
(C) Posting advertisements
(D) Updating a Web site

98 ?aMews-flaga*||g Hackers.co.kr
Questions 169-171 refer to the following letter.

Adam Suzuki
12 Westbourne Avenue
Hull, UK, HU2 7HF
November 22
Dear Mr. Suzuki,
I am pleased to announce a change to your North Bank account. Currently, you can overdraw up to £250
from your checking account when needed. — [1] —. As you have been a loyal customer with us over the
past three years and have an excellent credit rating, your overdraft limit has been increased to £750. The
increase will take effect at the beginning of next month. — [2] —.
In addition, I would like to let you know that you are eligible to apply for our Silver Spender and Graduate
Spender credit cards. — [3] —. Some details about the benefits of each card are listed below. If you are
interested in applying for either card, please fill out the attached form and send it to us. There is no need to
do anything else as you have already been preapproved. — [4] —.

m /
Silver Spender Credit Card
e
10% discount on movie admission fees
c o
c.
e
0% interest on installment plans for up to 24 months

o
0
£25 annual fee
Graduate Spender Credit Card
n g
0
e
Access to Greyline Air's VIP lounges in airports nationwide
0% interest on installment plans for up to 12 months
m s
g
0
£45 annual fee

k i n
Should you have any questions or concerns, please feel free to contact us at any time.
North Bank Customer Service Team

e a
ic sp
e
//to
169. What is the purpose of the letter? 171. In which of the positions marked [1], [2],
||p [3], and [4] does the following sentence

t :
(A) To ask for details regarding an

p application best belong?

ht
(B) To describe some beneficial "Other terms and conditions associated
developments with your account will remain the same."
(C) To explain changes in bank regulations
(D) To suggest switching to a newer (A) [1]
(B) [2]
product
(C) [3]
170. What is indicated about the Graduate (D) [4]
Spender Credit Card?
(A) It can only be applied for in person.
(B) It has a lower annual fee than the
Silver Spender.
(C) It involves a lengthy approval process.
(D) It offers interest-free installments for
up to one year.

GOONJOTHENEXTPAGE^

TEST 3 PART 7 99
Questions 172-175 refer to the following article.

Health First Magazine


April Edition

Leading Fitness Trends

Spring is always the busiest season for fitness centers as


individuals prepare to get in shape for the coming summer
months. And gym owners hoping to have a competitive edge over
other facilities will be pleased to know that popular health
equipment manufacturer RockTech has recently launched a new
line of fitness equipment with some impressive features.

The head of product design for RockTech, Nancy Flores, said that

m /
o
the new equipment is sleek in appearance and incorporates the
latest technological advances. The RockRunner treadmill, for

c.
instance, automatically adjusts settings, such as speed and incline,c
g o
in correspondence with satellite map routes to give users the
experience of jogging in real terrain.
s n
m
Furthermore, Ms. Flores noted that RockTech will soon release a

g
k i n
new line of weight training equipment. The machines will include
highly sophisticated, newly developed calorie counters that can be

e a
synchronized with wearable fitness trackers. The machines will

sp
also include calorie counters. "People focused on burning calories

e ic
can easily see their progress," Ms. Flores explained. She also
stated that RockTech plans to have this feature on all newly

//to
released products.

tp : These and other products are available on the company's Web site,

ht
www.rocktech.com, but they are presently only sold to gyms and
health clubs.

100 ¥^^1UMS] 20|JUh EfOiop|$ 5J MP3 Hackerslngang.com


172. What is the main topic of the article? 174. The word "edge in paragraph 1, line 3, is
closest in meaning to
(A) The opening of a fitness center
(B) The popularity of an exercise class (A) approach
(C) The release of workout products (B) sharpness
(D) The recall of a health equipment (C) advantage
(D) threshold
173. What is NOT mentioned about the
RockRunner? 175. What is indicated about RockTech?
(A) It makes use of recent technology. (A) It sponsors an assortment of athletics
(B) It corresponds with map data. competitions.
(C) It can be purchased in select locations (B) It will include a new feature in its
(D) It allows users to feel as if they were upcoming products.
jogging.
clubs.
m /
(C) It offers discounts to gyms and health

o
(D) It has regional offices all over the

c
c.
world.

g o
s n
g m
k i n
e a
ic sp
e
p : //to
ht t

GO ON TO THE NEXT PAGE

TEST 3 PART 7 101


Questions 176-180 refer to the following invitation and list.

Come to the SRA9s Annual Wine Tasting Fund-raiser!

Saturday, November S
from 6:00 p.m. to 11:00 p.m.
at the Lambdia Hotel Grand Ballroom
The Sight Right Association (SRA) invites you to our yearly fund-raiser. You can indulge in a
variety of gourmet foods and upscale wines specially selected to appeal to discriminating tastes.
Enjoy the musical stylings of Willis Gustavo's Jazz Ensemble, which will be on hand to perform
a number of tunes.
m /
c o
c.
During the fund-raiser, an auction will also be held, where guests will have the opportunity to
place bids on all of the wines served at the event. After a short welcome speech by an SRA

g o
representative, chefs will begin preparing food at stations placed around the Grand Ballroom.

s n
Guests are free to help themselves to food while they socialize. And don't forget to visit the
reception area, where you can enter a draw to win a free bottle of Miller-Ortega Riesling wine!
Tickets:
g m
$85 for SRA members/$145 for SRA member couples

k i
$95 for non-SRA members/$165 for non-SRA member couplesn
e a
* Must be reserved by calling 555-3509 no later than November 1.

ic
prescription lenses. sp
All proceeds raised during the event will be used to provide the needy with eye care and

e
p : //to
ht

t SRA Fund-raiser Food and Wirie List
1
f
Location Food Wines
33
Station One - Spinach and cranberry salad White wine
i - Blackberry compote with roasted chestnut cream Chateau 54 Sauvignon Blanc
Station Two - Focaccia bread with mozzarella and black olives White wine
- Mushroom risotto Mosdel Pinot Gris
;■
Station Three - Salmon with brown sugar and mustard glaze Red wine
Pewter Vineyards Pinot Noir
Dessert Station - Assortment of miniature pastries Sparkling wine
- Chocolate truffles Wildfelt Moscato
- Hot coffee and tea
Chefs will prepare small plates of food item that guests can pick up at each station with their wine.

102 Hackers.co.kr
176. What is NOT true about the event? 179. What is NOT indicated in the list?
(A) It is cheaper for SRA members to (A) Wines will be provided to diners at
attend. every station.
(B) A live band will entertain guests. (B) Guests will be served meals at their
(C) There is a deadline to reserve tickets. tables by staff.
(D) It will take place throughout the (C) Seafood dishes will be prepared in
weekend. Station Three.
(D) Guests will be able to enjoy hot
177. In the invitation, the word "upscale" in beverages at Dessert Station.
paragraph 1, line 2, is closest in meaning
to 180. What can be inferred about the auction?
(A) eligible (A) It will set the starting bids for available

/
(B) exclusive items,
(B) Items there have been donated by
m
(C) economical
(D) excessive SRA members.

c o
c.
(C) Participants will receive a box of
178. Why might guests visit the reception area? chocolate truffles.
(A) To sample a salad with some red wine
g o
(D) Sparkling wine will be available to
(B)
(C)
To enter to win a bottle of wine
To pay for tickets to the fund-raiser
s
bid on.
n
(D) To check out items available for
g m
auction

k i n
e a
ic sp
e
p : //to
ht t

GO ON TO THE NEXT PAGE

TEST 3 PART? 103


Questions 181-185 refer to the following notice and e-mail.

fftotiee t@ Visitors

We've had a great summer season here at the Newport County Amusement Park and Zoo, but as the
winter season approaches, we're going to be closing sections of the park for maintenance and repairs.

Starting on October 1, the Mach One rollercoaster will be closed for inspections and repairs until the
beginning of the next summer season. In addition, certain animal enclosures will be shut down
temporarily to allow workers to perform needed renovations. These measures are vital to ensure the
future safety and enjoyment of all park visitors.

To make up for the inconvenience, we have some new winter attractions coming up. The days may be
getting shorter, but that just gives us a great excuse to hold a fireworks show, every day at 7 p.m. from

m /
o
October 22 to December 4. On top of that, all of our restaurants will be serving our brand-new winter

c
c.
menu starting on September 30. The menu was created by celebrity chef Alexandre Mapouche and is
based on European winter delicacies.

g o
staff.
s n
If you have any questions about these changes, please ask any member of our helpful and informative

g m
k i n
e a
To: Customer Relations <relations@ncapz.com>

Subject: Feedback
ic sp
From: Sandra Higson <Sandra1973@zipmail.com>

e
p : //to
Dear madam or sir,

ht t I would like to report an issue I had at the Newport County Amusement Park and Zoo regarding
the closure of one of your rides. My son had wanted to visit your park for many months so he
could ride the Mach One rollercoaster. I finally found the time to take him, and he was very
excited. However, upon arriving at the park, we saw the notice outside the ticket office
announcing that the Mach One was closed for maintenance.

We decided to visit the park anyway, as we had traveled all the way from Colebrook, but my son
was very disappointed. I believe that, at the very least, we are owed an apology since you did
not announce the closure on your Web site; and perhaps we are even entitled to a refund of half
the amount we paid for our tickets because the attraction we were most Interested in was
| unavailable.

I look forward to your response.

Sandra Higson

104 esjiuhMP3 HackGrsIngang.com


181. What is suggested about the daily 184. What is implied about Ms. Higson?
fireworks show? (A) She was unable to see the fireworks.
(A) It will temporarily be halted during (B) She went to the park after October 1.
maintenance. (C) She plans to visit the zoo in December.
(B) It will take place in a new section of (D) She lives in the same city as the park.
the park.
(C) It is not usually held during the 185. What does Ms. Higson request?
summer. (A) A replacement ticket
(D) It costs visitors extra to watch. (B) A partial return of a payment H
m
in
(C) A coupon for free parking -1
182. What is mentioned about the winter menu? (D) A map of the amusement park o
(A) It was designed by a well-known chef.
(B) It was created upon the suggestion of
visitors.
m / o
r\j
i i n
(C) It will be available until December 4.

c o
c.
(D) It will not be served in some park
CD
restaurants.

183. Why was the e-mail written?


n go O
cn

(A) To suggest ways to improve service

m s O
CD

ing
(B) To point out some safety concerns
CD
\J
(C) To make a complaint about a situation

ak
(D) To get details about a roller coaster O
CO

pe CD
OD

ic s CD
e
p : //to j2
l>
Hfi

ht t n£
rid
aa
Jdo
LU
fDOJTO
a.
OQ3'

GO ON TO THE NEXT PACE..

TEST 3 PART 7 105


Questions 186-190 refer to the following article, notice, and form

OolJ
Tea Tree Eco-Resorts Set to Open in Mexico by Year's End
By Consuela Hidalgo

International hotel chain Tea Tree Eco-Resorts from whom we source our ingredients for
is in the final stages of building a 22-acre resort regional specialties, such as pork wrapped in
on Mexico's Mayan Riviera. The first visitors banana leaves and citrus-marinated chicken
are expected to arrive by December 21. Like soup, use any chemical pesticides at all," said
other Tea Tree Eco-Resorts in the Philippines, general manager Jose Perez. The plan is for the
Thailand, and Greece, the new hotel will be as facilities to be built around naturally growing
eco-friendly as possible. "We have solar panels trees and plant life, with structures blending in
generating electricity and heating the water, for with the jungle. Further details can be found at
instance. Furthermore, none of the local farmers www.teatreeecoresorts.com/mexico.
m /
c o
o c.
Tea Tree Eco-Resorts, Mayan Riviera
n g
m s
Welcome to our beautiful compound in the stunning Mayan Riviera! ^ jr
We have several activities scheduled for this week!

i n g
Monday, January 17: Sailboat tour ($22)

a k
Join us for a 2-hour sailboat ride that stops at local islands. There's even a chance of spotting

e
sp
some dolphins! Sign up in advance at the guest activities counter.

ic
Wednesday, January 19: Visit to wildlife park ($38)

e
Travel by bus to a local wildlife park to view indigenous animals and plants. Departure will be at

//to
9 a.m., and guests will be back by 5 p.m. Sandwiches will be provided at a restaurant near the park.
Advance bookings are necessary.

tp :
Thursday, January 20: Tour of ancient Mayan city ($57)

ht A bus will take guests to an archaeological site where a pyramid and other ancient structures can
be found. Departure will be at 8 a.m., and guests will be back by 5 p.m. Lunch is included in the
cost of admission, and bookings are required.
Saturday, January 22: Beach party (Free for all guests)
Visit the beach between 11:30 a.m. and 2:30 p.m. for an outdoor barbecue, and sample the
wonderful flavors in our grilled seafood, citrus-marinated chicken soup, and pork wrapped in
banana leaves!
*Prices indicated are in US dollars.

106 Hackers.co.kr
Tea Tree Eco-Resorts, Mayan Riviera
We hope you enjoyed your stay with us. Please take a few moments to provide us with feedback,
Return the completed card to a front desk staff member.
NAME Henry O'Neil DATE(S) OF STAY January 17-23
E-MAIL Henoneil@awesomepost,com PHONE (905)555-6948

Were you satisfied with the following during your stay? If the answer is "NO", please leave a short
comment at the bottom with an explanation.
Cleanliness of facilities YES m NO □ Service of staff YES m NOD
Meals and meal service YES ■ NO □ Daily activities YES □ NO M

COMMENTS:

/
My wife and I were interested in a sailboat tour but were informed that the boat only held 24 passengers and

m
o
was full. So, we inquired about a trip to the Mayan city, but it was also completely booked. Thankfully, we

available for trips, especially during high tourist season.


c. c
managed to go on the January 19 outing and enjoyed it very much. I recommend that more seats be

Thank you for your time!


g o
s n
186. What is NOT indicated about Tea Tree
g m
189. What does Mr. O'Neil mention about the
Eco-Resorts?
(A) Its new facility uses solar power as a
k i n Mayan Riviera hotel?
(A) It cancelled several planned daily
source of energy.
e a activities.

sp
(B) It has built several resorts around the (B) It was fully booked during his recent

ic
globe. stay.

e
(C) It posted details about a new branch (C) It offered satisfactory levels of

//to
on a Web site. cleanliness.
(D) Its Mexican resort is the company's (D) It needed improvement in regard to

tp :largest yet. meal service.

ht
187. In the notice, the word "spotting" in
paragraph 2, line 2, is closest in meaning
to
190. What can be inferred about Mr. O'Neil?
(A) He had an unpleasant experience on a
boat trip.
(A) circling (B) He and his wife saw some wild
(B) pointing animals.
(C) observing (C) He was unable to extend a hotel
(D) marking booking.
(D) He enjoyed touring an ancient Mayan
188. During which activity were guests served a city.
regional specialty?
(A) Sailboat tour
(B) Visit to wildlife park
(C) Tour of ancient Mayan city
(D) Beach party
GO ON TO THE NEXT PACE.

TEST 3 PART? 107


Questions 191-195 refer to the following letter, e-mail, and form.

February 4
Dear Ms. Montessori,
As you are probably aware, my contract will expire at the end of March. We have already
discussed my plan to retire, so hopefully this does not come as a surprise. This letter serves as
official notification of my intention to step down as the company's vice president of finance.
I would be happy to stay on for an additional month to train and assist any replacement you
choose.
I have truly enjoyed my time with Homestead Properties and appreciate all the support I've been
provided over the years. I will miss working with my wonderful colleagues, but I will stay in touch.
Thank you very much for all the opportunities you and the board of directors have afforded me.

m /
Sincerely,

c o
c.
Edward Grainger

g o
s n
TO
g
Florence Ingram <f.ingram@homesteaclprop.com>m
FROM
i n
Elizabeth Montessori <e.montessori@homesteaclprop.

k
SUBJECT
DATE
Party preparations
March 12
e a
ic sp
Hi Florence,
e
p : //to
You've likely heard the news about Mr. Grainger. I would like to organize a dinner
party in his honor. You did such a wonderful job of planning our awards dinner last fall,

ht tand I was wondering if you'd be willing to take care of the arrangements for this event

We plan to spend up to $6,000 on the party's location. The board and I will take care
of the gift. The date would be March 27, which is a Friday. I can send you a list of
people we would like to invite later today. I imagine 50-60 people will be attending.
As before, the company is happy to offer you an extra $500 on your next paycheck as
compensation. Please let me know if you're interested as soon as possible.

Regards,

Elizabeth Montessori
COO, Homestead Properties Inc.

108 ^31 ^^1MP3 Hackerslngang.com


mmmii

NAME Florence Ingram COMPANY (If applicable) Homestead Properties Inc.


PHONE (604)555-4995 E-MAIL f.ingram @ homesteadprop.com
EVENT DATE March 27 EVENT TIME 7 P.m.-10:00 P.M.
EVENT SITE 2nd Floor, Victoriana Event Hall

NUMBER OF GUESTS: 66
MEAL: Four-course dinner with beef, fish, and vegetarian options
BEVERAGES: Open bar with soft drinks, wine, and cocktails
SPECIAL REQUESTS: Microphone and projection system with sound
This form serves as confirmation of your reservation, and receipt of your $2,500 deposit. The remaining
funds of $2,500 is payable upon the conclusion of your event. In case of cancellation, the deposit is
non-refundable. The total amount of $5,000 is inclusive of a $500 service fee.
m /
c o
o c.
g
191. What is the main purpose of the letter? 194. What can be inferred about Ms. Ingram?
(A) To confirm an intention to resign from
a position
s n
(A) She reserved two rooms for the event.
(B) She invited fewer guests than
(B) To turn down an offer for an executive
g m expected.
job
(C) To notify an executive of a contract
k i n (C) She needs to submit a deposit to the
venue soon.
extension
e a (D) She booked an event venue under

sp
(D) To request some time off for personal budget.
reasons

e ic 195. What has Ms. Ingram requested from the


Grand-Mont Hotel?

//to
192. According to the e-mail, what will
Ms. Ingram be offered? (A) An invoice for the total cost

tp :
(A) A printed invitation for a party (B) Some recommendations for menus

ht
(B) A monetary incentive (C) Some audiovisual equipment
(C) A complimentary meal (D) A choice of two meal options
(D) A copy of an agreement

193. What did Ms. Montessori ask Ms. Ingram


to do?
(A) Arrange a corporate awards ceremony
(B) Plan a retirement celebration
(C) Purchase a gift for an employee
(D) Recruit volunteers to decorate a venue

GO ON TO THE NEXT PACE.

TEST3 PART? 109


Questions 196-200 refer to the following advertisement, text message, and notice.
4!
OcM

http://www.portkeyclassifieds.com/realestate/rentals

HOME CLASSIFIED ADS POST AN AD CONTACT

DOWNTOWN APARTMENT FOR RENT: 543 Mapleton Street

This 950-square-foot apartment is located in the city center in Portsmouth Residential


Towers. It contains two bedrooms, a living room, a kitchen and dining area, a bathroom,
and a balcony. The residence is conveniently located near a bus stop, a subway station,
and a shopping mall. The unit comes with a space in the building's parking garage, the
cost of which is included in the $1,900 monthly rental fee. New wallpaper will be installed
for the new resident. The apartment will be ready to move into as soon as this work is
m /
c o
c.
finished on August 30. Contracts are for one or two years with the possibility of an

o
extension. A one-month security deposit is required along with a reference from a previous

g
landlord. To book a viewing, please contact George Lee at georgelee@vendorrealty.com,

n
or call him at 555-0393.

m s
i n g IE90

a k
e
sp
From: Neil Kaplan (555-3004)
To: George Lee (555-0393)

e ic Received: May 25, 4:10 p.m.

//to Thank you for showing me the three

tp:
apartment units this morning. I'm interested in

ht
renting the one on Mapleton Street. I
understand that the owner needs a
reference prior to signing any agreement. He
is free to contact Miranda Chase at
mirandach@verymail.com. She's the
proprietor of the studio apartment I currently
rent in Tanner Hill Condominiums. I am
prepared to sign a two-year contract and
move into the unit as soon as the new
wallpaper has been put up. Thank you.

110 Hackers.co.kr
NOTICE: All Tenants

Tanner Hill Condominiums' underground parking facility will undergo resurfacing this week. The work
will begin on Monday, May 30, and last until Friday, The facility will be inaccessible not only during this
time but also on the weekend as the concrete requires time to set properly. Tenants may once again
park their vehicles in the garage on Monday, June 6.
Tenants should move their vehicles one day before the work begins, to the Downtown Parkade located
across the road at 168 Beauregard Avenue. Simply pick up a temporary pass from the facility's ticket
office, and display it on your vehicle's windshield.
We apologize for any inconvenience and disturbance the work may cause. For more details, contact the
building's administrative office at 555-4958 during its operating hours of 8 a.m. to 4 p.m., from Monday
to Friday.

m /
c o
196. What is included in the advertised
o c.
199. What will Mr, Kaplan probably have to do
apartment's rental fee? on May 29?
n g
(A)
(B)
A security system
A parking spot
m s
(A) Park his vehicle at the DownTown
Parkade
(C) Maintenance costs

i n g
(B) Move his belongings into an apartment
(D) Electricity charges

a k (C) Meet with a landlord to sign an


agreement
197. What is implied about Mr. Kaplan?
e (D) Purchase a pass at a ticketing office

ic sp
(A) FHe should pay the rent by bank transfer.
(B) He must renew a contract after one 200. What is indicated about Tanner Hill
year.
e Condominiums?

//to
(C) He will not be refunded a deposit, (A) It currently has no vacant units.

:
(D) He can move into a unit from August 30.

tp
(B) It charges monthly fees for parking.

ht
(C) It is situated nearby public transit.
198. Who is Miranda Chase? (D) It closes its office on weekends.
(A) The owner of a residential unit
(B) A representative of a real estate
agency
(C) The administrator of Portsmouth
Residential Towers
(D) A tenant of a Mapleton Street
apartment

This is the end of the test. You may review Part 5, 6, and 7 if you finish the test early.

S9p.324/g^Sftffip.327/Sl|-!t p.344 / Part 5&6 UfSfiZI


»c® ii]|o|x|oi| Self ilia aiiSl rn S*i| i0| ehei MAfli

TEST 3 PART 7 111


oH3^ 1000HI 3 READING

m /
c o
oc.
n g
m s
i n g
a k
I TEST
e 04

ic sp
e
//to
PARTS

tp:
01 PART 6

ht 0) PART 7

Self Ip apM

S'ZI! Ml—— 5! SIWH


1.^HBiggBsma? doii
2. Answer Sheet, 21, xi^7|f gd |ohSL|-o ? D oj|
3.A|7||tgd|SHaL|-s? □ 01[
OS gdpp 2t=£|Sl°2 *4# ttlil * MIAE# AlsfSilti

m mmafe70g- °°J _A| _^?JL|CK


MIAE Aiite#75mo|DI, m m as % tiew am# ?ISII A^lick
READING TEST
In this section, you must demonstrate your ability to read and comprehend English. You will be given a
variety of texts and asked to answer questions about these texts. This section is divided into three parts
and will take 75 minutes to complete.
Do not mark the answers in your test book. Use the answer sheet that is separately provided.

PARTS
Directions: In each question, you will be asked to review a statement that is missing a word or phrase.
Four answer choices will be provided for each statement. Select the best answer and mark the
corresponding letter (A), (B), (C), or (D) on the answer sheet.

© parts iFb'l-olAiy 11§

101. Meals at Moonlight Grill consist of various 105.


m /
all the entrepreneurs in Florida are
dishes for guests of all ages.
c o
in favor of the government's plan to cut
(A) enjoys

o c.
taxes, though a small minority is opposed.
(B)
(C)
enjoy
enjoyable
(A)
(B)
Solely
Certainly
n g
(D) enjoying

m s
(C)
(D)
Unusually
Nearly
102. Mayberry Research Institute offers staff

i n g
which include dental coverage and
a pension plan.
a k 106. AEG Co. is looking for sales associates
who demonstrate excellent communication

e skills and an ability to work in a

sp
(A) benefitted
(B) beneficially manner.
(C) benefits

e ic (A) cooperative

//to
(D) beneficial (B) cooperation
(C) cooperatively

tp :
103. Arranging an appointment to see (D) cooperate

ht
Dr. Menard one month in advance is
due to her busy schedule. 107. The unnecessary extra work on Sackler
(A) grateful Department Store's new building could
(B) conclusive have been if the design plans had
(C) precise been followed precisely.
(D) necessary (A) avoided
(B) proposed
104. Patrons who had their vehicles parked (C) cautioned
waited by the Canton Theater entrance (D) cultivated
while retrieved their cars.
(A) attending 108. Ms. Herrera could not sit in an aisle seat
(B) attends during her international flight because
(C) attendants there were available.
(D) attendance (A) hardly
(B) any
(C) mainly
(D) none
114 Hackers.co.kr
109. Mr. Holt's acceptance of the financial 115. Passengers to fill in their arrival
executive position is his satisfaction forms during the flight for quicker
with the salary offer. processing at the immigration checkpoint.
(A) in contrast to (A) remind
(B)dependent on (B) have reminded
(C) as opposed to (C) reminding
(D) on condition that (D) are reminded

110. Transport officials are still unsure of 116. Overdrive is a low-budget action film that
caused Train 580 to break down earlier has no to actor Sam Horton's
this morning. autobiography, which has the same title.
(A) each (A) relates
(B)those (B) related
(C) what (C) relate
(D) why (D) relation

m /
111. The Project Green report argues that 117. Without exception,

c o
employees must

c.
the best way to save endangered species undergo on-the-job training to enhance
is by placing them in protected wildlife
areas.
their skills.
(A) every
g o
(A) persuasive (B)
s all
n
(B)
(C)
persuade
persuasion
g m
(C)
(D)
a lot
few
(D) persuasively

k i n
118. The client was never in his office

e a
112. The guide told visitors touring the High many times Ms. Baxter tried to call him.

sp
Point Library that the facility by the (A) otherwise

ic
renowned architect Albert Grand. (B) however
(A) designs
e (C) hence

//to
(B) was designed (D) somehow

tp :
(C)
(D)
designing
will be designed 119. LocerTech's primary goal is to be as

ht
as possible to customer inquiries.
113. Sundersen Technologies 10 percent (A) shortened
from Mr. Garrison's bill since he was (B) accelerated
eligible for a bulk purchase discount. (C) responsive
(A) deducted (D) intentional
(B) substituted
(C) deposited 120. Rodgers Industrial celebrated the of
(D) submitted its 50th anniversary by hosting a banquet
for clients from around the world.
114. the recent economic upturn, (A) impression
unemployment is on the decline as more (B) integration
and more businesses are hiring full-time (C) occasion
employees. (D) gratitude
(A) In order that
(B) As a result of
GO ON TO THE NEXT PAGE..
(C) Despite
(D) Provided that
TEST A PARTS 115
121. Gilbot Grounds had to work to get 126. Although Southmoore Records has been
the landscaping project finished by the actively seeking a new CEO for three
deadline. months, they have to find a suitable
(A) intense candidate.
(B) intensely (A) yet
(C) intensive (B) never
(D) intensity (C) seldom
(D) seemed
122. Under the finance minister's leadership,
corporations recovered quickly from last 127. Many office workers frequent Stan's Bistro
year's losses and then began making because of its to the business district.
substantial
(A) proximity
(A) regulations (B) closure
(B) earnings (C) simplicity
(C)
(D)
entrants
estimations
(D) scheme

m /
o
128. The temperature this morning is cool and
c
c.
123. The provision of additional government pleasant, but it is expected to increase
grants for college tuition will make
education to more people from
steadily
o
the day.

g
around the country.
(A)

s
(B)
between
above n
(A)
(B)
exclusive
plentiful
g m
(C)
(D)
next to
throughout
(C) confidential

k i n
a
(D) accessible 129. Now that the city has hired additional

e workers, it can with the construction

sp
124. The current edition of the employee of the new civic center.

ic
handbook detailed guidelines for
(A) enhance

e
employee performance reviews. (B) concern

//to
(A) contain (C) replace

tp :(B)
(C)
was contained
contains
(D) proceed

ht (D) containing 130. -- — to be the best in the city, the Melise


Restaurant is booked for the next six
125. Shoppers looking for high-quality months.
electronics at low prices will find a (A) Reporting
selection at the new NorvelTech store,
(B) Report
(A) greatest (C) Reported
(B) greatly (D) Reportable
(C) greaten
(D) greater

116 ?SS2te!{JM9|H°|3Ah9O)°t7®5J30(a?IMP3 Hackerslngang.com


PART 6
Directions: In this part, you will be asked to read four English texts. Each text is missing a word,
phrase, or sentence. Select the answer choice that correctly completes the text and mark the
corresponding letter (A), (B), (C), or (D) on the answer sheet.

© PARTS ^RftolARt

Questions 131-134 refer to the following memo.

To: All staff members


From: Greg Birch, CTO

/
Subject: Server
Date: March 2

o m
As you may know, our company server
131.
Although the disruption was only temporary

c. c
o
and the server is now running, I am considering switching to a different system. ------- the

g
s n
server goes down again, it could create a serious problem. Because we want to prevent that

m
from happening, I've spoken with Jason, the head of our IT team. He can install a new server

g
that will be less
133.
to malfunctions.
134. i n
Thankfully, we will be able to transfer files from

k
e a
the old server to the new one without any difficulty.

ic sp
I will continue to send you updates as I receive more information from Jason.

e
//to
tp:
131. (A) disconnecting 134. (A) The breakdown is expected to be

ht
(B) was disconnected ||||| repaired over the next few days.
(C) has been disconnected (B) In addition, it will have a greater
(D) has disconnected memory capacity and better security.
(C) However, this solution is impractical as
132. (A) Before it is much too costly.
(B) But (D) Several companies in the affected area
(C) If reported similar outages.
(D) So

133. (A) vulnerable


(B) flexible
(C) dangerous
(D) faulty

GO ON TO THE NEXT PAGE.

TEST 4 PART 6 117


Questions 135-138 refer to the following letter.

■ osil
lLTr<?J
-y- Nicole Freemont
1452 Reservoir Road NW
Washington, D.C. 20057

Dear Ms. Freemont,

I am pleased to confirm your membership to the Georgetown Food Cooperative. , we


135.
have enclosed your membership card. By supporting us, you are helping to make food more

affordable for the community. This is because we get our products from local businesses with

lower shipping and manufacturing expenses.

m /
Flowever, we can only
136.
o
with the active assistance of all our members. That is why we

c
ask each member to provide four hours of voluntary work at our store every month.

o c.
g
Moreover, we encourage you to stop by and familiarize yourself with issues pertaining to the

n
s
costs of food and to learn about other ways you can help.

m
Best wishes,
i n g
a k
Edwin Krueger
e
sp
Membership coordinator, Georgetown Food Cooperative

e ic
//to
135. (A) Instead 137. (A) We are forced to increase membership

tp :
(B)
(C)
Accordingly
Ever since
|i|j| fees for this reason.
(B) We are delighted that you have

ht
(D) Unfortunately contributed so much of your time.
(C) The cooperative is still accepting bids
from local food suppliers.
(D) You can pay a monthly surcharge of
$20 if you are unable to do so.

138. (A) growingly


(B) grown
(C) growing
(D) grows

118 ?SS9)ifS-?|gsaJlB Hackers.co.kr


Questions 139-142 refer to the following press release.

New Territory for Aarhus Clothing

The British public is excited about the opening of Aarhus Clothing's first UK-based store.

The Danish company has gained a reputation for achieving consistently high rates of

customer It quickly became a well-known brand in its home country. ------


139. 140-

Aarhus Clothing launched its first German store two years ago, after which the company

quickly moved into Belgium and the Netherlands. With the inauguration of a flagship store on

London's Oxford Street next month, the company hopes to stand out in the ------ UK market.
I.

m /
Designer Mads Jensen told reporters, "Our products have been sought after in Europe
c o
for many years now because they are simple and
c.
As they are basic and long-lasting,
o
we expect them to be just as popular in the UK."
n g
m s
139. (A) satisfying
i n g
141. (A) multiple
(B)
(C)
satisfactorily
to satisfy
a k (B)
(C)
lucrative
duplicate

e
sp
(D) satisfaction (D) diligent

|||||
e ic
140. (A) Its popularity subsequently spread to
neighboring nations.
142. (A)
(B)
disposable
attentive

//to
(B) Therefore, the company has only one (C) collective

tp : store left in the country. (D) durable

ht
(C) Nevertheless, the peak of its fame was
reached in Britain last year.
(D) This had a negative effect on the brand
in the years that followed.

GO ON TO THE NEXT PAGE.

TEST 4 PART 6 119


Questions 143-146 refer to the following advertisement.

Turn Your Scuba Diving Dreams into a Reality!

The Estrella Diving Center's summer certification courses will begin at our Playa Del Carmen

facility on June 2. No prior is necessary because our trainers will cover all the basic
143.
techniques for beginner students. As always, enrollees must have no existing medical

conditions and be at least 13 years old. Those who cannot satisfy ----- of these requirements

will be unable to sign up.

All equipment will be provided, and training will be delivered in three sessions. Held on
146.
the same day, each lesson will last an hour with short breaks in between. Enrollment opens
m /
c o
c.
on Monday, May 3.

g o
143. (A)
(B)
negotiation
compensation s
145. (A) either
n
m
(B) whatever
(C)
(D)
instruction
registration
i ng (C)
(D)
extra
couple

144. (A) Certain criteria must be met before


a k 146. (A) coincidental
e
sp
||||| taking the course. (B) inadequate
(B) Divers have stated that they agree with (C) remote
the new rule,

e ic (D) consecutive

//to
(C) Such challenges are not easily
overcome.

tp :
(D) Diving instructor applicants will be

ht
contacted shortly.

120 ¥£Sat?!i!E^a°|^he0ig}?IS?!E!0ig!?|MP3 Hackerslngang.com


PART 7
Directions: In this part, you will be asked to read several texts, such as advertisements, articles, instant
messages, or examples of business correspondence. Each text is followed by several questions. Select
the best answer and mark the corresponding letter (A), (B), (C), or (D) on your answer sheet.

© part? ^ 10IMV 54£

Questions 147-148 refer to the following invitation.

The Committee for

/
World Education Development
Would like to cordially invite you to

o m
The 20th Annual Portland Study Abroad Fair

c. c
January 10 through 14, from 10 a.m. to 4 p.m.
Barring Exhibition Hall, 475 Wharton Drive
g o
Portland, Oregon
s n
g m
The Committee for World Education Development is proud
to host this event in downtown Portland. The fair will involve

i n
more than 2,000 representatives from universities, colleges,

k
e a
language schools, and other academic institutions from
around the globe.

ic sp
Join us on any day of the fair by showing this card at the
registration desk near the front entrance of the exhibition

e hall. The invitation entitles the recipient to bring one

//to
additional guest.

tp :
ht
147. What type of event is the invitation for?
(A) A city's tourism promotion fair
148. How can the invitation's recipient attend
the event?
(B) A conference on international (A) By presenting the card near a hall
languages entrance
(C) An organization's student awards (B) By making a donation to a scholastic
ceremony institution
(D) An exposition for overseas education (C) By providing the committee with
confirmation in advance
(D) By showing an identity card at the
registration desk

GO ON TO THE NEXT PAGE

TESTA PART 7 121


Questions 149-150 refer to the following notice.

Space Science Museum


NOTICE TO VISITORS

Please note that the main building of the Space Science Museum
at 290 Chamomile Street will be closed from June 4 to June 8 for
scheduled restoration work.

The work will improve the foundation and interior design of a


number of exhibition halls to better enhance museum experiences
for our guests. The museum's planetarium will also be closed, as
staff will be installing new equipment and upgrading seats.

However, the museum's auxiliary buildings will remain open during


m /
c o
c.
this period. Visitors with passes to the special exhibition, The
Moons of Saturn, may still access it at Orbit Hall. Likewise, those

g o
wishing to visit our space transport display may do so at Satellite
Hall.

s n
We apologize for the inconvenience.
g m
All museum facilities will officially reopen on Thursday, June 9.

k i n
e a
ic
149. What is the notice mainly about?
sp 150. What is stated about the museum?
(A)
e
Major exhibitions for the month (A) Its interior design work will last for a

//to
(B) Temporary facility closures month.

tp :
(C)
(D)
Scientific research programs
Museum foundation events
(B) Its entrance locations will be
temporarily changed.

ht
(C) Some of its venues will be open to
the public during construction.
(D) It postponed The Moons of Saturn
exhibition.

122 Hackers.co.kr
Questions 151-152 refer to the following e-mail.

To: Joseph Hillier <j.hillier@romanos.com>


From: Marilyn Sharpe <m.sharpe@romanos.com>
Subject: Burton Farms
Date: October 21
Attachment: Product list

Joseph,

As you know, we are in the process of negotiating an agreement with Burton Farms to sell some of
their dairy products at all our supermarket locations in the Midwest. Depending on how well the
products sell, we may distribute them later on to our other stores across the country.

In connection with this, I'd like your team to complete a presentation in time for a meeting we are

m /
o
having with Burton Farms on the llth of next month. You will need to do some research on their

c
c.
brand positioning and target market. Next, you'll have to come up with ideas for arranging the
products on our shelves and promoting them to our customers. I've attached the Burton Farms

g o
product list and highlighted the items we plan to sell. I'll be in the office all week if you want to
discuss this further.

s n
Thanks!

g m
Marilyn Sharpe
Vice president of marketing
k i n
Romano s Groceries
e a
ic sp
e
//to
151. According to Ms. Sharpe, what does 152. What has Mr. Hillier's team NOT been
Romano's Groceries plan to do? asked to do?

tp:
(A) Take over the operation of Burton (A) Finish a presentation for a meeting

ht
Farms (B) Perform some market studies
(B) Open further distribution centers in the (C) List dairy items to be sold
Midwest (D) Develop concepts for product displays
(C) Hold training sessions for its staff
members
(D) Enter into a supply arrangement with a
dairy

GO ON TO THE NEXT PAGE

TEST4 PART? 123


Questions 153-155 refer to the following announcement.

The Millsboro Water Department is calling on all area students to participate in its
third annual art contest. The theme of this year's competition is Water For All.
The contest is open to Millsboro students in grades 4 through 12. The rules call for
original artwork to be submitted on 75-centimeter by 110-centimeter poster board
or paper, with a 3-centimeter border on all sides. Entrants may use any traditional
medium they wish, such as pastel crayons, colored pencils, watercolor paints, cut ; j
paper, or fabric. However, computer-generated artwork and photography will not
be accepted. Students may receive guidance from a parent, teacher, or guardian
but must do all the work by themselves.
The grand prize winner of each grade level will receive $300 cash and a $200 gift
card from fellow sponsor Millsboro Shopping Center. Furthermore, three additional

m /
o
contestants will be awarded with $100 gift cards, also furnished by the shopping
center. Along with other entries, the winning pieces will be featured in next year's
student artwork calendar, which will be distributed free to clients of the Millsboro
c. c
Water Department at the beginning of next year.

g o
The deadline for entries is April 30. For more information, visit www.mwd.com/art_contest.

s n
•. . .r ,• ...•y-.-r-- - V, •; . . . . .. . . . . . , . . . . . . . - ;,.,v . • - .

g m . -"v.- -- OTWWffiS, -H'. OB "' WA

k i n
153. What is the purpose of the announcement?

e a 155. What will the Millsboro Water Department


do the next year?

sp
(A) To publicize a local event
(B) To introduce a public service (A) Sponsor student art exhibits at area
(C) To
ic
promote water conservation

e
schools
(B) Sell some artworks to raise funds for

//to
(D) To inform residents about a change
charity

tp :
154. What is NOT indicated about the Millsboro
Water Department?
(C) Expand its services to other locations

ht
(D) Hand out calendars with student art to
(A) It requires entries to have specific its clients
measurements.
(B) It will not accept submissions produced
on a computer.
(C) It released a series of posters on water
usage last year,
(D) It is cosponsoring an event with
Millsboro Shopping Center,

n
124 A yoiiPl MP3 Hackerslngang.com
Questions 156-157 refers to following text message chain.
A|
mm

Dawn Konrad 4:48 p.m.


Yoon-Hee, are you busy tomorrow? Ms. Jackson sent a
message saying that we need to rush the Lambert
project. I'll be working overtime on it tomorrow night
and could use your help.

Yoon-Hee Sung 4:51p.m.


I'm heading to the marketing conference tomorrow,
remember? I'll be out of town for two days.

Dawn Konrad 4:52 p.m.


Oh, that's right. It completely slipped my mind. Should I
m /
ask Mr. Martin?
c o
Yoon-Hee Sung 4:54 p.m.
o c.
I'd try Mr. Stewart. He's worked on the project longer
and knows Percy Lambert really well.
n g
m s
Dawn Konrad

i n g
I'll do that. Thanks, and have a good trip!
5:00 p.m.

a k
e
ic sp
e
156. What is true about Ms. Konrad? 157. At 4:54 p.m., what does Ms. Sung most

//to
(A) She will attend a marketing conference. fjjfe likely mean when she writes, "I'd try

:
(B) She will be assigned to a new project.

tp
Mr. Stewart"?

ht
(C) She will be working extra hours (A) She believes he can suggest a
tomorrow. replacement.
(D) She will ask Ms. Jackson for help. (B) She has already tried to contact
Mr. Martin.
(C) She thinks he is more qualified than
Mr. Martin.
(D) She is not well acquainted with Percy
Lambert.

GO ON TO THE NEXT PAGE

TEST4 PART? 125


Questions 158-160 refer to the following advertisement.

Send Yourself To The Next Level

Whether you're a professional athlete or just an enthusiastic


amateur, having the right shoe is crucial for improving your
performance. The Lynx sneaker is the result of 25 years of
research and experimentation into the perfect athletic shoe.
The Jet Shoe Company has worked with some of the finest
sports scientists and conducted tests with the world's top
athletes to develop the Lynx. And the effort shows in
everything you do while wearing them.

Our patented Absorbex technology cushions your joints from


m /
impacts as you run while providing a firm surface to spring
c o
off from. And the collar is specially designed to provide extra

o c.
g
stability to your ankle, reducing the risk of sprains. The Lynx

s n
is used by top athletes like sprinter Dwayne Harrison and
basketball player Andrew Singer. Top coach Dennis Potts of

m
the Baltimore Crabs football team recommends all his
g
i n
players wear shoes that use the technology when in the
gym.
k
e a
The Lynx is available at most general sports retailers. To find

sp
your nearest outlet, check our Web site at www.jetshoe.com.

ic
e
//to
tp:
158. The word "performance" in paragraph 1, 160. Where can customers purchase the
advertised product?

ht
line 3, is closest in meaning to
(A) production (A) In some gym facilities
(B) ability (B) At an upcoming event
(C) exhibition (C) On the manufacturer's Web site
(D) fulfillment (D) At athletic goods shop

159. What does the advertisement mention


about the Lynx sneaker?
(A) It protects the joints while in use,
(B) It is the cheapest training shoe
available,
(C) It supplies additional warmth in winter.
(D) It is sold in a wide variety of colors.

126 Hackers.co.kr
Questions 161-163 refer to the following letter.

WIMSPEAR HOTEL
3498 Ada Boulevard, Edmonton AB T6A 2C1
November 14
Paul Mansbridge
2405 Cottonwood Lane
Windsor, ON N9B 3P4
Dear Mr. Mansbridge,
Thank you for the letter dated November 12 regarding your stay at the Winspear Hotel in Edmonton.
— [1] —.I understand that you and a colleague were our guests from November 8 to 10 but had some
issues during your visit.
The manager on duty, Melanie Depuis, has notified me that one of the rooms you had reserved was
double-booked. — [2] —. Unfortunately, we could not correct our mistake after your arrival since the
hotel was at full occupancy. We are very sorry, and enclosed are two vouchers for a complimentary
m /
o
one-night stay at any Winspear Hotel nationwide—one for you and one for your associate, David Solomon.
c
o c.
You also said you thought our buffet breakfasts were complimentary. In regard to this, morning meals are
only free for those staying in our premium accommodations. — [3] —. As you and Mr. Solomon stayed in

- [4]
n g
a standard room, we had to charge you for your breakfasts. I apologize for any misunderstanding.

m s
We value the continued patronage of our guests, and we hope the vouchers will make up for any
inconvenience.
Sincerely yours,
i n g
'KCtH&esUey 'Sell
a k
Kimberley Bell
e
sp
General manager, Winspear Hotel

ic
Edmonton

e
p : //to
ht t
161. Why did Mr. Mansbridge most likely send
a letter on November 12?
163. In which of the positions marked [1], [2],
iBfe [3], and [4] does the following sentence
best belong?
(A) To confirm booking details
(B) To inquire about an extra room "As a result, you were forced to share a
(C) To ask about a travel program room with your traveling companion."
(D) To make a complaint about a stay
(A) [1]
162. What is true about Mr. Mansbridge? (B) [2]
(C) [3]
(A) He was charged extra for requesting
(D) [4]
an additional bed.
(B) His membership points were not
credited correctly.
(C) He will receive a complete refund for
his stay.
(D) He was given vouchers for a hotel
stay. GO OA/ TO THE NEXT PAGE.

TEST A PART? 127


Questions 164-167 refer to the following letter.

Pure Mix
PO Box 12525
Oregon City, OR 97045
Paul McCall
898 Manatee Lane
Madison, WI 53716
Dear Mr. McCall,
We have received your inquiry about our natural daily vitamin supplement. You asked if our product
contained any chemicals or nonorganic materials. Also, you inquired about some of the ingredients
used to make the vitamin powder, as you are allergic to strawberries and cannot ingest grapefruit for
medical reasons.
As advertised, we are a provider of certified organic products, so there are no chemical or unnatural

m /
materials in our products at all. Our organic ingredients are of the highest quality available, made only
from fruits, plants, and naturally raised animals.

c o
c.
As stated, our products do include fruits. However, the daily vitamin supplement does not contain

g o
strawberries or grapefruit as ingredients. You can ingest it safely without any worry about an allergic
reaction to strawberries or medication interaction with grapefruit. But some of our other products may

n
contain one or the other. For example, Pure Mix energy drinks contain grapefruit seed.
s
g m
I hope I have provided satisfactoiy responses to your questions. Enclosed, you'll find an informative
brochure on our most popular products. As always, should you plan to start a regular vitamin regimen,

i
we highly recommend that you first consult your physician.

k n
Sincerely,

e a
sp
Linda Mills
Customer relations manager

e ic
//to
164. Why was the letter written? 166. Why should Mr. McCall avoid drinking

tp :
(A) To place a vitamin order Pure Mix's energy drinks?

ht
(B) To obtain organic certification (A) They will cause him to have an allergic
(C) To inquire about an advertisement reaction.
(D) To respond to a prior inquiry (B) They have an ingredient he cannot
consume.
165. What does Ms. Mills say about Pure Mix's (C) They are prohibited for diabetics.
products? (D) They will give him digestion issues.
(A) They all have strawberries as
167. What does Ms. Mills suggest Mr. McCall
ingredients.
(B) They do not contain any animal do?
materials. (A) Research the benefits of natural
(C) They are all completely organic. medications
(D) They have yet to be released on the (B) Speak to a doctor before taking dietary
market. supplements
(C) Order vitamins from a manufacturer's
Web site
(D) Keep the product's package after
opening it
128 ¥^§^21 ^ twopi mpb Hackerslngang.com
Questions 168-171 refer to the following article.

A Very Fruity Idea

Our cities are filled with trees, and the vast majority
of those trees produce edible fruits. — [1] —. But
with no one to collect and process them, most 11 ^
simply fall to the ground and rot, wasting a
potentially excellent source of nutrition.

One new organization wants to change that. — [2] —. Fresh From The
Tree is a non-profit group that aims to collect as much fruit as possible
from the city's trees, and then make it into delicious snacks that can be
sold at low prices to local residents.

"We have been doing a survey of the locations of all the fruit trees in
town," says co-founder Michaela Anderson. "Once we have identified
m /
a promising tree, we contact the homeowners and ask if we can pick
c o
the fruit for them. — [3] —. They usually don't have time to manage
it themselves."
o c.
n g
m s
After the volunteers collect the fruit, they give half of it to the owners
of the residence, and keep the other half to turn into fruit pies, jams,

i n g
sorbets, and other tasty but healthy desserts. The group only has a
handful of volunteers at the moment but are hoping to recruit more in

a k
the coming months. — [4] —. They are also negotiating with the city

e
to collect fruit from trees on government property.

ic sp
e
168. What is the article mainly about? 170. According to the article, what do

//to
(A) The establishment of a firm homeowners receive from Fresh From

p
(B)

t : The opening of a fruit orchard The Tree?

ht
(C) The management of public land (A) A small payment for the food
(D) The work of an organization (B) Some of the collected produce
(C) Recognition at a public event
169. What does Fresh From The Tree do with (D) Free baked goods
the fruit it collects?
(A) Sells it to local restaurants 171. In which of the positions marked [1], [2],
(B) Turns it into snacks llll [3], and [4] does the following sentence
(C) Donates it to a charity best belong?
(D) Transports it to City Hall "Most of the residents accept the offer with
pleasure."

(A)[1]
(B) [2]
(C) [3]
(D) [4]
GO ON TO THE NEXT PAGE.

TEST 4 PART? 129


Questions 172-175 refer to the following online chat.

(|1| Cheryl Stone [3:48] I just got a message from Mr. Baker saying that his
flight has been delayed until 9 p.m. Weather conditions
are terrible there in Chicago.
Allan Franklin [3:50] Really? But he's supposed to make a presentation to
the executives from Harper Telecom today!
Maurice Wood [3:51] I've got all the visuals ready. But it would be
problematic if we don't have a speaker.
Allan Franklin [3:52] That's an understatement. We need this contract.
Otherwise, we may not meet our sales target for the

Cheryl Stone [3:54] Can't we do the presentation ourselves?

m /
Allan Franklin [3:55] I'm not sure. There are several firms competing for
c o
c.
the contract we are trying to get. Harper Telecom

g o
could decide to give the contract to someone else if
they aren't impressed with our presentation.
Cheryl Stone
s n
[3:57] I think I could do the speaking, but without Mr. Baker's

g m
technical background and expertise, our proposal
might not sound very convincing. What do you think?
Allan Franklin
k i n
[3:58] I agree with you. I think it would be better to call

e aHarper Telecom and try to convince them to listen to

sp
his pitch tomorrow morning. That might be our best
option.

e ic
(|^| Cheryl Stone [3:59] I'm on it. I'll let you know how it goes.

p : //to
ht t

130 Hackers.co.kr
172. What is suggested about the presentation? 174. What is mentioned about Mr. Baker?
(A) It targets a company based in Chicago. (A) He is preparing to go to Chicago on
(B) It is scheduled for tomorrow morning. business,
(C) It is going to be uploaded to a Web (B) He is proficient in technical matters.
site. (C) He previously worked at Harper
(D) It is aimed at securing a contract. Telecom.
(D) He produced all of the presentation
173. What can be inferred about Harper materials.
Telecom?
(A) They are unsatisfied with a 175. At 3:59, what does Ms. Stone mean when
presentation. |i|i| she writes, "I'm on it"?
(B) They are considering offers from (A) She will call Harper Telecom.
several businesses.
(C) They want Mr, Baker to participate in
(B) She will give a presentation tomorrow.
/
(C) She will wait for Mr. Baker's flight to
m
the presentation. arrive.

c o
c.
(D) They will not be available at a different (D) She will contact her absent colleague.
time.

g o
s n
g m
k i n
e a
ic sp
e
p : //to
ht t

GO ON TO THE NEXT PAGE

TEST A PART 7 131


Questions 176-180 refer to the following notice and letter.

..... . ... . .

CITY OF GRIGGSTON PUBLIC MEETING NOTICE

A city hall meeting has been scheduled to discuss proposed plans for two road improvements,
specifically the extension of Marigold Street from Route 103 and the road hazard repairs on Hartford
Street. The first project on Marigold Street will be conducted over two weeks while the latter is
expected to take five days. The Griggston Office of Transportation Planning has provided funding for
both projects.
The work is expected to cause some disruption to traffic and some disturbances to homes and
businesses in the area. The city has already put together a traffic rerouting plan for the duration of
both projects and will allow the usage of noisy machinery during daytime hours.
Please note the meeting details below.
Date: Tuesday, January 10
m /
Time: 6:30 p.m.
c o
Location: Griggston City Hall, Room 5A in basement level one
This room is fully-accessible for persons with disabilities.
o c.
n g
Representatives from the Office of Transportation Planning, along with project contractors and

m s
consultants, will present an overview of the roadwork while providing the opportunity for questions
from the audience. Should you have any inquiries about this meeting or comments regarding either

ng
project, they may be directed to our senior city planner Lois Oliveira at l.oliveira@griggstoncity.gov.

i
a k
e
Griggston City Hail
ic sp
e
100 West Lorry Road

//to
Griggston,TX 76645

tp :
Office ofTransportation Planning

ht
February I 5
Shawn Davis
Walnut Creek Contracting
58 Lilac Terrace
Fort Worth(TX 76120
Dear Mr. Davis,
I am sending this letter as an expression of my gratitude for the excellent work your contracting company
did on the Marigold Street construction work. You and your 10 crew members were a pleasure to work
with and were always timely, professional, and knowledgeable.
Already, there have been a lot of positive reactions about the work you've done from residents and business
owners in the vicinity.There have been many positive remarks about the beautiful brick sidewalks that were
installed.
This was now the fifth time for us to hire your company and I am delighted that your business and the

132 ^ a°|iiAf • tKWIS 0A eWi mpb Hackerslngang.com


City of Griggston have continued our ongoing partnership. I have wired the payment for your contracted
services to your company's bank account as per our regular agreement.
Also, I would like to let you know that my office is working on a similar project for Rosedale Avenue.
Nothing is confirmed as of yet, but I should have more details in a few weeks. I would also like to consider
your company for this task. It would take approximately the same amount of time as the work on Marigold
Street did and would possibly begin at the end of April, Let me know if you might be interested.
Thank you again, and we look forward to working with you in the future.
Sincerely,
Lois Oliveira
Senior city planner

m /
o
176. What is indicated about the City of 179. What can be inferred about Mr. Davis?
Griggston?
(A) It has city hall meetings once every
c. c
(A) He asked for extra money to cover
unexpected costs.
month.
g o
(B) He suggested using bricks for the city's
(B) It had some issues with its road
conditions.
s n
pedestrian walkways.
(C) He organized the transportation
(C) It has hired contractors for building
g moffice's meeting.
renovations,
(D) It will increase property taxes to pay
k i n (D) He supervised a group of 10 workers
on a project.
for upcoming projects.

e a 180. What is indicated about Walnut Creek

projects?
ic sp
177. What does the notice state about the city's Contracting?
(A) It only accepts service payments by

e
//to
(A) They will not create much noise, check.
(B) They will end three weeks after a city (B) It presented building plans during a

tp :council meeting. recent public meeting.

ht
(C) They will cause traffic to be detoured. (C) It submitted a contracting bid for the
(D) They will be voted on by the public. Hartford Street project.
(D) It may do some roadwork on Rosedale
178. According to the notice, how can residents Avenue for two weeks.
provide feedback on the roadwork
projects?
(A) By calling Lois Oliveira's personal
assistant
(B) By sending an e-mail to a city official
(C) By visiting project contractors
(D) By posting on the city's online forum

GO ON TO THE NEXT PAGE.

TESTA PART 7 133


Questions 181-185 refer to the following brochure and e-mail.

Are the countertops In your home or workplace worn or outdated? Let Carlson Counters give your
kitchen, office, bathroom, or reception area a brand-new look!

We offer a vast assortment of styles, colors, and materials to choose from! And our prices are some of
the best in town:
Laminate Starting at $80 per square meter
Tile Starting at $90 per square meter
Granite Starting at $130 per square meter
Marble Starting at $150 per square meter

All countertops listed above are available for customization at increased rates if requested at least 15
days in advance. In such cases, Carlson Counters charges $120 per square meter for laminate and tile
customization and $190 per square meter for specialized orders in granite or marble. Carlson Counters

m /
does require a 50 percent deposit for any custom order.

c o
c.
All you have to do is call us at 555-4933 or send an e-mail to rogcarl@carlsoncounters.com to request

g o
an on-site visit. Owner Roger Carlson will visit your space to get rough measurements and discuss
installation options with you. He will also describe the various materials we sell and provide you with cost

s
will immediately schedule the work and let you know how long it will take. n
estimates for the different countertops you can choose. Once you have made your selection, Mr. Carlson

g m
So, contact Carlson Counters today and find out how we can give your space a new and contemporary
look!

k i n
e a
ic sp
e
//to
TO: Roger Carlson <rogcarl@carlsoncounters.com>

tp :
FROM: BlaireThomson <bomson@gotomail.com>
SUBJECT: Countertops

ht
DATE: May 29,Thursday
Dear Mr Carlson,
Thank you for coming by my home this afternoon to check out my kitchen. I appreciate the information you
provided as it confirmed what your brochure said,
I have discussed the materials you suggested with my husband. Both of the recommendations are attractive, but
after talking about it, we have agreed that we should go ahead with the marble option for our countertops.
Please send us the total cost estimate for the project when you have time.
You also mentioned during your visit that custom countertops like the one we're requesting need to be
specially ordered and may require additional time for delivery. So, could you let us know how long it will take
to arrive?
t hope to hear from you soon, and thank you for your excellent service.
Regards,
Blaire Thomson

134 Hackers.co.kr
181. What is indicated about Carlson Counters? 184. How much will Ms. Thomson have to pay
(A) It provides services for residential for the new countertops?
spaces only. (A) $90 per square meter
(B) It accepts visit requests through (B) $120 per square meter
multiple channels. (C) $150 per square meter
(C) It is situated in a central part of the (D) $190 per square meter
town.
(D) It is currently offering a service at a 185. What most likely did Ms. Thomson do
discount. before Mr. Carlson's visit to her home?
(A) Discarded an old countertop
182. In the brochure, the word "rough" in (B) Looked through an informational leaflet
paragraph 4, line 2, is closest in meaning (C) E-mailed size specifications for an
to
(A) forceful
installation

m /
(D) Visited a branch for a free consultation
(B) general

c o
c.
(C) coarse
(D) uneven

g o
183. According to the brochure, what
information will NOT be provided during
s n
an on-site visit?

g m
(A)
(B)
The date of a delivery
Descriptions of materials
k i n
(C) Prices of products
e a
sp
(D) The duration of a project

e ic
p : //to
ht t

GO ON TO THE NEXT PAGE,

TEST 4 PART 7 135


Questions 186-190 refer to the following article, e-mail, and ticket.

The Regency Scotia School of Design Hosts Historic Royal Costume Display

Threads of Royal Histoiy, launched on Friday, is the latest exhibit from the Regency Scotia
School of Design (RSSD) in Glasgow. It is a treat for period costume enthusiasts. For the exhibit,
the school borrowed 52 outfits belonging to both historical and current monarchs from museums
around the world. Of particular note are the 4,000 year old headdress and accessories on loan
from the Cairo Museum in Egypt as well as several stunning robes from the Forbidden City
Museum in Beijing. In addition, fashion students from the school have created 38 replica outfits.
All of the items are beautifully displayed on specially designed mannequins.
The exhibit will last for a month, concluding on April 30. Regular admission is £6.50, but

/
students and faculty of the RSSD are admitted for free. Senior citizens pay £5.00, and children 10
and under pay £3.50. Inquire about student group rates by calling 555-3400. The exhibit space is

o
located in the school's Leighton Hall, which is open from 10 a.m. through 8 p.m., Monday through
m
Thursday.

c. c
g o
s n
TO Larry Dent <larden@ternngton.edu.uk>

g m
n
FROM Rhoda Augustine <rhoaug@terrington.edu.uk>
SUBJECT RSSD Exhibit
k i
1I
DATE April 20

e a ' —Ii
!|!•
IS
1
Hi Larry,

ic sp ji|
I
|
e
I saw an article in the paper about RSSD's current exhibit and am thinking of taking the students

//to
i| in my morning art classes there for a field trip, i called to find out about student group rates,

I
tp :
and if there are 30 or more students, the admission fee is significantly lowered to £4.00. But I
only have 24 in total for both my morning classes, so I was wondering if you and your students
1:1

|
!,j ht
might be interested in joining us. The community center's bus seats 55 people, so there should
be enough room to accommodate everyone. It sounds like a really interesting display, and I
think our students would learn a lot from it as well. There is no rush, but let me know whether
ij

j
you're interested by Friday so I can book the visit.

j Thanks!

i Rhoda

136 Hackerslngang.com
PAID IN FULL (cash)
Group Ticket for: Threads of Royal History Exhibition
Date: April 28 "This ticket can only be used on the indicated date.
Name: Terrington Secondary School
Rate: Student group rate
Participants: 48
Group Leader(s): Rhoda Augustine, Larry Dent
All visitors are required to proceed through the security checkpoint prior to
admission. Photography is strictly forbidden. Please do not touch any item on
display.

186. What is NOT true about Threads of Royal 189. What did Mr, Dent's students most likely
m /
History? do on April 28?

c o
(A)
(B)
It
It
includes items on loan.
has pieces from around the world.
o c.
(A) Viewed ancient accessories
(B) Toured some local historic sites
(C) It is open to the public on weekends.
n g
(C) Attended a fashion show
(D) It will close at the end of April.

m s
(D) Displayed some outfits they had
created
187. Why did Ms. Augustine want Mr. Dent's
students to join her group?
i n g
190. What are visitors to the exhibit asked to

a
(A) So that her group becomes eligible for
k do?
a free tour guide
e (A) Speak in low voices

sp
(B) So that she can obtain a discount on (B) Leave food and drinks outside

ic
admission fees (C) Avoid taking pictures

e
(C) So that the school can cover the (D) Reserve a tour guide in advance

//to
expense of a bus rental

tp :
(D) So that her students can enter their
designs into a competition

ht
188. How much were the Terrington Secondary
School students charged for admission?
(A) £3.50 per person
(B) £4.00 per person
(C) £5.00 per person
(D) £6.50 per person

GO ON TO THE NEXT PAGE.

TEST 4 PART 7 137


Questions 191=195 refer to the following flyer, review, and e-mail.

Wheel-Pals, the world's most popular carpooling program, has arrived in Auckland! Download the
application today onto your mobile phone, device, or computer, and find out how easy it is to save
money on transportation! The application connects you with other users in your area that are
interested in carpooling at specific times. All our members undergo a screening process and can post
reviews about other passengers and drivers. With fees starting as low as $6 per month for a basic
package, Wheel-Pals is sure to be as popular here in Auckland as it is in Brisbane, Sydney, and
Melbourne! Go to www.wheelpals.com for more details.

USER REVIEW: Wheel-Pals Carpooling Application


By Jason Diaz (jdiaz@vastpost.co.au)
m /
c o
c.
Wheel-Pals recently launched its service in the Auckland area, and having already used the

o
n g
application while living in Brisbane, I was one of the first to sign up. As before, I purchased the
basic package because, in the past, it had enough services to help me form carpool groups with

s
other drivers or riders that live nearby. However, I don't think Wheel-Pals has done enough to

m
g
promote its launch in this area. I sometimes find it difficult to find people to carpool with. In

i n
Brisbane, I only drove my car to work once or twice a week, but now I have to do it more often.

k
Also, few people that I know locally are even aware of the application.

e a
TO
ic sp
Jason Diaz <jdia2@vastpost.co.au>
e
//to
FROM Adele Simmonds <asimmonds@\vhcelpals.co.au>
SUBJECT Thanks for the review!

tp : DATE May 29

ht Dear Mr. Diaz,


My name is Adele Simmonds, and I'm a public relations representative for Wheel-Pals. I recendy came
across an online review 5'ou left about our applicadon. On behalf of die company, thank you for taking
the dme to leave die comment. We appreciate your concerns and have decided to take steps to market
the program further in your area.
As a token of thanks for your review, you will receive points that you can use like cash to pay for your
subscripdon or any of our other services. The points will show up in die applicadon the next dme you
log in.
Thank you again, and we hope 3^011 will continue to find Wheel-Pals useful.
Regards,
Adele Simmonds
Regional PR associate, Wheel-Pals Inc.

138 Hackers.co.kr
191. What is the purpose of the flyer? 194. Where does Wheel-Pals plan to do
additional product promotion?
(A) To publicize availability of a software
application (A) In Brisbane
(B) To announce price reductions for a (B) In Sydney
specified period (C) In Melbourne
(C) To request reviews from users of a (D) In Auckland
ride-share program
(D) To inform subscribers of special 195. In the e-mail, the word "leave" in
services available paragraph 1, line 3, is closest in meaning
to
192. What problem does Mr. Diaz mention in (A) exit
the review? (B) make
(A) People find a program complicated to
use.
(C)
(D)
remain
assign
m /
(B) Membership fees are too high.

c o
c.
(C) Local residents are unfamiliar with a
program.
(D) Drivers take on too many passengers.
g o
193. What is indicated about Mr. Diaz?
s n
(A) He has never used Wheel-Pals before.
g m
(B)
(C)
He drives his car up to twice a week,
He recently relocated to Brisbane.
k i n
(D)
a
He pays six dollars per month for his
e
sp
subscription.

e ic
p : //to
ht t

GO ON TO THE NEXT PAGE.

TEST4 PART? 139


Questions 196-200 refer to the following information, e-mail, and article.

Ool
http://www.corianderfinedining.com/services

HOME ABOUT SERVICES CONTACT MENUS


Coriander Fine Dining offers a selection of services for groups:

DELIVERY SERVICE: We provide free delivery service for orders over $80. A $5 delivery fee will be applied
to orders costing less than this amount. Delivery is possible within the Washington, D.C. city limits only.

CATERING SERVICE: Coriander Fine Dining can cater weddings, parties, and corporate or personal events
at any location. We will work with you to create a menu for groups of up to 800. Service dishes, plates,
cutlery, and glassware are also available for an additional charge. Contact our catering manager Cecilia
Shiraz at ceshiraz@corianderfinedining.com to make a reservation.

VENUE RENTAL: Let us host your gathering in our private function room! We can serve groups of any size
(up to a maximum of 80 diners). Contact Raj Kapoor at rajkapoor@corianderfinedining.com to inquire
m /
about available dates and event bookings.
c o
o c.
n g
TO
s
Cecilia Shiraz <ceshiraz@corianderfinecljning.com>

m
FROM
SUBJECT
Steven Potter <spot@spartantires.com>
Availability
i n g
DATE June 12

a k
e
Dear Ms. Shiraz,

ic sp
My name is Steven Potter, and I work for Spartan Tires. I was a guest at the banquet you
e
//to
catered for the opening of Lucas Industries' headquarters. We recently made a deal to
partner with a manufacturing company in India, and a team of their representatives are

tp :
going to arrive next week to finalize our agreement. We thought it might be nice to have a

ht
meal served after contracts are signed on Friday, June 18, here at our headquarters.
Everyone should be ready for dinner by 6:30 p.m. Including our representatives, there will be
30 people in total. I would like a buffet-style menu with chicken, fish, vegetables, dessert,
and salad. We can make arrangements for our own beverages. We do not have any dishes
or dining ware, so we will need those items as well. Could you let me know if you are
available for this date and, if so, what the cost would be?
Sincerely,
Steven Potter

140 ^MP3 Hackerslngang.com


Spartan Tires Joins Forces with IVlaha-Rubber
By Enid Wendall
Spartan Tires announced in a press statement yesterday that it will finalize a deal with India-based
manufacturer Maha-Rubber on June 18. The company will produce tires under the Spartan Tires
brand. Spartan Tires has 38 branches across the country, with an average of four new franchises
opening annually. However, it does not produce its own accessories or parts, carrying only those
made by other companies. Spartan Tires CEO Carlo Gregario said in the statement, "We don't
have the facilities to produce our own tires. Partnering with Maha-Rubber will allow us to do so
at a reasonable cost." The first of the new products will be available at Spartan Tires retail
outlets nationwide on August 1 of next year.

196. What is NOT true about Coriander Fine


Dining?
199. In the article, the word "carrying" in
paragraph 1, line 4, is closest in meaning
m /
(A) Its event room can accommodate
to
c o
up to 80 diners.
(B) It is available to provide meals for
(A)
(B)
enduring
transporting
o c.
various occasions. (C) retailing
n g
(C) It offers free delivery for any order
within city limits.
(D)

m s absorbing

(D) Its catering service can be booked


through Cecilia Shiraz.
i n g
200. Who might Ms. Shiraz be preparing a meal
for on June 18?

a k (A) Retiring executives at Spartan Tires


197. What does Mr. Potter ask about?
e (B) Visitors from Maha-Rubber
(A)
(B)
ic sp
The availability of an event venue
The price for catering a dinner
(C) Guests at an event held at Coriander
Fine Dining
(C)
e
The date of a final agreement (D) Participants in a celebration hosted by

//to
(D) The cost of renting out parking Lucas Industries

tp :
198. What will Mr. Potter have to pay extra for?

ht (A) Borrowing plates and cutlery


(B) Having several options for main
courses
(C) Getting food delivered outside the city
(D) Serving an assortment of appetizers

This is the end of the test. You may review Part 5, 6, and 7 if you finish the test early.

33 p,32A / 34 SHJS p.327 / «I3 P.B52 / Part 5S6 tfS SJ-g M7|
»Cfg E|0|x|0i| Ofe Self *||=i aiAMl ssn XKJO) sxfi to| EflE# ggsil mahs.

TEST 4 PART 7 141


m lOOOfll 3 READING

m /
c o
o c.
n g
m s
i n g
a k
e
sp
I TEST 05

e ic
//to
PART 5

tp : 01 PART 6

ht 0fr PART 7

Self X]\3.

Wm Eil^S S #°!A^
1.-gqima|?Bte^8& noil
2. Answer Sheet. 31, ^Hl Stl|oHaL^a? □ o||
3.A|2j|lS|6HaUfa? □ 0)1
OSga|7f StHEIOJODf =S KgBj o EjlAES A|5forL|Ch

^il mm Dl*lfe 70¥ #! _A| ¥tlLRK


EH ae ApJ0 # 75¥0|D1, M°i 2 S£ nA DIS# ?|SH
READING TEST
In this section, you must demonstrate your ability to read and comprehend English, You will be given a
variety of texts and asked to answer questions about these texts. This section is divided into three parts
and will take 75 minutes to complete,
Do not mark the answers in your test book. Use the answer sheet that is separately provided.

PARTS
Directions: In each question, you will be asked to review a statement that is missing a word or phrase.
Four answer choices will be provided for each statement. Select the best answer and mark the
corresponding letter (A), (B), (C), or (D) on the answer sheet.

© PARTS i!$«0l-M2f 11§

m /
101. Holden Advertising Agency won all 105. Audio-One is so

c o
about the quality of

c.
the major marketing awards this year. its products that it offers a money-back
guarantee for all items.
(A)
(B)
practical
practicality (A) beneficial
g o
(C) practically
s
(B) probable
n
(D) practicalities

g m
(C)
(D)
confident
productive
102. Mr. Harris neglected his name at the
bottom of his employment application.
k i n
106. After carefully analyzing the budget report,
(A) sign
e a Mr. Green could approve funding for

sp
(B) signs the expansion of Jewett Resorts.

ic
(C) to sign (A) final
(D)
e
was signed (B) finally

//to
(C) finalize

:
103. Eugene Rivera, who negotiated an (D) finals

tpagreement between the two firms,

ht
reported that the details would be settled 107. Ms. Warren is in charge of making sure
the next several days. that the clients receive everything
(A) within they need during their stay,
(B) close to (A) to visit
(C) afterward (B) visit
(D) nearby (C) visited
(D) visiting
104. Earlier this morning, Mr. Yang that
everyone in his department would receive 108. Kembil Co.'s board instructed James
a bonus. Holley to implement a versatile to
(A) announcing long-term planning for the next five years.
(B) announces (A) approach
(C)announced (B) approaches
(D) has announced (C) approachable
(D) approached

144 S2jiuh 901^7IS MP3 Hackerslngang.com


109. Building of the Beverly Office Complex can 115. Ventera Inc. was able to build strong
begin as soon as the firm all the relationships with its most valuable
necessary permits from City Hall. customers by communicating
with them.
(A) searches
(B) entrusts (A) marginally
(C) figures (B) comparably
(D) acquires (C) consistently
(D) indefinitely
110. A factor especially to the agricultural
industry is whether there are reliable 116. In order to satisfactory levels of
supplies of water and electricity. productivity in the factory, the floor
(A) knowledgeable manager regularly consults with workers
(B) convenient about mechanical problems.
(C) pertinent (A) persuade
(D) simultaneous (B) limit

m /
o
(C) maintain
111. the most cutting-edge cellular
phones may not satisfy the specific needs
(D) remain

c. c
of every user.

g o
117. The candidates for the job opening have
experiences and career objectives
(A)
(B)
Notwithstanding
Once
s n
despite all having worked in the same
(C) Nearly

g m
field.
(D) Even

k i n (A)
(B)
difference
differently
112. As it has been

e a
60 business days
since the purchase date, Branson Outlet
(C)
(D)
different
differ

(A) so much sp
will not allow an exchange or refund.

ic 118. Koester Company records all incoming

e customer calls with the of ensuring

//to
(B) more than
inquiries are handled appropriately.
(C) much longer

tp:
(D) as few as (A) status

ht
(B) reservation
113. Ms. Mitchell is looking for an apartment (C) association
a train station so that she will not (D) intent
need to commute by car.
(A) near 119. Coalport's management is not certain that
an advertising campaign will successfully
(B) toward
boost sales because it has launched
(C) adjacent
(D) between one before.
(A) never
114. Immediately after ordering a new office (B) also
desk online, Mr. Perez was issued (C) always
of the transaction. (D) still
(A) confirm
(B) confirmation
(C) confirms
(D) confirmed
GO ON TO THE NEXT PAGE.

TESTS PARTS 145


120. MediaCore's customers will have digital 126. Immigration gave Mr. Kim an on
to dozens of publications by the end his stay, allowing him to remain another
of the month. six months for work.
(A) access (A) attendance
(B) accessing (B) expertise
(C) accessed (C) operation
(D) accesses (D) extension

121. fashion design jobs appear 127. The reason a new security gate was
glamorous in popular media, most positions installed is that the old one had been
in the industry are demanding in reality. damaged by the storm.
(A) Although (A) thick
(B) Despite (B) quick

/
(C) Regarding (C) high
(D) Assuming (D) main

o m
122. Ryder Capital Bank now trains its advisors
more to make certain that they are
128. Submissions

c. c
meet Literati Magazine's
standard will be considered for publication
thoroughly familiar with the investment in our next issue.

g o
process.
(A) comprehensively
(A)these

s
(B)those n
(B) temporarily
g m
(C) that
(C)
(D)
wastefully
optionally
k i n (D) they

e
123. Once the project ends next year,a 129. Investor Link was able to the
financial crisis because Mr. Macintyre had
employee evaluations

ic sp
to assess
performance and decide on pay increases.
the foresight to develop a contingency
plan.

e
//to
(A) will conduct (A) advocate
(B) were conducted (B)engage

tp :(C) have conducted (C) persist

ht
(D) will be conducted (D) withstand

124. Wheelpoint's new line of tires performs 130. Ms. Kenner and Mr, Stone were both
under the most hazardous road invited to the meeting, but of them
conditions. has confirmed yet.
(A) habitually (A) most
(B) capably (B) few
(C) sizably (C) neither
(D) generously (D) whatever

125. Bransonville Beach is closed further


notice as the area is affected by strong
tides caused by Hurricane Thurston.
(A) from
(B) onto
(C) until
(D) during

146 JiSsensJig Hackers.co.kr


PART 6
Directions: In this part, you will be asked to read four English texts. Each text is missing a word,
phrase, or sentence. Select the answer choice that correctly completes the text and mark the
corresponding letter (A), (B), (C), or (D) on the answer sheet.

Q PART 6 it! cf #0I -M^t 8^

Questions 131-134 refer to the following e-mail.

To: Jennifer Mendez <jmendez@imshampoo.com>


From: Rick Holloway <rholloway@imshampoo.com>
Subject: Marketing
Date: March 13
m /
c o
Dear Jennifer,

o c. O
-O

n g
I just met with the new head of PPJ Marketing, Robert Pierson, and he is ------ to helping
1 ul .
|S||

m s
update the look of our product. Like me, he believes that our shampoos are overdue for
en

i n g
rebranding and thinks that marketing them to young, free-spirited people is an excellent idea. —
--J

k
in this regard, he feels that researching the needs of consumers between the ages of
a
e
132. g

sp
18 and 24 would be helpful for developing television advertisements. This will provide us with
O
LD

e ic
information to effectively
133.
this group. Over the next few weeks, he and his team will
o

//to
create concepts for 30-second commercials, which they will to us for consideration. —
134. s

tp :
ht
131. (A) accustomed 133. (A)convey
(B) committed (B) authorize
(C) indifferent (C) gather
(D) confused (D) target

132. (A) None of our shampoo advertisements 134. (A) be delivered


|||j| are presently directed to that group, (B) are delivering
(B) The new product line would (C) deliver
significantly expand our merchandise (D) have been delivering
selection.
(C) This has helped them make a better
decision about what to buy.
(D) Our biggest concern is increasing our
share in the international market. GO ON TO THE NEXT PAGE,

TESTS PART 6 147


Questions 135-138 refer to the following e-mail.
i!
To: Customer Service <cs.interair@interair,com>
From: Alex Hogan <ahogan@dallasmail.com>
Subject: Reservation #JHK2105
Date: June 21

To Whom It May Concern,

I apologize for the inconvenience, but a personal matter has come up, me from
135. 136.
flying to Budapest as planned. I would like to get my money back. If this is possible, can you

please my request? I am hoping to book a different flight to Sydney in less than five
137.
days. I understand that I will have to pay a
m
for this change. You may deduct this from /
138.

c o
c.
the amount that will be returned to me.

Please let me know if you need any other information,


g o
Sincerely,
s n
g m
Alex Hogan

k i n
e a
sp
135. (A) I am writing to inquire about having my 137. (A) renew

ic
|||| seat upgraded. (B) generate

e
(B) My frequent flier points have not been (C) expedite

//to
applied to my account. (D) prove
(C) My luggage was lost on a recent flight

tp:
with your airline, 138. (A) ticket

ht
(D) I would like to cancel my flight and (B) charge
obtain a refund for it. (C) royalty
(D) reward
136. (A) prevents
(B) prevention
(C) preventing
(D) preventive

148 9019^9 Seoi^iMPB Hackerslngang.com


Questions 139-142 refer to the following advertisement.

If you own a small or medium-sized business and are looking for professional accounting

services, Johnson and Eversham Accounting can help!

We our expertise to a wide variety of businesses. Established three decades ago,


139.
Johnson and Eversham Accounting began by assisting small firms in Manchester, New

Hampshire, and we have been expanding our reach ever since. --------

All of our accountants are by the Organization of Chartered Accountants, which allows
141.
us to guarantee that our clients always receive top-notch services. Each of our employees

m /
c o
in distinct business accounting fields, so we can handle any financial matter you might

c.
142.
have.

g o
s n
To find out more, contact us today at 555-6277 or go to www.jeaccounting.com.

g m
139. (A) provided
k i n
141. (A) careful
(B)
(C)
provide
will be providing
e a (B)
(C)
eligible
suspended

sp
(D) had provided (D) certified

e ic
140. (A) Our expansion was completed several 142. (A) specializes

//to
Ipl years ago. (B) specialize
(C) specialist
:
(B) These firms have announced that they

tp will form an association. (D) special

ht
(C) We now have 60 offices in cities
across the nation.
(D) This prevented us from launching our
first office in the region.

GQ ON TQ THE NEXT PACE_

TESTS PART 6 149


Questions 143-146 refer to the following notice.
|f!I
OS!
Notice of Homeowner's Association Dues Increase

At its last meeting, the board was asked to consider increasing the dues slightly.
143.
deliberations on this request, they proposed raising fees by $25 a month, from $375 to $400,

effective July 1. To the board, this action is justified for several reasons. First, the additional

funds will contribute to important , such as the installation of a new security system in
144.
the parking garage next year. Second, they will cover the rising cost of natural gas.

The timing of the dues increase seems reasonable


146.
there has not been one in four

m /
o
years. Tenants are invited to vote on the proposal at the Homeowner's Association meeting

in June.
c. c
Jordan Roper
g o
Accounting Manager
s n
m
Homeowner's Association

i n g
a k 145. (A) The homeowner's association

e WSk management made some repairs over

sp
the winter.

ic
(B) Tenants were satisfied with the estimate

e provided to them by the contractor.

//to
144. (A) evaluations (C) We had to pay more than usual last
(B) experiences

tp :
(C) eliminations
winter due to a steep hike in gas prices.
(D) We hope the building's improved security

ht
(D) expenditures system will attract more businesses.

146. (A) in case


(B) given that
(C) even as
(D) whether

150 ?ss<we-¥l?j8a*ilS Hackers.co.kr


PART?
Directions: In this part, you will be asked to read several texts, such as advertisements, articles, instant
messages, or examples of business correspondence. Each text is followed by several questions. Select
the best answer and mark the corresponding letter (A), (B), (C), or (D) on your answer sheet.

© PART 7 #0I MV 54^

Questions 147-148 refer to the following flyer.

Green Wave Consulting Services


Saving the planet while saving you money! ^ y

m /
We all know that we need to reduce the amount of

c o
c.
greenhouse gases being released into the atmosphere. But
few of us realize that our own residences are often to blame.

o
Homes are the third largest source of emissions in the entire
g
s n
country! Luckily, there's now a whole host of easy solutions to
reduce your environmental impact. And the best thing of all is
that doing so can save you money as well.

g m
• Installing wall insulation
k i n
Our money-saving green changes include:

e a
" Fitting homes with new, efficient water heaters
o Switching to energy-saving lightbulbs

ic sp
• Putting in window coverings that keep heat in
Call 535-9974 today to arrange for one of our technicians to
e
//to
visit you at home for a consultation. It's completely free, and
we'll get back to you within three business days with a detailed

tp : personal plan for making your home more energy efficient.

ht
For any other inquiries, visit our offices on 593 Rayburn Street
in downtown Portland, Monday through Saturday from 10 a.m.
to 7 P.M.

147. What is mentioned about greenhouse gas 148. How can customers receive a customized
emissions? energy saving plan?
(A) They recently went up by a large (A) By arranging a meeting with a consultant
amount. (B) By inputting their details on a Web site
(B) Residences are one source of them. (C) By visiting the Green Wave headquarters
(C) They are limited by the government. (D) By sending an e-mail to a specialist
(D) Meters can be installed to measure
them.

GO ON TO THE NEXT PAGE^

TEST 5 PART? 151


Questions 149-150 refer to the following text-message chain.

Marge Bledsoe 11:05 a.m.


I just remembered I have a client who'd like to see the house that
we've listed on 78 Crescent Lane. Unfortunately, I'm fully booked
today. Is anyone free this afternoon?
Evan Clark 11:08 A.M.
I can bring the client to the house at 2:30. I have another showing
nearby and can go there afterward.
Julia Anderson 11:14 A.M.
Marge, are you referring to Eva Teal? She called a short while
ago and said she was looking for a two-bedroom place downtown.
m /
c o
c.
Marge Bledsoe 11:15 a.m.
Yes, that's her. I think Evan can handle it. I need your help with
another client, Julia.
g o
Julia Anderson
Sure, just let me know later.
s n 11:15 a.m.

Evan Clark
g m 11:17 a.m.

i n
OK. Why don't I show her the other home nearby as well? It's just

k
a
a few blocks away from there and also has two bedrooms.
Marge Bledsoe
e 11:18 a.m.

ic sp
Good call. Will find out if she's interested in seeing it. I'll give her
your mobile number in case there's a problem. She can probably

e meet you there first before seeing the Crescent Lane house.

p : //to
ht t
149. Who most likely is Ms. Anderson? 150. At 11:18 a.m., what does Ms. Bledsoe
mean when she writes
(A) A building administrator Hi ' "Good call"?
(B) A real estate agent (A) She can confirm that Mr. Clark has the
(C) A housing inspector right information.
(D) An office receptionist (B) She is relieved to find out that
someone can replace her.
(C) She is pleased with the results of a
phone conversation.
(D) She likes the proposal that Mr. Clark
has made.

152 ?SSat°ii!E5(a°|il<.h90t^lS?J90|°f7|MP3 Hackerslngang.com


Questions 151-152 refer to the following e-mail.

mm\

Bob Fletcher <blletch88@speedmail.com>


Tropical Tours <customersupport@tropicaltours.com>
March 1
Subject Notice regarding Southeast Asia tour

Dear Mr. Fletcher,


We regret to inform you that Tropical Tours has been forced to call off the Southeast Asia tour
you booked from March 3 through March 10. You may have seen on the news that a severe storm
hit the region yesterday and has caused serious damage. The airports you were scheduled to use in
Jakarta, Singapore, and Kuala Lumpur remain closed to commercial planes for the time being.
Airport officials believe it will be four days until air traffic will return to normal. And as the storm
m /
travels, other tour destinations may be affected as well.
c o
o c.
At this point, we can offer you and your wife a complete refund for the tour package. Your total
purchase price will be refunded to the credit card you used to book the tour. Alternatively, we can

n g
book a tour for you at a later date. If you choose this option, we recommend waiting at least until

m s
May so that your visit to the different locations is as pleasant as possible. Should you decide to
take the tour at a later time, we will also provide you with hotel upgrades at all the locations on
the itinerary.

i n g
k
We sincerely apologize for the situation as we understand that it is a last-minute cancellation, and

a
e
you have probably already prepared for the trip.

sp
Sincerely,
Customer Services

e ic
//to
Tropical Tours

tp :
ht
151. What is the main purpose of the e-mail?
(A) To advertise tour packages to Asia
(B) To notify a traveler of airport transfer
152. What will happen if Mr. Fletcher travels at
a later date?
(A) He and his spouse will have to take
changes some flights separately.
(C) To solicit feedback from tour (B) He will be given superior
participants accommodations.
(D) To announce the cancellation of a trip (C) He will be unable to travel to all the
destinations.
(D) He will receive a partial refund of the
charges.

GO ON TO THE NEXT PAGE.

TEST 5 PART 7 153


Questions 153-154 refer to the following information.

SAFETY INSTRUCTIONS

This product was prepared from meat that was inspected and
passed by the National Food Safety Board. Nevertheless, some food
products may become contaminated with illness-causing bacteria if
mishandled or cooked improperly. For your protection, follow the
steps below:
0
Store meat in a chilled environment as soon as possible after
purchase. The recommended minimum temperature settings are
-18 degrees Celsius for freezers and 4 degrees Celsius for
refrigerators. Avoid thawing meat in an environment of greater
m /
than 20 degrees Celsius.
c o
c.
6
Keep raw meat separate from other types of food.

o
• Wash all surfaces that come into contact with meat, as well as
g
0
your hands and cooking utensils.

s
Cook meat thoroughly and keep warm before serving. n
g m
k i n
appear?
e a
153. Where would the information most likely 154. What are cooks advised against doing?

sp
(A) Cooking at low temperatures
(A) On a food product package (B) Storing leftovers for too long
(B)
e ic
In a kitchen appliance manual (C) Touching food items with bare hands

//to
(C) In a recipe book for meat dishes (D) Defrosting meat in certain
(D) On a display in a dairy section temperatures

tp :
ht

154 ^ *11# Hackers.co.kr


Questions 155-157 refer to the following article.

Stretch Makes Plans to Expand

Stretch Corporation announced yesterday that it keep up with this increase so that we can continue
plans to expand in a much larger location on the providing our target customers with quality
outskirts of the city. The athletic wear company's athletic wear and equipment." Moreover, Stretch's
headquarters will remain at its current location at move is also good news for the local economy.
the corner of Dupont and Beverly Street, but — [2] —. "Even though domestically manufactured
production facilities will be moved to Haverford, products cost more than those manufactured
around 15 kilometers from the city center. CEO overseas, our customers are happy to pay a little
Alex Cruz said, "For the past few years, the extra," said Cruz. — [3] —. "For this reason, we
number of people who enjoy exercising has grown
/
intend to keep production nearby and have no reason

m
significantly. — [1] —. Our move will allow us to

c o
to outsource to anywhere else." — [4] —.

o c.
155. What is the purpose of the article?
n g
157. In which of the positions marked [1], [2],
(A) To discuss a company's plans for

m s
lip [3], and [4] does the following sentence
best belong?
growth
(B) To encourage fitness among youth
i n g
'This is because the relocation is expected
(C) To describe a new line of products

a k to create 150 additional jobs for the

e
(D) To report on the opening of a store residents of Haverford."

156. For whom most likely are Stretch

ic
Corporation's products made? sp (A)
(B)
[1]
[2]

e (C) [3]

//to
(A) International businesspeople
(D) [4]
(B) Fans of sports teams

p
(C)

t : Working parents

ht
(D) Fitness enthusiasts

GO ON TO THE NEXT PAGE.

TESTS PART? 155


Questions 158-160 refer to the following notice.

NOTICE: Fashion-It Employment Opportunities

Fashion-It is now accepting applications for positions at our new headquarters.


We manage one of the most popular clothing chains in the country. Applicants
will find that our wages and benefits are comparable to or even better than our
competitors'. Our work environment is second-to-none, with quality facilities and
a caring, supportive staff. Our thousands of employees can testify that Fashion-It
is one of the best companies to work for.
Currently, we are seeking:
5 Building Maintenance Technicians

m /
I Inventory Analyst

c o
c.
6 Receiving, Shipping, and Warehouse Associates
1 Product Manager
2 Security Officers
g o
s n
Applicants should log in to our Web site at www.fashionit.com/jobs. Choose one
or more jobs of interest and fill out the applicafion(s). After you submit the form,

g m
you will receive a confirmation via e-mail. If we are interested in interviewing you,

i n
we will contact you within 14 days. All applications will be kept on file for
12 months.
k
e a
ic sp
e
//to
158. What is NOT indicated about Fashion-It? 160. According to the notice, what will happen

tp :
(A) It offers its employees excellent within two weeks of an application being
submitted?

ht
working condi tions.
(B) It stores job applications for one year. (A) More job openings will be posted.
(C) It is recruiting employees for several (B) Fashion-It will begin training new
branches. members of staff.
(D) It operates an online recruitment page. (C) The new store will officially open to
the public.
159. The word "comparable" in paragraph 1, (D) Applicants who have been selected
line 3, is closest in meaning to will be notified.
(A) affiliated
(B) similar
(C) significant
(D) balanced

156 ^ yWI MP3 Hackerslngang.com


Questions 161-163 refer to the following e-mail.

To: Jason Minkovski <admin(a)hepfordrealestate.com>


From: Angela Johnstone <president(a)hepfordrealestate.com>
Date: April 23
Subject: Conference arrangements

Hello Jason,
I've been invited to speak at the Great Lakes Real Estate Convention between June 18 and
21. It's taking place in Milwaukee, so could you book a flight for me that departs the day
before the conference begins? An economy class ticket is fine as Milwaukee is just a short
flight from here in Chicago,
Also, while I'm in the Midwest, I should stop by the Minneapolis branch and see how the
new regional manager. Beryl Twispe, is getting along in her new position. Could you
m /
contact her and find out if this would be a good time for me to visit? If so,

c o
c.
I would prefer traveling to Minneapolis by train, but I am not sure if it is a reasonable
option. However, if a train trip will take too long, go ahead and make reservations for a
flight out of Milwaukee.

g o
s n
I'd like to depart the day after the conference ends, and I'll also require accommodation in
Minneapolis. You don't have to take care of the arrangements for my stay in Milwaukee

g m
as I was informed by the organizer of the convention, Brenda Orson, that she will deal
with my hotel, local transport, and meals. If you need to coordinate with her, you can
call (612) 555-3991-

k i n
a
Thank you, and let me know the details when you have them.

e
sp
Best wishes,

ic
Angela Johnstone

e
p : //to
t
161. Why did Ms. Johnstone write to

ht Mr. Minkovski?
(A) To invite him to be a guest speaker at
163. What has Mr. Minkovski been asked to
find out?
(A) The duration of a trip to the Midwest
an event (B) The cost of hotel accommodation in
(B) To request that he make arrangements Milwaukee
for a trip (C) The availability of a regional manager
(C) To suggest he register for a convention for a visit
(D) To ask for a progress report from the (D) The location of the upcoming
Minneapolis branch convention

162. When will Ms. Johnstone leave Milwaukee?


(A) On June 17
(B) On June 18
(C) On June 21
(D) On June 22

GO ON TO THE NEXT PAGE,

TEST 5 PART 7 157


Questions 164-167 refer to the following online chat discussion.

^ Tom Estrada 18:42 Good afternoon from Apex Cable TV. How may I help you today?

Alyssa Munro 18:43 Hi. I need help accessing movies using Movies On Demand.

^ Tom Estrada 18:44 OK. Please give me a moment to open your account details on my
computer.

Tom Estrada 18:45 Thank you for waiting. It appears that you are not subscribed to
our Movies On Demand service.

Alyssa Munro 18:45 Oh, really? I thought it was part of my subscription.

^ Tom Estrada 18:46 Well, our records show that you have the standard plan. This
doesn't give you full access to our movie channels.

m /
Alyssa Munro 18:47 Will I need to change my plan if I want Movies On Demand?

c o
Tom Estrada 18:47
c.
You can add the service to your present plan. It costs an extra

o
$8.99 a month. Would you like me to add it now?

Alyssa Munro 18:48


n g
Do you need my credit card number? I don't have it with me now.

Tom Estrada 18:49


m s
That's all right. We can add the charges to your bill, beginning
next month.

i n g
Alyssa Munro 18:50
k
OK. Let's go ahead and do that then.

a
e
ic sp
e
164. What does Ms. Munro need assistance with? 166. What can be inferred about Ms. Munro?

//to
(A) Setting a favorite channel (A) She will pay a higher monthly fee.

tp :
(B) Renewing a subscription (B) She will cancel a membership.

ht
(C) Gaining access to a service (C) She cannot get a good cable signal.
(D) Repairing a faulty connection (D) She was not approved for a credit
card.
165. What is indicated about the standard plan?
(A) It is valid for a period of one year. 167. At 18:49, what does Mr. Estrada mean
(B) It costs less than competitors' « when he writes, "That's all right"?
packages. (A) He will ask about the credit card
(C) It is being offered at a promotional number later.
rate. (B) He thinks a charge will be canceled.
(D) It provides limited access to movie (C) He has received payment for a bill.
channels. (D) He does not require some information.

158 Hackers.co.kr
Questions 168-171 refer to the following advertisement.

Satisfy your Craving for Sweets


at Katie's Cupcakes!

For the most delicious cupcakes you'll ever taste, head to Katie's Cupcakes at 637 Elm Street in
downtown Lavington! — [1] —. We can make over 50 flavor combinations of cake, filling, and
frosting. Whether you like more traditional cupcake flavors or more exotic lands, like mint and
pumpkin, Katie's Cupcakes can fulfill your craving without ruining your diet. Nearly half of our
items are low in both sugar and fat, making them great for those limiting their caloric intake.
— [2] —. We promise that, upon your request, any of our delicious treats can be baked free of
sugar, gluten, lactose, or nuts or can be made specially for diabetics.

These days, many people use cupcake arrangements instead of traditional cakes for weddings,
m /
c o
birthdays, and other celebrations. — [3] —. Katie's Cupcakes can customize an arrangement in

o c.
any color and flavor, for any occasion. Stop by any time to check out examples of our work. Those
placing an order for a cupcake arrangement in June will receive a 10 percent discount!

n g
Katie's Cupcakes is open Monday through Saturday from 8 a.m. to 4 p.m. Samples are always

s
available, so drop by soon and satisfy your taste buds. — [4] —.

m
i n g
a k
e
168. What is NOT indicated about Katie's 170. How can customers get a discount on

sp
Cupcakes' products? cupcake arrangements?

article.
e ic
(A) They were promoted in a magazine (A) By attending a sampling session
(B) By ordering during a specific period

//to
(B) Some of them come in low calorie (C) By submitting a coupon at an

tp :varieties.
(C) They can be tailored to customers'
establishment
(D) By answering a customer

ht
preferences. questionnaire
(D) Some of them are appropriate for
diabetics. 171. In which of the positions marked [1], [2],
|j|fe [3], and [4] does the following sentence
169. According to the advertisement, what does best belong?
Katie's Cupcakes do for its customers?
"And for those with dietary restrictions,
(A) Provides delivery to them for a small we offer an array of options."
fee
(A) [1]
(B) Allows them to taste products for free
(B) [2]
(C) Prints ingredients on packaging
(C) [3]
materials
(D) [4]
(D) Transports special arrangements to
event venues

GO ON TO THE NEXT PACE,_

TESTS PART 7 159


Questions 172-175 refer to the following article.

Everything Video Opens Store


By Kelly Warren

Everything Video, a pioneer in the used electronics marketplace,


recently opened its fourth store, this time in San Diego. With
popular branches in Anaheim, Los Angeles, and San Bernardino,
the chain's new store, located in the Seaside Mall, is its largest
location yet. The space boasts an incredible variety of thousands
of games for multiple platforms that date back to the 1980s.
There is also a smaller section of game consoles, handheld

m /
players, and an assortment of other vintage video game devices.

c o
c.
Used movies on videotape or DVD are also available.

o
Kendra Brown, founder of Everything Video, says, "A lot of

g
s n
people miss owning physical objects." She explained that the
shift in recent years to playing games and watching movies

g m
online has indirectly created a demand for physical products.

i n
"Often, our customers buy these items for sentimental reasons."

k
e a
Not only can customers do some shopping, but Eveiything Video

sp
buys stock as well. If you have games, movies, or devices they're

e ic
interested in, staff offer cash payments or store credit to use at
any of their locations in exchange for them.

p : //toAccording to branch manager Dane Cruz, as an introductory

ht t special, shoppers who buy three games or more will receive a 20


percent discount off their total purchase amount. The offer is
valid until August 1 and is only available at the San Diego
branch.

a
160 A MP3 Hackerslngang.com
172. What is the article mainly about? 174. What is NOT indicated about Everything
Video?
(A) The launch of a software development
firm (A) It has a Web site where customers can
(B) A sale on new video games rent games and videos.
(C) An expansion of a retail chain (B) It purchases some items from its
(D) The release of a gaming device customers.
(C) It is offering a special promotion at its
173. What is NOT mentioned as a product type San Diego store.
that will be sold at the new store? (D) It provides store credit for use at —1
m
various branches. LO
(A) Secondhand movie DVDs H
(B) Videotaped television series g
The word "shift" in paragraph 2, line 3, is
(C) Classic video games "
(D) Handheld game consoles
closest in meaning to
(A) period
m / CD
nj

(B) relocation

. c o o
LU

oc
(0) swipe
CD
(D)change 4>

n g
m s O
CD

in g a
-u

a k a
CO

pe o

ic s LD

e O

//to
or

tp : j2
!>"
Kn

ht
nit
rid
oo
2do
LU
X)
mfu
a.
rj'

GO ON TO THE NEXT PAGE^

TEST 5 PART 7 161


Questions 176-180 refer to the following online contact form and e-mail.

Cyprus Software

If you experience any difficulty with our products, let us know! Simply fill in your contact
details below, write us a short message, and then click "send." One of our staff will send you
an answer as soon as possible. You may also go to www.cyprussoftware.com/help to get live
online assistance or to check out our troubleshooting forum to find answers for your
questions from our staff or other users.

NAME Amos Poison


E-MAIL ampol@digimail.com
PHONE (509)555-3984

m /
c o
c.
I recently purchased and downloaded a photo-editing program from your site called
Picto-Master. I have tried installing the software, but at the end of the process, a window

o
popped up Informing me of an error. I was not notified this might happen when I

g
n
purchased the software, so I am unsure how to proceed with the installation.

regard to this specific issue.


m s
I checked the online forum to find a possible solution, but I was unable to find any post in

i n g
Please contact me about this issue as soon as possible, as I need the program for a
photography project I'm working on.

a k
Thank you.
e
ic sp , SEHD 'H

e
p : //to
ht t TO: Amos Poison <ampol@digimail.com>
FROM : Client services <cHentserv@cyprussoft\vare.com>
SUBJECT: Your inquiry
DATE: September 17

Dear Mr. Poison,


Thank you for your message and for purchasing one of Cyprus Software's products. First, I would like to
apologize for any inconvenience.
Generally, the problem you described only occurs when the user's operadng system is incompadble with
our software. Please check the list of supported operadng systems on our Web site. If your operadng
system isn't listed, you will need to run a separate repair program in order to install Picto-Master. After
you have run the program, you may try downloading Picto-Master again. The repair program may be
downloaded from our Web site as well.

162 ^ Hackers.co.kr
If you experience die same problem, make sure that you have downloaded the correct file from our site,
and not the business or trial versions of the photo-editing software.
Should you require further assistance, please don't hesitate to respond to this e-mail or call our 24-hour
hotline at 1-800-555-3300.

176. Why did Mr. Poison fill in the online 179. Why did the pop-up window probably
contact form? appear on Mr. Poison's computer?
(A) To purchase a computer program (A) His payment was not accepted by the
(B) To request installation assistance vendor.
(B) His operating system is not compatible
/
(C) To put in a work order for photography
(D) To ask about a home security system with the software.

o
(C) He typed in a required access code
m
177. What is NOT mentioned as a way clients
can get assistance from Cyprus Software?
incorrectly.

c. c
(D) He is using an unauthorized version of
(A) Browsing an online forum a program.
g o
(B)
(C)
Submitting an online form
Visiting a branch office
s n
180. What is NOT indicated about Cyprus
(D) Using a chat service
g m
Software?

178. What is mentioned about Cyprus


k i n (A) Its programs can be downloaded
multiple times.
Software's Web site?

e a (B) It offers on-site service visits via online


request.

sp
(A) A repair program to assist with
installation is available there. (C) Its photo-editing program comes in

ic
(B) It includes instructional manuals for
e
different versions.
(D) It can be contacted for assistance

//to
its products.
(C) It promotes a number of items 24 hours a day.

tp :
currently being offered at a discount.

ht
(D) Some of Cyprus Software's older
products can be found there.

GO ON TO THE NEXT PAGE

TEST 5 PART? 163


Questions 181-185 refer to the following schedule and e-mail.

Swimming lesson schedule

Beginning on July 16, the Cornwood Condominium Homeowners Association, in partnership with the
Irwindale Athletics Club (IAC), is hosting a series of swimming lessons for the summer, the details of
which are below.

Age and Skill Level Instructor Schedule


Iliii
Beginners Coach Eric Moss Mondays and Thursdays 2 to 6 p.m. $100
6-12 years
Advanced Coach Ty Warren Wednesdays and Fridays 8 to 10 a.m. $150
6-12 years
Beginners
12-17 years
Coach Eric Moss Wednesdays and Fridays 2 to 6 p.m. $150

m /
Advanced Coach Kay Sanders Saturdays 2 to 6 p.m.

c o
$200

c.
12-17 years
Adult Coach Liza Simmons Saturdays 9 to 11 a.m. $250
18 years and older
g o
*A $20 monthly fee for using the pool is included in the fees.
s n
m
Notes: To join a class, please complete a registration form at the administration office and pay the

g
k i n
corresponding fees. IAC members or groups of four or more are eligible for a course fee discount of
15 percent. For inquiries about the swimming lessons or other activities for residents this season,

a
contact Stan Macaulay at s.macaulay@cornwoodhoa.org.

e
ic sp
e
//to
tp:
'

ht
,

Dear Mr. Macaulay,

My name is Allan Carpenter, and I just recently moved into the Cornwood Condominium. I have two teenage
daughters, Tamara and Kristin, who are interested in signing up for the swim lessons being offered this
summer. Tamara is 13 and is already quite an accomplished swimmer, having been a varsity member at her
previous school for two years. The younger one, who is 11, has had less experience as a swimmer, but she is
certainly not a beginner.

I am planning to visit administration office this week with my daughters to sign them up, but I wanted to
find out whether you accept credit cards as a method of payment. Please let me know as soon as it is
convenient. Thank you!

Allan Carpenter

164 MP3 Hackerslngang.com


181. What is suggested about the Cornwood 184. Which coach will most likely teach Kristin
Condominium Homeowners Association? Carpenter's class?
(A) Its members do not have to pay course (A) Eric Moss
fees. (B) Ty Warren
(B) It is raising funds to build a new pool. (C) Kay Sanders
(C) Its office is located in another building. (D) Liza Simmons
(D) It has arranged several summer
activities. 185. What has Mr. Macaulay been asked to
provide?
182. What is NOT indicated about the (A) A complete schedule of activities
swimming lessons? (B) Information about a payment method
(A) They may sometimes take place at an (C) The contact details of an instructor
outdoor pool. (D) Directions to a registration desk

m /
(B) They are a joint effort of two

c o
c.
organizations,
(C) They include a fee for using the pool.
(D) They are discounted for members of
g o
an athletics club.

s n
183. What does Mr. Carpenter mention about
his eldest daughter?
g m
(A) She already signed up for class,
k i n
(B) She
a
recently graduated from school.

e
sp
(C) She joined a swim team in the past,
(D) She plans to become an IAC member.

e ic
p : //to
ht t

GO ON TO THE NEXT PAGE.

TEST 5 PART 7 165


Questions 186-190 refer to the following invitation, e-mail, and information,
Ifll
Ooli

You are cordially invited to the opening night of


Dance of the Daffodils
An original ballet written by Michelle Adams
Choreographed by Lucas Pasdar, Music by Amy Lin
Performed by the Fraulein Danza Company New York City
The Gladstone Theater on 473 Broadway Avenue will open its doors for the event on Friday
May 9 at 7:30 p.m. The choreographer will say a few words about the performance before the
curtains rise at 8 o'clock. Following the performance, lead female dancer Sofia Pinsky and
lead male dancer Igor Petrovich will greet guests and sign autographs at a special reception
with cocktails and appetizers. Formal clothing is recommended and confirmation of
attendance is required (e-mail events@frauleindanza.com). This invitation is for the holder
and a guest and must be presented upon entering the theater.
m /
c o
o c.
i:j TO Lena Reid <lenareid@generlcamail.com>
n g
H
I
FROM
SUBJECT
Shonda Dixon <shodi@minepost.com>
Re: Ballet
m s
||
yl
DATE May 2
"
i n g " if
j| Hi Lena,

a k
jj
e
Thanks so much for the invitation! I would love to go with you to the ballet premiere. Fraulein Danza

sp
| Company has done amazing work at the shows I've seen before, so I'm sure this performance will be !

ic
| fantastic as well. Besides, I'm a really big fan of Sofia Pinsky. You must have had a lot of fun interviewing |j
|
e
her and Igor Petrovich at their studio yesterday.

//to
[I _ |
j| I can meet you at the theater at 7:30 on May 9 if you want, or maybe we can get together earlier for
3
I
p :
dinner. There is a nice place right next to the Gladstone Theater called Carlotta's Bistro. Let me know if
you'd like to do that.
t
j

ht
I I
3 Thanks again. I am really looking forward to it!
i i
3 Shonda

Prima Ballerina: Sofia Pinsky


The main dancer of this evening's premiere performance, Dance of the Daffodils, is prima ballerina Sofia Pinsky
from Ukraine. Ms. Pinsky plays the title character of Princess Daffodil, leading a troupe of 30 professional
dancers. She trained at the prestigious Kiev Classical Dance Academy and went on to study under famed dancer
Alexi Petrov in Moscow. Under his instruction, she entered the European National Ballet Competition in
St. Petersburg and won second place, which was the first trophy of her career. She has been the lead dancer in
performances in cities around the world, including London, New York, and Tokyo. Ms. Pinsky joined the Fraulein
Danza Company two years ago and has been performing for them ever since.

166 *||g Hackers.co.kr


189. What is suggested about Shonda Dixon?
186. What is NOT indicated about the event
on May 9? (A) She will see Ms, Pinsky perform the
role of Princess Daffodil.
(A) It will start 30 minutes before a
performance begins. (B) She will contact Fraulein Danza
Company about a ticket.
(B) It has a suggested dress code for
guests. (C) She does not plan on attending a
reception.
(C) It requires attendees to present
invitations. (D) She read a positive review of Dance of
the Daffodils.
(D) It includes a reception with dancers
at 8:00 p.m.
190. Where did Ms. Pinsky win her first dance
award?
187. What is true about Carlotta's Bistro?
(A) It stays open 24 hours a day.
(B) It is situated near a performance
(A)
(B)
In Kiev
In St. Petersburg
m /
venue. (C) In Moscow
c o
c.
(D) In London
(C) It will be catering a reception.
(D) It provides discounts to theater

g o
n
patrons.

188. What did Ms. Reid do on May 1 ?


m s
(A) Attended a premier performance

i n g
(B) Met with two lead dancers from a show

a
(C) Wrote a review of Dance of the Daffodils k
e
(D) Confirmed her attendance for an event

ic sp
e
p : //to
ht t

GO ON TO THE NEXT PAGE..

TEST5 PART? 167


Questions 191-195 refer to the following announcement, review, and registration form

Dynamic Performance
Improve your acting skills at the region s top acting school by enrolling in one of our spring or summer
courses! We have programs for both our regular acting students and for anyone with an interest in
learning the craft.
Spring Intensive
A comprehensive course designed to teach the fundamentals of acting. Learn to read scripts, use vocal
techniques, and move dynamically. Runs from May 1 to July 1. Available at all campuses.
Tuition: $2,250
Summer Intensive
Similar to the Spring Intensive, with the addition of classes in improvisational techniques. Students will
learn to respond to other actors without a script. Runs from July 5 to August 13. Available at all
campuses.
Tuition: $3,495
m /
Summer Youth Ensemble
c o
c.
Explore the world of dramatic acting through a fun approach. Open only to students aged 15 to 18.

o
Runs from July 11 to August 5. Stamford and Brookline campus only.
Tuition: $2,500

n g
s
Summer Bridge Program
Designed exclusively for advanced students enrolled in our full-time acting program, this three-week

m
course will deepen your understanding of acting. Available at Charleston campus only. Runs from July
g
10 to July 31.
Tuition: $1,800.
k i n
a
To apply, e-mail your registration form to admissions director Floyd Mink at f.mink@dynamicperform.com.

e
ic sp
e
//to
tp:
•V
www.uspeak.com

ht HOME | ABOUT | REVIEWS


Category: Short Programs > Acting > Dynamic Performance
| REGISTER | FAQ

Awesome program!
by Cody Norris
Best three weeks of my life! Got the classes as a birthday present and couldn't have been happier Learned a
lot about acting and made a ton of new friends. Looking forward to enrolling full-time.,. More
Not a bad experience ☆☆
by Liz Hershowitz
My expectations were high for the Spring Intensive given the overwhelming number of positive comments
about it on this site. Overall, it was a good experience, but 1 wasn't fully satisfied with the instructor.. More
Mostly good
by Mandy Berger
1 generally enjoyed myself quite a bit and am glad 1 enrolled. 1 think i benefited from some of the course work
and liked everyone in the class, but 1 felt that the improvisation classes were a little outdated... More
—! Ty
bi mm H

168 -ysq a2j:a^• tfoicPIS" MP3 Hackerslngang.com


Dynamic Performance
Registration Form
Date: July 4
Name: Hannah Boyle
Telephone: 555-2092
Address: 410 Fayette St, Savannah, GA 31405
E-mail: hannabee@mailhaul.com
Which program are you interested in?
□ Spring Intensive □ Summer Intensive
□ Summer Youth Ensemble S Summer Bridge Program
How did you first hear about us?
m /
□ Online □ Print 0 Mail □ Referral

c o
c.
What do you hope to achieve by participating in a program?
I want to receive professional acting advice.

g o
s n
191. For whom is the announcement most likely
intended?
g m
194. What is indicated about Mr. Norris?

(A) Parents of young children


k i n (A) He gave Dynamic Performance the
highest possible rating.
(B) Fans of a theater production

e a (B) He joined a program with a group of

sp
(C) New and experienced actors his friends.
(D) Staff at a production studio (C) He took the Spring Intensive class

e ic
192. What is mentioned about classes at
a second time.
(D) He selected a program based on its

//to
Dynamic Performance? positive reviews.

tp :
(A) They were designed by a well-known
195. What is suggested about Ms. Boyle?

ht
acting coach.
(B) Only those who pass an audition can (A) She has been in several theatrical
take part in them. productions.
(C) Floyd Mink receives the applications to (B) She paid her tuition fee for a class
register for them. online.
(D) They are being offered in the summer (C) She will be taking a course in
for the first time. Charleston.
(D) She is ineligible for Summer Youth
193. How much was the class Ms. Berger took? Ensemble.
(A) $1,800
(B) $2,250
(C) $2,500
(D) $3,495

GO ON TO THE NEXT PAGE,

TESTS PART 7 169


Questions 196-200 refer to the following advertisement and e-mails.

FOR SALE: Kitchen Appliances!

I am moving into an apartment unit that already has kitchen appliances, so I am looking to sell the following
items:

Cucina Stove/Oven (model #CS4095834). This stainless steel appliance was purchased three years ago but
is still in good condition. It has a 4-burner range and a full-size oven. Asking $300 (but willing to negotiate).

Lava-Sud Dishwasher (model #40958374850). This dishwasher was purchased two years ago but is in good
condition. It has two racks for dishes in addition to cutlery slots. Asking $250 (but willing to negotiate).

Preserve-Mate Refrigerator (mode! #R-2343209). It was purchased a year and a half ago but is in excellent

freezer, fridge, and a dispenser for water and ice. Asking $l ,500 (but willing to negotiate).
m /
condition. It is still under warranty for another six months and has no damage. It is stainless steel and has a

c o
c.
The items must be removed from my current apartment by July I. Send an e-mail to Laurie Henner at
lauriehen@mostmail.com for details.

g o
s n
TO Lance Volstead <lancev@postaway.com>
g m
FROM
SUBJECT
Laurie Henner <lauriehen@mostmail.com>
Re: Query about advertisement
k i n
DATE May 29
e a
Mr. Volstead,

ic sp
e
Thank you for responding to my advertisement. The refrigerator is still available. As for the measurements

//to
you asked about, you can check out the precise size of the model on the Preserve-Mate Web site. But from
the way you described your kitchen, I think there will be plenty of space for the refrigerator.

tp :
Finally, I will accept your offer of $1,300 under one condition; that you pick up the item from my

ht apartment within the next two days as I'm moving out quite soon. Please let me know if you want to
proceed.
Regards,
Laurie Henner

TO Laurie Henner <laurlehen@mosnostmai


FROM staway.
Lance Volstead <lancev@posta\
SUBJECT >ement
Re: Re: Query about advertlsem
DATE May 29

Ms. Henner,
Thank you for getting back to me so quickly. The measurements will work perfectly for me. I'm fully

170 Hackers.co.kr
booked tomorrow, but I can come by the day after, I could be at your building by 4 p.m. If that time
doesn't work, let me know when you are available.
Also, could you possibly send me your phone number? That way, I can call you if I'm delayed or
something comes up. My number is (402)555-3049, Please let me know your apartment number, too,
I will bring a cart to move the refrigerator, and my friend will accompany me to give me a hand. If there
is any particular place I should park my truck, you can notify me of that as well.
I will bring cash and pay you right away, I hope to hear from you soon.
Lance Volstead

196. What is suggested about Ms. Henner? 199. According to the second e-mail, what has
Ms. Henner NOT been asked to do?
m /
(A) She is willing to consider lower prices

c o
c.
than listed. (A) Provide her availability
(B) She is preparing to remodel her (B) Give out contact information
kitchen. (C)
o
Tell about a parking situation
g
(C) She purchased all her appliances
online.
(D)

s n
Get a friend to help move an appliance

(D) She has owned her home for three

g m
200. What is indicated about Mr. Volstead?

n
years. (A) He will visit Ms. Henner's home during

k i his vacation.
197. When did Ms. Henner buy the appliance
Mr. Volstead is interested in?
e a (B) He will be moving into an apartment

sp
on May 31.
(A) Less than a year ago (C) He will hire someone to transport his
(B)
(C)
e
Two years agoic
One and a halt years ago purchase.
(D) He will give Ms. Henner $1,300 in

//to
(D) Three years ago cash.

tp :
198. According to Ms. Henner, what information

ht can be found on the Preserve-Mate Web


site?
(A) A product's exact size
(B) The price of some merchandise
(C) A product warranty's conditions
(D) The availability of a specific model

This is the end of the test. You may review Part 5, 6, and 7 if you finish the test early.

FT
^ p.3E5 / p.327 / oH^ p.361 / Part 5&6 6^ M7| IM
* cm miomoii ?ij= spif mau xkAiqi °t|| =01 ikajtu

TEST 5 PART 7 171


1000X11 3 READING

m /
c o
o c.
n g
m s
i n g
a k
e
sp
I TEST 06

e ic
p ://to PART 5

ht t PART 6

§ PART 7

Self ||3

1.°EH^o|^#mLfm? □ oj]
2. Answer Sheet, oil, T|^m ^u|5H^o? n o||
3.A|7j|l^H|S^o? coil
□- 2hSE|^ODi [CHI 1 El)AE^ A|^L|EK

§A1| #0|#mife ApJ^ 70g °e! _A| _fe?JL|EK


EH AE A§^ # ys^oini, AI^ gs 2 sa si H □m# ?ioii II-^&LIEK
READING TEST
In this section, you must demonstrate your ability to read and comprehend English. You will be given a
variety of texts and asked to answer questions about these texts. This section is divided into three parts
and will take 75 minutes to complete.
Do not mark the answers in your test book. Use the answer sheet that is separately provided.

PARTS
Directions: In each question, you will be asked to review a statement that is missing a word or phrase.
Four answer choices will be provided for each statement. Select the best answer and mark the
corresponding letter (A), (B), (C), or (D) on the answer sheet.

© PARTS iFEt #o|A|y ng

m /
101. Ms. Berkin nominees their invitations
o
105. Local transport regulations permit city
c
c.
for the Outstanding Citizenship Awards buses to have 32 seats for
Gala last week. passengers.

g o
(A)
(B)
sends
sent
(A)

s
(B)
as for
up to n
(C)
(D)
is sent
sending
g m
(C)
(D)
next to
regardless of

k i n
a
102. Patients undergoing a complete medical 106. In response to the feedback that her
examination are asked
e
eating solid manuscript was too lengthy, author Dawn

sp
food for 12 hours prior to their appointment. Wentz the number of pages.
(A)
(B)
avoid
to avoid
e ic (A) reduction
(B)reduce

//to
(C) avoiding (C) reduced

tp :
(D) avoidably (D) are reducing

ht
103. Strong winds knocked a tree down on the 107. Zoo-Croft Fashion's corporate income and
road and created a potential for expense documents were delivered to the
speeding motorists. outside accountant to get an unbiased
(A) treatment financial
(B) hazard (A) analysis
(C) expectation (B) analyst
(D) deliberation (C) analyzed
(D) analyze
104. The finance director noticed that the
reports had submitted to the board 108. Mr. Cantrell realized he had overbilled the
were not the final version. client and contacted her to explain
(A) she the error.
(B) her (A) prompt
(C) hers (B) prompted
(D) herself (C) promptly
(D) prompts

174 Hackers.co.kr
109. According to the survey, a greater number 115. Southern Mutual Insurance clients to
of residents than ever before in the sign up for long-term plans as they have
city's recycling program. lower rates.
(A) participation (A) was encouraged
(B) are participating (B) encouraging
(C) participating (C) encourages
(D) participants (D) is encouraged

110. Just one day after announcing that a new 116. The Loughton Youth Club offered to
factory would be opened, KerbCo help organize a community cleanup day
Manufacturing received applications and to recruit local volunteers. —1
m
i r\
for jobs. H
(A) manually
(A) several (B) arguably g
(B)none
(C) every
(C)
(D)
suggestively
generously
m / o
r\j
(D) each
117. Mr. Pullman has asked for
. c o
time to
o
LU

oc
111. After three months of intense negotiations, turn in his report because he requires o
•£-
for the construction of new power
plants were signed.
n g
some additional information.
O
LTI
s
(A) any
(A) agreed (B) a few
(B) agreeable
g m
(C) more
(C)
(D)
agreements
agreeably
k in (D) many o

e a 118. Mayor David Lee's introduction of Senator o


CO

updated business centers are


s p
112. Reviews of the Mardison Hotel chain's Laura Moncton her speech, giving
an overview of her track record of political o
LD
positive.

e ic reforms.
o

//to
(A) overwhelming (A) required
(B) preceded or
(B) overwhelmingly

p
(C)

t : overwhelmed (C) performed


3
D-
Kn

ht

(D) overwhelm (D) accomplished Kt
r*
oo
113. Mr. Holden's outgoing is beneficial to 119. Album sales for rock band Trifecta have o
LU
him as his job requires a lot of networking been low their concert tickets have 73
and socializing. been selling out. DJa.
OQZ3
(A) person (A) if only
(B) personality (B) as long as
(C) personable (C) even though
(D) personally (D) provided that

114. Mr. Henry gave his staff additional training


on the new office policy, but several of
them were still
(A) confusing
(B) confused
(C) confuse r*r\ nhi Tn
UU Ur/ IU TUP mpvt/ PA HP
/nc /VcA
(D) confusion

TEST6 PARTS 175


120. Ms. Davies could only hear of what 126. Due to their limited experience in Asia,
was discussed since the restaurant KDM's executives made the choice
chosen for the meeting was so noisy. to collaborate with a well-established firm
in the region.
(A) fragments
(B) excerpts (A) sense
(C) summaries (B) sensing
(D)shares (C) sensible
(D) sensibly
121. There were malfunctions with the
agency's updated software, so management 127. The media have named several
decided to use the old version temporarily. medications that can be dangerous when
(A) competent taken without a doctor's prescription.
(B) cautious (A) inconveniently
(C)
(D)
persistent
imperative
(B)
(C)
expressly
supportively
m /
(D) comparably

c o
c.
122. In an emergency, an alarm in Hamley
Towers will sound, indicating that everyone 128. Jessup Inc. chose Bill Torres to represent
inside must the building immediately.
g o
the firm in an upcoming meeting his
(A) oppose
(A) n
success in past negotiations.

s likewise
m
(B) dismiss
(C)
(D)
vacate
assemble
i n g (B)
(C)
instead of
apart from

a k (D) given

e
123. The accommodations at the Grandview

sp
Hotel seemed less than when 129. The machinery in Templex's factory has
compared to the attractive, reasonably become so outdated that funds have been

ic
priced options in town.
e
to purchase new ones.

//to
(A) desirable (A) exempted

:
(B) responsible (B) hosted

tp (C) extensive (C) conducted

ht
(D) unlimited (D) designated

124. Madsen Industries' new microwave ovens 130. The manager did his best to make
are now available high-quality decisions that took the finance division
kitchen appliances are sold. members' suggestions into
(A) by the time (A) recommendation
(B) as soon as (B) calculation
(C) wherever (C) appliance
(D) throughout (D) consideration

125. The sequel to last year's film Galactic


Express made $400 million worldwide,
is more than the first movie earned.
(A) this
(B) what
(C) whose
(D) which

176 my SojUAf • ^ eoiopl MP3 Hackerslngang.com


PART 6
Directions: In this part, you will be asked to read four English texts. Each text is missing a word,
phrase, or sentence. Select the answer choice that correctly completes the text and mark the
corresponding letter (A), (B), (C), or (D) on the answer sheet,

© PART6 aStSoUPJ 8S

Questions 131-134 refer to the following letter.

Dear Ms. Clark,

Thank you very much for your gift to The NewEd Foundation. As you requested, your ------
lu I ■
m /
will support the education of children enrolled in public school systems in the local area.
c o
o c.
At NewEd, we work to provide educational institutions with funding for after-school programs,

n g
materials, and training for instructors. We also strongly ------ salary increases for teachers

and specialists employed at educational institutions.


1 Ol..

m s
i n g
a way of thanking you for supporting our cause, The NewEd Foundation welcomes you

a k
e
to join our annual conference on science education. I have enclosed an invitation for you and

a guest.
134.
ic sp
e
//to
Thank you again for your ongoing financial support of our organization.

tp :
Kindest regards,

ht
Marianne Lexington
Program coordinator, The NewEd Foundation

131. (A) expertise 134. (A) Simply present it at the entrance on


(B) volunteering the day of the event.
(C) establishment (B) We appreciate your offer to give a talk
(D) contribution at this fundraiser.
(C) Those who attended agree that the
132. (A) assess event was a great success,
(B) restore (D) Provide the volunteers with instructions
(C) promote when you arrive.
(D) place

133. (A) Due to


(B) As
(C) By GO ON TO THE NEXT PAGE,
(D) Even if
TESTS PARTS 177
Questions 135-138 refer to the following e-mail.
|A||
OS!
To; Andrea Gershon <a.gershon@wwconsult.com>
From: Takeshi Miyagi <t,miyagi@wwconsult.com>
Date: Friday, October 10
Subject: Furniture delivery

Flello Andrea,

I ordered a new chair and desk on October 5 to replace the ones I've been using since I

started working for the company. A couple of days ago, I received an e-mail from the

manufacturer stating that the items It said that they had been shipped and delivery was

expected within the week.


135.

m /
c o
c.
I have yet to receive a call from the delivery person, I suppose I won't get my furniture

o
until next week.
137. g
When the delivery comes, could you please sign for me and then call

n
s
Mike Flarrison in the maintenance division? Fle's responsible for putting furniture together and

m
removing
138.
i n g
items, so he will get everything set up in my workspace.

Thanks a lot.
a k
e
Takeshi

ic sp
e
135. (A)
//to
had arrived 137. (A) I didn't expect project to be delayed by

tp:
(B) have arrived Wm as much as one week.

ht
(C) will be arriving (B) As soon as the items are ready, send
(D) arrived them immediately.
(C) Unfortunately, I will be out of the office
next week due to a conference.
(D) The new desk and chair are now in the
storage room.

138. (A) immeasurable


(B) nonessential
(C) imprecise
(D) impartial

178 Hackers.co.kr
Questions 139-142 refer to the following memo.

MEMO

DATE: March 31
TO: All operations employees
FROM: Robin Evans, Chief of operations
SUBJECT: Reports review

Last week, I distributed a report detailing the department's performance in the first quarter.

There are some items in the report that I think should be addressed.

First of all, it seems that costs in the maintenance department have


139.
since last quarter. I

m /
o
have asked Terrence Howard's team to provide some details related to specific increases in

c
c.
maintenance expenses. Also, I am quite surprised that certain project ----- have not been

o
met.
141. g
Lastly, inventory levels are currently lower than projected, I would like to set up a

n
m
142.s
meeting with Carmine Gosford's group to discuss our ----- for managing inventory going

forward.

i n g
a k
We will be doing other performance reviews throughout the year, and I hope that this will help

e
sp
us become a more efficient and productive department.

e ic
//to
139. (A) stayed over 141. (A) I understood the feedback and will

tp :
(B)gone up
(C) dropped down
|||i| implement your suggestions in the
future.

ht(D) evened out

140. (A) objected


(B) The only thing left to do is evaluate
these concluded sessions.
(C) Project managers should act on them
(B) objectively as soon as possible.
(C) object (D) Fortunately, everyone on the team was
(D) objectives able to meet the revised deadline on
short notice.

142. (A) confirmation


(B) attention
(C) procedure
(D) registration

GO ONTO THE NEXT PAGE

TEST 5 PART 6 179


Questions 143-146 refer to the following e-mail,
l*r
OS]
To: Marilyn Farris <marilyn_farris@madmail,net>
From: Lara Smith clara.smith@lpranalytics.com>
Date: September 22
Subject: Job application

Dear Ms. Farris,

Thank you for your interest in the position of technology adviser. We would like to schedule

an ----- between 3 p.m. and 4 p.m. next Thursday. We hope to find out more about your

qualifications and experience in relation to the role you have applied for.

m /
o
In addition to photocopies of ---—- certificates, we ask that you arrange for three references

c
to be sent to us in sealed envelopes.
145.
c.
We want to hear about past experiences which

o
are relevant to the position.
146. g
, we will talk about what your responsibilities will be should

n
you get the job.

m s
i n g
Please respond to this e-mail to confirm your availability. We look forward to meeting with you

a k
e
Sincerest regards,

ic sp
Lara Smith
e
//to
Hiring manager, LPR Analytics

tp :
ht
143. (A)
(B)
(C)
examination
interview
assignment
145. (A) We were unable to contact one of your
mm listed references,
(B) Please also be prepared to give a short
(D) investigation presentation about your career,
(C) I will send them to you using the
144. (A) valid address you provided.
(B) inaccurate (D) The information provided a lot of useful
(C) optimistic material for our discussion.
(D) secretive
146. (A) Afterwards
(B) Nevertheless
(C) Therefore
(D) Conversely

180 ?sS2J°!^£e|S^mp90®7|g5iB0i°PIMP3 Hackerslngang.com


PART 7
Directions: In this part, you will be asked to read several texts, such as advertisements, articles, instant
messages, or examples of business correspondence. Each text is followed by several questions. Select
the best answer and mark the corresponding letter (A), (B), (C), or (D) on your answer sheet.

© PART 7 m #01 WJ 11 54^

Questions 147-148 refer to the following advertisement.

/
SOFTWARE CLASSES AT BRAMBLE LIBRARY

o m
Do you want to be more tech-savvy? Are you interested in learning how to
create spreadsheets, presentations, and other common documents? If so, stop
c. c
g o
by Room 303 at the Bramble Library on Tuesdays at 8 p.m. between March 2

s n
and April 15. Richard Hernandez, a trained IT executive, will be teaching people
how to make spreadsheets, graphs, and presentations with frequently used

g m
software applications. Classes cost $20 per session. To sign up, visit www.

i n
bramblelib.org/programming/software. A complete schedule for the classes will

k
be posted shortly.

e a
sp
To learn more about the educational opportunities we offer, from literary

ic
classes to sessions on public speaking, please visit www.bramblelib.org/

e
programming.

p : //to
ht t
147. Why was the advertisement written?
(A) To advertise a Web page design
148. What will Richard Hernandez discuss?
(A) Making effective presentations
service (B) Studying literary classics
(B) To promote some recently updated (C) Writing informative speeches
software (D) Repairing electronic devices
(C) To publicize a course at an
establishment
(D) To encourage enrollment in an online
class

GO ON TO THE NEXT PAGE.

TESTS PART 7 181


Questions 149-150 refer to the following text message chain.

Ool

DeanStnvers [10:31a.m.]
11 Sylvia, are you busy? Upper management just requested
ll an urgent meeting with the research department in
about an hour.

Sylvia Flores [10:33 a.m.]


I'm available, but my team is at one of our main
1
branches downtown. They won't be back until at least
2:00.
Jl Dean Strivers [10:35 a.m.]
Well, do you think you could handle it yourself? I think
m /
it'll go well even if your team members aren't there.
c o
Sylvia Flores [10:36 a.m.]

o c.
g
It shouldn't be an issue. Do I need to prepare anything?

|| Dean Strivers
s n
[10:40 a.m.]

m
They want to ask questions about our upcoming product

i n g
launch, so bring the market analysis, financial estimates,
and other reports that you and your team have worked

a k
on. The meeting will be in Conference Room 510.

e
sp
M
|j Sylvia Flores [10:41 a.m.]
I'll be there.

e ic
p : //to
ht t
149. At 10:36 a.m., what does Ms. Flores mean
f||| when she writes, "It shouldn't be an
issue"?
150. What has Ms. Flores been asked to
prepare?
(A) Some product samples
(A) She is pleased with the progress of the (B) Videos from an event
product launch. (C) Some business documents
(B) She cannot deal with an urgent (D) A list of potential clients
problem.
(C) She will call back some coworkers.
(D) She is capable of managing a
discussion alone.

182 ifSBWSHaSMJlg Hackers.co.kr


Questions 151-152 refer to the following invoice.

Harwood Services
3372 South High Street
Bloomington, Indiana, 47404

For attention of: Johnathan Mackay, Midwest Consulting Group, 105 East Hillside Drive,
Bloomington, Indiana, 47403

Date: March 15

Reference: MCG104

Services: Providing the following at the Indiana Agricultural Subsidy Seminar on March 10:
• Soup and sandwich lunch, soft drinks/fruit juices included
$350, for 27 guests
m /
• Two refreshment breaks with tea, coffee, pastries, and snacks
c o
$280, for 27 guests
• Evening bar/cocktail service, trays of appetizers
o c.
$220, for 35 guests*

n g
• Serving and preparation staff for duration of event
$240, three staff members
m s
Total cost: $1,090
i n g
Deposit prepaid: $500
a k
Amount due: $590
e
ic sp
* NOTES: Please note that there were eight extra people for the evening event at the conclusion of

e
the seminar. As you had previously indicated that there might be some additional executive guests,

//to
the price has been adjusted accordingly as per our agreement,

p :
All prices include tax. Payment is due within one week of receiving this invoice. Late payment may

t
ht
incur a penalty as noted in the contract for our services. Should you have questions regarding this
invoice, please call us at 555-3920.

151. What type of company most likely is 152. What is indicated about the Indiana
Harwood Services? Agricultural Subsidy Seminar?

(A) A beverage manufacturer (A) It featured two complete meal services.


(B) A wedding planning business (B) It included more than one intermission.
(C) A catering company (C) It was not charged extra for service or
(D) A fresh produce supplier preparation staff.
(D) It was attended by less individuals than
expected.

GO ON TO THE NEXT PAGE^

TEST 6 PART? 183


Questions 153-155 refer to the following memo.

^ MEMORANDUM

Date: January 27
To: All marketing and communications employees
From: Charles Mikkola, Head of Marketing and Communications
Subject: Organizational change

As many of you are already aware, very shortly I will be leaving my position as
the head of the marketing and communications department here at Thumston
International. I'm proud to say that during my time in this position, Thumston
International has made some tremendous advancements. Working together, we
have enhanced our customers' experiences, expanded service offerings,
strengthened relationships with new partners around the globe, increased
m /
revenues, and made our business practices more standardized,

c o
I am pleased to announce that Ms. Hannah Orburg will be taking over my

o
position as of February 1. Hannah has more than 15 years of experience in c.
n g
sales and marketing for businesses in the corporate consulting field. Prior to

m s
being recruited by Thumston International, Hannah served as the Chief of
Marketing and Operations at Diskledge, a large firm offering technological

i n g
consulting to over 500 corporate clients internationally. With such stellar

k
achievements relevant to our main business operations at Thumston, I'm

a
confident that Hannah is the ideal person to replace me.

e
sp
Once again, it has been truly wonderful working with you all, and I wish you all

ic
the best in your professional endeavors.

e
//to
tp:
153. What is being announced in the memo? 155. In what industry is Thumston International
most likely involved?

ht
(A) An executive change for a company
division (A) Technical staff recruitment
(B) A firm's operational plan (B) Global financial investments
(C) A temporary supervisory staffing (C) Computer hardware marketing
change (D) Corporate advisory services
(D) An available managerial position

154. What did Mr. Mikkola do for Thumston


International?
(A) Expanded its offices in various
locations
(B) Formed stronger international
partnerships
(C) Increased total customer numbers
(D) Helped it adopt new technology

184 ¥SSst?!i!sejH°)iuhE.K)(°f7®si90|°p|MP3 Hackerslngang.com


Questions 156-157 refer to the following article.

Art Council to Host Summer Street Festival

Back by popular demand, the Los Ramos Art Council (LRAC)


will once again host Wine, Dine, and Design this year on July
12 and 13. The street festival will take place in downtown Los
Ramos in the well-known Franklin Avenue neighborhood, This
year, more than 120 local artists and artisans have registered to
showcase their work during the festivities.

But the fair isn't just for art lovers — it's for food and wine
enthusiasts as well. Wine tastings will be set up by 12 different
local vineyards, and some of the city's most popular chefs will
also be on hand to serve delectable regional treats.
m /
c o
c.
Spokesperson for the LRAC Rosita Santiago said that popular
folk music group Bella Vista Band will perform during the final

g o
evening of the fair along with many other artists. "In addition to

s
days. So, you're sure to find something you'll enjoy!" n
music, dancers from local schools are set to perform on both

g m
A complete calendar of specific activities, vendors, and events

i n
is available at the LRAC office or at www.losramosartcouncil.
k
e a
org. The street fair is free of charge, with activities starting at
11 a.m. daily and concluding at midnight. Extra parking will be

ic sp
made available at Los Ramos Middle School.

e
: //to
156. What does the article indicate about the

p
157. What will take place on July 13?

ht t
Los Ramos Art Council?
(A) It will host a series of cooking courses.
(B) It organizes a community festival.
(A) A festival in a downtown area will be
canceled.
(B) A group will play some music during
(C) It offers financial assistance to local a celebration.
craftspeople. (C) A list of vendors will be distributed to
(D) It is affiliated with a national organization. visitors.
(D) A short talk will be given by an LRAC
representative.

GO ON TO THE NEXT PAGE.

TEST 6 PART? 185


Questions 158-160 refer to the following schedule.

Daily schedule for: Melissa Traynor


Managing Editor, The Green Bay News

Thursday, April 19
9:30 A.M. Meet with Harold about last month's advertising revenue
10:30 A.M. Talk to Ray about upcoming series of articles on state finance
11:00 A.M. Give a short training session to an incoming editor
11:30 A.M. Look at tomorrow's scheduled stories before today's lunch meeting at Mariano's
12:00 P.M. Grab lunch with Sarah to talk about her new layout idea
2:00 P.M.
3:00 P.M.
Talk with Greg about the sources for his new feature article
Check e-mails, return missed calls
m /
c o
c.
4:00 P.M. Call Peter to arrange a place to meet
5:00 P.M. Meet with all editors about tomorrow's issue

g o
6:00 P.M.
8:00 P.M.
Review articles
Meet with Tim about copy editing
s n
9:00 P.M. Send the newspaper to be printed
g m
k i n
Overtime hours from 6 p.m. to 9 p.m. have been approved by management. Should you be unable to

e a
make any of the appointments, please notify your assistant, Joel Heartley, who will deal with any

sp
necessary rescheduling.

e ic
Please note that there will be no schedule for you tomorrow, as you will be leaving on your
business trip to New Delhi.

p : //to
ht t
158. What is suggested about Ms. Traynor?
(A) She was promoted to copy editor.
(B) She is creating a new layout design for
160. Why will Ms. Traynor contact Peter?
(A) To set up a meeting
(B) To offer feedback on an article
a magazine. (C) To respond to his e-mail
(C) She is responsible for checking articles (D) To report an appointment cancellation
prior to publication.
(D) She writes feature stories for a local
newspaper.

159. What is Ms. Traynor supposed to do at


Mariano's?
(A) Review next month's stories
(B) Meet with Tim about upcoming issues
(C) Have a work-lunch with Sarah
(D) Talk about sources with Greg

186 sans-sagMJig Hackers.co.kr


Questions 161-163 refer to the following e-mail.

TO: Fatima Sukarno <sukfat@ballnesiaindustries.com>


FROM: Vinnie Deniro <vlnde@vertbIeu.com>
SUBJECT: Upcoming visit
DATE: January 29
ATTACHMENT: product specifications

Dear Ms. Sukarno,


Thank you for responding so quickly to my inquiry about visiting your manufacturing plant in
Jakarta. I will ask my assistant Theresa Montovani to go ahead and make travel arrangements
for me and my colleague Sheryl Stein to travel there from New York on February 24.
As Vert-Bleu Beauty is interested in pursuing a production contract with your company, I think it
is important that we meet with you and see your facilities in person. — [1] —.
m /
c o
c.
Our cosmetics are guaranteed to be completely natural and environmentally sound, so our
production guidelines would be rather strict. — [2] —. I am attaching a list of specifications for

g o
the manufacturing process of our beauty products. You and your executive staff can have a look
through them and see if everything is clear.

s n
g m
Once our flights have been booked, Ms. Montovani will send you, or your secretary, a final
itinerary. — [3] —. We thank you kindly for offering us hotel accommodation and a driver

i n
service during our stay, and gratefully accept. — [4] —. Our assistants can coordinate to work

k
a
out any details.

e
I am looking forward to meeting with you, and I do hope that we will be able to work together
on this venture.

ic sp
e
Regards,

//to
Vinnie Deniro

tp :
Product development director, Vert-Bleu Beauty

ht
161. Who most likely is Fatima Sukarno? 163. In which of the positions marked [1], [2],
(A) A distribution firm executive |||| [3], and [4] does the following sentence
(B) A travel agent best belong?
(C) A factory representative "This will also help you get a better idea of
(D) A beauty item salesperson our needs and vice versa."

162. What is mentioned about Vert-Bleu (A) [1]


Beauty? (B) [2]
(C) [3]
(A) Its headquarters is located in Jakarta.
(D) [4]
(B) It plans to build its own manufacturing
plant.
(C) It follows strict production guidelines.
(D) It is unprepared for an upcoming
inspection. GO ON TO THE NEXT PAGE.

TEST 6 PART 7 187


Questions 164-167 refer to the following online chat.
Al
Ool
1

Helen Gatlneau [3:35] I'm sure you've all heard that we're going to be merging with Fresh Food
Wholesalers at the end of next month. We will have a meeting to discuss
transition plans next week. But in the meantime, I will try to answer questions
you may have about the merger.
Leo Kinghorn [3:36] Everyone in the purchasing department is wondering how we will be affected
by the change. Are there any firm details available?
Mimi Hanway [3:38] It's my understanding that administrative functions will be carried out in only
one building for efficiency's sake. As a result, administrative personnel,
including me, will be moving to the Fresh Food Wholesalers' building in the

Helen Gatineau
coming weeks.
[3:39] That's right. But, we are going to maintain separate warehouse facilities at
m /
their present locations.

c o
Jude Mason [3:41] Any idea how that will change my routine?

o c.
Helen Gatlneau
g
[3:42] You'll stop at both facilities before taking your deliveries to the retail store,

n
Jude. We'll hire additional warehouse workers to speed things up so you have

s
time to visit both spots. They will help you load items.

m
Leo Kinghorn

ng
[3:45] You know, the workers at Fresh Food Wholesalers will probably need to take a
health and safety seminar before they can handle meat. I believe that Fresh
i
k
Food Wholesalers hasn't dealt in meat products yet.

a
Helen Gatlneau
e
[3:47] That makes sense. I'll arrange for instructors to visit as soon as possible.

ic sp
e
164. What is the online chat mainly about? 166. In what department does Mr, Mason most

//to
(A) Adjustments for a change of operations likely work?

tp :
(B)
(C)
The reassignment of job duties
Advantages of a corporate merger
(A)
(B)
Human Resources
Administration

ht (D) A visit to a new office building

165. What are some personnel required to do?


(C)
(D)
Marketing
Shipping

(A) Contact a list of clients 167. At 3:47, what does Ms. Gatineau most
(B) Revise some company policies likely mean when she writes, "That makes
(C) Relocate to a different office sense"?
(D) Arrange a warehouse visit (A) She agrees that some workers will
need to attend a class.
(B) Her knowledge of meat products could
be improved.
(C) She is willing to consider proposals to
expand a product line.
(D) Her team can organize a welcoming
event for new staff members.

188 a^UTAt. E!0)°p|9 % 90i?PI MP3 Hackerslngang.com


Questions 168-171 refer to the following foreword of a book.

Starting Up
A Foreword by Michael Lawman
Starting Up is Anne Schubert's first nonfiction publication, tracing the events in her life that led her to
become a successful businessperson. Schubert reminds us through her compelling life story that
creating a start-up company is not something that one can do without planning, — [1] —. Sufficient time
and hard work are both necessary to make one's own business profitable, and the process of how to do
that is outlined in detail throughout this book,
Schubert discusses how even from a young age she had always thought herself something of an
inventor. Trying to come up with original products that others would find useful in their daily lives gave
her inspiration. — [2] —. She also had to overcome the multiple challenges that go along with
promoting a new product and ultimately landed contracts to supply Venus Footwear and Constant
Sports Equipment,
m /
c o
And it didn't take Schubert long to open a start-up company in Seattle called Swift World, which

c.
eventually led to her becoming CEO of Swift Enterprises International, Now with over 280 shoe stores

g o
across six different countries, we learn in the book how Schubert came from nearly nothing and
developed Swift Enterprises into one of the nation's most lucrative retailers. — [3] —.

s n
Not only is Starting Up incredibly insightful and educational for entrepreneurs, but it is an entertaining

g m
read with lots of personal stories about the highly competitive world of product development and retail.
I've been fortunate enough to be Schubert's professional peer for many years and have benefitted a lot

i n
from her insight and intuition. — [4] —, Now, with Starting Up, you too can have a look inside
Schubert's world of start-up success.
k
e a
ic sp
168. What is stated about Starting Up? 170. What is suggested about Ms. Schubert?

e
(A) It is on an international best-seller list. (A) She has an educational background in

//to
(B) It motivated Mr. Lawman to pursue a marketing.

tp :career in business.
(C) It is about the career of an entrepreneur.
(B) She went to university with Mr. Lawman.
(C) She experienced success in retailing

ht (D) It was published prior to the opening of


Swift Enterprises.
shoes,
(D) She plans to increase the number of
her company's branches.
169. What is mentioned about Swift World?
171. In which of the positions marked [1], [2],
(A) It was developed in collaboration with
a partner. |||| [3], and [4] does the following sentence
(B) It has seen its sales levels decline in best belong?
recent years. "However, she learned quickly that idea
(C) It carries a wide selection of used creation alone was simply not enough to
sports products. bring a product to market."
(D) It expanded globally after opening in
(A) [1]
Seattle.
(B) [2]
(C) [3]
(D) [4]

GO ON TO THE NEXT PAGE.

TEST B PART 7 189


Questions 172-175 refer to the following article.

A New Hotspot for Business Travel


The western Chinese city of Chengdu has, for a long time, been overshadowed by
more famous eastern cities like Shanghai and Beijing. But with China increasingly
becoming the world's economic powerhouse, cities like Chengdu are becoming
important destinations for business travel. However, conference halls and
boardrooms aren't the only things visitors should see in this amazing and attractive
metropolis. If you have some spare time during your trip, why not go and see
some of the sights?
• The panda sanctuary is the biggest draw for tourists in the city, and the only

/
chance you'll have to see these wonderful and endangered creatures up close.
English-speaking tour guides are available, and there are around 60 pandas
including newborns. Note that the animals tend to be asleep between noon and
o m
c.
2 p.m. because of the heat. Due to its popularity, advance ticket purchases and
reservations are recommended. c
g o
• Luodai Ancient Town is a region of Chengdu that has survived the demolition

s n
and development of recent decades. It contains a number of traditional houses,
and local Sichuan street food is available at stalls. Bus and train services to

g m
Luodai are available, although the bus takes longer due to traffic.

i n
• Mount Emei is located just outside Chengdu and is one of the most famous
k
e a
mountains of Chinese Buddhism. If you have an opportunity to go, take the cable
car to the summit and enjoy a long but easy walk back down.

ic sp
• If you have several days available, your hotel will be able to help you arrange a
trip to some of the smaller Tibetan towns that surround the city. In these towns,

e
//to
you can ride horses, stay in traditional Tibetan accommodations, and visit the
many temples and libraries.

tp : All of these sights (except the Tibetan villages and Mount Emei) are easily

ht
accessed by a taxi from the center of the city, which should cost no more than
50RMB,

190 Hackers.co.kr
172. Where would the article most likely 174. The word "overshadowed" in paragraph 1,
appear? line 1, is closest in meaning to
(A) In a library brochure (A) enclosed
(B) In a history text book (B) dominated
(C) In a travel magazine (C) modified
(D) In a company newsletter (D) governed

173. What does the article suggest about 175. What is NOT mentioned as an activity
Chengdu? tourists can do?
(A) It has a large Tibetan population. (A) Trying the food in a nearby town
(B) It needs more tour agencies for (B) Taking a train to a mountain area
international visitors. (C) Observing animals with a tour guide
(C) It hosts many professional meetings. (D) Traveling to a historic area by bus

m /
o
(D) It attracts more travelers than other
nearby cities.

c. c
g o
s n
g m
k i n
e a
ic sp
e
p : //to
ht t

GO ON TO THE NEXT PAGE,

TEST 6 PART 7 191


Questions 176-180 refer to the following advertisement and e-mail.

Sunfield Limousines
San Diego's Premier Limousine Service
Now Serving The Greater San Diego Area
Reservations are available
24 hours a day, 7 days a week, 365 days a year
via walk-ins, phone, or on our Web site.
Transportation is available to over 50 areas and
neighborhoods in the greater San Diego area.
We offer airport transport, wedding and special event services, and much more!
New low-mileage sedans, SUVs, stretch limousines, and party buses are available.
m /
c o
o c.
Contact one of our professionals now

n g
to reserve your chauffeured ground transportation services today.

m s
Special Deal For First-Time Customers

i n g
Enter coupon code BN23X5 on www.sunfieldlimo.com
to receive 15 percent off your first limousine reservation.

a k
Sunfield Limousines

e
sp
35 Arroyo Drive I San Diego, CA 92093

ic
www.sunfieldlimo.com I 555-7488

e
Check out our new mobile application: SunfieldApp!

p : //to
ht t
TO: Henry Karrington <h_karrington@sunfieldlimo.com>
FROM: Fred Puccini <fpuccini6@farmail.net>
SUBJECT: Refund request
DATE: December 14

Dear Mr. Karrington,


I am writing to express my disappointment in your limousine services. As a first-time customer that was
hoping to take advantage of your company's special discount offer, I am sorry to say that I experienced
several issues with the service provided.
I had requested that a driver and stretch limousine meet some important visitors at the airport for
Saturday, December 10. However, after waiting for nearly an hour in the arrivals area, nobody picked them
up, and they were forced to call your company. They were informed that the car and driver were waiting at
the domestic terminal instead of the international one. I clearly indicated in my reservation form that the
flight would arrive at 6:30 p.m. in Terminal 1. Unfortunately, all of these delays meant our visitors waited
around for an extra hour and a half.

192 ^aA MP3 Hackerslngang.com


The driver then took our visitors to the Crown Diamond Airport Hotel instead of the branch located
downtown. This added another 30 minutes to the trip. These visitors are potential investors in my company,
and I was very embarrassed by their first impressions,
I booked and paid for a return trip from the Crown Diamond Hotel to the airport for December 16 at 10:45 a.m.,
but at this point I wish to cancel. And I would appreciate a refund for the return trip. You may send the
funds to the corporate credit card used to pay for your services,
I hope to receive a response from you shortly. Please e-mail me a confirmation as soon as the refund is
processed.
Sincerely,
Fred Puccini

m /
179. What occurred on December 10?
c o
c.
176. What is stated about Sunfield Limousines
in the advertisement? (A) The Sunfield Limousines company
(A) The driving area is restricted to
g o
sent a driver to the incorrect location.
suburban towns.
(B) Different types of vehicles are available
s n
(B) Mr. Puccini's clients were unable to
reach an airport in time for a flight.
to clients.
g m
(C) Mr, Karrington's staff did not respond
(C) Limousines can only be booked during
certain seasons.
k i n to telephone calls from visitors.
(D) The driver offered a small discount on

a
(D) Services are not provided on national
e
driving services to make up for an

sp
holidays.

ic
177. What is Sunfield Limousines now offering?

e
180. What is indicated about Mr. Puccini?

//to
(A) A program for mobile devices (A) He made a reservation on Saturday.
(B) Complimentary phone consultations (B) He has no time in his schedule to meet

p :
(C) Coupon books for returning customers

t
some arriving guests.

ht
(D) A worldwide service (C) He came to an agreement with some
corporate investors.
178. How did Mr. Puccini most likely make his (D) He will not use Sunfield for the return
reservation? trip.
(A) By telephone
(B) In person at a store
(C) On a Web site
(D) Through a friend

GO ON TO THE NEXT PAGE^

TEST B PART? 193


Questions 181-185 refer to the following notice and letter.

NOTICE: Upcoming maintenance


Posted on May i

County Line Bank's mission is to do everything we can to earn the trust and respect of our
customers. We extend that mission to our online services as well. To do this, we must
continually update our computer systems and software. Unfortunately, making these
necessary improvements requires that we shut down parts of our online banking services for
short periods of time.
In this regard, on May 22 from 5 p.m. to 10 p.m, some online banking features will become
temporarily unavailable. During this time, customers will be unable to check balances, pay
bills, transfer money, stop payments, or deposit money for checking and savings accounts
registered on our online system.

m /
We apologize for any inconvenience this may cause you. To take care of time-sensitive

c o
c.
transactions during the shutdown period, you may call our 24-hour customer service line at
1-800-555-3278. You will be required to provide your account number and may also be asked

o
security questions to verify your identity. Alternatively, you may conduct all regular

g
n
transactions available through any automated teller machine or visit any County Line branch

continued business.
m s
for personal service from one of our professional tellers. Thank you for your patience and your

i n g
a k
e
sp
May 8
y, |

e
Mr. Roger Black ic
County Line Bank Headquarters

//to
950 Crestside Drive
Q

tp :
St. Louis, MO 63112

ht
m Dear Mr. Black,
< . . . ..... S!
i am an account engineer with Datup Solutions, specializing in online systems. I have learned that you
are the director of information technology for County Line Bank. I would like to discuss briefly with
you how our company is helping clients like you to minimize their system downtime.
Datup recognizes the financial consequences for companies or institutions who regularly have to take
parts of their systems offline for maintenance, as your bank does with its site. With this in mind,
Datup's main business focus has always been to keep systems running during upgrades and upkeep.
We have been successfully helping financial institutions, insurance firms, and online retail companies
do exactly that for four years now, with most clients reporting substantial returns on their
!V| investments.
•vj i
If you are interested, I would like to meet with you and any other key personnel related to your
system s operations. Ideally, it would be a good idea to have an appointment one week before your
upgrade takes place. We can review the bank's current system, and my associate Michaela Owens
and I can explain to you face-to-face how Datup can help to improve the situation. We are available to
meet with you any day this week apart from Thursday, which is the day of a company conference. And
f\ you can also select any time between 8 a.m. and 7 p.m. Please call me at 555-6420 to make §
arrangements.

194 Hackers.co.kr
II
I do look forward to meeting with you and possibly working together with you and County Line Bank in
the future.
Sincerely,
Jason Richards
Datup account engineer

181. For whom is the notice most likely 184. What is mentioned about Datup Solutions?
intended? (A) It offers free assessments to clients
(A) Staff members at a bank using its maintenance programs,
(B) It is developing an operating system
(B)
(C)
A team of maintenance crew members
A financial institution's clients for computers.

m /
(D) A company's software developers

c o
(C) It regularly participates in technology

c.
conferences.
182. What is stated about the upcoming (D) It has an online retailer as a client.
maintenance work?
g o
(A) It is carried out on all bank systems
on a monthly basis.
s n
185. What does Mr. Richards want to do?
(A) Dispatch an assistant to perform an
(B) It will not affect the operation of
g m inspection
automated teller machines.
(C) It will take an entire day for technicians
k i n (B) Meet with Mr. Black sometime before
May 22
to complete it.
e a (C) Hold a conference call on the coming
Thursday

sp
(D) An outside service team has been
hired to deal with it. (D) Send a job application form to County

e ic
183. What most likely is Datup Solutions?
Line Bank's headquarters

(A)

p
(B)
: //to
A computer repair center
A corporate equipment retailer

ht t
(C)
(D)
An information technology company
A conference organizer

GO ON TO THE NEXT PAGE,

TEST 6 PART 7 195


Questions 186-190 refer to the following flyer, form, and review.

Segovia Paints
Here are some of our featured brands that are currently on sale:
Vlbra waterproof paint
o
Regularly priced at $24 per gallon
• Retain their color for long periods even under severe weather conditions
• Get 50 percent off pastel red and ivory when you buy at least two gallons
Densen metallic paint
• Regularly priced at $34 per gallon
• Specially formulated to give oft a deep and luxurious shine (indoor use only)
• Get 25 percent oft dark pink, cobalt blue*, amber yellow*, and emerald green
Fulmax blended paint
• Regularly priced at $30 per gallon
® Made with our computerized paint-mixing machine
m /
• Get 25 percent off any blend of two or more colors
c o
Alpha basic paint
• Regularly priced at $18 per gallon
o c.
• Eco-triendly and adheres to most surfaces

n g
• Get 30 percent off cool gray*, warm gray, and steel gray
*Co/or unavailable in other brands
m s
Need more help? Stop by one of our locations:

i n g
2490 Taylor Street
Conway, AR 72034
a k
405 Knoxville Avenue
Russellville, AR 72801
107 Division Street
Morrilton, AR 72110
(Open Mon. - Sat.)
e (Open Mon. - Fri.) (Open Mon. - Fri.)

sp
Our team is always ready to assist you! All special otters on this flyer are available until April 30 only.

ic
e
p : //to
https://www.segoviapaints.com/feedback/form

ht t Order No.: 12798-130524


Name: Samantha Eggers
E-mail address: sameggers@fastmail.com
Date: April 28
Please provide all the details about the problem you experienced, and we will answer you within
24 hours.

I recently ordered some paint on promotion from your store, but it seems that the wrong paint
color was delivered. I got cobalt blue instead of warm gray. Perhaps one of your attendants
confused the two since they have some similarities in their packaging. Could you look into this
and take whatever actions are necessary? If possible, I'd like to return the wrong item and have
the correct one sent to me without any additional charges for shipping. Thank you.

Ell

196 MP3 Hackerslngang.com


Do not patrosiize this store! B

Review posted on May 9 by S. Eggers


I
This is a review for Segovia Paints. Somehow, they mixed up my order and sent me the i

wrong item. When I contacted them about it, they didn't get back to me for a week! Since I |
i
needed the paint right away, I asked if they could send me the correct one using expedited d
H
delivery, but they refused to cover the cost. After wasting more time arguing back and m
in
H
forth, I decided to drive to their store on a weekend and speak to a manager in person. I
was able to get the paint I wanted, and a partial refund, but the whole experience left me g

;i
feeling completely dissatisfied. I do not recommend this store!

m / :
o
r\j
1 --r J

. c o \—\J
UJ

oc
o
186. What is stated about Segovia's paint 189. In the review, the phrase "get back to" in 4>
products?
g
paragraph 1, line 2, is closest in meaning

n O
U1
(A) Some of them emit no smeli.
(B) They can be delivered free of charge.
to

m
(A) reply s
(C) Some of them are on sale for a limited

in g (B)recover
CD
--j
time.

a k
(D) They are all environmentally friendly.
(C)repay
(D) retrieve a

pe CO

ic s
187. What is stated about Vibra paint?
(A) It has been the store's best-selling
190. What is suggested about Ms. Eggers?
(A) She changed her mind about a paint
o
ID

e color. O

//to
paint for years.
(B) It can be blended with more than two (B) She had to visit a store in Conway, or

tp :
colors. (C) She misplaced the receipt for her
order.
l>
(ffl

ht
(C) It requires a minimum purchase for a J2
rui
discount. (D) She has paid full price for her item. ra
o
o
(D) It is normally the most expensive paint. o
-il
UJ
188. What brand did Ms. Eggers most likely ft)X)
o.
receive initially? oa
(A) Vibra waterproof paint
(B) Densen metallic paint
(C) Fulmax blended paint
(D) Alpha basic paint

GO ON TO THE NEXT PAGE..

TEST 6 PART 7 197


Questions 191-195 refer to the following announcement, schedule, and e-mail.

Desmond-Hilbert Community College


Canter, Connecticut
Are you interested in enhancing your skills to increase your employability
and competitiveness in the job market? The Desmond-Hilbert Community
College is now offering courses for both employed and unemployed
individuals interested in obtaining certifications or improving their skill
sets.
The program offerings include certification in real estate brokering,
automobile safety, and computers. In addition, we have two new
certification courses this semester in basic nursing and building
inspection.
m /
If you would like more information, please visit our Web site at www.
c o
c.
desmond-hilbert.edu/vocation. An online form is available for those
who would like to enroll for the upcoming school term. Once you

g o
submit the form, we will respond within 48 hours. You may also enroll

s n
at the registrar's office. Deadline for enrollment is August 12.
Enrollment beyond the deadline is at the discretion of the instructor.

g m
k i n
e a
sp
Course Schedule for Fall Term August 24 to December 21

e ic Desmond-Hilbert Community College

//to
COURSE SCHEDULE INSTRUCTOR

tp:
Real Estate Brokering Mon and Wed, 9 a.m. - 4 p.m. Ms. Wendy Dufresne

ht
with a 1 -hour lunch break
Automobile Safety Tues and Thurs, 9 a.m. - 1 2 noon Mr. Harold Singh
Building Inspection Mon and Fri, 9 a.m. - 11 a.m. Mr. Ted Angelotti
Computers and Tablets Tues and Thurs, 2 p.m. - 4 p.m. Ms. Sandy Packard
Basic Nursing Mon to Fri, 8 a.m. - 5 p.m. with a 1 -hour lunch break Mr. Paul Leong
Information on payment of tuition and fees will be provided via e-mail if you register using
the online form.

To:Joey Partido <joeypartido@swiftmail.com>


From; Gail Bowe <registrar@desmond-hilbert.edu>
Subject: Registration
Date: August 9

198 Hackers.co.kr
Dear Mr. Partido,
This is to confirm that we have received your registration form. Your tuition and fees total $2,200. The
enrollment process will be complete once you have submitted your payment.
You indicated that the firm you work for will pay for your training.The amount may be transmitted to the
following account: Oak Valley Bank, Account No. 381 1245-67-981 I. Your employer may also pay by credit
card through our Web site.
Once your payment has been received, you will be issued a receipt via e-mail. Print it out and bring it on the
first day of class. Please be reminded that the course you enrolled in has full daytime sessions every day
except Saturday and Sunday. If you have any questions, you may reach me by e-mail. Thank you.
Sincerely,
Gail Bowe
Registrar
m /
c o
o c.
191. What is the announcement mainly about?
n g
194. Based on the e-mail, what is true about
(A) Job vacancies for teachers
(B) Testing guidelines for students
m s
Mr. Partido?
(A) He has paid for all of his tuition tees.
(C) Professional skills training

i n g
(B) He plans to take the same course he

k
(D) Course schedule changes has taken before.

e a
192. In the announcement, the word "discretion"
(C) He is currently working for a company.
(D) He has an account with Oak Valley

to
ic sp
in paragraph 3, line 6, is closest in meaning Bank.

e 195. What class did Mr. Partido most likely

//to
(A) substitute
enroll in?
(B) choice

tp:
(C) attention (A) Real Estate Brokering

ht
(D) disparity (B) Automobile Safety
(C) Computers and Tablets
193. Which instructor is teaching a new course? (D) Basic Nursing
(A) Ted Angelotti
(B) Harold Singh
(C) Sandy Packard
(D) Wendy Dufresne

GO ON TO THE NEXT PAGE,

TEST 5 PART 7 199


Questions 196-200 refer to the following information, form, and memo.
rtn
|o5l
Lakeshore Recreational Complex
Our facilities are suitable for a range of activities and events. To rent a facility, fill out a request form and
send it to booking@lakeshorerec.com. From April to September, we are open daily from 7 a.m. to 9 p.m. From
October to March, we are open Tuesday through Sunday from 8 a.m. to 7 p.m.

Name Facility Rental costs (per hour)


Zone I 1 Softball field, 1 Soccer field, 1 Oval-shaped track available $110
Zone 2 2 Basketball courts, 2 Volleyball courts, 3 Badminton courts available $55
Zone 3 1 Gym available for dance, yoga, martial arts $45
Zone 4 1 Hall available with facilities for bowling and billiards $35

Additional fees apply for staffing, equipment, and other services.


m /
Restrooms are situated throughout the complex. A cafeteria and vending machines are located in the recreation hall.

c o
c.
A 50 percent deposit is required upon booking, with the remainder due on the rental date. Companies that
have used our services before may be eligible for a 20 percent discount if they book a facility during off-peak
hours.
g o
s n
g m
i n
Lakeshore Recreational Complex

k
a
Facility Rental Request Form

e
sp
Organization: Markovich Insurance Date: July 11
Contact: Lois Butler Telephone: 555-3858

ic
Dates required: Saturday, August 29

e
//to
Times required: 10:00 a.m. to 4:00 p.m.

:
Facilities requested:

tp (Mark one or more selections below)

ht
S Softball field □ Soccer field 0 Oval-shaped track
□ Basketball court (Quantity: )
□ Volleyball court (Quantity: )
□ Badminton court (Quantity: )
□ Indoor gym □ Recreation hall
* multiple facilities available

Additional comments:

A small group of us will be at the venue one hour before our official start time to put up signs
and set up tables and chairs. Also, is it possible to reserve parking for our group? I'm not
certain what your conditions are in regard to this. We will probably need about 15 spaces in
total.

200 ^ ao^. a, yo^opi Mp3 Hackerslngang.com


Markovich Insurance
Date: August 22
To: All staff
From: Lois Butler
Subject: 9th Markovich Insurance Sports Day
Hey everyone! Just reminding you that our annual sports event is going to be held next week on Saturday,
August 29, at the Lakeshore Recreational Complex, We have lots of fun, food, and surprises in store, including
games and prizes! By now, you should have received one of four team assignments. Be sure to wear your
team's designated colors! Our first event for the day will be a softball match, so we will meet first thing at the
venue's softball field at 10 a.m. If you need help with transportation, please contact Dave Begley. See you all
there!

m /
c o
196. What is a feature of the Lakeshore
o c.
199. In the memo, the phrase "in store" in
Recreational Complex?
g
paragraph 1, line 2, is closest in meaning

n
(A)
(B)
Locker rentals
A dining area
to

m
(A)s preserved
(C)
(D)
Membership privileges
Discounts on catering
i n g(B) arrived

k
(C) donated

e a
197. What does Ms. Butler inquire about on the
(D) prepared

sp
form? 200. Where will the employees most likely meet
(A)
(B)
The
The
e ic
availability of temporary staff
price of renting equipment
on the morning of August 29?
(A) Zone 1

//to
(C) The possibility of extending rental hours (B) Zone 2
(D)

tp : The venue's policies on parking (C)


(D)
Zone 3
Zone 4

ht
198. What is indicated about Markovich
Insurance?
(A) It is bringing its own sports equipment.
(B) It requires the assistance of additional
staff.
(C) It is renting a facility during its off-peak
hours.
(D) It must settle all rental charges by
August 29.

This is the end of the test. You may review Part 5, 6, and 7 if you finish the test early.

smm

p.325 / p.327 / SIH p.369 / Part S&6 61® U|S M71


• qg mioixioj ojfe Self *113 aiAmi g4H xkjoi gxfi #o| ^14 cnsf ggsH MAjia

TEST 6 PART 7 201


6P^ m1 ^ 1000J1I 3 READING

m /
c o
o c.
n g
m s
i n g
a k
e
sp
I TEST 07

e ic
p ://to PART 5

ht t § PART 6

J-sa) PART 7

Self n\3. a|A

S® Efl^S S ztVMQ
l.^H^omo!§TL>^o? □oil
2. Answer Sheet. ^1. T|^7H# ^uloH^L-^? □ oil
3.A|7l|t^H|o^o? con
s.€ ^Upl" OD1 ss [[j§ oEllAE^Al^LlCh

^11 #0|# XI^EI 70^ °°I _A| _^Lm.


E1|AM i 75£0|[]1, A|^ gs 1§& SS 5! M Ol?J# ?|6|| A^L|Ch
READING TEST
In this section, you must demonstrate your ability to read and comprehend English. You will be given a
variety of texts and asked to answer questions about these texts. This section is divided into three parts
and will take 75 minutes to complete.
Do not mark the answers in your test book. Use the answer sheet that is separately provided.

PARTS
Directions: In each question, you will be asked to review a statement that is missing a word or phrase.
Four answer choices will be provided for each statement. Select the best answer and mark the
corresponding letter (A), (B), (C), or (D) on the answer sheet.

Q PART 5 o' S0! Mi! Its

m /
101. Mr. Finney paused during his speech 105.
c o
gas containers in a cool environment
to change the slide projected on the screen.

o c.
to prevent them from exploding.

g
(A) moment (A) To store
(B)
(C)
momentarily
momentous
(B)

s
(C)
Store
Stored n
(D) momentary

g m(D) Storing

102. All of the presenters at the social media


k i n106. Ms. Brendon received a to see a
marketing seminar introduced
audience before beginning to speak.
e a
to the skilled accountant specializing in tax
preparation from her coworker.
(A)
(B)
they
theirs
ic sp (A)
(B)
refer
referral

e
//to
(C) them (C) refers
(D) themselves (D) referable

tp :
103. Rattan Furniture House's new production 107. If the animation convention had not been

ht plant is expected to be fully


April.
(A) operate
by early canceled, the Richelieu Hotel
considerable amount of business.
(A) has had
a

(B) operator (B) has


(C) operation (C) is being had
(D) operational (D) would have had

104. Petra Towers is located just of the 108. The Society for Ecological Excellence is
downtown area, close to a subway station. currently accepting of environmentally
friendly organizations for its upcoming
(A) outside
(B) beyond awards ceremony.
(C) away (A) reputations
(D) across (B) nominations
(C) concentrations
(D) assumptions

204 Hackerslngang.com
109. Ms. Crawford accepted the most job 115. The campus parking lot is used by so many
offer she received after considering all her students that even those with parking
options. passes are not guaranteed a spot.
(A) prefer (A) insistently
(B) preference (B) measurably
(C) preferable (C) necessarily
(D) preferably (D) markedly

110. Southbound traffic Highway 14A will 116. Ram Builders' supervisor the client
be slow for the next two weeks while road that the supplies needed for construction
repairs take place. would arrive on time.
(A) among (A) bargained
(B) all (B) concerned
(C)
(D)
along
aboard
(C)
(D)
predicted
assured
m /
c o
c.
111. Greil Manufacturing began a partnership 117. Schilling Investors Group has not
with FRN Inc. that benefited both found a suitable candidate to replace
companies.
g o
Mr. Macmillan, who resigned two weeks
(A) mutually ago.

s n
m
(B) delicately (A) almost
(C)
(D)
densely
preventively
i n g
(B)
(C)
yet
far

a k (D) only

e
112. The expense for fixing the vehicle was

sp
higher than the cost given by the 118. The athlete recently beat the world record
auto mechanic. for the marathon less than a minute.
(A) estimating
e ic (A) beneath

//to
(B) estimated (B) above

tp :
(C)
(D)
estimation
estimates
(C)
(D)
at
by

ht
113. Logan Home Appliances' annual
promotion is now underway, the store is
119. First Canadian National Bank charges a
commission when cash from its
far busier than usual. machines by non-customers.
(A) But that (A) to withdraw
(B) Due to (B) is withdrawn
(C) Notwithstanding (C) withdraws
(D) Since (D) is withdrawing

114. Experts say that a rise in property values in 120. The city of Myerstown plans to turn an
the Bedford area will be the outcome 80-acre of land near Weller River
of the increase in local population. into a public park next year.
(A) susceptible (A) period
(B) transferable (B) stretch
(C) probable (C) degree
(D) questionable (D) collection
GOON TO THE NEXT PAGE.

TEST? PARTS 205


121. The woman sitting Mr. Heath at the 126. Customers who subscribe to Travelog's
meeting was kind enough to lend him a online magazine will receive discounts
pen at his request. their subscriptions are for six months
(A) apart or a year.
(B) opposite (A) rather
(C) from (B) either
(D) closely (C) even
(D) whether
122. Once the vendor his payment,
Smith & Cooper Wholesalers will ship his 127. Barton Electronics will have to boost
order out. production to meet the demand for
(A) substitutes its newest dishwasher model.
(B) leases (A) bulky
(C)
(D)
transfers
enforces
(B)
(C)
proficient
sizable
m /
(D) wealthy
c o
c.
123. The board of directors agreed to
change Two-Tone Media's logo as all the
members considered it outdated,
g o
128. Mr. Sampson must renew his gym
membership by December 31, he
(A) inseparably
s n
will be unable to access the facility.
(B)
(C)
unanimously
elaborately
g m (A)
(B)
but
or
(D) intimately

k i n (C)
(D)
so
not

e a
124. Airport officials have announced that more

sp
security checks will be adopted to 129. ReliaCorp's sale of some surplus property

ic
eliminate any potential threats. is with saving the company from

e
(A) reluctant bankruptcy.

//to
(B)mundane (A) agreed

tp :
(C) obtainable
(D) rigorous
(B)
(C)
motivated
reminded

ht
(D) credited
125. The organizers of the Brock County Fair
have decided to the event due to the 130. otherwise stated, all course
poor weather conditions. materials will be available at the campus
(A) back down bookstore two weeks before the first day
(B) give away of class.
(C) keep out (A) While
(D) put off (B) Still
(C) Unless
(D) Whereas

206 Hackers.co.kr
PART 6
Directions: In this part, you will be asked to read four English texts. Each text is missing a word,
phrase, or sentence. Select the answer choice that correctly completes the text and mark the
corresponding letter (A), (B), (C), or (D) on the answer sheet.

© parts A|y 8^

Questions 131-134 refer to the following e-mail.

To: Mimi O'Hare <mohare_1 @mymail.ca>


From: Super Wash Center <customersupport@superwashcenter.ca>
Subject: Your inquiry
Date: June 2
m /
c o
c.
Dear Ms. O'Hare,

We received the message stating that you


g o
your pre-paid rechargeable card. You
131.

s n
mentioned that the missing card had a $50 balance and that you hoped to get it back.

g m
Unfortunately, we cannot fulfill this request because our records show that you failed for

k i n
balance protection, which allows us to track card balances.

e a
sp
It is a service we offer to recover money left on your card in the event that it is
133. 134.

ic
misplaced or stolen. If you get it, we'll be able to secure your balance in the future and send

e
//to
you a replacement card immediately.

tp :
We are sorry that we cannot be of more help,

ht
Mac Benson, Super Wash Center

131. (A) damaged 133. (A) You can also collect and spend points
(B) returned HHl with the card.
(C) lost (B) We will begin distributing it in the
(D) sold coming weeks.
(C) We recommend signing up for it next
132. (A) registers time to prevent this problem,
(B) to register (D) You did not have enough money on
(C) registered your card to cover the cost.
(D) be registering
134. (A) what
(B) every
(C) some of
(D) any GO ON TO THE NEXT PAGE

TEST 7 PART 6 207


Questions 135-138 refer to the following article.

KYR Commonwealth Trust Application Period to Open August 1

Organizations funding may be in luck. The KYR Commonwealth Trust has announced
135.
that grant applications for local businesses will soon be available. According to spokesperson

Meryl Pond, proposals will be accepted from August 1 to September 30. Applicants
136.
should note that one aspect of the initiative has changed. Businesses have traditionally used

the funding to supplement their existing programs. , they will now be expected to use
137.
the money to develop new services for the community.

m /
"We're willing to give grant recipients more than before. But we expect more in return
c o
c.
138.

o
for this additional money. We want to inspire the design of better, more productive programs

g
that can make a bigger difference for more people," she said.

s n
g m
135. (A)
(B)
in
in
charge of
opposition to
k i n
137. (A)
(B)
Consequently
Otherwise
(C) in search of
e a (C) However

sp
(D) in lieu of (D) Therefore

for the event.


e ic
136. (A) They have already begun fundraising 138. (A) support
(B) training

//to
(B) They can be submitted through the (C) information

tp : foundation's Web site. (D) concern

ht
(C) The application process was revised
after this point.
(D) The funding organization is in need of
capital itself.

208 51 B0^7| MP3 Hackerslngang.com


Questions 139-142 refer to the following notice.

The city's Waste Management Division has been cleaning Brentridge's streets twice a month.

However, in response to the rapid accumulation of trash in several districts, we have decided

to increase the frequency of this service to once a week starting May 1 , we hope to
139.
improve the appearance of public areas while removing substances that could contaminate

the environment.

There are a few things that you need to be aware of. First, each neighborhood has been

assigned a day of the week for its streets to be cleaned. Additionally, residents must

make sure to keep their curbs


140.
at these times. This is vital as our cleaning vehicles will
m /
141.

c o
c.
be unable to reach them otherwise. We would truly appreciate your with these changes.
142.

g o
139. (A) For instance 141. (A)
soccupied n
(B) Until now (B)

g m separate
(C)
(D)
On the other hand
In this way
k i n (C)
(D)
accessible
flexible

e a
140. (A) Refrain from disposing of toxic materials 142. (A) association
||||
sp
outside of these designated areas.

ic
(B) Please visit our Web site to see when
(B) qualification
(C) cooperation

e
this will occur in your area. (D)sequence

//to
(C) The mayor will address citizens'

tp :
concerns at the next town hall meeting.
(D) The city plans to implement a new

ht recycling program to reduce waste.

GO ON TO THE NEXT PAGE

TEST 7 PART 6 209


Questions 143-146 refer to the following article.
| A||
OS!
-r- Baiter Food Reaches Agreement with Lexi's

July 21—National grocery chain Baiter Food ----- organic produce company Lexi's.

Negotiations began in October of last year, and the transaction came to a close last Monday.

Lexi's was founded nearly six decades ago but struggled to recover from losses for quite a

while. Its attempt to enter the California market with the opening of new locations proved

aa This caused Lexi's to fall behind its competition.


"

Baiter Food representatives say that 15 of the 29 Lexi's stores are now in the process of
m /
c o
c.
being renamed "Baiter." The performance of these stores will be monitored closely to
145.

o
determine whether they should undergo additional modifications. According to consultants,

g
this is the best strategy as loyal Lexi's customers may need to
n during the transition.

ms
146.

i n g
143. (A)
(B)
will have acquired
has acquired
a k 145. (A) Some suggest that the renovations
|||| were an excessive expenditure.

e
sp
(C) will acquire (B) The new brand will represent Baiter's
(D) to acquire most recent innovation.

144. (A)
e ic
unsuccessful
(C) All other branch names will be changed

//to
gradually over the course of a year.
(B) incomparable (D) The market for grocery stores has

tp :
(C)
(D)
unintentional
inaccurate
become flooded in recent years.

ht 146. (A)gauge
(B) adjust
(C) regulate
(D) finalize

210 Hackers.co.kr
PART 7
Directions: In this part, you will be asked to read several texts, such as advertisements, articles, instant
messages, or examples of business correspondence. Each text is followed by several questions. Select
the best answer and mark the corresponding letter (A), (B), (C), or (D) on your answer sheet.

© PART 7 ^#01 ApJ 54^

Questions 147-148 refer to the following notice.

m /
UURLT6RS M6MORIRL RIRPORT

c o
c.
March 12

g o
The Walters Memorial Airport's parking garage B is closed for structural repairs

s n
that are expected to take six months. Travelers may park in garage A or in one
of the facilities outside the airport. The airport management has doubled the

g m
number of shuttle buses from each of the outer parking lots, including Well's

i n
Parking Lot and Aviation Road Parking Lot. Our shuttles will pick up passengers

k
e a
near each lot's customer service booth. The drop-off and pickup point for the
airport terminal is located in front of Terminal A's main doors. All shuttles are

ic sp
free to ride and will run continuously throughout the day. We apologize for any
inconvenience this situation may cause our visitors.

e
p : //to
ht t
147. What is mentioned about parking garage B? 148. What is offered to airport visitors using
Well's Parking Lot?
(A) It has recently been repaired.
(B) Its service booth was relocated. (A) Free parking passes
(C) It will be enlarged to hold more vehicles. (B) A ride to a terminal entrance
(D) It may not be used for a period of time. (C) Additional pickup points
(D) A schedule for local buses

GO ON TO THE NEXT PAGE

TEST 7 PART 7 211


Questions 149-150 refer to the following text-message chain.

SsJ

Riya Shankar [9:10 A.M.]


I received a customer complaint about our Web site yesterday. I
was wondering if you have time to look into it now.

Noel Mitchell [9:13 A.M.)


Was the complaint about our Frequently Asked Questions page?
We finally finished fixing it last night. Everything should be okay.

/
Riya Shankar [9:14 A.M.]

m
That's good to hear, but the problem actually concerns the store
content page. The customer mentioned that several images of

c o
c.
our latest products aren't loading.

Noel Mitchell [9:18 A.M.]


g o
s n
If it was yesterday evening, it was probably because we were
updating the pictures. But I'll double-check the page and get
back to you right away.
g m
Riya Shankar
k i n [9:20 A.M.

a
Fantastic. Thanks for your help!

e
ic sp
e
p : //to
149. At 9:13 a.m., what does Mr. Mitchell most
likely mean when he writes, "Everything
150. What problem did a customer have with
a Web site?

ht tshould be okay"?
(A) He remembers loading an updated
(A)
(B)
Locating a customer inquiry menu
Understanding a return policy
map to the site. (C) Reading user feedback
(B) He believes a customer will write a (D) Viewing pictures of some goods
positive review.
(C) He is sure that a Web page is
functioning properly.
(D) He has a suggestion for addressing
a complaint.

212 A!£qj sojjT/pyojopixh □! Q^opi MP3 Hackerslngang.com


Questions 151-152 refer to the following flyer.

The snow is melting and that can mean only one thing-it's spring!
It's time to clear away all the dirt that's built up over the winter
and throw away all the old things you don't need. These can be
challenging tasks, so why not get Tidy Up Time cleaners to do the
work for you?

We charge hourly or daily (7 hours) rates for a one-time cleaning


and provide a 10 percent discount to customers who pay for
m /
regular (weekly or monthly) cleanings. Our standard package

c o
c.
includes sanitizing your entire home, which involves dusting every

g
charge, we can help you gather up unwanted items and put themo
surface and wiping down all the windows. In addition, at no extra

s n
into bags or boxes. For an extra fee, we can also perform other

m
services, such as deep cleaning carpets, mattresses, and upholstery,

g
k i n
as well as washing and ironing clothes, towels, and linens.

a
For more information on prices and services, head over to our

e
Web site at www.tidyuptime.com.

ic sp
e
//to
151. What is stated about Tidy Up Time? 152. What service is provided for an additional

p :
(A) Its employees do not clean bathrooms.

t
charge?

ht
(B) It only services certain types of (A) Cleaning garments
houses. (B) Disinfecting rooms
(C) It does not offer services on weekends. (C) Collecting unwanted items
(D) It provides discounts for regular (D) Wiping down windows
services.

GO ON TO THE NEXT PAGE^

TEST 7 PART? 213


Questions 153-155 refer to the following information from a manual.

Using the RoadRunner Sedan I2,s dashboard has never been easier. As with last year's model,
the car features an innovative touch screen interface that allows you to control various
operations. — [i] —. These include playing music, finding your way with a Global Positioning
System (GPS), talking on a speaker-phone, controlling the temperature, and much more. The
first time you put the keys in the ignition, the interface will appear and prompt you to select a
language. Once you have done this, you are free to use the system.

Each feature of the interface is represented by an icon. For example, to access the GPS, simply
press the map icon. This will bring up a digital map with several buttons. Simply type in your
destination or say it out loud, and several routes will be suggested. — [2] —. To access other
functions, press "MENU" or hit the back arrow to return to the main page.
m /
c o
c.
To connect your mobile phone or any other device to the dashboard, plug one end of a USB

g o
cable to the device and the other to the car's USB port. — [3] —. Once it is inserted, a message
reading "DEVICE FOUND" should appear. If you want to play music, simply select the song
on your device and hit "PLAY."
s n
g m
Please note that the dashboard and all functions switch off automatically when the vehicle's

i n
ignition has been turned off. — [4] —. This is done to prevent the car's battery from losing

k
power.

e a
ic sp
e
153. According to the manual, what can the 155. In which of the positions marked [1], [2],

//to
dashboard interface do? IBfe [3], and [4] does the following sentence

tp :
(A) Display fuel consumption best belong?

ht
(B) Give driving directions "Select your preferred path from the
(C) Read the outside temperature choices provided."
(D) Provide parking assistance
(A) [1]
154. What does the manual say about connecting (B) [2]
a cell phone? (C) [3]
(D) [4]
(A) It may take a few minutes to set up.
(B) It may be incompatible with some
models.
(C) It is a feature that costs extra.
(D) It requires the use of a cable.

214 Hackers.co.kr
Questions 156-159 refer to the following Web page.

International Interpreter Network

Find the best interpreter for your needs.


Discover resources for document and Web site translation as well.
We offer translation services in over 30 languages.
Search by language (Type in box)
General site search (Type in box)

Home Services Offered Languages Covered Request an Interpreter

We have compiled a vast network of trained experts who speak English and one or more additional
languages. All of our experts can translate oral communications to and from English. — [1] —.A select few
m /
can also translate written materials such as books, manuals, and Web sites. Each interpreter in our

c o
network has passed thorough language testing and completed interpretation skills training. After each

assurance and ensure that we continue to offer the best services available.
o c.
concluded project, we collect evaluations from our customers. — [2] —. This is to facilitate quality

g
The whole process for securing our services can be carried out on our Web page, making it easy for clients

n
s
to locate the translator or interpreter best suited for the job. — [3] —. Fill out the online form and tell us
about your project or event needs. We'll obtain the best interpreters in our network for you. Prices may vary
depending on the interpreter available and the job requirements.
m
i n
Do you want to become an interpreter-translator in our network? g -. Click here.

a k
e
ic sp
156. What is the main purpose of the Web page? 158. How does the organization maintain the
(A)
e
To promote a language course
quality of its services?

//to
(B) To announce a job vacancy (A) By hiring workers with teaching
(C)

tp
(D): To explain pricing for a service
To provide a description of a business
certificates
(B) By holding monthly training sessions

ht
(C) By gathering assessments from clients
157. What is true about translators at (D) By implementing a peer evaluation
International Interpreter Network? system
(A) All of them are fluent in at least three
languages. 159. In which of the positions marked [1], [2],
(B) They majored in language instruction ||p [3], and [4] does the following sentence
best belong?
in college.
(C) Some of them can translate spoken "To begin, click on the "Request an
and written communications. Interpreter" button."
(D) They have published their own
(A) [1]
textbooks more than once.
(B) [2]
(C) [3]
(D) [4]

GO ON TO THE NEXT PAGE.

TEST 7 PART 7 215


Questions 160-162 refer to the following article.

Restructuring in Preparation for Farnsworth Capital Bank

The government has approved the merger between Capital Status


Bank and Farnsworth Regional Bank. Both banks have exhibited
average profitability when compared to other financial institutions,
and each bank has strengths that the other does not. The combined
board of directors of the new institution, Farnsworth Capital Bank,
expects profitability to start improving in about five years.

Bob Altman from Capital Status Bank has been selected to be the
chief executive officer of the new organization, and Emily Carter from
Farnsworth Regional Bank will serve as the chief financial officer.

m /
The board of directors has not yet appointed a chairman, but

c o
c.
spokesperson Ryan Salazar has stated that someone from outside both
organizations will be hired,

g o
Mr. Altman said that because of the region's rapid growth, employee

s n
layoffs will be kept to a minimum. Instead, some employees may be

g m
asked to move to a new location or accept a different position. No
changes are expected in regard to the number of bank branches

i n
serving customers. As for where the new headquarters will be located,

k
a
details have yet to be announced.

e
ic sp
e
160. What is the article mainly about? 162. What is NOT mentioned about the

//to
(A) The profitability of a new venture merger?

tp :
(B)
(C)
A consolidation of two firms
The effect of a government regulation
(A) The number of bank branches will not
change.

ht
(D) An industry's long-term outlook (B) Staff may be requested to fill different
roles.
161. What is indicated about Farnsworth (C) Some employees will transfer to other
Capital Bank? offices.
(A) It plans to move its headquarters to a (D) It will have been completed before the
new city, new year.
(B) It will keep wages at current levels.
(C) It is not expected to earn more money
right away.
(D) It will reduce the size of its branch
network.

216 ?SSrt?!i!Sa|H°UUf 90B7|5f^Etot°f7|MP3 Hackerslngang.com


Questions 163-166 refer to the following online chat discussion.
Al
ss
1*1 John Mallet [3:45 P.M.) Hi, everyone. You'll be happy to know that management gave me
approval to send some of our people to the career fair at Duluth
University next month.
Janet Chan [3:48 P. M That's excellent news. A lot of students have probably heard of our
company, since Go-C Tech is such a short distance from the campus.
John Mallet [3:52 P.I Exactly, however, our budget is limited, so we need to decide which
departments should be represented. We can't send everyone.
Melissa Kovac [3:55 PM, My marketing team recruited quite a few new people just last month.
You can leave me out.
|r| John Mallet [4:00 PM.] I'll keep that in mind. And I think the accounting department should
participate. Management feels that they will need more help when our
purchase of Stanfield Incorporated moves forward. I'll speak to Susan
m /
o
Edwards about it.
Peter Mercer [4:01 P.M.]

c. c
I need to supervise product testing in the research and development
department for the next few months. Can I send someone in my place?

o John Mallet [4:03 P.M.]


Of course. Who do you choose, Peter?
g o
Peter Mercer [4:05 P.M.]
s n
Amy Lintan, She doesn't have a lot of experience, but she's quite
knowledgeable about our department's needs.

o John Mallet [4:08 P.M.]


g m
That's fine. Perhaps someone who has been employed for a shorter

i n
time can provide a valuable perspective on what it is like to join our
company.
k
e a
sp
Send

e ic 165. What is implied about Go-C Tech?

//to
163. What is indicated about Duluth University?
(A) It is located near Go-C Tech. (A) It lost several employees in the past

p :
(B) It offered Go-C Tech a sponsorship

t
month.

ht
deal. (B) It recently released a new product.
(C) It is well-known for its scientific (C) It will be closing down a research
research. facility soon.
(D) It holds a career fair annually. (D) It is preparing for an expansion.

164. At 3:55 p.m., what does Ms. Kovac mean 166. Who will delegate a representative to the
when she writes, "You can leave me out"? career fair?
(A) She needs time to train some staff. (A) Amy Lintan
(B) She does not want to attend an event. (B) Peter Mercer
(C) She has already participated in a (C) Janet Chan
career fair. (D) Melissa Kovac
(D) She is not familiar with a recruitment
process.

GO ON TO THE NEXT PAGE

TEST 7 PART 7 217


Questions 167-169 refer to the following schedule.

The Lake Point Small Business Association (LPSBA)


Small Business Grant Proposal Workshop
Wednesday, August 11
8:00 A.M. - 5:30 P.M.
Lake Point Center
Schedule of Events:

8:00-8:30 A.M. Sign-in and continental breakfast


8:30-9:00 A.M. Welcome and introductory comments
by Carol Summers, LPSBA President, Owner of Smartphone Repair Shop
9:00- 10:00 A.M. "Researching and Selecting Applicable Grants for your Business," followed by a
15-minute break
Presented by Dr. Brian Simon, Professor of Business Development, University of
m /
Jefferson
c o
10:15 A.M.-
12:00 P.m.
"Grant Proposal Writing Fundamentals: Writing a Winning Proposal"

o c.
Presented by Lucy Haggerty, Administrator, Government Business Development Office
12:00- 1:00 P.M. Lunch in the Edward Gray Lounge
n g
1:00-2:30 P.M.
s
A review of business grant proposals. Sessions will be conducted in large groups

m
divided by type of business, followed by a 15-minute break

n g
Facilitated by Carol Summers, Oliver Fleadley, Frances Connors

i
2:45-5:00 P.M.
k
Optional grant proposal review by all presenters during one-on-one sessions

a
5:00-5:30 P.M.
e
Closing remarks by Carol Summers

ic sp
The Lake Point Small Business Association wants all participants to get as much out of the workshop as
possible without distractions. During the workshop, please switch off mobile phones and other devices that

e
make sounds. Breaks will be provided for attendees. If you need to take a call or respond to a message

//to
outside of these breaks, please exit the room quietly and conduct your business in the lobby. Thank you.

tp :
ht

218 Hackers.co.kr
167. What is scheduled immediately after 169. What does the LPSBA ask attendees to
Dr. Simon's talk? do?
(A) A video presentation (A) Share experiences starting a business
(B) A brief intermission (B) Register for a follow-up workshop
(C) A talk on government projects (C) Avoid causing noise disturbances
(D) A one-hour lunch service (D) Use meeting rooms for private
discussions
168. What is NOT indicated about Ms. Summers?
(A) She will be involved in reviewing
proposals,
(B) She is the proprietor of a business.
(C) She teaches courses at a university.
(D) She is the final speaker of the event.

m /
c o
o c.
n g
m s
i n g
a k O

e CO

ic sp O
ID

e O

//to
tp:
l>
Kn

ht
j2

CD
a
a
2d
UJ
fDOJX)
a.
OQ=3*

GO ON TO THE NEXT PAGE

TEST 7 PART 7 219


Questions 170-173 refer to the following memo.

MEMO
To: All Keenan Company Production Workers
From: Karen Pollack, Director of Manufacturing
Date: May 27
Subject: Overtime work opportunity
i
We are very excited to announce that we have received a large order from the Saturn Moon
Corporation. Saturn Moon has sold a lot of our products and foresees a continued growth in demand,
which is why they have requested that we fulfill a much bigger order for them than we normally do.
The order is due for shipment at the end of September.

We are unable to complete the order without making some changes. In this regard, we have two
options. We can stay open longer from July 1 through September 30 and assign employees to work
m / IJ

c o
extra weekly shifts. Our second option would be to hire temporary employees, train them for some of

o c.
the less complex tasks, and dismiss them when the job is done. We need input from each employee,
so please think about what you'd prefer. You will be meeting with your managers individually sometime
this week to let them know your preference.
n g
s
If we have enough workers willing to work overtime, we will adjust your schedules. Otherwise, we will

m
I

i n g
resort to hiring temporary workers. Thank you for your participation in this process.

a k
e
170. What is being announced in the memo? 172. The word "dismiss" in paragraph 2, line 4,

employees
ic sp
(A) A plan to take on new permanent is closest in meaning to
(A) disregard

e
//to
(B) A new company incentive program (B) surrender
(C) A need to produce more goods than (C) eradicate

tp : usual (D) discharge

ht
(D) A revised monthly staff pay scale
173. What will take place if not enough staff are
171. What does Ms. Pollack request that willing to work overtime?
workers do?
(A) Employees' salaries will be altered.
(A) Take two options into consideration (B) An order request will have to be
(B) Train the short-term helpers denied.
(C) Recommend training topics they would (C) Managers will fill in as provisional
prefer workers.
(D) Check their pay statements (D) Short-term staff will be taken on.

220 ¥esaK!i!S5)HepuhtB|^®5!E!0iof7|MP3 Hackerslngang.com


Questions 174-175 refer to the following Web page.

http://www.doorstepdelectable.com/home

HOME MENU FEEDBACK REGISTER CONTACT

Doorstep Delectable... The perfect way to order food packaged just for you!

Doorstep Delectable delivers customized meals to your home once each week with easy
preparation instructions. Our meals are healthy and delicious, containing no artificial ingredients.
In addition to providing the foods you enjoy, our meals are tailored to meet your special dietary
needs, whether you're diabetic, vegetarian, or on a gluten-free or low-carbohydrate diet.

Our ordering process is simple! First, click on "REGISTER" above and provide your contact
information. You'll also be asked about your specific dietary needs and preferences. You can

m /
then browse through our meal menus and select the ones you prefer and how many you would

c o
c.
like each week. We will ask you for payment, and once it has been processed, a confirmation
e-mail will be sent to you including an estimated delivery date.

g o
s n
It really is that easy, so what are you waiting for? Place your first order today!
If you are pleased with our company's product, let us know by posting a comment
in our feedback forum.
g m >, ^

k i n lEBU

e a
sp
174. According to the Web page, how often can 175. What is NOT indicated about the company's
products?

ic
customers receive deliveries?
(A) Every day
e (A) They can be ordered through a Web

//to
(B) Twice weekly site.

tp :
(C) Every week
(D) Twice monthly
(B) They are available in meatless
varieties.

ht
(C) They contain only natural ingredients.
(D) They can be paid for after delivery.

GO ON TO THE NEXT PACE.

TEST 7 PART 7 221


Questions 176-180 refer to the following notice and e-mail.

Tribcott Manor House


Tribcott Manor House was built for the Tribcott family in 1786, and today it is the largest
stately mansion in the southwest of England. While the upper floors of the house are still
inhabited by the family and are not open for public visits, a large number of rooms can be
seen. These include the original master bedroom, the servants' quarters, and the grand dining
hall. An adult entry ticket is £8.50, and a child's ticket is £3.00. From Tuesday to Saturday, our
opening hours are between 10 a.m. to 5 p.m. On Mondays, we are open from 9 a.m. to 12 p.m.
Our ground-floor facilities have been recently renovated so that they can be used for small
meetings or private events. The following rooms are available to rent by the hour at
reasonable fees.
o
©
The
The
Cornwall Room: Seats up to 70 people. Contains a large stage at the front of the room.
Devonshire Room: Seats up to 40 people, with wheelchair accessibility.
m /
® The Somerset Room: Seats up to 40 people. Contains a kitchenette.
c o
c.
® The Wiltshire Room: Seats up to 25 people. Includes audiovisual equipment.

including the catering options that we provide.


g o
Please contact us at queries@tribcottmanorhouse.co.uk to discuss prices and further details,

s n
I ■

g m
k i n
TO:
a
<queries@tribcottmanorhouse.co.uk>

e
sp
FROM: Angela Carter <ancart(5)corsoindustries.co.uk>

ic
SUBJECT: Venue inquiry
DATE:
e
March 28

p : //to
Dear madam or sir,

ht t My company is planning to host a celebratory meal for a longstanding employee who is being
promoted to CEO. For the occasion, we require a room that seats 30 people comfortably and allows
easy access for an individual in a wheelchair. We would need it for about three hours on Monday, April
30. Is such a room available on that date?
If so, a colleague and I would need to come over and view it to assess its suitability. Please let us know
when would be a good time to do this. If you require more information, you will need to contact Paul
McGuire at 555-7263, or p.mcguire@corsoindustries.co.uk because I will be occupied for the next few
days attending a seminar. He is fully aware of our plans and can make any necessary arrangements on
the company's behalf. Thank you and we look forward to hearing from you.
Best wishes,

222 Hackers.co.kr
176. What is NOT indicated about Tribcott 179. What is implied about Mr. McGuire?
Manor House? (A) He was satisfied with the services at
(A) Food services can be arranged for an Tribcott Manor House.
event. (B) He is considering taking another
(B) The servants' area remains accessible. position.
(C) Guided tours are available upon (C) He was unable to book the caterer he
request. regularly uses.
(D) Events can be held in some of the (D) He works with Ms. Carter,

180. In the e-mail, the word "assess" in


177. Why was the e-mail written? paragraph 2, line 1, is closest in meaning
(A) To seek recommendations for a to
restaurant (A) judge
(B) To announce an executive's recent (B) secure

m /
promotion (C) establish

c o
c.
(C) To make arrangements for a special (D) complete
occasion
(D) To inform participants of a scheduled
g o
meeting

s n
178. Which room will Ms. Carter most likely be
interested in renting?
g m
(A) The Somerset Room
k i n
(B) The Cornwall Room

e a
sp
(C) The Devonshire Room
(D) The Wiltshire Room

e ic
p : //to
ht t

goonjvthene^PAGE^

TEST? PART? 223


Questions 181-185 refer to the following Web page and review.

www.BizzPro.com/BizzSec/update
About I Update I Download I Customer Reviews
Our best-selling software BizzSpec has now been updated to version 4.0! Just like with BizzSpec 3.9,
you can create comprehensive business reports and presentations. At the same time, the new version
imports data, text, and images from various sources, which can be inserted into a format that you
design. In addition, the upgrade improves the functionality, look, and features of the program, allowing
it to start up and import files faster than ever before while completing tasks more effectively. Now.
users can customize the home page and main menus to reflect the functions that are used most.
The updated version is now available to download for existing users of BizzSpec 3.0-3.9 at no cost.
Updating is easy. Simply click here to go directly to our download page, where we'll ask you a few
questions to ensure that the upgrade is compatible with your computer's operating system. Download
and installation time is about 30 minutes. Please e-mail upgradehelp@bizzservices.com if you have
m /
any questions.

c o
available for a limited time.
o c.
*BizzSpec 4.0 is also available for new customers. Please check the Web site for special rates that are

n g
m s
.
i ng S
Reviewer: Johnny Nestor

a k
e
I recently installed BizzSpec 4.0 and have worked with it for two weeks. I

ic sp
liked the features in the 3.0 version I had before but used it only
occasionally as it was slow and difficult to navigate. The new version

e
//to
addresses both of these problems. The ability to customize the menu
means that you can remove functions that you don't use and arrange the ■!

tp:
\
icons in a way that works for you. The functions aren't deleted from the

ht
program, so if one is needed at a later date, you can retrieve it. Whether
you customize the main menu or not, the program starts quickly and
moves seamlessly between tasks.

However, I have two minor complaints. First, the length of the setup time
stated in the company's Web page was inaccurate as the process took
much longer than that. Secondly, there are a few older file types that the
updated version is unable to read.

Overall, it's a great tool that is now easier to use. If you create a lot of
reports and presentations, I recommend the update.

224 iieq a°j:DAh EKWlg eoio[7| MP3 Hackerslngang.com


181. What is the main purpose of the Web 184. What does Mr. Nestor say about the setup
page? process?
(A) To invite customers to rate a product (A) It has too many complicated steps.
(B) To give instructions for removing a (B) It took more than half an hour.
computer virus (C) It did not require the import of files.
(C) To notify customers of a product (D) It tends to pause intermittently.
improvement
(D) To provide information about a 185. What is suggested about Mr. Nestor?
presentation (A) He prefers the appearance of the 3.0
product version more.
182. What is indicated about BizzSpec 4.0? (B) He got a new program version free of
(A) It can convert one file type to another, charge.
(B) Unused functions are automatically (C) He uses the software program mostly
deleted, at home.

m /
(C) The version is only available for
o
(D) He recommended the software to his

c
c.
existing customers. colleagues.
(D) Tasks are processed faster than in
earlier versions,
g o
183. In the review, the word "navigate" in
s n
paragraph 1, line 3, is closest in meaning
to
g m
(A) examine
k i n
(B)
(C)
guide
operate
e a
(D) plan

ic sp
e
p : //to
ht t

GO ON TO THE NEXT PAGE.

TEST 7 PART 7 225


Questions 186-190 refer to the following list, information, and e-mail.
I ill
ota
MUSICAL RECORDINGS BYi SI IA KRALI
Midsummer Day's Dream
Krall's first recorded album from the 1980s is a collection of mostly self-written songs, with some composed by
collaborator Ken Lewes.The album sold 3.5 million copies and propelled Krall to fame.
Listen Here
In her second album, Krall collaborated again with Ken Lewes, and two songs they wrote together became
number one hits.The album's strong social commentary appealed to many fans.
The Temptation
Many called Krall's third album her best work yet, citing the interesting musical arrangements and intelligent
lyrics written by Ken Lewes. Despite receiving the most critical acclaim, the album sold only moderately well
and "Bright Days Ahead" was the sole song on it to make the top ten list.
Little Thoughts
m /
c o
After taking a break from the industry, Krall came back with this reflective album. Critical reviews were mixed,

were written in collaboration with Ken Lewes.


o c.
but fans appreciated its similarity to some of Krall's early work.The album sold 2.8 million copies, and all songs

n g
On The Whole: For her fifth release, Krall has re-recorded ten of her most famous songs with a live orchestra.
The album contains two new songs written by Krall and Lewes.

m s
i n g
a k
e
http://www.musictalkmelissa.com

ic sp
HOME | ABOUT | NEXT BROADCAST | EPISODES | CONTACT

eJoin us every Sunday at 7:30 p.m. for a livestream broadcast of

//to
Aius/c Talk with Melissa I Catch up on old shows by clicking on the

tp : "Episodes,/ tab above.

ht
Weekly show for: August 9

Join Melissa Oliver tor song reviews, music news, and a live
interview with musician and songwriter Shawna Krall. Born in
Halifax, Nova Scotia, Krall's career has spanned decades. She will
be here to perform one of two new songs from her fifth album,
which was written with partner Ken Lewes. Entitled 'The Beginning
of the End," the song is already at number three on world music
charts. She will also discuss her previous work and plans for an
international tour starting on August 28. The episode will be
rebroadcast at the same time on August 1 0 and 1 1.
TO: Melissa Oliver <Moliver@mustalkmelissa.com>
FROM; Brian Rivers <Rivers_Br@musicnetwork.com>
SUBJECT: Work offer
DATE: August 12

Dear Ms. Oliver,


I produce and direct documentary programs for The Music Network. I am currently working on
preparations for a series about popular musicians that have been culturally influential. We hope to
interview many artists like Ms. Krall on our show. I missed your livestream interview with
Ms. Krall but caught the episode on its next showing and thoroughly enjoyed it. I was very impressed with
your rapport with Ms. Krall.

m
I am looking to cast an interviewer for our series. My colleagues and 1 would like to know if you would be/
interested in filling this role. If so, I can provide you with further details.

c o
Regards,

o c.
Brian Rivers

n g
m s
186. What is stated about Ms. Krall?

i n g
189. What does Mr. Rivers ask Ms. Oliver to
do?
(A) She has written songs for Ken Lewes's
albums.
a k (A) Provide him with a musician's contact
(B) She collaborated with the same
e information
composer on all her albums.

ic sp
(C) She has yet to have a number one hit
(B) Post the newest episode of a talk show
(C) Work on an upcoming program
song.
e (D) Become a producer for a film

//to
(D) She recorded all her songs with live

tp :
orchestras. 190. When did Mr. Rivers watch Ms. Krall's
interview?

ht
187. Which of Ms. Krall's albums did critics like (A) On August 9
best? (B) On August 10
(A) The Temptation (C) On August 11
(B) Listen Here (D) On August 12
(C) Midsummer Day's Dream
(D) Little Thoughts

188. What is true about On the Whole'?


(A) It won an award for the best album

(B) It is disliked by fans despite receiving


critical acclaim.
(C) It has achieved the highest sales
among all of Ms. Krall's albums.
(D) It contains a song ranked third on a
global chart. GO ON TO THE NEXT PAGE.

TEST 7 PART 7 227


Questions 191-195 refer to the following advertisement, form, and e-mail.

Subscribe to News Incorporated!


Who says people don't read newspapers anymore? They still do, only they've found
a more convenient medium—the Internet. And when it comes to online news, smart

readers turn to News Incorporated. Why? For one thing, they get news that is
updated continuously and is easily accessible on computers, mobile phones, and
tablets. Another reason is that we post dozens of pictures online related to current
events. What's more, you'll have full access to our archive of news articles dating
back more than five years. And, of course, you'll be able to post your opinions and
read those of others.
I /1
News Incorporated provides hundreds of news stories each day, all of which are
researched and written by some of the best journalists in the field. We make sure
that all of our news is accurate and timely. When you subscribe to News
m /
Incorporated, you'll get complete coverage of a wide range of topics, including
c o
c.
politics, sports and entertainment. Readers can also gain access to bonus materials
such as videos by subscribing to our monthly Web magazine.

g o
Visit www.newsinc.com and sign up for a month-long subscription to our online

s
newspaper before July 10 to get another month of access for free.
n
g m
k i n
News Incorporated
e a
sp
www.newsinc.com

e ic
//to
Home Digital Subscription Services
Please choose the service you want to subscribe to, read and agree to the

tp :News
World
terms and conditions, then click "NEXT." For more details, click on the package

ht
or service.
Local
Politics Regular Packages
Sports Web Only
Entertainment □ $2.75/week
Technology □ $11/month
Culture/Lifestyle □ $110/year
Web/Mobile Phone
Videos □ $3.85/week
□ $15/month
My Account □ $150/year
Web magazine $8.00/month □
Subscription
Terms and conditions (Read)
Contact Us I have read and agree to the terms and conditions. □

NEXT

228 my y £0)op| MP3 Hackerslngang.com


To; News Ina
From: Katheri
Subject: Subs
Date; Jul

Dear Customer Service,


I purchased a month-long subscription package on your Web site yesterday, and although I got access to your
content on my computer immediately, I am unable to view it on my mobile phone, I downloaded the application as
instructed, but whenever I log in, my access is restricted. If you check my account details, you will see that I have
paid to have access on my mobile phone as well. Could you please correct this error or send me instructions on
what to do?
Sincerely,
m /
Katherine Andres
c o
o c.
191. What is NOT mentioned as a feature of
g
194. What is suggested about Ms. Andres?

n
News Incorporated?
(A) Award-winning writers
m s
(A) She is unfamiliar with the features of
her phone.
(B)
(C)
Access to images
Continuous updates
i n g
(B) She will receive online news content
for two months.
(D) A user comments section
a k (C) She spoke to a technician over the

e phone.

sp
192. How can subscribers access video footage? (D) She used to subscribe to the
(A)
(B) ic
By entering a promotional code

e
By paying for a full year's subscription
publication's print edition.

//to
(C) By signing up for an online magazine 195. How much did Ms. Andres pay for her
(D)

tp : By subscribing to the printed


publication
subscription?
(A) $2.75

ht
193. Why did Ms. Andres write the e-mail?
(A) To express appreciation for a Web
(B)
(C)
(D)
$3.85
$15
$110
service
(B) To alter her original subscription
package
(C) To inquire how to download the
application
(D) To report her inability to access content

GO ON TO THE NEXT PAGE

TEST 7 PART 7 229


Questions 196-200 refer to the following e-mails and invoice.

TO: Gene Lipton <geneIipton@beetleadagen


FROM: James Nichols <jamesnichols@beet
SUBJECT: Change
DATE: April 8
ATTACHMENT: New script

Hi Gene,
I got the revised script for the Tungsten motorcycle television commercial this morning. The client
understands that giving us a new version now is short notice as we are scheduled to shoot on Thursday, but
the changes appear to be very minor. Anyway, I will forward the new script to the cast as some of their
lines are different, and I've attached a copy for you, too.
Also, I spoke to Noah Pierce at the filming venue and asked him if we could extend the shoot until
8 o'clock. He said there is another shoot after ours at 6 p.m., so it wouldn't be possible on that date.
However, if we booked the day after, he could extend our shoot. I am fine with delaying filming by a day.
If you're in agreement, let me know, and I will confirm the rescheduling with Noah. It will give you the
m /
extra filming time you requested.

c o
c.
I'll be out of the office for the rest of the day, so just send a text message to my mobile phone if you have
any questions. Thanks, and I'll see you at the production meeting tomorrow afternoon.
James Nichols
g o
Accounts representative
Beetle Advertising Agency
s n
g m
k i n
||
e a
TO: Noah Pierce <npierce@hlproductionstudios.com>

sp
H FROM: Malory Bateman <mbateman@hlproductionstudios.com>
| SUBJECT: Re: Schedule change
|
1
DATE: April 9

e ic i?

//to
i!] Hi Noah, I

tp :
I got your message about the scheduling change. I understand that James Nichols wants to delay filming by a day.

ht
;1 I'm free to do hair and makeup for the TV commercial on April 12 instead of on Thursday, but I have to leave by ji
I 6:30 p.m. I have a function at my children's school to attend. This should be fine as I've checked out the script and I
i :l photos of the actors, and the makeup and hair are going to be straightforward. Additionally, the cast for the
j production isn't very big, and Juan Carlos will be there to do any necessary touch-ups on hair and makeup anyway.
If you need someone there for the entire shoot, maybe ask Diana Watson to fill in, as I don't think she's scheduled ;|
I for anything on Friday.
I i
sj Let me know sometime today whether or not you'd like me to do the job.
I Thanks!
Malory Bateman
I Production staff, Hinterland Production Studios

Invoice Date: April 12


From: Hinterland Production Studios
303 South Van Ness Avenue, Los Angeles, CA 90020

230 Hackers.co.kr
To: Beetle Advertising Agency
289 W. 33rd Street, Los Angeles, CA 99328
SERVICES RENDERED PRICE
Rental of film studio (April 12, 10 a.m.-8 p.m.) $2,200.00
Rental of sound equipment $280.00
Hiring of lighting and sound operators (two staff at $400 each) $800.00
Hiring of makeup and hair artists (two staff at $600 each) $1,200.00
SUBTOTAL $4,480.00
TAX $480.00
AMOUNT OWED $4,960.00
Payment for the total amount is due within seven days of the billing date. Payment may be sent via direct
transfer or made by credit card. For payments by corporate check, please visit our offices at the address

/
above between 9 a.m. and 6 p.m. from Monday through Thursday,

o m
196. What has Beetle Advertising Agency been

c. c
199. What can be inferred about Juan Carlos?
asked to do?
(A) Find some different cast members
g o
(A) Diana Watson will replace him for part
of the day on April 12.
(B) Select a venue for an upcoming shoot
s n
(B) He was unavailable to work extended
(C) Produce different versions of a
commercial
g m hours on the requested date.
(C) Beetle Advertising Agency was
(D) Use an updated version of a script

k i n charged $600 for his services.


(D) He was forwarded a copy of a script by
197. What is indicated about Mr. Lipton?
e a Ms. Bateman.

Friday.
ic sp
(A) He agreed to film a commercial on
200. What is true about Beetle Advertising

e
(B) He spoke to Mr. Pierce about a Agency?

//to
schedule. (A) It sent Mr. Nichols to Van Ness Avenue

tp :
(C) He wanted to make some minor script
changes.
to complete a payment.
(B) It rented lighting equipment from

ht
(D) He extended the rental of a venue by a Hinterland Production Studios.
day. (C) It received a discount on some staffing
services.
198. In the second e-mail, the word "function" in (D) It is located in the same city as
paragraph 1, line 3, is closest in meaning Hinterland Production Studios.
to
(A) duty
(B) event
(C) purpose
(D) result

This is the end of the test. You may review Part 5, 6, and 7 if you finish the test early.

igii

p.325 / Sits p.327 / StH p,378 / Part SS6 6||M HfS fi7|
• c® liiioi jioii Self *l|a a|AMi §611 xhjo| s*ii m "m EflsLU ggsn axiia

TEST 7 PART 7 231


oH3H± m1 ^ lOOOIil 3 READING

m /
c o
o c.
n g
m s
i n g
a k
e
sp
I TEST 08

e ic
p ://to PART 5

ht t 0^ PART 6

01 PART 7

Self ilk a|AE

EII^E *! IfOjAHh
1. me| SSS p^Wfi? o oil
2. Answer Sheet. 21. ^NlSulSHaLfo? n
3.A|7j|lSH|6H!3L|S? coil
□ = gapi 2tsEisi°2 cc«a 4 eiiams

^|:#0|1. Dj^lxg^EI 70^ ^0J _A| .MbJM'-K


ApJ^ # 75^0|01, A|°t #5 2 5^a ge □] «ohx| 0^# ?|6|| A^L|cK
READING TEST
In this section, you must demonstrate your ability to read and comprehend English. You will be given a
variety of texts and asked to answer questions about these texts. This section is divided into three parts
and will take 75 minutes to complete.
Do not mark the answers in your test book. Use the answer sheet that is separately provided.

PARTS
Directions: In each question, you will be asked to review a statement that is missing a word or phrase.
Four answer choices will be provided for each statement. Select the best answer and mark the
corresponding letter (A), (B), (C), or (D) on the answer sheet.

© PARTS S^tolAltr 11g

m /
101. The meteorologist reviews the daily
o
105. After Benton Enterprises adopted the

c
c.
weather patterns and makes for of allowing its staff to work flexible
temperatures and conditions.
(A) predicts (A) practice o
hours, productivity began to rise.

g
(B) predictions
s
(B)
n
practically
(C)
(D)
predicted
predictably
g m
(C)
(D)
practiced
practitioner

102. Sommerland Shopping Mall is situated


k i n 106. Evergreen Hotel was built at a time when
ten minutes away from the
e a Baldwin City strong growth, but it

sp
downtown area. may close if the recession continues.
(A)
(B)
e ic
approximating
approximated
(A)
(B)
undergoes
is undergoing

//to
(C) approximately (C) will undergo

tp :
(D) approximate (D) was undergoing

ht
103. All airlines are required by law to keep 107. The display coordinator's role is to ensure
inflatable life jackets located that everything sold in the store is
passenger seats in case there is an arranged to look appealing to customers.
emergency. (A) barely
(A) following (B) namely
(B) next (C) formerly
(C) except (D) carefully
(D) underneath
108. Many consumers agreed that Edgewood
104. Mr. Katz was confident that could do Limited's greatest strength was its in
a better job with database maintenance maintaining high levels of quality.
than his highly skilled team. (A) comparison
(A) less (B) component
(B) few (C) consistency
(C)those (D) conclusion
(D) whatever

234 Jig Hackers.co.kr


109. The human resources manager called a 115. Customers with packages exceeding 250
meeting on Monday everyone about centimeters in length are additional
his plan to hire new employees in the shipping fees by Bowden Couriers,
coming months. (A) charging
(A) informs (B) charged
(B) informed (C) chargers
(C) be informed (D) charges
(D) to inform
116. Participants will be given ample time after
110. Mary Rose was asked to join the the presentation to any concerns
information access team because she they may have about the marketing plan.
to similar projects previously. (A) raise
(A) was contributed (B) discharge
(B)
(C)
had contributed
contributes
(C)
(D)
screen
invest
m /
(D) will contribute

c o
c.
117. The new road from Batik Enterprise's
111. The accounting department's current warehouse to the post office has been a
software program is not for the tasks great
g o
for employees in the shipping
that need to be performed.
(A) adequate
department.
(A)
s n
registration
(B) competent
g m
(B) increment
(C)
(D)
comforting
proficient
k i n (C)
(D)
movement
advantage

e a
sp
112. Chef Alan Peralta's of classic French 118. The launch of Blanca Restaurant was
dishes is considered both unique and successful , but the proprietor wished

ic
daring in the culinary world.

e
the event had attracted greater attention

//to
from local media.
(A) obligation
(B) calculation (A) rather

tp :
(C) subtraction (B)enough

ht
(D) interpretation (C) soon
(D) yet
113. Work from artist Leah Mills' newest
collection was displayed at the 119. Edmonton Supply is to extend a
Beech Gallery in Atlanta. discount when customers place a
minimum order for 1,000 units of the
(A) exclusion
camping accessories.
(B) exclusively
(C) excludes (A) capable
(D) exclude (B) respective
(C) compatible
114. wishing to work overtime this month (D) willing
is reminded to advise the supervisor
before the end of the week.
(A) Whoever
(B) All
(C) Those
GO ON TO THE NEXT PAGE.
(D) Anyone

TESTS PARTS 235


120. Renovations can begin on Bounty Bank's 125. a few members opposing the plan, the
main offices the board authorizes the executive board has decided to go through
project. with the investment in Diehl Electronics.
(A) also (A) Notwithstanding
(B) pending (B) Consequently
(C) unless (C) Between
(D) once (D) Throughout

121. Ms. Wilson left behind a career in law to 126. An international body has the
her dream of running a formation of a protected area covering
bed-and-breakfast in Tuscany. large sections of the Amazon rainforest.
(A) aspire (A) notified
(B) pursue (B) approved
(C)
(D)
withdraw
contend
(C)
(D)
deducted
signified
m /
c o
c.
122. The staff in charge of writing press 127. Steeltop's machinery is solidly built and
releases should get all facts will operate for years without the
thoroughly before sending an
g o
need for costly maintenance and repairs.
announcement to the media. (A)
s explicitly
n
m
(A) checked (B) regretfully
(B)
(C)
checking
check
i n g
(C)
(D)
reliably
attentively
(D) checks

a k
e
128. The spokesperson for Beaumont Industries

sp
123. Ms. Reyes considered the amount of made an official apology the company
luggage she was taking on her beach for the way it dealt with a delivery delay.

e ic
excursion before deciding on car to (A) as soon as

//to
rent. (B) according to

:
(A) these (C) in spite of

tp(B) where (D) on behalf of

ht
(C) other
(D) which 129. Copyright have the exclusive right to
use, modify, and distribute the images they
124. A top competitor withdrew from the upload to Westforth Corporation's Web site.
international tennis tournament due to a (A) held
injury. (B) holds
(A) captivating (C) holding
(B) prescribing (D) holders
(C) towering
(D) lingering 130. the weather is pleasant, the
company's social gathering will occur at
Ogilvy Park this weekend.
(A) Rather than
(B) Assuming that
(C) Owing to
(D) Hence

236 ¥esat?!i!Esiao|^htK>i?}?IS5!90)S!?IMP3 Hackerslngang.com


PART 6
Directions: In this part, you will be asked to read four English texts. Each text is missing a word,
phrase, or sentence. Select the answer choice that correctly completes the text and mark the
corresponding letter (A), (B), (C), or (D) on the answer sheet.

© PART6 S%hw0l ^ se

Questions 131 -134 refer to the following article,

Royal Exchange Building Reborn as Hotel

Nationwide hotel operator Verdant Group niillions of pounds over three years

m /
converting the Royal Exchange Building on Quay Street, a historical landmark, into a hotel.

c o
o c.
"We wanted to preserve the building's key architectural elements," says Verdant's CEO Gaile

n g
Edwards, "That's why we hired a group of historical experts to work with our team."

m s
The building retains much of its original charm as the 18th century structure and stonework

i n
remain intact. However, the building's interior has all the g found at any other five-star

accommodations, such as a pool and spa.


a k 133.

e
sp
The combination of the hotel's modern facilities with its historic appearance makes the venue

ic
a major
e
for tourists. Already, suites are fully booked for its opening weekend, which is

: //to
scheduled for late next month.

p
ht t
131. (A)spends 133. (A) activities
(B)spent (B) priorities
(C) will spend (C) opportunities
(D) spending (D) amenities

132. (A) It will take another year before the 134. (A) issue
||j| hotel opens its doors. (B) accomplishment
(B) This is Verdant Group's first project (C) attraction
that is a joint venture. (D) commitment
(C) Judging from the results, it seems that
they were successful.
(D) Consequently, the construction firm's
initial proposal was rejected.

GO ON TO THE NEXT PAGE_

TESTS PART 6 237


Questions 135-138 refer to the following letter.

July 15
Allison Morita
Vestige Insurance
4186 Maryland Avenue
Pinellas, FL 34624

Dear Ms. Morita,

I am writing to you in the hope that you can my insurance claim. I spoke with general
135.
claims agent Gary Fink on July 6, , at the time, explained the process and
136.
recommended that I write this letter.

m /
c o
Last month, on June 20, I suffered an injury when I slipped and fell in my kitchen. The impact

o c.
caused me to break my wrist, which forced me to undergo surgery. Does my policy cover

injuries of this nature?


n g
I expect to be reimbursed. Currently, my medical expenses

amount to about $900.


m s
i n g
k
Please respond as soon as you review my documentation.

a
e
sp
Thank you.

Sincerely,
e ic
p
June Miller

: //to
ht t
135. (A) cancel 138. (A) I have enclosed receipts to support this
(B) handle ill claim
'
(C)change (B) Only half of the amount has been paid
(D) summarize by your company.
(C) It is difficult to determine who was at
136. (A) when fault for the accident.
(B) who (D) Let me know when my insurance
(C) how contract has been authorized.
(D) why

137. (A) If so
(B) Until then
(C) After that
(D) On condition of

238 Hackers.co.kr
Questions 139-142 refer to the following announcement.

Welcome to Redstone National Park

For the protection of the park, all visitors are asked to observe some basic
139.

Redstone National Park officially closes at 8 p.m. there are a number of campsites

situated throughout the park for those who wish to stay overnight. It is important to note that

this option is only available to those with permits.

We also ask that all visitors be thoughtful about maintaining the premises. Please make sure

that rubbish and anything brought into wildlife areas is taken out upon leaving or disposed of
m /
in the appropriate receptacles.
c o
o c.
Following these rules will help to ensure the
g
of the park's beauty for future visitors.

n
For any questions or concerns, please call 555-9092.
m s
i n g
139. (A) preventions
a k 141. (A) We project that these campsites will be
(B) demonstrations
e |||| completed by the end of the year.

sp
(C) policies (B) It must be closely monitored by park

ic
(D) corrections rangers at all times,

e (C) The easiest way to get to the park is

//to
140. (A) Previously by taking a shuttle bus.
(B) Besides

p
(C)

t : However
(D) These can be obtained at the visitor
center every day until noon.

ht
(D) Moreover
142. (A) preserves
(B) preservation
(C) preservative
(D) preserved

GO ON TO THE NEXT PAGE^

TEST B PART 6 239


Questions 143-146 refer to the following e-mail.

To: Janet Boyle <jboyle55@overmail.net>


From: Customer Service <service@lagoonair.com>
Subject: Your inquiry
Date: July 29
Attachment: Baggage claim form

Dear Ms. Boyle,

This is in reply to your inquiry about baggage. Problems involving luggage on domestic
143.
flights must be reported to airline personnel at an airport within 48 hours of flight arrival.

However, if you have flown in from outside the country, you may report any destruction to
m /
your luggage to claims@lagoonair.com using the attached form. Claims can also be
c o in

c.
144.
person at an airline office.
145.
o
, The airline will not grant any claim made more than 14 days

g
following your flight.

s n
g m
Lagoon Airlines is not liable for any harm to luggage that is of poor quality or possesses an

inherent defect.
k i n
, reimbursement for repairs is not offered for minor wear and tear.
146.

e a
sp
Sincerely,

e ic
Lagoon Airlines Customer Service

p : //to
t
143. (A)

ht
(B)
(C)
delayed
damaged
unattended
145. (A) You will receive confirmation of your
flight reservation by e-mail.
(B) Refer to your ticket to view the
(D) allowable baggage allowance for this flight.
(C) We will deliver the bag to your address
144. (A) submitted after it has been recovered.
(B) retrieved (D) Please note that there is a deadline to
(C) denied apply for reimbursement.
(D) waived
146. (A) Thereafter
(B) Nonetheless
(C) Additionally
(D) Otherwise

240 ^• E!01d7Io aAMP3 Hackerslngang.com


PART 7
Directions: In this part, you will be asked to read several texts, such as advertisements, articles, instant
messages, or examples of business correspondence. Each text is followed by several questions. Select
the best answer and mark the corresponding letter (A), (B), (C), or (D) on your answer sheet.

© PART 7 m «ol xpj 54©

Questions 147-148 refer to the following memo.

ARGENTA SCIENCE LABORATORIES

Date: August 27

m /
To: All personnel
c o
c.
From: Thomas Sutton, facility manager
Subject: Water crisis

g o
s n
As you know, the state is in the middle of a water crisis, and has
asked that all residents and businesses adopt immediate conservation

g m
measures. Therefore, we are asking everyone to manage their

i n
individual consumption responsibly and avoid unnecessary use,

k
particularly in the kitchenette and in the restrooms. For the time

e a
being, I have also had the drinking fountains in the hallways shut off,

sp
but bottled water for consumption will be made available in staff

ic
lounges, Thank you for your cooperation, and feel free to notify me of

eany concerns or questions.

p : //to
ht t
147. What is the purpose of the memo?
(A) To announce the results of a
148. What did Mr. Sutton recently do?
(A) Met with a government official
scientific study (B) Renovated some laboratory facilities
(B) To report the findings of a water (C) Purchased new equipment
system inspection (D) Restricted access to an amenity
(C) To request that staff monitor their
water consumption
(D) To explain a measure to treat water
contamination

GO ON TO THE NEXT PAGE

TESTS PART? 241


Questions 149-150 refer to the following notice.

Notice of New Location

As of August 31, the Send It Right packing and shipping center will no longer be located at
this site. While we have enjoyed serving customers at this location situated inside the Jovan
Financial Building, we have grown beyond the space we have. On September 1, you will find
Send It Right across Centerville Highway in the Perkins Plaza, Suite 112. Our regular hours
of 7 a.m. to 10 p.m. from Monday through Friday will resume on that date.

Send It Right will have a much larger space that was specifically designed for our needs,
including a new self-packing zone for our customers who prefer to bring in their items and

/
boxes from home and do their packaging on site. Customers can also purchase our boxes,
offered in various sizes and shapes. We'll supply the tape, scissors, and worktable.

o m
c. c
We will continue to offer all the same services as before, but will assist you more quickly as
the additional space will allow us to hire more employees to work during peak hours. As a

o
reminder, our services include packing, shipping, mailing, faxing, and notarizing items as well

g
s n
as renting an expanded selection of individual mailboxes. We will continue to offer both
domestic and international courier services, and will send packages or documents through

m
our regular shipping services as well. We hope you continue to do business at our new

g
location.

k i n
e a
ic sp
149. What will Send It Right most likely do on
August 31?
150. What is suggested about Send It Right in

e
the Perkins Plaza?

//to
(A) Close its establishment for relocation (A) Its customers are permitted to park

tp :
(B) Interview applicants for a new facility
(C) Sign a rental agreement with Perkins
for free.
(B) Some current services will not be

ht
Plaza offered.
(D) Begin offering international shipping (C) It may employ more personnel than
options previously.
(D) It will extend its hours of operation.

242 xJIg Hackers.co.kr


Questions 151-152 refer to the following text messages.
A!
m Wlm

Sandra Fuller [2:23]


Thank you again for planning such a good client visit with
the representatives from IPD Toy Incorporated. I'm quite
impressed.

Brent Cavanaugh [2:25]


I'm glad to hear that. They seemed interested in placing an
order for some of the toy lines that I described during my
presentation.

Sandra Fuller [2:27]

m /
Oh, didn't you know? They contacted us this morning to say

c o
c.
they have decided to carry our Happy Abbey Doll line in all
their stores globally.

Sandra Fuller
g o
[2:28]

s n
They think that there may be a market for the dolls and have
agreed to order 500 units to start.

g m
Brent Cavanaugh

k i n [2:29]
That's great! This could really help us establish our brand
internationally.
e a
ic sp
e
: //to
151. At 2:27, what does Ms. Fuller mean when

p
152. What can be inferred about IPD Toy

t
^||| she writes, "Oh, didn't you know"?

ht(A) She thinks Mr. Cavanaugh should


have contacted a client.
Incorporated?
(A) It will request some changes to a
product's appearance.
(B) A customer's transaction was canceled (B) It operates retail outlets in different
at the last minute. countries.
(C) She expected Mr. Cavanaugh to be (C) It specializes in educational toys and
aware of a decision. games.
(D) A message was sent to the wrong (D) It manufactures most of the items it
person by mistake. sells.

GO ON TO THE NEXT PAGE.

TEST 8 PART 7 243


Questions 153-154 refer to the following e-mail.

'IQ " —"z— "" mz— "~~zizzzz ——-—■' ■ ' „ ,,,, ,,, ^■|-- - i

To: Kay Fine <kay.fine@beebonnethaU.com>
From: Joe Warner <joe.warner@sammonsproductions.com>
Date: November 17
Subjects: Sammons Productions booking change

Dear Ms. Fine,

Last month, I booked your facility to host our company's upcoming employee meeting and
luncheon on January 30. Unfortunately, I have just learned that our president will be out of
the country on that date. So, I would like to reschedule for the following week, if possible. My
first choice of date is February 5 at the same time as before, 12 p.m. Please notify me as soon
as possible if you have the space available. If not, let me know what dates are free.
m /
c o
c.
Also, you provided a selection of menu options for the luncheon, and I have had some time to
look through them. I would like to choose the grilled chicken along with spring pasta and

g o
tossed salad for the main course. For dessert, I will go ahead and select your cherry

s n
cheesecake. Also, I forgot to ask you about what types of beverages are offered, and I don't
see them on the list of menu options. Could you send me information regarding the options as
well?
g m
k i n
Feel free to reply to this e-mail or, if you prefer, you may call me at 555-6758. I look forward
to hearing from you shortly.

e a
sp
Sincerely,

Joe Warner

e ic
//to
; ijg)

tp:
153. Why does Mr. Warner want to change his 154. What does Mr, Warner ask Ms. Fine to

ht
company's reservation? do?
(A) An occasion's date has not yet been (A) Suggest a good restaurant
confirmed. (B) Provide a list of drinks
(B) An initially booked space will not be (C) Arrange a large room
available. (D) Send a revised invoice
(C) Another meeting was scheduled for
the same day.
(D) An executive will not be available to
come.

244 ?SSa)?!ilse|H5)iAh90i?f7l5(5!E!0)of7|MP3 Hackerslngang.com


Questions 155-157 refer to the following information.

extosie 900

Please read the enclosed user's manual before assembling the product. Moreover, retain these
instructions for future reference. Do not use the product if any parts are missing. Contact us at
the number provided to have any missing parts shipped to you.

Before each use, check that all pieces are securely fastened. You risk serious injury if the machine
has not been attached correctly. Components may come loose after an extended period of use, so
it is vital that you regularly check that everything is in order and make repairs as necessary

Caution:
1. Always consult a qualified doctor before undertaking any exercise program. Excessive or
incorrect training on this apparatus may cause injury

m /
o
2. Make sure the equipment rests on a flat surface and that all bolts, pins, and fasteners are secure

c
c.
before beginning your workout.
3. Wear suitable clothing when using the product. Avoid loose attire and jewelry of any kind, as
such items may catch when exercising with this equipment.
g o
s n
4. Ensure there is enough clearance around the machine to permit unrestricted use.
5. At no time should young children be allowed access to the machine, even when it is not in use.

g m
n
For general care and maintenance, wipe upholstered surfaces with a warm, damp cloth. Do not

k i
use the Flextone 900 if signs of damage are present.

e a
ic sp
155. What type of product most likely is the 157. What is NOT a recommendation for users?
Flextone 900?
e (A) Adjusting the settings for children

//to
(A) A workout machine (B) Checking that components are secure
(B)

tp : A kitchen appliance (C) Keeping sufficient space around the

ht
(C) Factory equipment machine
(D) A health supplement (D) Wearing appropriate types of clothing

156. What is stated about the Flextone 900?


(A) Some of its pieces are sold separately.
(B) It is not for use in the home.
(C) It requires some maintenance.
(D) It must be attached to a charger.

GO ON TO THE NEXT PAGE,

TESTS PART7 245


Questions 158-161 refer to the following online chat discussion.
o fl
■-= a
h OiSI i

Blake Dunlap 10:28 a.m. Hi, everyone. I'm happy to announce that Alstrop's board of directors
has decided to open a sixth branch in Cleveland. Consequently, diey'd
like to transfer existing employees so that the branch is launched as
quickly as possible. In fact, I need each of you to submit some staffing
recommendations.
Cathy Schiiltz 10:30 A.M. Would the transfers be temporary? Many of our staff members have
lived here in Inglewood their entire lives. They won't be happy about
this.
Blake Dunlap 10:31 A.M. That will depend on the preference of each employee. Naturally,
performance is also a factor.
Roy Reese 10:31 a.m. When's our target opening date?
Blake Dunlap 10:32 a.m. The branch must be fully operational by November 15 to take
advantage of our year-end tire sale.
m /
o
George Kesterson 10:32 a.m. If you're looking for an office administrator, I think Tammy Roselli
c
c.
would do a great job. She does have a family here though, so I'm
unsure if she'd want to move.
Roy Reese
g o
10:34 a.m. And for sales director, Charles Kang would be a good choice.
Blake Dunlap
s n
10:36 a.m. Great suggestions! Any ideas for a purchasing clerk? We will need
someone to start ordering merchandise right away.
Cathy Schultz
g m
10:38 a.m. What about Carson Drake? He's been working in the same position

Blake Dunlap
k i n
for years and he needs a challenge.
10:40 a.m. That is a good idea. Anyway, there are a number of other roles to fill,

e a
and I'll forward you all a list later today. Message me or drop by

sp
human resources if you have any other ideas.

e ic
//to
tp:
158. What kind of business most likely is 160. At 10:38 a.m., what does Ms. Schultz mean
Alstrop? Ull when she writes, "he needs a challenge"?

ht (A)
(B)
(C)
A
A
A
retailer of tires
home builder
staffing firm
(A) She believes finding the right
employee will be a challenge.
(B) Mr. Drake wishes to be assigned a
(D) A moving company recruitment task.
(C) She thinks a staff member should take
159. What is mentioned as a concern about on a new responsibility.
staff transfers? (D) A purchasing clerk is deserving of a
(A) The cost of relocating a workforce is promotion.
high.
(B) Staff may not want to move away from 161. What will Mr. Dunlap do later in the day?
their current homes. (A) Go through some phone messages
(C) There is insufficient time to carry out a (B) Supply a list of open positions
move. (C) Meet with human resources staff
(D) New administrative employees have (D) Forward some suggestions to a
limited experience. superior

246 Hackers.co.kr
Questions 162-164 refer to the following bill.

INVOICE
Clean Genie
3102 Flamilton Boulevard Date of invoice: August 8
Allentown, PA 18103 Job # C6512-2
555-7681
www.cleangenie.com
Bill to:
Vasco's Bistro
501 Broad Street, Emmaus, PA 18049
555-0219
Description of Services Price
Fully inspected premises and issued a report dated July 24 $9.00
Disinfected all surfaces with Clean Genie Surface Wash
Vacuumed carpeting in main dining area and applied Clean Genie Protect
Sanitized air in main dining area with Clean Genie Fresh Burst
m /
Removed items marked for disposal
c o 9HI

c.
Performed follow-up inspection and cleaning on August 1 •J
•II
Issued laboratory-certified cleanliness report dated August 5
Provided Lehigh County Association of Food Retailers membership discount
g o ($15.00)
Comments
Client requested service as part of obligation to meet permit
s n Sub-total
Additional discounts
$111.50
($15.00)
requirements of the Emmaus Borough Department of Food

g
Sanitation. Additional discounts applied for using Clean Geniem Tax
TOTAL
$9.00
$105.50
patented formulations.
Payment information
k i n Customer Signature
Payment method:
e a Signing your name below indicates your

sp
□ Cash acceptance of the job as performed and
^ Check no.: 201-98942-2 agreement with all charges.
□ Credit card no.:

e ic Svcbttt P7H<wie
Evelyn Moore, supervisor

p : //to
ht t
162. What is indicated about Clean Genie?
(A) It did not charge for some services.
164. Who most likely is Evelyn Moore?
(A) A city sanitation official
(B) It makes some of its own cleaning (B) A dining facility manager
products. (C) A professional cleaner
(C) It holds membership in a food industry (D) An event organizer
association.
(D) It has visited Vasco's Bistro three times.

163. Why did the client request cleaning services?


(A) It received some customer complaints.
(B) It is preparing its venue for an
important event.
(C) It received a discount coupon in the
mail.
(D) It has to comply with cleanliness
standards. GO OA/ TO THE NEXT PAGE,

TEST 8 PART? 247


Questions 165-167 refer to the following e-mail.

To: Edward Morton


From: Antonio Parri
Subject: Yo
Date: July 2
■ W

Dear Mr. Morton,


Greetings from Alive magazine. I am very pleased to inform you that your entry for our
Readers Travel Writing Competition, A Three-Day Weekend in Mexico City, has been
selected to appear in print in next month's issue. — [1] —.
As you may know, Ms. Josephine Tan, who writes for the travel guide Pathways to the
World, was invited to judge all the entries. As the author of the winning entry, your name
will be announced in our September issue. You will also receive a $500 voucher from www.
bookingpros.com, redeemable at one of over 2,000 hotels and resorts around the world.
m /
— [2] —. Two runners-up will also be chosen by Ms. Tan, with winners receiving a $150

c o
c.
voucher from Goliath Luggage.

g o
Our editor, Mr. Jason Carter, has also instructed me to ask if you have any photographs of
your trip. — [3] —. If so, please e-mail them to me as we may choose to print a few of
them as well. — [4] —. Visuals would really help bring your story to life.

s n
magazine for years to come.
g m
Congratulations once again, and we hope you continue to be a loyal subscriber to Alive

Yours truly,
k i n
Antonio Parrish
Marketing associate
e a
Alive magazine

ic sp
e
//to
165. Who is Ms. Josephine Tan? 167. In which of the positions marked [1], [2],

tp :(A) A magazine publisher wSm [3], and [4] does the following sentence
best belong?

ht
(B) A public speaker
(C) A professional writer "You will find it alongside other selected
(D) A literary critic entries in our regular section on travel."

166. What can be inferred about Mr. Morton? (A) [1]


(B) [2]
(A) His photographs will be featured on a
(C) [3]
cover.
(D) [4]
(B) He will meet Ms. Tan at an awards
celebration.
(C) He has traveled to Mexico City before.
(D) He will have to submit a revised draft.

248 51 B01?PI mpb Hackerslngang.com


Questions 168-171 refer to the following article.

Sonorum Brings Vixo Mob One Step Closer to Reality


By Albert Lepke, music correspondent

Music giant Sonorum, which holds the rights for over 80 record labels, has struck a global licensing deal
with online streaming provider Vixo, paving the way for the launch of Vixo Mob, a paid music subscription
service. Vixo already has the rights to stream content from several major record labels. — [1] —. But until
now, it lacked access to Sonorum's vast catalog of contemporary music.

The agreement with Sonorum will significantly increase the amount of content Vixo will be able to offer,
making Vixo Mob's long-awaited introduction more likely to occur. The premium service has been in
development for two years. — [2] —. "We're clearly excited about the prospect of seeing our artists' work
distributed through Vixo's Internet and mobile platforms," said Sonorum CEO Sandra Scheine. "While we

m /
appreciate our existing partnerships with some of Vixo's competitors, none of them have the reach that Vixo
has."

c o
o c.
Vixo has more than 180 million regular users, whereas the next largest streaming service, SoundStonn, has
only 20 million. — [3] —. The way the sites are used explains the disparity. Most people visit Vixo to watch

n g
free videos rather than to stream music. Meanwhile, over 48 percent of Soundstonn's users subscribe to the

be willing to do the same.


m s
site for a monthly fee so they can listen to music. — [4] —. It is unknown whether Vixo Mob's users would

i n g
a k
e
168. What is indicated about Vixo Mob? 170. What does the article mention about

sp
(A) It will be offered for free for a limited SoundStorm?

ic
time. (A) It is fast approaching the same level of

e
(B) It focuses on a particular genre of popularity as Vixo.

//to
music. (B) Almost half its users have signed up

tp :
(C) It has yet to be released to the public.
(D) It is largely dependent on advertising.
for a service.
(C) It achieved profitability in a short

ht
amount of time.
169. According to the article, what competitive (D) Some of its former executives now
advantage does Vixo have? work for Vixo.
(A) It has a substantial user base.
(B) It offers a flexible payment scheme. 171. In which of the positions marked [1], [2],
(C) It is available in several languages. mm [3], and [4] does the following sentence
(D) It provides excellent customer service. best belong?
"Once it is launched, users will be able to
stream a wider range of both music and
videos in a high-quality digital format."

(A) [1]
(B) [2]
(C) [3]
(D) [4]

GO ON TO THE NEXT PAGE.

TESTS PART? 249


Questions 172-175 refer to the following information.

Open Waters Cruises Cancellation Policies

Cruise travel involves long-term planning and preparation for


both cruise lines and passengers. We recognize that travelers 1
pi ']
may occasionally need to change their bookings and have
created the following cancellation policies. We established these
regulations after taking into consideration the costs incurred by
11 H
Open Waters Cruises along with the needs of our customers.
! ] ; 1
Clients may send an e-mail to bookings@openwaters.com or
write a letter to our reservations department at 98 Capri
Boulevard, Miami, FL 33124 to cancel their trip with us.
Customers may also make such a request by calling 1-800-555-
m /
c o
c.
2978. In all cases, you must provide our representative with a
phone number where you can be reached. All requests for

o
rescheduling to other cruise dates are handled on a case-by-case
g
I •;

s n
basis. Cancellation refund details are listed below. j j
Days until departure
15 or less
g m
Refund amount
None
16 to 30
i n
25 percent cruise package refund

k
31 to 60

e
61 or more a 50 percent cruise package refund
Full package amount

ic sp
For inquiries, call our reservations department at the toll-free

e
number indicated above.

p : //to
ht t

250 Hackers.co.kr
172. What is the main purpose of the information? 174. What is NOT indicated in the information?
(A) To promote a new service (A) Cancellation requests may be
(B) To give instructions on ticket purchase submitted over the phone.
methods (B) Passengers can change their cruise
(C) To notify passengers of a canceled dates if needed.
cruise (C) Additional charges may be imposed by
(D) To describe booking modification travel agencies.
procedures (D) Refund amounts vary based on days
from departure.
173. The word "established" in paragraph 1,
line 4, is closest in meaning to 175. According to the information, what is
(A) instituted required for cancellation?
(B) restored
(C) contained
(A)
(B)
A passenger number
Contact information
m /
(D)convened (C) A reservation voucher
c o
c.
(D) Signed forms

g o
s n
g m
k i n
e a
sp
O
ID

e ic O

//to
or

tp : !>•
mi

ht r*
CDo
UJ
ft)XI
CDo.
OQZD

GO ON TO THE NEXT PAGE

TESTS PART? 251


Questions 176-180 refer to the following advertisement and order form.

SAN FRANCISCO CUSTOM T-SHIRTS


Want a quick, cheap, and simple way to create a cool T-shirt? Visit San Francisco Custom T-Shirts
on 18th Street on weekdays between 9 A.M. and 5 P.M. Our team of design specialists will help
you create any T-shirt you could possibly imagine using clip art, a wide variety of fonts, and
thousands of color choices! Not just that, but we can also print photographs, or any other types of
art files that you bring us, provided that they are under 50 megabytes.
You can order as many T-shirts as you want! And we give a 12 percent discount for large orders
of 200 pieces or more. We can complete most jobs within one to three days, with larger orders
usually ready for pickup in one week!
i
All our shirts are made of finely-woven American cotton, and our prices are some of the lowest in
the city, with our basic white model with four-color printing starting at $12 per item. You can also
choose the color of material and select different styles including long and short sleeves as well as
m /
regular and V-neck collars, all of which can be provided in any size you need!

c o
c.
We also offer a free delivery service for orders of 20 items or more to the cities of San Francisco,

g o
San Jose, and Oakland. Shipping to any other destination can also be arranged for an additional
charge. To learn more about our product offerings, services, and prices, please visit www.
sanfrantshirts.com.

s n
g m
k i n
e a
Name
ic Harry Okata sp Company/Affiliation North Park Elementary School

e
//to
Order date March 14 Pick-up/Delivery date March 16
Phone number 555-1121 E-mail hokata@npeschool.edu

tp:
Delivery address 1812 Telegraph Avenue, Payment method Bank transfer

ht
(If applicable) Berkeley, CA 94701
Order Description: 20 children's basic white T-shirts with blue capital lettering saying NORTH PARK
INTRAMURAL BASKETBALL TEAM on the rear side in Sportnik font. The front side will display the school
logo in blue, black, and yellow (art file has been provided). Sizes needed are 5 large, 11 medium, and 4
small.
4 adult's basic white T-shirts for coaches with the same specifications. Sizes needed are 2 medium and 2
large.
Total number of products ordered: 24
Order taken by: Sam Boswell
NOTE: No refunds for T-shirt returns are permitted. If you wish to file a customer complaint, please call us
3t 555-5531 or visit our location at 2389 Tennant Boulevard in downtown San Francisco.

0
252 eo|iui-■ A eoi?PI MP3 Hackerslngang.com
176. How can customers receive a discount? 179. What is NOT indicated about the T-shirts
(A) By placing a bulk order requested by Mr. Okata?
(B) By filling out a survey (A) They will be paid for by a credit card.
(C) By using an existing design (B) They will be worn by elementary
(D) By visiting a new store school students,
(C) They will be delivered within a week.
177. What is indicated about Mr. Okata? (D) They will feature a school's symbol.
(A) His delivery was expedited upon
request. 180. What should customers do if they are
(B) He is the coach of a school basketball dissatisfied with their T-shirts?
team. (A) E-mail a completed complaint form
(C) His T-shirt design was created by a (B) Call an establishment's number
faculty member.
(D) He will pay an extra fee for his order,
(C) Return their purchases to a service
counter
m /
o
(D) Submit an original receipt to
c
c.
178. In the advertisement, the word "create" in Mr. Boswell

o
paragraph 1, line 3, is closest in meaning
to

n g
(A)
(B)
equip
reverse
m s
(C)
(D)
appear
produce
i n g
a k
e
ic sp
e
p : //to
ht t

TEST 8 PART? 253


Questions 181-185 refer to the following e-mails.

To: Martha Cale <mcale@speedmail.com>


From: Mark Summers <msummers@speedmail.com>
Subject: Furniture
Date: June 25
Attachment: Furniture Images

Dear Martha,
Thank you so much for the e-mail yesterday containing photographs and details of some items you have
found that may be appropriate for my new house. I can see that you have a clear idea of the design and
look I'm going for, and I appreciate your effort. I have selected some of the items you recommended, as I
think they will match well with the contemporary look of the home, and the furnishings and decor items I
already have:
m /
For the living room: The Pendleton ottoman and an armchair
c o
c.
For the dining room: The Regency International oak high-backed chairs and dining table
For the kitchen; The Best-West set of two wood chairs and a small circular table

g o
For the music room: The leather couch from Fulton Furnishings and the two-person sofa in black from
Eversmith Interiors
s n
g m
Attached to this e-mail are photos of some of the furniture items I will be bringing with me to my new
house. I hope that you can incorporate them into your final designs.

k i n
I also visited Bards Flardware Supplies to check out the selection of paint as you recommended. I do like

e a
the hues you suggested for the kitchen and dining area, but I think I would prefer something less colorful
than yellow for the living and music rooms. I think I would like something more neutral like white,
brown, or gray.

ic sp
I look forward to meeting with you again next week to see your design proposals and to leam about some
e
//to
of the other features you plan to include.

tp :
Regards,

ht
To: Mark Summers <msummers@speedmail.com>
From: Martha Cale <mcale@speedmail.com>
Subject: Re: Furniture
Date: June 25

Hi Mark,
Thanks for sending me the photographs. You have some lovely pieces, and I've
already started incorporating them into your home design plan.
I don't really have any Issues with the furniture you've selected to purchase, but I
will say that the two-person sofa seems far too big for the space you have. You may
want to consider a one-seat armchair of the same style produced by the same

254 Hackers.co.kr
company, as it would fit better in that location.
With regard to the paint colors, I will go ahead and place an order for the blue and
yellow paint for the kitchen and dining room, respectively. If you'd like something
neutral for the other two rooms you mentioned, I would say that a beige or ivory
color would go very well with the furnishings you have selected for them. However,
if you prefer brown or gray, I can work with those preferences as well.
Anyway, my team and I are already hard at work on this project. I also have some
fabric samples for curtains and cushions, which we can go through together during
our meeting next week on June 29.
In the meantime, please feel free to get in touch if you have any other requests or
requirements.
Best Regards,
m /
Martha
c o
o c.
n g
181. What is mentioned about Mr. Summers'
house? (A)
m s
184. Who most likely is Ms. Gale?
A furniture store employee
(A) It has had previous occupants.
i n g
(B) A hardware supply representative
(B)
(C)
It
It
is a one-story building.
has modern characteristics.
a k (C)
(D)
A home interior decorator
A commercial photographer

e
sp
(D) It was built last year.
185. What will Martha Gale do next week?

e ic
182. What did Mr. Summers send to Ms. Gale?
(A) Show some material samples to

//to
(A) Interior design plans for a home Mr. Summers in person
(B) A set of paint samples (B) Move some furniture for a customer
(C)

tp : Images of his belongings (C) Request a paint color change from a

ht
(D) A list of furniture stores supplier
(D) Ask her team to develop some design
183. What is indicated about the two-person concepts
sofa?
(A) It was a secondhand item.
(B) It will not fit well in the music room.
(C) It is incompatible with other furniture.
(D) It is entirely white in color.

GO ON TO THE NEXT PAGE_

TESTS PART 7 255


Questions 186-190 refer to the following announcement, form, and e-mail.
jll
osal
■Til
Dayton Bank
Dayton Bank regularly subsidizes outside skills-development courses for its employees. Beginning April 1,
Pittman Training will once again make the following courses available to us, and these can be taken at the
training facility or online. Employees who complete a course will receive a cerdficate from Pittman Training.

Course Instructor/Description Days/Time


Effective Teller 3 weeks Sam Penner: This course will enhance skills in effective Mon./Wed./Fri.
Operations teller operations and dealing with customer problems. 8:30-11:30 A.M.
There are no prerequisites.
Bank Employee 2 weeks Lee Ching: This course focuses on more complex bank Mon./Wed./Fri.
Development transactions, such as international wire transfers. 8:30-11:30 A.M.
Participants must pass Effective Teller Operadons before
taking diis course.
Tues./Thur.
1:30-6:00 p.M.
m /
Credit Risk 1 week Jean Barker: This course teaches how to minimize risk
c o
Mon.-Fri.

c.
Management when reviewing loan applicadons. Only those who have 1:00-5:00 P.M.
worked for Dayton Bank for two years or more are
eligible to enroll in this class.
g o
s n
For more information and to register for a course, please visit Pittman Training's Web site at www.
pittmantraining.com.

g m
k i n
e a
sp
www.pittmantraining.com/registration

Course Registration

e ic
//to
Please fill out the form completely and review your information before clicking on "Next."

:
* Name: Bernard Hinds

tp* Street Address: 121 Rosemont Avenue, Chicago, Illinois, 60615

ht
* E-mail address: bernardhinds@smail.com
* Telephone number: 555-2143
* Sign me up for the following course: Bank Employee Development
* Preferred schedule: Tues./Thur.
* Please indicate whether you meet the prerequisite requirement: Yes H No □ N/A (Not Applicable) □
Payment information
Click Online payment to pay by credit card. If you wish to pay by bank transfer, please note the following
account information:
Bank name: First Standard Bank
Account Number: 3948-293
Payments by check should be mailed to Pittman Training, 9000 South Emerald Avenue, Chicago, IL
60620.
Credit card and bank transfer payments are processed immediately, and your enrollment will be
confirmed here after clicking "Next." If you send a check, we will send you an e-mail confirming your
enrollment within three business days of receiving your payment.

lESil

256 MP3 Hackerslngang.com


To: Pittman Training inquiries M
From: Bernard Flinds <bernard IB um
Subject: Follow u
Date; March 26

To Whom it May Concern,


I would like to follow up on my registration for the Bank Employee Development training course. I submitted
my payment one week ago and received an e-mail confirming my enrollment. However, I still haven t received
the course syllabus. Please send it to me as soon as possible so that I can begin preparing for the class.
Also, a colleague of mine is interested in taking the same course. I realize that it may be too late to register
since the deadline has passed, but I am wondering whether an exception can be made if the class isn't already

m /
o
full. Thanks,
Bernard Hinds

c. c
g o
186. What is NOT true about the courses at
s n
189. What is one purpose of the e-mail?

m
Pittman Training? (A) To ask about enrolling another student
(A) They include a certificate of completion.
(B) None of them are scheduled for a
i n g
(B) To inquire about paying for another
course
weekend.
a k (C) To find out a registration deadline

e
(C) They can be taken over the Internet. (D) To announce a planned absence

ic sp
(D) All of them take place in the afternoon.
190. What can be inferred about Mr. Hinds'

e
187. What is indicated about course registration? payment?

//to
(A) It requires applicants to visit a Chicago (A) It was submitted after a deadline.

tp :
office.
(B) Its price is discounted if completed
(B) It must be processed by the end
of April.

ht online.
(C) It can be canceled after the payment
is processed.
(C) It was mailed to an office on South
Emerald Avenue.
(D) It was not made in full.
(D) Its confirmation may take more than a
day.

188. What is suggested about Mr. Flinds?


(A) Ffe has been with Dayton Bank for at
least two years.
(B) He will be participating in classes on
weekday mornings.
(C) He has taken the Effective Teller
Operations course before.
(D) He will be traveling to a different city
for his classes.
GO ON TO THE NEXT PAGE.

TESTS PART 7 257


Questions 191-195 refer to the following memo, schedule, and e-mail.

Vora Advertising
MEMO
To: All staff
From: Joseph Tran, Human resources director
Subject: Staff retreat
Date: August 1
I'd tike to remind everyone that our annual staff retreat is coming up, This will take place from August 20
to 22 at the River Ranch Resort in Holbrook, which is a three-hour drive away. The company has
rented a bus that leaves Phoenix at 8 a.m. on the 20th. For those of you who cannot make it at that
time, Michelle Salazar will pick you up at around 2 p.m. Please contact her to make arrangements,

m /
o
We've planned several activities for this outing, and copies of the agenda have been circulated among

c
c.
the departments. We will be giving everyone the day off on the Monday right after the retreat. If you
have any questions, feel free to call me at extension 39.

g o
s n
g m
Vera Advertising Annual Staff Retreat

k i n
River Ranch, Holbrook, AZ
Friday, August 20

e a
sp
11:00 a.m. Arrive at River Ranch
12:00 P.M.

e ic
2:00 P.M. Leisure activity: Horseback riding with a guide

//to
5:00 p.M. Team building: Improving communication skills
6:00 P.M. Team building: Creative problem solving

tp : 7:00 P.M. Dinner

ht
Saturday, August 21
8:00 a.m. Breakfast
10:00 a.m. Seminar: Embracing innovation
12:00 p.m. Lunch
2:00 p.m. Leisure activity: Guided Whitewater rafting
7:00 p.m. Dinner
8:30 p.m. Staff meeting: Outlook for the next quarter
Sunday, August 22
8:00 a.m. Breakfast
9:00 a.m. Closing remarks from CEO Kent Vera
10:00 a.m. Group photo
10:30 a.m. Depart for Phoenix

All meals will be served in the lunch room located in the main building. All
outdoor leisure activities are subject to change depending on the weather.

258 Hackers.co.kr
To: Joseph Tran <j.tran@veiaadvertising.com>
From: Steven Oliver <s.oliver@holbrookriverranch.com>
Subject; Re: requests
Date: August 12

Hi Joseph,
I'm writing this e-mail to let you know that I carried out all of the requests in your last message. As you
asked, we booked three double rooms for Vera's top management, and nine other standard rooms for
the staff. We have also made sure that vegetarian and gluten-free menu options are available in
addition to our regular ones. Finally, I asked the kitchen staff to keep the lunch room stocked with hot
and cold beverages, as well as light refreshments, throughout each day of the retreat. There is one issue
you should be aware of, though. The main building will be closed for repainting on Sunday, August 22.
So, I recommend that you use the annex for your activities on that day instead. If there is anything
m /
more I can do at this time, please let me know. Thank you!
c o
Steven Oliver

o c.
n g
191. What can be inferred about Ms. Salazar?
m s
194. According to the e-mail, what has NOT
(A) She volunteered to bring her own
vehicle.
i n g
been requested by Mr. Tran?
(A) Preparing meals to satisfy different

a
(B) She will not participate in horseback
k preferences
riding,
e (B) Arranging specific rooms for staff

ic
her accommodations. sp
(C) She will be reimbursed for the cost of members
(C) Furnishing an annex with tables and

e
(D) She is in charge of arranging a staff chairs

//to
meeting. (D) Stocking refreshments throughout

p :
192. In the memo, the word "circulated" in
t
the day

ht
paragraph 2, line 1, is closest in meaning 195. What does Mr. Oliver suggest that
to Mr. Tran do?
(A) distributed (A) Finish a corporate event earlier than
(B) revolved scheduled
(C) transferred (B) Confirm reservations for a particular
(D) exhibited activity
(C) Flave breakfast at another location
193. What will occur twice during the staff (D) Make arrangements for additional
retreat? transportation
(A) A photo shoot
(B) A speech by an executive
(C) A communication skills workshop
(D) A guided leisure activity

GO ON TO THE NEXT PAGE

TEST B PART? 259


http ://www. botwe 11 sto re. co m/news/d utyfree

HOME CONTACT MEMBERSHIP PROMOTIONS HOURS/LOCATION

Coming Soon ... Duty-Free Shopping at Botwell Department Store


Botwell Department Store, situated in the heart of Cape Town, is pleased to announce that on April 12 it
will be opening a new duty-free store designed for all international travelers. Located on Botwell's fifth
floor, the establishment will feature luxury goods such as cosmetics, perfumes, liquors, jewelry, and
more. In celebration of the opening, all items by Moreno Luggage and Lydia Cosmetics will be marked
down by 20 percent until May 15. To buy any product being sold in the duty-free store, customers will
need to provide a passport and a ticket that will be used for an international departure within 28 days.
Those who are members of the Botwell Frequent Shopper program can receive an additional 10 percent
off all duty-free products.

m /
c o
BOTWELL DEPARTMENT STORE
o c.
n g
I'm very grateful to everyone for helping to make the first week of our duty-free store a success.

important things to take note of.


m s
Overall, I am pleased with your efforts and with how well the store ran. However, there are a few

i n g
First, I received a number of complaints about our voucher program. Please recall that it is our

a k
policy to give vouchers to customers who make purchases totaling $500 or more. These vouchers
allow them to take advantage of discounts on our partner establishments' Web sites. Please
e
sp
remember to give one to every eligible customer.

ic
Second, if a member of our frequent shopper program has lost his or her card, we cannot offer the

e
reduced price. However, shoppers may keep their receipts and get reimbursed for the difference

//to
once the card has been located or replaced.

tp :
Thank you for your attention to these matters, and keep up the good work.

ht
partment Store Receipt
Issued: April 19
To: Emilia Fortich Citizenship: Spanish
Passport number: XCV81324
Flight: Vela Airways VI342
Date of departure: April 21 Destination: Madrid, Spain

Item Quantity Price


Riley leather handbag (plum) 1 $160.00
Lydia Cosmetics lipstick 1 $46.00
Leganz digital camera 1 $258.00
Daphne Boutique scarf 1 $62.00
TOTAL $526.00

260 ^aA yof^PI MP3 Hackerslngang.com


Please note:
Cape Town Airport Authority (CTAA) regulations prohibit the transport of
more than 100 milliliters of liquids and gels onboard unless the products
have been purchased at a duty-free store and secured in a plastic bag
bearing the official CTAA seal. Furthermore, note that such items must be
removed from luggage when passing through a security checkpoint.

196. What is NOT mentioned about Botwell's 199. What is true about Ms. Fortich?
new establishment? (A) She misplaced her frequent shopper
(A) It is situated on the fifth floor of a card.
department store, (B) She signed up to be a member of a
(B) It requires the presentation of travel
documents.
program in April,
(C) She is eligible to receive a voucher for
m /
(C) It provides extra discounts to members a purchase.

c o
c.
of a program. (D) She must return to Botwell's store to
(D) It is opening a branch at an airport
terminal.
retrieve her items.

g o
197. What is the purpose of the notice?
s n
200. In the receipt, the word "secured" in
paragraph 1, line 3. is closest in meaning
(A) To announce future regulation changes to
g m
(B) To provide information about current
policies
k i n (A)
(B)
obtained
attached

e a
(C) To remind security workers about a (C) enclosed

sp
procedure (D) guaranteed

ic
(D) To suggest some solutions for a

e
system error

: //to
198. Which product did Ms. Fortich probably
buy at a discount?
p
ht t
(A) A
(B) A
camera
lipstick
(C)A handbag
(D) A scarf

This is the end of the test. You may review Part 5, 6, and 7 if you finish the test early.

aa p.325 / 94 Shtts p.327 / SJ-iil p,387 / Part 5&6 4S SHi ffS fi7| IPJ'-ACrv
* q-s hioixioii oifc Self *l|3 a|4Sl m *tdai m to| arxiq ehei ggsN axiia

TESTS PART 7 261


6P^ 10001 3 READING

m /
c o
o c.
n g
m s
i n g
a k
I TEST
e 09

ic sp
e
//to
PART 5

tp : «PART 6

ht ■M PART 7

Self *||H ai^

E1|^ E g ifojAHJ-
1.°c(|g$|o|gojSriMLfo? QO||
2. Answer Sheet. 'Eli!. x|°7Hir ^uloK3L^? □ oil
3.AIW^o|5^o? noil
S- -H|7^fi}-HE|04OD1 ^4^ilil o Ellaem A|^l|CK

n #01# mm ited mmE\ k ^ —Ai


EilAE A|7hS§75^0|D1,1® #5 2 5"° gg- ge □! gohX| °|S1| A^o}L|Ch
READING TEST
In this section, you must demonstrate your ability to read and comprehend English. You will be given a
variety of texts and asked to answer questions about these texts. This section is divided into three parts
and will take 75 minutes to complete.
Do not mark the answers in your test book. Use the answer sheet that is separately provided.

PARTS
Directions: In each question, you will be asked to review a statement that is missing a word or phrase.
Four answer choices will be provided for each statement. Select the best answer and mark the
corresponding letter (A), (B), (C), or (D) on the answer sheet.

© PARTS 11^

m /
o
101. Food manufacturers are responsible for 105. People who donate more than $50 to the
the safety requirements set by the
government.
c. c
Children's Cancer Group will receive a
T-shirt.
(A) fulfillment (A) compliment
g o
(B)
(C)
fulfilling
fulfilled
(B)

s
(C) n
compliments
complimented
(D) fulfill
g m
(D) complimentary

102. A number of people are lining up in the


k i n
106. To move items that are fragile and
Sherwood Hotel lobby
technology symposium.
e a
in the two-day breakable, the manager has rented

sp
specialized from Relocation
(A) enroll Shippers.
(B) enrolls
e ic (A) equipment

//to
(C) enrollment (B) condition
(D) to enroll (C) background

tp : (D) quality

ht
103. Tel-Corn Corp.'s merger with Voice
Messenger Inc. at a press conference 107. The Grenville Heritage Museum
on Monday last week. visitors from taking any photos or videos of
(A) was announcing the exhibition pieces.
(B) to announce (A) differentiates
(C) was announced (B) presents
(D) announced (C) prohibits
(D) releases
104. Mr. Kim went to Beijing by for a
digital media conference and met several 108. If you lose your room key, please it
potential clients there. to reception staff to get a replacement.
(A) he (A) reports
(B) him (B) report
(C) his (C) reported
(D) himself (D) reporting

264 ?SSar2!i!se(as|mf go|°i7®^9CH?}7|MP3 Hackerslngang.com


109. Reyman Steel is building a new warehouse 115. The Russian tennis player is confident that
the street from its main manufacturing she can defeat her opponent by a
plant. wide margin.
(A) without (A) slightly
(B) between (B) effortlessly
(C) across (C) cautiously
(D) inside (D) diversely

110. Dr. Germain dozens of research 116. The Magic Attic, a newly released movie
papers on electronic commerce before she by director Ron Speilman, is a family film
retired a few years ago. that is for all ages.
(A) will write (A) resistant
(B) writing (B) appropriate
(C) writes (C) simultaneous
(D) had written (D) equivalent

m /
111. The CEO is in the office but
o
117. Excursion Travel Agency has numerous

c
c.
communicates with his staff through listings for priced accommodations
frequent e-mails.
(A) immediately
within city limits.
(A) reasonable
g o
(B) similarly
s
(B) reasonably
n
(C)
(D)
rarely
concisely
g m
(C) reasonability
(D)reasoned

112. The federal government abandoned a plan


k i n
118. Reports that Shadco will be acquiring

e a
to build a pipeline through Alton City due Durbania remain at best as neither

sp
to overwhelming public company has confirmed any such plans.
(A)
(B)
opposition
oppose
e ic (A)
(B)
tentative
perpetual

//to
(C) opposes (C) formal
(D)

tp : opposing (D) consistent

ht
113. The Ergonicore office chair is , so it 119. All goods by ship must be thoroughly
can comfortably seat individuals of varying inspected by customs officials before
heights. entering the country.
(A) occupied (A) arrive
(B) productive (B) arrives
(C) adjustable (C) arriving
(D) committed (D) have arrived

114. The presentation will commence all 120. Brilla Boutique staff may refund a
the audience members have taken their customer's clothing purchase after a
seats. customer has presented a receipt.
(A) even if (A) when
(B) as soon as (B) only
(C) prior to (C) yet
(D) by means of (D) still
GO ON TO THE NEXT PAGE^

TEST 9 PARTS 265


121. Attending the shareholders' meeting takes 126. All Swift-Dent electronic toothbrushes are
for the president of Goodman warranty for a year from the time
Company despite his hectic work schedule. they are purchased.
(A) outlook (A) after
(B) effect (B) among
(C) priority (C) under
(D) direction (D) behind

122. Quest Airlines allows registered members 127. Production has been on Grant Auto's
to accumulate mileage points they new line of cars while engineers resolve a
buy a plane ticket. problem with the brake system.
(A) whatever (A) attracted
(B) instead (B) subtracted
(C) whenever (C) confronted
(D) that (D)suspended

m /
123. Offering rooms that can about 40
c o
128. Tai Shing Electronics usually outsources

c.
persons, Bean's Playpen is ideal for small manufacturing to foreign partners its
parties and events.
with demand.
g o
domestic facilities are unable to keep up
(A)
(B)
convey
accommodate
s
(A) but for n
(C)
(D)
furnish
familiarize
g m
(B)
(C)
in case
so that

k i n (D) up to

a
124. the unfavorable feedback Borebrooke

e
University received in the past, this year's 129. Patrick Jolson will be interviewed

sp
survey results show that students are three other promising candidates for the

ic
generally satisfied. position of research and development
(A) As far as
e head.

//to
(B) Compared to (A) even though

tp :
(C)
(D)
Given that
Rather
(B)
(C)
up until
in honor of

ht
125. Because the Web site for Workmates
Consulting was not updated, it listed
(D) along with

130. Eastwood County likely has a of


several job vacancies that had already natural resources given its proximity to
been filled. both a mountain range and the ocean.
(A) regularly (A) deliberation
(B) respectively (B) dedication
(C) numerically (C) wealth
(D) casually (D) leisure

266 ?ase«e-¥|SgMJi|g Hackers.co.kr


PART 6
Directions: In this part, you will be asked to read four English texts. Each text is missing a word,
phrase, or sentence. Select the answer choice that correctly completes the text and mark the
corresponding letter (A), (B), (C), or (D) on the answer sheet.

© PART6 to| XRt

Questions 131-134 refer to the following letter.

March 15

Dear Mr. Weber,

m /
I would like to thank you for your comprehensive
o
Your talk on creative problem-solving

c
was informative and very enjoyable, not only for me but for all my
132.
o c.
students. I especially

g
appreciated that you incorporated problem-solving examples applicable to several different

n
fields of learning into your session. My students,
133. s
major in various disciplines, all rated

m
i n g
the session highly as they were able to gain something from your insights. Accordingly,

k
I would like to invite you back next month. If you're available, please let me know so we can

a
e
sp
discuss the details.

Thank you again.

e ic
//to
Mila Hyatt, Ph.D.

tp :
Rappleton University Professor

ht
131. (A)
(B)
discovery
examination
134. (A)
importance to the university.
(C) submission ^ (B) I honestly believe you are my mos
(D) lecture successful guest speaker to date.
(C) My sessions lasted longer than tb
132. (A) attend ones.
(B) attended (D) There were some points that I too
(C) attending issue with.
(D) attendance

133. (A) who


(B) their
(C) whose
(D)they
GO ON TO THE NEXT PAGE.

TEST 9 PART 6 267


Questions 135-138 refer to the following memo.

To: All deans of Mount Westerly University


From: Patricia Griffin, Chief academic officer
Date: April 29
Subject: Promotions for next semester

The time has come for us to determine which faculty members will be promoted.
135.
Therefore, please consider the potential candidates carefully. We advise you to keep in mind

their teaching performance and academic accomplishments before writing letters of


136.
recommendation for the board to take into account. You will also need to submit some

evaluative documents. Please answer all of the questions as ----- as possible. I will forward

m /
the requisite paperwork to you later today with some instructions. These should

c o the

c.
138.
decision-making process.

g o
135. (A) The criteria for recruitment will be
s
137. (A) specificn
modified by the manager.
g m
(B) specifically
(B) We have received your applications
and will let you know the results soon.
k i n (C)
(D)
specify
specification
(C) The choices we make will have a

e a 138. (A) impede

sp
significant impact on the university.
(D) Some of you failed to fully complete (B) negotiate

ic
the recommendation form.

e
(C)
(D)
prolong
guide

//to
136. (A) courteous

tp :
(B)
(C)
infamous
descriptive

ht (D) previous

268 Hackerslngang.com
Questions 139-142 refer to the following information.

Farmer's Market Rules of Operation for All Vendors

The market opens to the public at 8:30 a.m. but will be accessible to vendors at 6:00 a.m.

Vendors should arrive than 30 minutes before opening time in order to set up
139.
their stalls.

Also, please be reminded that vendors must adhere to the state regulations on food safety

and customer protection listed on our Web site. It is the vendor's responsibility to stay

informed about these guidelines.


140.

m /
Furthermore, any applicable fees should be paid on time. The amount will depend on your
c o
stall's
141.
Those closer to the entrance will be charged more.
o c.
Finally, it is illegal to leave any litter in your sales area.
n g
all vendors should have a

receptacle on hand to dispose of any refuse.


m s
i n g
139. (A) shortly
a k 141. (A) design

e
sp
(B) no more (B) location
(C) already (O) merchandise
(D) no later

e ic (D) dimension

//to
140. (A) Those who do not comply with them 142. (A) Meanwhile

tp :
may lose their vending license.
^ (B) You may request an application form if
(B)
(C)
Fortunately
Therefore

ht you are interested.


(C) Some of the food items have been
identified as top sellers.
(D) Regardless

(D) We believe the status of our regular


vendors has changed.

GO ON TO THE NEXT PAGE_

TEST 9 PART 6 269


Questions 143-146 refer to the following article.

■»
*41
^ Sports Complex Put to a Vote

Gainesburg, June 3—Gainesburg residents ----- on whether a sports complex should be

built in town. It was concluded that 82 percent are of the project. In consideration of the
144.
results, the city approved its construction, and development plans are expected to commence

in the coming months.

Gainesburg's one existing fitness club was last year after 30 years of use. This left the
145.
area without any functional leisure facilities at all, and residents had been pushing for a sports

complex ever since.


m /
c o
146.
o c.
"Having a fitness center will make athletic resources readily accessible to a wider

n g
range of people. It will also help unite the community in a fun and healthy way," said council

member Claire Faukes.


m s
i n g
(A) will vote
a k 146. (A) Opposition to the plan remains
e
sp
(B) are voting llpl prevalent at this point in time.
(C) have been voting (B) Most believe such facilities will be
(D) voted

e ic highly beneficial for Gainesburg.

//to
(C) The mayor delivered an inspiring
(A) supportive opening speech.

tp :
(B) support (D) Some modifications were required

ht
(C) supportable before the decision was made.
(D) supports

(A) looked over


(B) paid for
(C) fixed up
(D) shut down

270 saws-ifiassi JJg Hackers.co.kr


PART?
Directions: In this part, you will be asked to read several texts, such as advertisements, articles, instant
messages, or examples of business correspondence. Each text is followed by several questions. Select
the best answer and mark the corresponding letter (A), (B), (C), or (D) on your answer sheet.

© PART 7 ^ #o| A|7| 54^

Questions 147-148 refer to the following invitation.

You are Invited


to the Launch of Ricardo's Corner!
. CL
m /
c o
at 335 West Laughlin Drive

o c.
on Sunday, March 18
between 10 A.M. and 6 P.M.
n g
m s
After four months of construction, we are finally ready to

i n g
welcome customers. Ricardo's Corner is the first store in the
neighborhood to specialize in ingredients for Latin American

a k
cuisine. Our products are imported from throughout Mexico,

e
the Caribbean, and Central and South America. Now you can

sp
cook your favorite dishes from these areas the authentic and

ic
traditional way!

e In celebration of this occasion, our first 50 customers will get a

//to
$50 discount on any purchases totaling over $200. Stop by on

tp : Sunday to meet owner Ricardo Fuentes, and sample a variety


of foods, including homemade tortillas and salsa.

ht
147. What type of event is the invitation for? 148. What is true about Ricardo's Corner?
(A) A community food fair (A) It operates multiple stores.
(B) An annual banquet (B) It will not sell goods on weekends.
(C) A grand opening (C) It sells ingredients from different
(D) A cooking competition regions.
(D) It will offer coupons to all visiting
customers.

GO ON TO THE NEXT PAGE.

TEST 9 PART 7 271


Questions 149-151 refer to the following notice.

Renfrew Gymnasium: For all Members

At Renfrew Gymnasium, we always do our best to ensure that you have


a pleasant and safe visit. To that end, we would like to remind all
members about our policies regarding personal items. — [1] —•
The gym provides lockers in the changing rooms to store your
possessions while you work out. However, please do not leave highly
valuable items in your lockers for security reasons. Renfrew Gymnasium
is not responsible for the loss of any items left in lockers. — [2] —. You
may leave valuables with our front desk staff, who will put them in a
safety deposit box. — [3] —.
Should you lose any belongings during your visit, please notify a staff
m /
member at the front desk. You may also ask them to see if anyone has
c o
turned your item in to the lost-and-found.

o c.
g
— [4] —, Thank you for your cooperation in these matters, and speak to

concerns.
s n
one of our helpful employees should you have any questions or

Management
g m
k i n
e a
sp
149. What is the purpose of the notice? 151. In which of the positions marked [1], [2],

ic
[3], and [4] does the following sentence
(A) To describe a gymnasium's added

e
service facilities
best belong?

//to
(B) To announce changes to a staff "They will provide you with a ticket which

tp : member work policy


(C) To provide details on emergency
must be presented to retrieve your stored
items."

ht safety procedures
(D) To inform members of guidelines
concerning belongings
(A)
(B)
(C)
[1]
[2]
[3]
150. According to the notice, what should gym (D) [4]
users do when reporting a loss?
(A) Visit the facility's front desk
(B) Fill out a complaint form
(C) Write to a head office
(D) Post a note on a bulletin board

272 £z\°l ho]^.qj yo^opi mp3 Hackerslngang.com


Questions 152-153 refer to the following text message chain.

Colin McKay [8:45 A.M.]


Hey Brenda, what time are you supposed to have your yearly
health checkup tomorrow?

Brenda Chan [8:51 A.M.]


At 3:30 in the afternoon. Why do you ask?

Colin McKay [8:52 A.M.]


Mine is at 1:00, and I was wondering if you could possibly
switch with me. I've got a lunch meeting, and time might be a
bit tight.

m /
Brenda Chan [8:53 A.M.]

c o
c.
I wish I could. But I have to go to the plant at noon to get some

o
information from the supervisor about the increased output.
Couldn't you reschedule the meeting for an hour earlier?

n g
Colin McKay

m s [8:54 A.M.]
I guess I will have to do that. In fact, I'd better phone the client
now.

i n g
Brenda Chan

a k [8:56 A.M.]
Good luck!
e
ic sp
e
: //to
152. At 8:53 a.m., what does Ms. Chan most

p
153. What will Mr. McKay most likely do next?

t
likely mean when she writes, "I wish I (A) Meet with a factory manager

ht
could"? (B) Cancel a plant visit
(A) She hopes to attend a meeting. (C) Check some sales figures
(B) She is unable to switch times. (D) Rearrange an appointment
(C) She does not want to accept a
proposal.
(D) She wants to leave the office early.

GO ON TO THE NEXT PAGE,

TEST9 PART? 273


Questions 154-155 refer to the following e-mail.

To: Meredith Patton <meredithpatton@sqmail.com>


From: Arthur Compton <artcompton@cherrylanepottery.com>
Date: October 13
Subject: Order Number P76559

Dear Ms. Patton,


Thank you for your order. We have three of the four pieces you ordered available for immediate shipment.
Unfortunately, the cookie jar you selected is out of stock due to its popularity. You have three options on how
to proceed. You can cancel the cookie jar and get a refund for it. Alternatively, you can choose another item
that is in stock instead, or wait until the item is available to ship in approximately three weeks. Please reply
to this e-mail to let me know which option you prefer. I can go ahead and ship the other items now. But
should you decide to wait for the out-of-stock item, I can hold off and send everything together. That way
you will avoid an extra delivery charge.
m /
c o
c.
Sincerely,
Arthur Compton
Customer Service Manager
g o
s n
154. What is the problem with Ms. Patton's
g m
155. What is NOT an option suggested by
order?

k i n Mr. Compton?
(A)
(B)
e a
An item was damaged during transit.
A selection is unavailable now.
(A)
(B)
Delaying a shipment
Replacing a selection
(C)
(D)
A price has been changed.

ic
A shipment has been lost. sp (C)
(D)
Canceling a single item
Returning a defective product

e
p : //to
ht t

274 Hackers.co.kr
Questions 156-157 refer to the following schedule.

Production: Fire Across the Sand


Produced by: Cinemavent Studios
Directed by: Karen Greene
KAREN GREENE'S SCHEDULE
October 1 to 12

Monday Tuesday Wednesday Thursday Friday


1 2 3 4 5
2 P.M. 3:30 P.M. 1:30 P.M. 1:30 P.M.
Production meeting Consult with Meet with Attend first
to address budget scriptwriter Joel production designer rehearsal with
matters Mabe about
changes to ending
to approve
wardrobe changes
actors

m /
scene

c o
8
2:30 P.M.
9
9 A.M.
10
2 P.M.
11

o
12
c.
7 A.M.
Discuss opening Review final draft Attend rehearsal
n g Begin shooting film
sequence
with director of
of screenplay with with actors
principal cast
m s
photography

i n g
a k
e
ic
156. What is NOT a task arranged forsp 157. What time will the second rehearsal

e
//to
Ms. Greene? commence?

:
(A) Reviewing alterations to a script (A) At 1:30 p.m.
(B)

tp Discussing a film's production costs (B) At 2:00 p.m.

ht
(C) Auditioning main actors (C) At 2:30 p.m.
(D) Talking about a shot sequence (D) At 3:30 p.m.

GO ON TO THE NEXT PAGE.

TEST 9 PART 7 275


Questions 158-161 refer to the following article.

Gordon Institute Publishes Study on Americans' Eating and Exercise Habits

In its last survey conducted a decade ago, the Gordon Institute found that many Americans were not as
active as they should be and often ate excessively. Now, the Atlanta-based nonprofit organization has
released the comprehensive results of a new survey on the same topic.
According to the survey, 45 percent of Americans exercise once a week for 30 minutes a day or more.
An additional 25 percent exercise three times a week for 30 minutes a day or more, and a further
10 percent exercise at least 30 minutes every day of the week. — [1] —.
Of the Americans who do exercise, 65 percent say they jog slowly, 25 percent say they lift weights,
and the remaining 10 percent say they walk long distances or perform other activities. — [2] —. About

60 percent say they strive to eat healthily at least some of the time. — [3] —.
m /
40 percent of this group say they don't make any effort to regulate their food intake, while about

c o
These results make for an interesting comparison to a similar survey the Gordon Institute published

o c.
20 years ago. In that survey, only 23 percent of Americans said they exercised at least once a week.
Furthermore, the percentage of those monitoring their food intake and endeavoring to make smart food
choices has risen 11 percent. — [4] —.
n g
m s
158. What can be inferred about the Gordon
i n g
160. What does the most recent survey indicate
Institute?
a k about Americans?

e
(A) It releases survey results every five (A) They have the tendency to overeat.
years.

ic sp
(B) It has produced at least three studies.
(B) They are making better choices about
their health.

e
(C) It advises the government on medical (C) They got more exercise a decade ago.

//to
concerns. (D) They spend most of their spare time

tp : (D) It holds seminars on maintaining a


healthy diet.
exercising.

ht
161. In which of the positions marked [1], [2],
159. What type of exercise did most survey mh [3], and [4] does the following sentence
participants prefer? best belong?
(A) Taking lengthy walks "The rest of them say they do not work out
(B) Participating in weight training at all."
(C) Running at a slow pace
(A)[1]
(D) Engaging in competitive sport
(B) [2]
(C) [3]
(D) [4]

276 ^|oi ^ MP3 Hackerslngang.com


Questions 162-164 refer to the following Web page.

Super Ticket Exchange Not a member?


www.supertix.com Sign in to receive updates on
your favorite types of events.
HOME | SEARCH TICKETS | SELL TICKETS | HELP | CONTACT US
Super Ticket Exchange is the best place to buy and sell tickets online. We have the widest selection on offer
.with tickets listed at prices well below original cost.

Search by City: Philadelphia Event Type: Concert Date Range: October 1-30
Your Results:
Click on an event name for more details.
> University Symphony Orchestra
October 3, Finkle University
m /
> Stars of Country Music featuring Stacy Bellows and the Howling Coyote Band
c o
October 12, Anchor Arena
> Guest Soloist: Haley Hammond
o c.
October 15, Windwell Auditorium, Windwell College Campus
n g
> The Wandering Noisemakers
October 26, Cougar Stadium
m s
n g
If you have any problems with your transaction or tickets and would like your money back, it is necessary to

i
k
contact us within 30 days of purchase. Our e-mail address is customerservice@supertix.com.

a
e
Selling tickets instead? Click here to go to our sellers' page and enter the requested information.

^1

ic sp K.B

e
//to
162. What is true about Super Ticket Exchange? 164. What are ticket sellers required to do?

tp :
(A) It operates a Philadelphia office.
(B) It offers refunds for a limited amount of
(A) Send an e-mail to confirm an address
(B) Make a phone call to provide payment

ht
time. details
(C) It charges a small commission on (C) Fill in an online form with some details
every transaction. (D) Create an advertisement to post on the
(D) It sells tickets at the lowest prices on site
the market.

163. On what date will a school group give a


performance?
(A) October 3
(B) October 12
(C) October 15
(D) October 26

GO ON TO THE NEXT PAGE_

TEST9 PART? Ill


Questions 165-167 refer to the following questionnaire.

Wembley Motel, Chicago

Thank you for visiting our business center here at Wembley Hotel, Chicago. We hope you found
our services and facilities satisfactory and that our staff assisted you with all your needs. To help
us improve the quality of our services, we kindly ask all guests to take a few moments to fill out this
short questionnaire. Once it is completed, simply hand it to a business center employee. Those
doing so will be entitled to take advantage, free of charge, of the business center facilities at any
Wembley Hotel upon their next visit.
o
What was the purpose of your visit to Chicago?
due'meee meeting with eome cWente
® What did you use the business center for?
I printed out a few contracts, made a conference call, and ueed the Wi-Fi.
m /
• How could we improve our facilities?
c o
The printar you have now doeen't work very well. It printed elowly and eomet'mee
stopped altogether. I think purchae'mq a new one mi^ht he helpful
o c.
o
How was the service you received?
n g
wao exoollent.
m s
Your staff mgmbgr Erin Minors was fn'gndly and assisted me with the printgr. Samcg

Name: Jamos Gould


i n g
Phone: 555-5049
a k
E-mail: iamoo^ould^p^fmvoot.com
e
ic sp
e
//to
165. What is indicated about Wembley Hotel? 167. What is NOT mentioned about Mr. Gould?

tp :
(A) It charges a fee to use the business
center.
(A) He traveled to Chicago on business.
(B) He was pleased with a member of

ht
(B) It has branches located in many staff.
different countries. (C) He printed out some documents.
(C) It recently updated many of its facilities. (D) He recommended the hotel to a
(D) It is considering introducing a new coworker.
service.

166. What did Mr. Gould have difficulty with?


(A) An inoperable phone system
(B) A malfunctioning device
(C) An occupied meeting venue
(D) A poor Internet connection

278 Hackers.co.kr
Questions 168-171 refer to the following newspaper article.

After a Long Absence, Thierry Martin is Back!

Thierry Martin used to be among the best-known But now he is back making new recordings.
and most admired guitarists in the world. His last According to an announcement on Martin's Web
album, La Defense (named after the Parisian site, his next album, Normandy, will be released
business district), had sold more than 1 million this November by BBS Records. Featuring 20 new
copies and was still listed at the top of music charts songs and contributions by several famous
after being there for six weeks. In addition, he was vocalists including British pop star Carly Fey and
scheduled to contribute to the albums of several American singer James Norwell, it's almost certain
award-winning vocalists. to be a best-seller this holiday season.

Then three years ago, while skiing during a holiday

m /
To build excitement for Normandy, Martin released
in Colorado, he broke his left arm. He canceled all
o
the first single from that album. Entitled "Summer

c
c.
scheduled interviews and appearances and had to Squall," it's a six-minute instrumental piece,
postpone his impending world tour. Fans followed incorporating hard rock influences. The popular
his progress anxiously as he slowly recovered, and
g o
music blog MP3Stream called the new single "one
it seemed like he had given up playing altogether
for a time. s n
of his all-time greatest tracks."

g m
k i n
168. What is stated about La Defense?

e a 170. What is posted on Mr. Martin's Web site?

sp
(A) It had lower sales than Mr. Martin's (A) Tour schedules

ic
previous albums. (B) A song by Carly Fey

e
(B) It ranked highly on some charts for a (C) Ways to contact his agent

//to
period of time. (D) Album release information
(C) It consists of recordings with other

tp:
artists. 171. How has Mr. Martin promoted his

ht
(D) It was produced at a recording studio upcoming album?
in Paris. (A) By putting out a song in advance
(B) By performing free concerts
169. What is implied about Mr. Martin? (C) By being interviewed by journalists
(A) He hosted his own televised talk show (D) By writing about it on his music blog
for a time.
(B) He built a recording studio near his
home in Colorado.
(C) He has not performed in public for
three years.
(D) He produced a best-selling album for
James Norwell.

GO ON TO THE NEXT PAGE,

TEST9 PART? 279


Questions 172-175 refer to the following online chat.

O Andrew Walker [4:45] Revenco is growing faster than we could ever have imagined, and
although we receive resumes all the time, the majority of applicants
do not seem appropriate for our needs.
Jolene Gibson [4:46] I agree. We need to focus on recruiting talented people who already
have experience. WeVe participated in job fairs in the past, but the
problem is that most people at those sorts of events are recent
graduates.

o Andrew Walker [4:47] To attract potential candidates, we need to get the word out that
we're hiring and offer a competitive compensation package. Does
anyone have any ideas?
Luke Gamier [4:49] Well, I suppose we could actively browse profiles on employment
Web sites and directly contact people who meet all our criteria.
m /
c o
c.
Alice Page [4:50] I like the idea of reaching out to specific candidates who we would

o
want to work for us. There's just one thing. I think corresponding

g
back and forth with each of them could take a lot of time.

n
o Andrew Walker
Alice Page
[4:51] What do you propose then?

m s
[4:52] We can hold an open house either here at headquarters or at the

n g
Pendleton branch and only invite people we're interested in hiring.

i
a k
They can get a chance to see our offices, meet the staff, and learn
about some of the projects we're working on.

e
sp
Jolene Gibson [4:54] That sounds like a really good idea. It would help with the screening

ic
process, too. Those who respond to our invitation and come will

e show us that they're genuinely interested.

//to
o Andrew Walker [4:55] I like it, too. Alice and Luke, I'd like you to start making a list of

tp : names. We'll work on organizing an open house once that's done.

ht
Alice Page [4:57] Great. I'll get started right away

Send

280 my % t}o^7| mpb Hackerslngang.com


172. Why is Mr. Walker asking for ideas? 174. At 4:50, what does Ms. Page mean when
(A) A department is preparing to train new she writes, "There's just one thing"?
employees. (A) She believes there is only one way to
(B) A committee has been asked to review deal with a lack of applicants.
a list of applicants. (B) She is not sure that people will be
(C) A company is having trouble finding interested in the position.
qualified personnel. (C) She thinks that candidates should
(D) A team has been given the task of have to fulfill a certain condition.
developing a Web site. (D) She does not fully agree with a
colleague's proposal.
173. What is NOT mentioned about Revenco?
(A) It has tried to recruit new employees at 175. What does Mr. Walker ask Ms. Page to
do?

/
job fairs.
(B) It is experiencing rapid growth at the (A) Organize a meeting with Mr. Garnier
present time.
o m
(B) Compile a list of people to invite
(C) It recently stopped running a job
advertisement. event
c. c
(C) Register the company for a recruitment

(D) It has more than one office location.


g o
(D) Give candidates a tour of the workplace

s n
g m
k i n
e a
ic sp
e
p : //to
ht t

GO ON TO THE NEXT PAGE

TEST9 PART? 281


Questions 176=180 refer to the following schedule and e-mail.

New Developments in Ancient History


Royal Harrington Hotel, Melbourne, July 25

TIME EVENT LOCATION


9-10 A.M. Registration Lobby
10-11 A.M. Panel discussion of the latest research in ancient history Main Conference Hall
11 A.M.-12 P.M. Smaller sessions:
o
Jonathan Taylor, "How the Pyramids Were Built" • Tufnell Salon
• Grace Li, "Temples in Ancient Korea" • Hardwell Room
• Zoe Harris, "Public Space in Classical Greece" • Mandala Room
12-1 P.M. Lunch: Main Conference Hall/
The main dining hall is occupied for another event on this day. Hampton Conference
However, participants may find seats and tables in either the
Main Conference Hall or the Hampton Conference Room.
Room

m /
Options for those with special dietary needs will be available,
c o
c.
but only in the Hampton Conference room. Please inform us

1-2 P.M.
ahead of time of such requirements.
Smaller sessions:
g o
s
• Michael Jones, "Popular Attire in the Roman Empire"
n • Tufnell Salon
® Hardwell Room

m
® Andrew Ware, "Agriculture in Mesopotamia"
• Mandala Room
2-2:30 P.M. Refreshments
i g
® Angela Cartwright, "The Druids and Stonehenge"

n Lobby
2:30-4 P.M.
k
Panel reconvenes to discuss what has been learned and
a
Main Conference Hall

e
where future research should be directed

ic sp
e
//to
To: Organizational Team <org@ancienthistory.eclu.au>

tp :
From: Jane Christie <jane.c@southaustraliacollege.edu.au>
Subject: Conference issues

ht Date: July 20

Dear madam or sir,


I am scheduled to attend your conference at the Royal FHarrington Hotel next week. However, due to the
timing of my bus from Adelaide, I will be unable to arrive at the venue until around 10:15 a.m., after the
registration period has officially closed. Could you let me know if there is any way I can still register when
I arrive?
I would also like to mention that I am a vegetarian and will need that to be taken into account at the
lunch.
Thanks for your assistance in these matters.
Best wishes,
Jane Christie

282 Hackers.co.kr
176. What is the schedule mainly about? 179. Why will Ms. Christie arrive after the
registration period?
(A) An event to promote enrollment at a
university (A) She is taking transit that arrives at a
(B) A tour of some historical buildings later time.
(C) An academic gathering regarding (B) She has another event to attend first.
history (C) She needs to check in to her hotel
(D) A series of presentations from room.
graduate students (D) She has a work assignment to
complete.
177. Which speaker will talk about the clothing
of an ancient culture? 180. Where will Ms. Christie most likely eat
lunch?
(A) Jonathan Taylor
(B)
(C)
Grace Li
Zoe Harris
(A)
(B)
In
In
the
the
main dining hall
Hampton Conference Room
m /
(D) Michael Jones (C) In the
o
Main Conference Hall

c
c.
(D) In the lobby
178. What is mentioned about the Royal
Harrington Hotel?
g o
(A) It is located in the suburbs of Melbourne.
(B) It is offering complimentary
s n
accommodations to guests.
g m
(C) It is one of the largest hotels in the city.
(D) It is hosting more than one event on
k i n
July 25.
e a
ic sp
e
p : //to
ht t

GO ON TO THE NEXT PAGE

TEST 9 PART? 283


Questions 181 -185 refer to the following article and e-mail.

Showing the Way Forward for Cyclist

Since it was first developed, Global Positioning maps that display less congested routes for a
System technology, or GPS as it is more safer and more pleasant riding experience. In a
commonly known, has been focused on finding move designed to make Ped-Routes appealing to
more efficient and accurate ways to direct drivers more people, Two Wheels has also provided
to their destinations. The average GPS available routes accessible to pedestrians.
is able to tell anyone in a vehicle the quickest
Adam Jorgeson, the director of Two Wheels, said
and most efficient route from point A to point B,
"We're very pleased with the new project and
but it is unable to provide cyclists or pedestrians
expect the first version to be launched before the
the same information.
end of the month. We are working with FGG on
A new mobile application called Ped-Routes,
funded by the cycling association Two Wheels,
future versions that will provide cyclists with the
option of taking the most scenic routes."
m /
aims to change all that. Developers from FGG
c o
Ped-Routes is downloadable for a cost of $4.99,
Software have used GPS technology to create
c.
which gives users access for a period of one year.

o
n g
ms
EDH

To:
From:
Melissa Hawk <techsupport@twowheels.com>

i
Georgina McGrath <gjmcgrath@zoommail.com>
n g
Subject:
a
Problems with Ped-Routes Mobile App k
e
sp
Date: April 30

Dear Ms. Hawk,


e ic
p : //to
I recently downloaded your group's mobile application and wanted to mention an issue I
encountered. I tried to use it to find my way around Providence. I just moved here last month for a

ht t job, so I'm still unfamiliar with the area. While I found that the app successfully guided me down
some quiet, pleasant streets, it also once suggested that I take a much longer route than necessary.
I followed the suggested route, and I later calculated that it took me over 40 minutes longer than
the most direct route would have.

I appreciate that the application is primarily designed for bicyclists, but I still feel that this flaw
needs to be addressed to make the software more useful. With that said, I would like to commend
your organization for the general quality of the interface and the impressive speed at which the
app works.

Best wishes,

Georgina McGrath

284 Hackerslngang.com
181. What is the main topic of the article? 184. What can be implied about Ms. McGrath?
(A) A bicycle parking facility (A) She uses the software mostly for
(B) An upgraded fitness tracking device bicycling to work.
(C) A new navigation program (B) She received a discount for renewing
(D) A searchable city map her membership.
(C) She paid for a one-year subscription to
182. What is Two Wheels? the application.
(A) A bicycling group (D) She will be charged an extra fee to see
(B) A motorbike manufacturer walking routes.
(C) A government agency
(D) A software developer 185. What is indicated about Red-Routes?
(A) It has an interface that is hard to use.
183. What has Ms. McGrath recently done?

/
(B) It operates at a fast speed.
(C) It is a trial program for cyclists.
(A)
(B)
Deleted a mobile application
Relocated for work (D)
o m
It costs less than similar software.
(C)
(D)
Hiked in the mountains
Rented a car
c. c
g o
s n
g m
k i n
e a
ic sp
e
p : //to
ht t

GO ON TO THE NEXT PAGE,

TEST 9 PART? 285


Questions 186-190 refer to the following flyer, Web page, and e-mail.

ADVERTISE IN THE SHELTON MEDIA COMMUNITY NEWSLETTER


Shelton Media cares about Birmingham, Alabama. That's why the monthly newsletter we produce is
sent exclusively to residents of this community. If you are an entrepreneur in the area who would like
to join the network of diverse local enterprises that we support, consider placing advertisements in
our publications. Doing so will not only help your business gain exposure but also entitle you to free
advertising at the many events we host, such as the annual Birmingham Community Picnic and
Parade and the biannual New Semester Career Fair.
*Rates indicated are per issue of the newsletter:
size
No. of issues^^^__^ 1/4 page 1/2 page Full page
6 $135 $175 $275
12 $125 $165 $255
m /
24 $115 $150
o
$230

c
o c.
We allow businesses to purchase front-page advertisements and inserts as well. If you are interested
in a long-term contract of more than 24 issues and would like to find out about our discounted rates,

g
contact us at ads@sheltonmedia.com or call 555-4311 to speak to Daniel Kelly. Payments by credit
n
s
and debit card, cash, and check are accepted. Make checks payable to Shelton Media.

m
i n g
a k
SHELTON MEDIA
e
www.sheltonmedia.com

ic sp
Home | About Us | Our Publications | Subscriptions | Advertising | Contact Us

e
//to
Contact Shelton Media

tp :
Should you need to communicate with us, visit the address below or call us at 555-4311 from

ht
Monday through Friday from 9 a.m. through 6 p.m. Dial one of the departmental extension numbers
listed below to connect to the appropriate representative.
Main office
1510 Dennison Avenue, Birmingham, AL 35211
Departments
Community sales: Rhonda Bullock, extension 11
Advertising sales: Daniel Kelly, extension 18
Layout: David Peretti, extension 24
Accounting: Avery Lee, extension 28
Distribution: Nadine Turner, extension 30
Additionally, we encourage you to tell us what you think of our site and our services. Click here to
fill out a comment form.

286 Hackers.co.kr
To: Charles Walker <c.walker(a)lawlerscapes.com>
From: Eva Loeb <e.loeb@lawlerscapes.com>
Subject: Advertisements
Date: February 2
Attachment: Report

Charles,
I compiled a report regarding the advertisements we ran in Shelton Media's community newsletters
over the course of the past 12 months in an effort to reach more local clients. I've attached it for your
convenience.
Based on the number of service inquiries we received from clients who had seen the ads, I
recommend we advertise in the newsletter again for the same amount of time. We paid $233 per ad
last time, but I think we should print our new one in the next size smaller to save some money. There
m /
o
is also the smallest size, which is even cheaper, but I think it would be difficult to fit all the content we
c
c.
need into a space that limited. I believe we should get our new ads into the newsletter as soon as
possible because it's almost spring and our services will be more in demand.

g o
Eva Loeb
Marketing manager
s n
Lawler Scapes

g m
k i n
e a
sp
186. What is NOT true about Shelton Media? 189. What does Ms. Loeb indicate about Lawler
Scapes?

ic
(A) It accepts multiple methods of payment.

e
(B) It targets residents of a particular (A) It plans to sign a two-year agreement.

//to
community. (B) It has more business during the spring.
(C) It prints one newsletter issue weekly. (C) It will increase its budget for advertising.

tp :
(D) It hosts events for job seekers. (D) It is a new company in the region.

ht
187. What extension number should clients dial
to inquire about discounted rates?
190. How much does Ms. Loeb suggest
spending on each advertisement?
(A) 11 (A) $125
(B) 18 (B)$150
(C) 24 (C) $165
(D) 30 (D) $175

188. What can customers do on Shelton


Media's Web site?
(A) Provide feedback on services
(B) Order products from a catalogue
(C) Enter a monthly draw
(D) Chat with a company representative

GO ON TO THE NEXT PAGE.

TEST 9 PART 7 287


Questions 191-195 refer to the following Web page, online form, and review.

BORDERLAND TOURS
For the adventurer

Home About Us Promotions Testimonials Booking

Borderland Tours offers extraordinary travel adventures that very few dare to undertake.
This winter, between December 1 and February 28, we are offering a 30-day trip through the
Sahara at 20 percent off our regular rate. This is a trek through one of the harshest but most scenic
landscapes on Earth.
The route goes through four North African countries: Morocco, Western Sahara, Mauritania, and
Mali. You can read about each country in more detail by clicking here.

m /
o
You will be required to provide a certificate of health signed by your medical provider proving that

c
c.
you are physically sound and capable of completing the trip. Otherwise, you will not be permitted to
go. To this end, please make sure you have read our Terms and Conditions.
If you would like to make a reservation, click here for booking information.
g o
n
ms
a E

i n g
a k
e
sp
BORDERLAND TOURS
For the adventurer
Home
e ic About Us Promotions Testimonials Booking

//to
Fill out the booking form below and send us a deposit of $500 to reserve your spot. It will be applied

tp :
toward the total cost or reimbursed if the reservation is canceled at least 30 days before your trip. If
a cancellation is made fewer than 30 days before, only half the amount will be returned.

ht Name; Enrique Turner


Contact Number: 555-8991
E-mail Address: enriqueturner@swiftmail.com
Name of Tour/Itinerary: 30-day Sahara Promotional Tour
Dates: January 1 to 30
Mauritania Tour Option:
I read that the tour members will have several different routes to choose from once we reach
Mauritania and that we are supposed to come to an agreement and go together. The option that
appeals most to me is the one that involves traveling by train. However, I am willing to go along
with whatever the rest of my group decides as all the options look exciting.
I have read the terms and conditions and agree to comply fully. 0

SUBMIT

288 Hackerslngang.com
A Once-in-a-Lifetime Journey! *★★★☆
Posted by Enrique Turner on February 10

I recently got back from Borderland Tours' 30-day Sahara Tour. At first, all I could focus on was
what we were going to eat. I had been excited to try African cuisine, but I generally found it to be
disappointing. Because we didn't stop anywhere for very long, the tour company provided most of
the meals, which became repetitive. Anyway, the journey started in Morocco. We drove through
Marrakech, Meknes, and Fez before traveling by camel to where the desert meets the ocean.
From there, we crossed the border into Western Sahara and continued along the coast to
Mauritania. In Mauritania, travelers can choose to go south to Nouakchott, camp out in Banc
d' Auguin, take a train to Atar, or join a camel caravan to Tagant. The majority of our group
wanted to go by caravan, so that's what we ultimately did. It wasn't my first choice, but I was
happy just the same. We passed some breathtaking scenery on the journey Read more...

m /
c o
191. What is NOT true about the trip through
o c.
194. What aspect of the trip was Mr. Turner not
the Sahara? satisfied with?

n g
s
(A) It can be purchased at a discount. (A) The impoliteness of a staff member
(B) It passes through at least three (B) The uncomfortable accommodations
countries.
g m
(C) The incompatibility of the group
(C) It is offered by Borderland Tours
year-round.
k i n members
(D) The lack of variety of meal options

e a
(D) It is scheduled to last for about a

sp
month. 195. Which destination in Mauritania did
Mr. Turner originally want to visit most?

e ic
192. According to the online form, what are (A) Nouakchott

//to
travelers required to do upon booking? (B) Banc d' Auguin
(C) Atar
:
(A) Indicate their meal preferences

p
(B)
t
Transmit a partial payment (D) Tagant

ht
(C) Provide a home address
(D) Arrange for necessary visas

193. What is suggested about Mr. Turner?


(A) He frequently goes on trips to exotic
locations.
(B) He is in a healthy state of physical
fitness.
(C) He took a safety course before his trip.
(D) He had to pay extra for changing his
reservation.

GO ON TO THE NEXT PAGE

TEST 9 PART? 289


Questions 196-200 refer to the following article, letter, and e-mail.

Autumn Leaf to Open Second Location in Buenos Aires


By Leonard Oldinan
Autumn Leaf Hotels has announced plans to open a second branch in Buenos Aires' Liniers district.
Autumn Leaf public relations director Angelo Suarez said that the new hotel will be more like a
temporary apartment building for businesspeople visiting the city. "There is a lot of demand from
visitors who need to stay in Buenos Aires for long periods of time but don't wish to spend a fortune on
a full-service hotel," he said. He went on to say that the hotel will consist of large units with living
areas as well as kitchens so that guests can prepare their own meals. Although the new hotel will not
house a restaurant or offer room-service like the main branch in Monserrat, there will be a 24-hour cafe
on the ground floor, housekeeping staff, and a concierge desk. The hotel will open sometime in August,
and prospective guests may make bookings or inquire about rates at www.autumnleafhotels.com.

m /
c o
April 9
o c.
Leonard Oldman
n g
World-Biz Magazine
Unit 542, Nelson Building, 432 W. 24th Street
m s
New York City, NY 10013

Dear Mr. Oldman,


i n g
a k
Thank you for writing the article about our hotel chain's expansion. We appreciate the publicity.

e
sp
However, there were a couple of errors. The article mentioned that bookings for the new hotel can
already be made on our Web site. We have been receiving inquiries from readers of your article

e ic
about this, but unfortunately, your information isn't accurate. We will only start accepting reservations
several weeks before our opening date, which hasn't been established yet. Your article specified the

//to
month the hotel would likely open, but we are actually planning for somewhat later than that.

tp :
I would be grateful if your publication printed corrections in an upcoming issue.

ht
Sincerely yours,
Angelo Suarez
Public Relations Director, Autumn Leaf Hotels
angsuar@autumnleafhotels.com

TO: Angelo Suarez <angsuar@autumnleafhotels.com>


FROM: Leonard Oldman <l.oldman@worldbizmag.com>
SUBJECT: Sincere apology
ATTACHMENT: draft
DATE: April 13

Dear Mr. Suarez,


I would like to apologize for the errors in the article I wrote. I must have misunderstood one of the

290 Hackers.co.kr
people I interviewed at Autumn Leafs main branch last month. I have spoken to my supervisor, and
we will print a retraction in our next issue. Please review the draft I've attached with the corrected
information.
Also, I would appreciate if you could let me know once the hotel's opening date has been settled as the
magazine might send me there to review it.
Once again, I am sorry for the mistakes.
Regards,
Leonard Oldman

196. According to the article, why is Autumn


Leaf opening a new hotel?
199. What is suggested about Mr. Oldman?
(A) He interviewed Mr. Suarez in New
m /
(A) To make up for the closure of its first York.
c o
one
(B) To satisfy a need for extended stays
(B) He took a translator with him to
Buenos Aires.
o c.
(C) To accommodate an excess of
n g
(C) He visited Monserrat sometime in
customers
(D) To take advantage of a district's
m s
March.
(D) He was hired by Autumn Leaf to write
growing popularity

i n g his article.

a
197. What is indicated about Autumn Leaf's
k 200. In the e-mail, the word "settled" in
new hotel?
e paragraph 2, line 1, is closest in meaning

sp
to
(A) It will rent rooms for events.
(B)
(C)
e ic
It will serve meals in guests' rooms.
It could open later than August.
(A) paid
(B) finalized

//to
(D) It may offer a valet parking service. (C) located

tp :
198. What does Mr. Suarez state about the
(D) relieved

ht
article?
(A) It could be reprinted in a company
newsletter.
(B) It is the second one written about
Autumn Leaf.
(C) It failed to mention a special promotion.
(D) It prompted readers to contact Autumn
Leaf.

This is the end of the test. You may review Part 5, 6, and 7 if you finish the test early.

eh: S

33 p.326 / 34= StS p.327 / 61I3 p.39B / Part 5&6 UfS M7| S
• c® ni|0|x|0|| Self *13. e| AS1 #sn gxi| #o| ^34 ehe# ggsii suiifl.

TEST 9 PART? 291


6H3H^ 1000X11 3 READING

m /
c o
o c.
n g
m s
i n g
a k
TEST
e 10

sp

e ic
//to
PART 5

tp : PART 6

ht PART 7

Self a|AE

^WH Ell^E g SEOJAHf


1.^HS^ISSfTiAjL^? □ on
2. Answer Sheet. 21, SU|6Hd4a? □ oj|
3.A|7jHgbi|SH34o? □o||
eg si®tS£|SJ°2 cciSSJ 4 EIIAE# AIHH

#0|1 D^lb APJS Xg^Ei 70" °°J _A| _^L|EK


E||AE X|yl # 75MO|ni, X|°j §5 g sgg gg 51 om# A^°}L|EK
READING TEST
In this section, you must demonstrate your ability to read and comprehend English. You will be given a
variety of texts and asked to answer questions about these texts. This section is divided into three parts
and will take 75 minutes to complete.
Do not mark the answers in your test book. Use the answer sheet that is separately provided.

PART 5
Directions: In each question, you will be asked to review a statement that is missing a word or phrase.
Four answer choices will be provided for each statement. Select the best answer and mark the
corresponding letter (A), (B), (C), or (D) on the answer sheet.

© parts *0I IIS

m /
o
101. Body-Wise Gym closes for minor 105. Colorlast makes a line of outdoor paints
renovations and cleaning.
(A) periodic
that are

c. c
formulated to withstand
years of exposure to sun and precipitation.
(B) periodical (A) special
g o
(C)
(D)
period
periodically
(B)

s
(C)
specialty
speciallyn
g m(D) specialization
102. The reporter forecasted sunny weather
the Midwestern region for tomorrow.
k i n 106. Athlete Tim Ferguson has made a
(A) among
e a recovery from his injuries and will be able
to play in his team's next match.

sp
(B) throughout
(C) without (A) reverse
(D) until

e ic (B) preferable

//to
(C) remarkable
103. The faulty card reader located at the side (D) negligible

tp :
entrance of Carter & Associates'

ht
headquarters in the coming week. 107. Prospective candidates for the position will
(A) was repaired be notified of the final decision by June 1
(B) repairs at the .
(C) will be repaired (A) late
(D) has been repaired (B) latest
(C) lately
104. The accounting department revised (D) later
estimates for expenditures after the
news that the budget would be changed. 108. Dr. Watanabe was seated the
(A) to receive keynote speaker and the foundation
(B) receives president during the banquet.
(C) receiver (A) about
(D) receiving (B) between
(C) versus
(D) except

294 Hackers.co.kr
109. Ms. Larrington's speech about how 114. The dean announced that an esteemed
to improve customer satisfaction has professor working at the university for
motivated service specialists across the three decades would be retiring.
country. (A) rarely
(A) perishable (B) frequently
(B) portable (C) newly
(C) enthusiastic (D) soon
(D) indifferent
115. Torres Architecture, the agency that
110. Verdant Island can only be visited in the planned the Steadman Tower, won an
daytime since of it is covered by award from Building Life Global for its
water at night. design.
(A) much (A) distinction
(B) few (B) distinctiveness
(C)
(D)
less
enough
(C)
(D)
distinctive
distinctively
m /
c o
c.
111. Extra staff were called in from nearby 116. To increase the stability of large data
transfers, the technical crew will an
branches to help plant crews -
production deadlines for an unexpected
g o
upgraded software program on Friday.
order. (A)
sdetach
n
(A)
(B)
meet
enter
(B)

g
(C)m install
convert
(C)
(D)
relieve
commit
k i n (D) capture

e a 117. The Cottonwood Gallery guarantees that

sp
112. Officials at Lextar International were of the items it sells is an original and

ic
unable to agree a definite solution to comes with an official certificate of
authenticity.
e
the problem of steadily declining profit

//to
margins. (A) many
(A)

tp
(B) : at
that
(B)
(C)
each
both

ht
(C) by (D) all
(D) on
118. If Jennifer hits all of her sales targets for
113. According to the Ministry of the the next quarter, there is a good chance
Environment, updated on water she will
consumption will go into effect next month. (A) promote
(A) regulation (B) have promoted
(B) regulate (C) be promoted
(C) regulating (D) be promoting
(D) regulations

GO ON TO THE NEXT PAGE

TEST 10 PARTS 295


119. The developers at Staycor test new 125. Responding to declining sales figures,
software multiple times as their overall Senturia Automotive concluded that they
objective is to detect and eliminate any had to the M5 Sedan line.
defects.
(A) disembark
(A) almost (B) disobey
(B) barely (C) discontinue
(C) suddenly (D) dismiss
(D) typically
126. The Chicago /-/era/d gives a 40 percent
120. Gigantica Studio's new action movie has discount to existing who sign up for
some scenes which may be inappropriate access to its online edition.
for under the age of 15.
(A) subscription
(A) whoever (B) subscribed
(B) them (C) subscribes
(C)
(D)
these
those
(D) subscribers

m /
o
127. Sunny River Vacations endeavors to satisfy

c
c.
121. After years of difficulty doing it herself, every customer by personalizing travel

o
Ms. Pace had her taxes through an details to ensure that each trip is
accounting agency.
(A) memory
n g
(A)
(B)
filing
file
m s
(B)
(C)
memorable
memorizing
(C)
(D)
filed
to file
i n g (D) memorials

a k 128. Rowan Training Center has added more


122.
e
expectations, the band's greatest business classes, which brings the total

sp
hits CD sold a million copies shortly after number of sessions to nearly double

ic
its release. that of last year.
(A)
e
Recommending (A) schedule

//to
(B) Perceiving (B) scheduled

tp :
(C)
(D)
Evolving
Exceeding
(C)
(D)
schedules
scheduling

ht
123. Mr. Conrad posted advertisements on
several recruitment sites, including the
129. Ms. Jacobs' managerial style is democratic
she includes her staff in the
regarding salaries and benefits. decision-making process.
(A) permissions (A) in that
(B) configurations (B) for example
(C) particulars (C) so that
(D) institutions (D) rather than

124. Top executives from Home Stock gave 130. MEG Electronics has a page on its Web
members of the local media a tour of the site dedicated to from users of its
new store its grand opening. products.
(A) next to (A) licenses
(B) ahead of (B) testimonials
(C) aside from (C) accomplishments
(D) in front of (D) tendencies

296 ?sge|°!t!M°|H2l3W(.90|oj^l5f5!90)g(7|MP3 Hackerslngang.com


PART 6
Directions: In this part, you will be asked to read four English texts. Each text is missing a word,
phrase, or sentence. Select the answer choice that correctly completes the text and mark the
corresponding letter (A), (B), (C), or (D) on the answer sheet.

© parts #0I A|y 8®

Questions 131-134 refer to the following advertisement.

Do you find your home more trouble than it's worth? If so, call Wimberley's—the most
131.
trusted housekeeping service in Kent. Since opening five years ago, we have consistently

received top ratings from our customers.


m /
c o
c.
Our crew takes care of window and floor washing, vacuuming, and tile polishing. We
132.

g o
use state-of-the-art equipment to get rid of the odors, dirt, and stains you've been struggling

with. Furthermore, if you need to remove large


s n
items such as furniture or appliances,
133.
the Wimberley's staff can do this for a small additional fee.
g m
k i n
We
134.
a
your home at whatever time is most convenient for you! So, if you're interested,

e
sp
please call us at 555-9190.

e ic
(A)
(B)
p :
buying
//to
cleaning
133. (A)
(B)
extensive
foreseeable

ht t
(C)
(D)
furnishing
decorating
(C)
(D)
unwanted
infrequent

(A) You may join the crew by sending in 134. (A) are visiting
your application. (B) had visited
(B) We also sell a wide range of cleaning (C) have visited
products. (D) can visit
(C) Wimberley's does all the tiresome jobs
that you don't have the time for.
(D) Our service schedule will be posted
online once we resume operations.

GO ON TO THE NEXT PAGE,

TEST 10 PART 6 297


Questions 135-138 refer to the following e-mail.

To: Talia Moroz <talia.moroz@homemail.com>


From: Miles Kaufmann <m.kaufmann@coldmaker.com>
Date: July 31
Subject: Your request

Dear Ms. Moroz,

I am writing in reply to your inquiry about the Coldmaker refrigerator you purchased recently.

In your e-mail, you stated that the compartment does not close properly. issue was
135.
brought up by other customers in the past. Our technicians have since discovered what the

problem is.
136.
m /
c o
c.
As indicated in the manual, to ---— the door, you must push it forward until a clicking sound

o
is heard. If you simply close the compartment without holding the tab, the door will not remain
g
closed.
s n
g m
Could you please try this method first? If you continue to have a problem, please contact a

service center on our Web site.


k i n
138.

e a
sp
Thank you for writing, and I hope I have been of service to you.

ic
Sincerely,
e
p :
Miles Kaufmann
//to
ht t Warranty service agent

135. (A) This 137. (A) enclose


(B) Their (B) fill
(C) The other (C) obstruct
(D) Such (□) secure

136. (A) We are glad that you have found a 138. (A) lists
solution to the problem. (B) listing
(B) It appears that users are not following (C) listed
product manual guidelines. (D) to list
(C) A serviceman will visit your home in a
day or two.
(D) The appliance you purchased is no
longer for sale due to this defect.

298 saga HIS Hackers.co.kr


Questions 139-142 refer to the following e-mail.

To: Henry Chan <h.chan@followmail.com>


From: Gleeson Consulting <hr@gleeson.com>
Date: March 14
Subject: Re: Application for project manager position

Dear Mr. Chan,

We received your application for project manager at Gleeson Consulting. The company had a

large number of applicants, so for the position was rather strong. I therefore regret to
139.
inform you that you have not been shortlisted for this post. we have closely examined

your qualifications and consider you eligible for the project assistant position. •

m /
c o
c.
Interviews for this vacancy will take place on March 20 from 10 a.m. to 3 p.m. Please get in

o
touch with me to let me know if you are interested in the job and can come in between these

g
times. If you are not
s n
perhaps we can arrange a date that is more convenient for you.

Yours truly,
g m
Jacqueline Hart
k i n
Human resources manager
e a
sp
Gleeson Consulting

e ic
//to
(A) supposition 141. (A) Some of the documents we requested
(B)

p
(C)
t : allocation
competition
|||| were missing from your application.
(B) We do not have vacancies at this time,

ht
(D) entry but may consider your application in
the future.
(A) Likewise (C) Your offer to find a suitable applicant
(B) Nonetheless was accepted.
(C) Therefore (D) We hope you will consider being
(D) Henceforth interviewed for this opening.

142. (A) anxious


(B) determined
(C) fortunate
(D) available

GO ON TO THE NEXT PAGE

TEST 10 PART 6 299


Questions 143-146 refer to the following announcement.

CALL FOR PAPERS

The Department of Sociology at the University of Northern Wisconsin an academic

conference on social transformation in the Midwestern United States from June 7 to 9.


144.
Interested professors and lecturers are asked to submit their abstracts before May 1.

You will be sent a notification from the department if your is selected. Once you receive
145.
this notice, you will be required to submit the full paper for review and approval.

Submissions are welcome from all universities, and we look forward to seeing a range of

m /
topics. the conference's focus on the Midwest, we welcome work from other regions as

c o
c.
146.
long as it relates to the needs of the target area.

g o
s n
143. (A)
(B)
can be held
has held
g m
145. (A)
(B)
outline
drawing
(C) will be held

k i n (0) certificate
(D) is holding

e a (D) assistant

sp
144. (A) A number of applicants were awarded 146. (A) Prior to

ic
grants for this project. (B) During

e
(B) In this regard, we are now accepting (C) Despite

//to
proposals related to this topic. (D) Up until
(C) We are pleased to have you on the

tp : committee and will provide needed

ht
support.
(D) In addition, it featured speakers from
regional education establishments.

300 ?ssa|?!i!Ee)a°|iy.h90iS7B5J90i?t7|MP3 Hackerslngang.com


PART 7
Directions: In this part, you will be asked to read several texts, such as advertisements, articles, instant
messages, or examples of business correspondence. Each text is followed by several questions. Select
the best answer and mark the corresponding letter (A), (B), (C), or (D) on your answer sheet.

© PART 7 m 10I ApJ 54£

Questions 147-148 refer to the following text message.


m
lh
< Messages Bergman Glazer Details
o

Received June 14, 1:00 p.m.

m / o
pu

o
Dear Ms. Crowder: Always shop at Bergman Glazer for great o
UJ
deals on fine home furniture and accessories. Enter the
promotional code 57HPDQ1X at www.bergmanglazer.com
c. c o

g
before checking out to receive free shipping on any order of
o o

n
$100 or more. This code may be used only once and cannot be

s
shared. It expires on June 30 at 11:59 P.M. This message has been
LTI

g m
sent because you are a registered customer of www.

n
o

k i
bergmanglazer.com. If you are not interested in receiving similar
promotional messages in the future, text "STOP" as a reply to

e a
this message. For assistance or inquiries, text "HELP." Standard
o
CO

sp
messaging rates apply. o
LD

e ic o

//to
147. What is the purpose of the message? 148. What is indicated about Ms. Crowder?

tp:
(A) To advertise a sale on secondhand (A) She recently placed an order worth at

ht
items least $100.
(B) To confirm registration on a Web page (B) She signed up as a customer on a
(C) To promote a new line of home Web site.
furnishings (C) She entered a draw to get a discount
(D) To encourage shopping at an online coupon.
store (D) She works for a furniture design
company.

GO ON TO THE NEXT PAGE

TEST 10 PART? 301


Questions 149-150 refer to the following notice.

Notice to All Library Users

Please note that over the next month, the West Springs Library will be
moving from its current location to a much larger space that will allow
us to provide more and better services to our users. We have signed a
lease to occupy the former Benchmark Cinema building on the corner
of Wentworth Rise and 9th Avenue, and we will be renovating the
structure before relocating our collection there. This building is in a
more central location, which will allow us to reach more library users.

Unfortunately, the process of moving will create some temporary


disturbances to our services. Users will not be able to borrow any items
from the library after April 6, and we ask that all items currently out on
m /
loan be returned by April 13. After this, the library will be closed from

c o
c.
April 14 until May 1, when our new location is expected to open. We
are sorry for any inconvenience this may cause. The nearest public

g o
library available to you during the closure will be the facility in Coach
Hill.

s n
Thank you for your understanding.

g m
Calgary Library Services
k i n
e a
ic sp
e
149. What is the purpose of the notice? 150. What is indicated about the West Springs

//to
Library?
(A) To announce a change of location for a

tp : facility
(B) To report the permanent closure of a
(A) Its current building will be used as a
cinema in the future.

ht
library (B) The closest public library to it is in
(C) To remind users about book return Coach Hill.
policies (C) Its renovations will last over three
(D) To inform members of an increase in months.
library fines (D) The facility is located in downtown
Calgary.

302 Hackers.co.kr
Questions 151-152 refer to the following letter.

March 3
Judy Mayer
Whole Nutrition Pet Foods, Inc.
1930 Industrial Street
Lawton, OK 73501
Dear Ms. Mayer,
I am writing to apply for the position of manufacturing plant electrician for your Lawton
location. I have been a certified electrician for 15 years. I recently left my job of three years
at the Oklahoma City facility of Loyalty Limited to relocate to Lawton. I am now seeking
employment and your advertised job would be a good fit for me.

m /
I worked on the general electrical needs of the factory and its equipment for my most
recent employer. My assignments required a lot of problem solving as well as electrical
c o
knowledge. The equipment setup and adjustments I carried out regularly involved
computer technology.
o c.
n g
About eight years ago, I earned an associate's degree in computer technology so that I

m s
could work in the capacity of an equipment electrician. Since then, I have kept up with
developments in the field through training courses. I have included more details about my

i n g
accomplishments in my resume, which is enclosed with this letter.

a k
Given my latest experience working at a company that manufactures similar products to
yours, I think I could make an easy transition. In fact, it is highly likely that your company
uses much the same equipment.
e
ic sp
I thank you for your consideration and do hope you will contact me for an interview at
rilaney@postmail.com or at 555-9483.

e
//to
Sincerely,

tp : Rick Delaney

ht
151. Why was the letter written? 152. What does Mr. Delaney suggest about
(A) To request a transfer to Oklahoma City Loyalty Limited?
(B) To apply for a technical position (A) It has employed him for the past
(C) To register for a degree program 15 years.
(D) To inquire if a job is still available (B) It produces products for animals.
(C) It offers financial support to staff
earning degrees.
(D) It operates a branch plant in Lawton,

GO ON TO THE NEXT PAGE

TEST 10 PART 7 303


Questions 153-156 refer to the following Web page.

IIP 1111 ^Ifii


http://www.cornucopiaindustries.com/companyoverview

HOME | COMPANY OVERVIEW | PRODUCT INFORMATION | EMPLOYMENT | CONTACT

Cornucopia Industries has grown alongside the development of technology in workplaces across the
globe. No matter what our various corporate customers' specific needs are, our company provides
the highest quality telecommunications solutions and customer service. Cornucopia has earned a
positive reputation with clients across North America over the years for its business practices.
-[1]-.
Cornucopia continues to be a total solution provider, delivering a variety of telecommunications
products for businesses in all types of industries. Some of our primary sellers include analog and
digital switches, fiber optic cables, modems, routers, and other local area network (LAN) equipment.
-[2]-.
m /
c
Cornucopia Industries is currently preparing to become a global leader by doing research into new
o
o c.
fields such as satellite technology and mobile networking. — [3] —. Our team of dedicated employees
also continually endeavors to engage clients abroad to foster relationships with a diverse array of
organizations.
n g
m s
It is the number one goal of Cornucopia Industries to become a revolutionary company that develops
and enhances communications techniques for another half a century, and even longer! We are

n g
grateful to all of our partners and clients for their ongoing support. — [4] —.

i
Ol
a k K> a

e
ic
153. What is mentioned about Cornucopia
sp 155. What is Cornucopia Industries' plan?

e
Industries? (A) To acquire another communications

//to
(A) It has been recognized with several company

tp : business awards.
(B) It is known for providing excellent
(B) To upgrade technological equipment at
its factory

ht
customer service. (C) To get involved in other fields within
(C) It derives most of its business from the industry
government contracts. (D) To hire a customer service training
(D) It sold a share of its business to a specialist
foreign firm.
156. In which of the positions marked [1], [2],
154. What can visitors most likely do on |||| [3], and [4] does the following sentence
Cornucopia Industries' Web site? best belong?
(A) View information on job vacancies "To achieve this, we must apply decades'
(B) Place orders for equipment of technical experience to the latest
(C) Obtain details on a refund policy technologies available today."
(D) Find local retailers of its merchandise
(A) [1]
(B) [2]
(C) [3]
(D) [4]

a
304 A yolSPI mpb Hackerslngang.com
Questions 157-159 refer to the following survey form.

The Granriver Hotel


Guest Stay Survey
We ask all of our guests to please take a moment to fill out this form to provide us with your valued feedback and
comments, which will help us improve our services. Please mark all responses with an (X).
NAME: Cordon Pence DATE(S) OF STAY: August 18-19
PHONE: (404) 555-3984 E-MAIL: gorpen@gotomail.com
Disagree Somewhat Agree
Agree
1) Overall, my stay at The Granriver Hotel was ...
pleasant, peaceful, and comfortable.
an excellent value for the price of my room.
2) Upon check-in, I was ...
immediately greeted by a front desk staff member.
given all necessary information for my stay.
X

m /
assisted with my baggage, if necessary.
c o
3) When entering my room for the first time, it was ...
neat, tidy, and orderly.
o c.
a comfortable and appropriate temperature. X
n g
4) I found the facilities at The Granriver Hotel to be ...
convenient in regard to my specific and personal needs.
m s
detailed accurately in the hotel directory.
5) In general, the staff at The Granriver Hotel were
i n g
easy to contact when needed.
a k
e
friendly, understanding, and attentive. X

sp
able to effectively resolve any issues I had.

e ic
6) Please specify any other comments below:
Overall, I am quite pleased with my experience at your hotel. I did have a small issue after check-in, though. I

//to
had several large bags, but the front desk clerk informed me that a bellhop was not currently available. My room

tp :
was clean and spacious. However, upon entering I noticed that, for some reason, the air conditioner was turned
down to a chilly 16 degrees Celsius. The indoor pool was fantastic and very relaxing, but I had a hard time

ht
getting in touch with staff on a few occasions when asking about where to pick up and dispose of pool towels.

157. What does the hotel NOT request 159. What is suggested about the guest's
feedback on? room?
(A) The attitude and behavior of employees (A) The air conditioning had been left
(B) The state of the room upon initial entry running.
(C) The accessibility of the hotel (B) The bed had not been properly made.
(D) The correctness of items in the directory (C) The bathroom did not contain any
towels.
158. What did the guest most likely do following (D) The hotel directory was not easily
hotel check-in? found.
(A) Ordered a room service meal
(B) Used an outdoor swimming pool
(C) Carried luggage to his room by himself
GO ON TO THE NEXT PAGE
(D) Made a complaint to a travel agency

TEST 10 PART? 305


Questions 160-162 refer to the following notice.

Steam Railway Jamboree - Friday, May 2


to Sunday, May 4

As always, the first weekend in May will see our annual Steam Railway Jamboree
take place. The Concord to Marlborough steam railway was first built in 1847 and
fell out of use in the early 20th century. But the trades are still in place, and once a
year the Massachusetts Historical Preservation Society spends three days running
the traditional trains through the beautiful landscape.

Trains will run through seven historic stations, departing from Concord every
hour. You can get on and off wherever you like, and there are great hiking trails
m /
available from Maynard and Hudson Stations. Or take the train to Marlborough

c o
c.
station, at the end of the line, where a wide range of vendors will be selling food
and drinks. Entertainment will be provided throughout the carriages, and we
encourage visitors to arrive in 19th century costumes!
g o
s n
Tickets are available in advance at www.massivehistory.com or can be bought at

m
Concord Station. Passes are valid for unlimited rides on the day of purchase, and

g
n
prices are $14 for adults and $7 for children.

k i
e a
160. Why was the notice written?

ic sp 162. What can visitors do during the first


weekend of May?

e
(A) To advertise membership in a historical

//to
society (A) Participate in a charity fundraiser near
(B) To announce an upcoming regional Hudson Station

tp : event (B) Enjoy refreshments at a train's final

ht
(C) To explain a change to transit destination
regulations (C) Borrow historical costumes at the
(D) To provide summer train service event venue
updates (D) Purchase souvenirs at Concord Station

161. What is indicated about the steam railway?


(A) It only operates on a special occasion.
(B) It is funded by a history preservation
group.
(C) It recently renovated its facilities.
(D) It offers a discount to the elderly.

306 Hackers.co.kr
Questions 163=164 refer to the following text message chain,

Kate Hendrickson [5:01 p.m.]


Jackie, have we heard back from the Loflin Institute yet? You
were supposed to ask them if their Hall D was available for our
end-of-year banquet.
Jackie Dalton [5:03 p.m.]
They said another company has already made a booking on
December 28th. But they mentioned a new ballroom they
opened last month that we could use for the party.

/
Kate Hendrickson [5:06 P.M.]
Oh, hadn't you heard? It was moved to the 27th.
Jackie Dalton [5:07 p.m.]
o m
That's news to me. call back and check for that day.
c. c
Kate Hendrickson
o
[5:08 p.m.]

g
to the staff before December 15.
s n
Great. Once that's settled, please send a notice about the event

Jackie Dalton
g m [5:09 p.m.]

i n
I'll still be on vacation then. Is the following day okay?

k
Kate Hendrickson

e a
Sounds good. Thanks a lot!
[5:11 p.m.]

ic sp
e
: //to
163. At 5:07 p.m., what does Ms. Dalton mean

p
164. When will staff receive the notice?

t
when she writes, "That's news to me"? (A) On December 15

ht
(A) She wishes she could help arrange a
banquet.
(B) She was not aware of a schedule
(B)
(C)
(D)
On
On
On
December 16
December 27
December 28
alteration.
(C) She has other plans for her holiday.
(D) She did not hear about a change in
venue.

GO ON TO THE NEXT PAGE

TEST 10 PART? 307


Questions 165-167 refer to the following notice.

/
i
The Watertown Recreation Committee invites you to join
The Fourth Annual Rock River Walking Challenge
Saturday, April 16, 9 a.m. to 1 p.m.
Each year, this race is organized by the City of Watertown as a way to promote community cooperation
and to celebrate Watertown's heritage. Open to people of all ages, the event promises a day of fun in
the great outdoors. The main event will be followed by an afternoon of merriment at the Jefferson
County Festival Grounds, where locals and visitors alike can enjoy good food, live music performances,
and a selection of beverages from the state's finest breweries. For the young and young-at-heart, games
and activities will be hosted by volunteers from the Anne Witmer Children's Foundation, which will also
be providing on-site childcare facilities.
To Join:
Simply sign up before 9 a.m. on the day of the event.
Participation costs are $10 for individuals or $5 for those participating in our group/team event.
m /
Children under 12 may participate for free.
c o
o c.
Cash prizes will be given to the individual and group that finish in first place.
And others finishing in the top 10 will receive gift vouchers from local businesses.

n
All participants will receive a commemorative T-shirt.
g
m s
If you would like to organize a team through your business, family, school, or other group, please contact the
program coordinator before April 16. A Walking Challenge Support Kit will be provided for each group,

n g
containing a guide, health forms, and handouts that you may distribute to your group's members.

i
Contact:
a k
Special thanks to our sponsors:
Rachel Bocher
e The Watertown Chamber of Commerce, The Anne Witmer Foundation,

sp
Program Coordinator The National Health Service, Green Vine Groceries, Mailman's

ic
rbocher@watertown_vt.org Pharmacy, and Jefferson Rural Bank

e
//to
v j

p :
165. What will NOT be offered at the event?
t
167. According to the notice, who should

ht (A) Locally-made drinks contact Ms. Bocher?


(B) Live performances (A) Race volunteers
(C) Day care services (B) Visiting tourists
(D) Use of a shuttle (C) Prospective vendors
(D) Group organizers
166. What is mentioned about the participants?
(A) They may attend an orientation session.
(B) They will each receive a complimentary
item.
(C) They must submit an online registration
form.
(D) They will need to wear T-shirts for the
race.

308 ^£^1 eojUAr • yojopis- □! yojopi MP3 Hackerslngang.com


Questions 168-171 refer to the following article.

There May be Hope for Small Regional Retailers After All!


By Doris Bentley

In the highly competitive world of retail sales, it's Phoenix-Star Properties provided half the funds for
difficult for inexperienced entrepreneurs to turn a the structural changes, and the rest came from the
profit when the cost of renting a store or shop is so city development budget, which is funded by local
high. But regional retailers don't need to look any taxes. — [3] —. "We are really excited about the
further now than the Bradford Retail Cooperative opening! And given Phoenix-Star's incredible
building, opening on March 1. — [1] —. success doing similar projects with other abandoned
buildings in locations across the country, I am sure
The cooperative is located in the building formerly
we will experience the same popularity here," said
occupied by the Mega-B-Mart superstore on the
city government spokesperson, Beatrice Guzman.
outskirts of Bradford City. In partnership with the city
government, Phoenix-Star Properties has updated the
structure and renovated it to house 160 small retail
The retail units will be available for rent to artisans,
craftsmen, and private businesses owners from the
m /
booths. Director of Phoenix-Star, Brenda Nixon, said,
o
region. With rental rates starting at as low as $500
c
c.
"the old structure was an eyesore ever since the store per month, Phoenix-Star has confirmed that
it once held went bankrupt. — [2] —. Through a very
small investment, we have been able to revitalize the
g o
90 percent of the spaces have already been filled. It
seems clear that the Bradford Retail Cooperative
building into a location that is not only useful to the
community but will help improve local economic -[4] -.
s n
building is already popular with local entrepreneurs.

conditions as well."
g m
k i n
168. What is the topic of the article?
e a 170. What is NOT true about the Bradford
(A)
(B)
A property development project

ic
A city's decline in constructionsp Retail Cooperative building?
(A) It has already secured tenants for

e
//to
(C) A retail branch closure many of its units.
(D) An increase in regional rents (B) It will house numerous small

tp :
169. What is indicated about Phoenix-Star
businesses.

ht
(C) It will be rented by the city government.
Properties? (D) It was partially financed by local taxes.
(A) It has been contracted by Bradford City
on several occasions. 171. In which of the positions marked [1], [2],
(B) It has transformed some vacant Mm [3], and [4] does the following sentence
structures into retail facilities. best belong?
(C) It provided the majority of the "However, it still remains to be seen
investment capital for the project. whether local shoppers will like the place
(D) It will complete its renovation work in as much."
three weeks.
(A) [1]
(B) [2]
(C) [3]
(D) [4]

GO ON TO THE NEXT PAGE.

TEST 10 PART? 309


Questions 172-175 refer to the following text message chain.

0 Alison Cambie 4:12 P.M. Just finished the first day at the Sacramento Annual Job
Fair. With only three of us at the company's booth, there
were more visitors than we could handle. Are any of you
willing to help out tomorrow? We will receive the overtime
pay rate for the hours we work.

Justin Bostwick 4:15 p.m. I can lend you a hand. But I am only available in the morning
because I have an important errand in the afternoon.

Reena Kumar 4:15 p.m, Sorry, but I have to inspect a job site for one of our clients.
Apparently, there is a problem that could delay our

/
construction of their building.

Angus O'Hara 4:17 p.m. I have nothing urgent planned for tomorrow. What time do I

o m
4:19 p.m.
need to be at the conference center?

c. c
I understand, Reena, and good luck with that! And thanks to
o Alison Cambie

g o
you others for volunteering. The doors of the exhibit center

help set up.


s n
open to visitors at 10. Please make sure you arrive early to

Justin Bostwick 4:20 p.m.


g m
I'll be there 30 minutes before the event starts.

Angus O'Hara 4:21p.m.


k i n
Do we need entry tickets?

e a
sp
o Alison Cambie 4:22 p.m. Oh, thanks for reminding me. I will leave them for you at the
administrative office located near the main entrance.

e ic
//to
Send

tp :
ht

310 Hackers.co.kr
172. What kind of company does Ms. Kumar 174. What does Ms. Cambie say she will do?
most likely work for? (A) Recruit volunteers for a job
(A) A retail store (B) Buy some tickets at a counter
(B) A moving company (C) Leave some admission passes at an
(C) A construction firm office
(D) A vehicle manufacturer (D) Wait for her colleagues by a venue's
entrance
173. At 4:15, what does Mr. Bostwick mean
when he writes, "I can lend you a hand"? 175. What is mentioned about the job fair?
(A) He can send extra staff to assist (A) It attracted few visitors on the first day.
visitors. (B) It is being held at a new building.
(B) He has some time tomorrow to work at (C) It is specific to an industry.
the booth. (D) It takes place once a year.
(C) He will be able to fill in for a coworker.

m /
(D) He is willing to register a colleague for

c o
c.
an event.

g o
s n
g m
k i n
e a
ic sp
e
p : //to
ht t

GO ON TO THE NEXT PAGE,

TEST 10 PART 7 311


Questions 176-180 refer to the following e-mails.

To: Janice Vespino <jvesp@farlington.net>


From: Mike Duff <mduff@farlington.net>
Date: February 3
Subject: Account information

Hi Janice,
This is Mike from Information Technology Services. After you gave me your list of newly hired
employees yesterday, I created usernames and passwords for them. Before I finish setting up their
personal account pages, however, I want to confirm that all their names and corresponding job
titles are correct. Here is the information I have:

John Firth: Personnel Associate


Margret Wurst: Account Executive
m /
Leslie Yang: Assistant to the Marketing Director

c o
c.
Fred Braxton: Junior Personnel Associate
Jamie Martinez: Intern

g o
I need to make sure I have their correct job positions, as those details determine what information

s n
each person has access to. For example, an account executive will be able to view our database of
clients, e-mails, whereas an intern would not be allowed to see that information. Please let me

g m
know at your earliest convenience what changes, if any, are needed.

i n
As soon as I hear back from you, I can get to work on the account pages and have them up and

k
running in just a few hours. Once that is done, each employee will need to log in with the

e a
temporary password I will send to their company e-mail accounts. They will then be prompted to

sp
enter a new password, which must be eight characters long, containing a mixture of numbers and
letters. You may also want to recommend that they change their passwords quite regularly.
Best Regards,
e ic
//to
Mike Duff

tp :
htTo: Mike Duff <mduff@farlington.net>
From: Janice Vespino <jvesp@farlington.net>
Date: February 4
Subject: Re: Account information

Hi Mike,
Thanks for the message you sent yesterday. All of the information regarding job titles is correct. Please go
ahead with your assignment, and let me know when the personal pages are running.
However, you mentioned that interns do not have access to our client e-mail database. Generally, this is
policy, but I would like to authorize an exception. The intern we have selected is actually going to be
sending a significant number of e-mails to our clients, so it would help if she had all their contact
information on hand and could view past communications. I can make her sign a confidentiality agreement
if necessary.

D
312 ¥£82^ A MRS Hackerslngang.com
I do have one further request for you, though. Could you make sure that these new employees will be able
to log on to their account pages in the NetSurfer Web browser? I know we've had trouble accessing
account pages with that browser, but as it is installed on most office computers now, please make certain
there are no similar issues for the new staff.
I appreciate your checking in with me.
Best,
Janice Vespino
Director of personnel

176. Why did Mr. Duff write to Ms. Vespino? 179. What does Ms. Vespino indicate in her
(A) To inform her of some new employees
(B) To check some information regarding
message?
(A) She discovered errors in the list of
m /
new staff names provided.
c o
(C) To update her login name and password
(D) To announce the cancellation of her
o
than one department. c.
(B) She will be directly supervising more

account

n g
(C) She needs Mr. Martinez to install a

177. What do Mr. Firth and Mr. Braxton have in


m s
browser on new office computers.
(D) She wants Mr. Martinez to have
common?
(A) They have signed a confidentiality
i n g access to clients' e-mails.

agreement.
a k 180. What is mentioned about the company's

e
(B) They will both work for Ms. Vespino. user account pages?

sp
(C) They have worked on the same project (A) They are set up by employees' direct

ic
before. supervisors.

e
(D) They were hired to do temporary work. (B) They automatically record login and

//to
password information.

:
178. In the first e-mail, the word "prompted" in (C) They were sometimes inaccessible

tp
paragraph 3, line 3, is closest in meaning using a particular Web program.

ht
to (D) They list privileges and benefits
(A) convinced provided to each staff member.
(B) hastened
(C) instructed
(D) activated

GO ON TO THE NEXT PAGE,

TEST 10 PART? 313


Questions 181-185 refer to the following article and e-mail.

New Peckham Gift Store Aims For Quality Above All


The latest addition to the up-and-coming area of Peckham is London Gifts. Opened just three weeks ago, the store
sells a variety of products created by independent artists, craftspeople, and publishers from across the country. The
store primarily specializes in books, artwork, and clothing—but everything has a British theme that appeals to
locals as well as tourists.
Most of the goods sold so far have been from London-based companies, but the shop has already begun
expanding its offerings, selling products from Finlay Street Books of Birmingham, Southern Clothing of
Southampton, and the Manchester-based artist John Stamp. London Gifts manager Stephanie Blanc is planning to
increase the variety even more in the future, giving shoppers an even greater selection of unique souvenirs and
gift items.
"There are already a lot of stores aimed at tourists in London," said Blanc, "but they tend to sell mass-produced
/
items. Our aim at London Gifts is to sell well-designed, beautifully crafted pieces that people will be interested in

m
for years to come after they return home."

c o
c.
London Gifts can be found at 12 Goldsmith Road, a 10-minute walk from Queens Road station, and is open
Monday through Saturday from 9 a.m. to 8 p.m.

g o
s n
To: Stephanie Blanc <shop(d)londongifts.co.uk>
g m
From: Sarah Broadhurst <sarah(d)oceanside.co.ul<>
Subject: Business proposal
k i n
Date: February 3

e a
Dear Ms. Blanc,
ic sp
e
My name is Sarah Broadhurst, and I run a small business called Oceanside Woodworks. We are

//to
based a few buildings down from one of your suppliers, Southern Clothing, and it was the

tp :
manager of that business who gave me your e-mail address.

ht
At Oceanside Woodworks, we make a number of products—including coasters, placemats, and
artistic pieces—and many of them would complement the selection you currently sell in your
store. In particular, we have a range of products with pictures of characters from classic British
books engraved on them. I have enclosed our catalog for this year so that you can get some
idea of the merchandise we produce.
I am always looking for retailers willing to sell our quality products in their establishments and
believe that a partnership could be of benefit to both our businesses. If you are interested in
discussing this possibility further, I would be happy to travel up to London next week with
some samples to show you. Please let me know by e-mail or by calling me at 555-6366.
Thank you for your consideration, and I hope to hear from you soon.
Sincerely,
Sarah Broadhurst
Proprietor
Oceanside Woodworks

314 Hackers.co.kr
181. What is indicated about London Gifts? 184. What does Ms. Broadhurst mention in her
(A) It opened for business a few weeks e-mail?
ago. (A) She plans to offer bulk orders to stores
(B) It is situated across from a subway needing a supplier.
station. (B) Her business has a Web site with
(C) It is owned by a large corporate product descriptions and prices.
franchise. (C) She will send some samples to
(D) It plans to launch more branches. Southern Clothing in the mail.
(D) Some of her items have a British
182. What is mentioned about the products sold theme.
at London Gifts?
(A) Some of them have been exported 185. What does Ms. Broadhurst offer to do for
abroad. Ms. Blanc?
(B) They are intended to have a higher (A) Deliver some housewares from a store

m /
o
quality than most souvenirs. in Manchester
(C) Some of them have been designed by
Stephanie Blanc. c
(B) Draw up a tentative supply contract

c.
(C) Bring selections of merchandise to
(D) They are more expensive than items London

g o
available at tourist sites.

sservices n
(D) Do some personalized engraving

183. Where does Sarah Broadhurst work?

g m
(A)
(B)
In
In
Birmingham
London
k i n
(C)
(D)
In
In
Manchester
Southampton
e a
ic sp
e
p : //to
ht t

GO ON TO THE NEXT PAGE.

TEST 10 PART 7 315


Questions 186-190 refer to the following e-mail, Web page, and review.

To: Rica Foust <ricafoust@genmail.com>


From: Casey Whitmore <caseywhitmore@planetvoyage.com>
Subject: Review
Date: Augusts

Fli Rica,
I have something you may be interested in. It relates to solo travel, which has become increasingly
popular lately. To keep up with this growing trend, we would like to feature a review of a cruise that
offers solo travel in our next issue. Would you be willing to take this assignment? Naturally, we will
cover all expenses. Since we are pressed for time, you will have to sign up as soon as possible. Your
review would be published in the September issue, so we would need the first draft by August 13.
Let me know if this is possible.

m /
Casey Whitmore

c o
c.
Managing editor
Planet Voyage

g o
s n
g m
k i n G
Seabird Ocean Liners

e a
sp
HOME TRIP PACKAGES DESTINATIONS CONTACT

ic
For single travelers: 3 nights to Baja from LA

e
//to
Enjoy a trip to the famous beaches of Baja aboard one of our ships. Relax in our spacious,

tp :
comfortable, smoke-free cabins. Spend your days by the pool or on our sports deck. Or if you prefer

ht
more luxury, book one of our staterooms, which include a wide array of amenities. Each has a TV
with hundreds of channels, computer with Wi-Fi, private shower, and dressing table. Prices for
staterooms are based on single occupancy and rates are from $437/person.

Itinerary
Day Location Arrives Departs
1 Los Angeles 5:30 P.M.
2 Baja 7:00 A.M.
3 Baja 5:00 p.M.
4 Los Angeles 8:00 A.M.

CLICK ON A SAIL DATE TO MAKE A RESERVATION


Sails: Aug. 3, Aug. 7, Aug. 13, Aug. 16, Aug. 22
o
SI Km

316 S2|i!Ah90iop|& 51 yoi°p| MP3 Hackerslngang.com


Planet Voyage September Issue
The Solo Traveler: Seabird Ocean Liners
By Rica Foust
Seabird Ocean Liners offers getaways from Los Angeles to a variety of destinations on the West Coast
of North America. I booked myself a three-night cruise from Los Angeles to Baja, and must say that the
experience was well worth the cost of the trip. The rooms were modern, comfortable, and spacious for a
cruise ship. Housekeeping was exceptional and my stateroom was always clean.
Although smaller than some cruise liners, the vessel I took boasts a modest pool, a restaurant, and a
recreational deck. Because of the vessel's size, I found it easy to get the crew's attention and to get to
know my fellow passengers. In fact, I made several new friends on the trip. If you're not the outgoing
type, the crew organizes fun activities designed to help passengers relax around one another. Now,
whether your goal as a solo traveler is to enjoy time alone or to meet new people, I think you will be

m /
satisfied with the overall experience. Taken against the backdrop of Baja's beautiful scenery, it was one
I will remember for years to come.
c o
o c.
186. Why did Mr. Whitmore write to Ms. Foust?
g
189. In the review, the word "exceptional" in
n
(A) To complain about a trip he took
(B) To schedule an interview for an article to
m s
paragraph 1, line 4, is closest in meaning

(C) To assign an article for a future

i n g
(A) superb
publication
(D) To ask her to attend a business
a k (B)
(C)
abnormal
crucial
meeting
e (D) expected

ic sp
187. Which sail date did Ms. Foust most likely 190. What can be inferred about Ms. Foust?
reserve?
e
//to
(A) She had to pay extra for some
(A) August 3 activities.
(B)

tp : August 7 (B) Her room was smaller than those of

ht
(C) August 13 fellow passengers.
(D) August 16 (C) She plans on returning to Baja on
another cruise.
188. What is indicated about Seabird Ocean (D) Her room was equipped with a
Liner? television.
(A) Its cruise ships were built earlier in the
year.
(B) It accepts trip bookings online.
(C) It has added extra destinations.
(D) Its trips to Baja will be discontinued
soon.

GO ON TO THE NEXT PAGE.

TESTIO PART 7 317


Questions 191=195 refer to the following advertisement, order form, and e-mail.

Today only! Our special offer will expire in [o]days, [13hours, and UTIminutes.

BRILLIANT TECHWARE is offering Flash-805 distance recorders at 40% off the


regular price.
The device's onboard microphone is outstanding at picking up any sound and reproducing it
faithfully. Plus, our patented waveform analyzer technology reduces distortion automatically.
This amazing recorder lets you remove unwanted sounds such as traffic noise and
background chatter so that you only capture the sounds you want and need. It even has
50 percent more battery life than our previous models. The Flash-805 is a truly versatile tool
with multiple applications for students, researchers, and reporters.
^Brilliant Techware provides free delivery on orders of $40 or above, except for items on
m /
promotion. For orders below $40, we charge $5 for delivery within North America and $10

c o
c.
for deliveries to most other parts of the world. For expedited shipping, there is a $5
surcharge.

g o
s n
g m
Order Form

k i n service@briltechware.com

a
www.brillianttechware.com

e
sp
BRILLIANT TECHWARE
755 Orange Park
Miami, FL 32003

e ic
//to
Customer Name: Chris Altman
Address: 11 Cornish Hill, Hesbitt, Bournemouth, UK

tp :
E-mail address: caltman@ukmail.com
Phone no.: 555-9100

ht Date: 10/17 11:23 a.m.


ORDER No.: 7891000

Product Code Description Price/unit Quantity Sub-total


3101805 Flash-805 distance recorder $145.00 1 unit $145.00
Less 40% discount ($58.00)
Subtotal $87.00
Shipping $10.00
Surcharge $5.00
TOTAL $102.00
Each item can be discounted once only. Your purchase will appear on your credit card bank
statement as BrilliantTech. All orders will ship from 3-4 business days of purchase. Cancellations
may be made within 24 hours of a purchase.
Any customs taxes must be paid by the customer.

318 Hackers.co.kr
To: service@briltechware.com
From: caltman@ukmail.com
Subject: Inquiry
Date: Thursday, October 17

To Whom It May Concern,


I ordered a Flash-805 distance recorder on your Web site this morning. My order number is 7891000.
I told my coworker about my purchase, and she is also interested in ordering the same item since you
offer the lowest price she has found on the Web. In addition, she was intrigued by its ability to reduce
distortion automatically. 1 was wondering whether I could simply add her recorder to my current
order. If so, would there be an additional fee to do that? We just wanted to see if we could save on
shipping. Even if you cannot change my order, my friend will go ahead with the purchase anyway.
I hope to hear from you today so that my friend can take advantage of this limited-time offer.

m /
Sincerely,

c o
Chris Altman

o c.
n g
191. What is NOT mentioned about the
Flash-805 distance recorder?
s
194. What is the purpose of the e-mail?

m
(A) It can be assembled manually by the
i n g
(A) To complain about the high shipping
cost
user.

a k (B) To inquire about changing an order

e
(B) It accurately duplicates the original (C) To request a cancellation of a recent

sp
sound. purchase

ic
(C) Its battery lasts longer than older models. (D) To point out a defect in a product

e
(D) It is capable of deleting undesired

//to
noise. 195. What is true about Mr. Altman's colleague?

:
(A) She used to own a Brilliant Techware
192. What must customers do to take

tp product.

ht
advantage of a special offer? (B) She has written to customer service
(A) Download an online coupon several times.
(B) Purchase a quantity of two or more (C) She read a positive online review
items about Brilliant Techware.
(C) Place an order within specific amount (D) She is interested in the Flash-805's
of time patented feature.
(D) Provide proof of residence within
Bournemouth

193. What is suggested about Mr. Altman?


(A) He will have three days to cancel his
order.
(B) He wants to use another delivery
company.
(C) He requested expedited delivery.
(D) He is working on a research project. GO ON TO THE NEXT PAGE

TEST 10 PART? 319


Questions 196-200 refer to the following announcement, form, and message.

Enter O-Deli's "Name Our Sandwich'' Competition and Win $1,000!

O-Deli's will be introducing its newest sandwich sensation on February 1! This delicious
combination of toasted rye bread, roasted turkey, spicy sauce, melted cheddar cheese, and
fresh lettuce and onions still doesn't have a name! And that's where you come in. Customers
can head to any O-Deli's branch in New York State and try out this incredible new menu
item. Taste it, come up with a name for it, and submit an entrance form at any one of our
sixteen branches. The winner not only will receive a cash prize of $1,000, but all three
finalists will receive a voucher for a weekly complimentary sandwich at any O-Deli's location
for one year! And upon submission of an application form, everyone entering will also be
given a coupon for a free medium size beverage at our establishments.

m /
c o
r
O-Deli's "Name Our Sandwich" Competition: Application form
o c.
i Name: Cheryl Rosen
n g
E-mail: cherposen@anypost.com
Mobile: 555-0293
m s a
n
Date of Application: February 18

i ng
i
Location of Outlet: 45th Avenue, New York City

a
Describe the sandwich to us in your own words: k i

e i

sp
This delightful sandwich makes a spicy, fresh addition to any lunch. The tangy cheese i
complements the meat, and its $4.89 price is a great value.

ic
i

e
Please give us your suggestion for a sandwich name (5 words or less): i

//to
Red Hot Turkey Sandwich

tp :
Three finalists will be notified by text message and receive invitations to a special promotional event to

ht
celebrate the launch of the new sandwich. The event will be held on February 28, when the winner will |
be announced. ||
L1 ■■ HK 1 mm <^m ' ■■ ' HI - ■■ IH1 ■■§ HI - 1 mm urn ■■ hh bm ■■ ■■■ HB ^m J

FROM: O-Deli's (555-2737)


TO: Ray Colsen (555-0320), Amos Vaughn (555-3928), Fiona Henry (555-6554)

Hello finalists! Thank you all for confirming your attendance to the event. Unfortunately, there has been a
change we need to notify you of. We had planned to hold the event at our 59th Street location, but that
branch has been experiencing some electrical problems. So, we will announce the winner at our 45th
Avenue branch instead. Please confirm as soon as possible whether or not you can still attend. If you
need assistance with transportation to the new site for the event, let us know and we can make
arrangements for you. We apologize for the inconvenience and hope to see you all there.

D
320 ^ A t^opi mps Hackerslngang.com
196. What is the announcement mainly about? 199. What can be inferred about Mr. Vaughn?
(A) A new franchise branch (A) He did not get an invitation letter sent
(B) A naming contest in February.
(C) A discounted menu item (B) He agreed to attend an event at 59th
(D) A cooking competition Street on February 28.
(C) He is a frequent diner at O-Deli's
197. What is indicated about O-Deli's on the restaurants.
announcement? (D) He sent a message requesting special
(A) It operates multiple locations within a assistance.
state.
(B) It recently announced plans to open 200. Why has O-Deli's changed its plans for an
new branches. event?
(C) It is currently not open for business. (A) An important executive will be unable
(D) It regularly sponsors monthly to attend.

m /
competitions. (B) A space is not large enough for

c o
c.
participants.
198. What did Ms. Rosen probably receive? (C) A technical concern has occurred at a
(A) A personalized sandwich venue.
g o
(B)
(C)
A
A
cash prize for a contest
coupon for a complimentary drink
s
unavailable. n
(D) A transport provider became

(D) A list of new menu items

g m
k i n
e a
ic sp
e
p : //to
ht t

This is the end of the test. You may review Part 5, 6, and 7 if you finish the test early.

0 0
lA
m p.326 / p.327 / P.A04 / Part 5&6 ^ ^ ^ S7|
♦ nuowoil Ol^ Self m EL| AE1 £*1| #0| EH^l M*i|o

TEST 10 PART? 321

You might also like